Ilovepdf Merged

You might also like

Download as pdf or txt
Download as pdf or txt
You are on page 1of 248

SSLC Mathematics Solutions – Part -1 YK

S.S.L.C Contents
Part -1

Sl.No. Chapters Page No.

1 Arithmetic Progression 1 - 31

MATHS
Yakub Koyyur 2 32 - 62
Triangles
GHS nada
Belthangady 3 Pair of Linear equation in two variables 63 - 98
taluk
D.K. – 574214 4 Circles 99 - 107
Email:
yhokkila@gmail.com
English Medium 5 Area Related to circles 108 - 122

Part-1 6

7
Constructions

Coordinate Geometry
123 - 132

133 - 149

All Solutions 8 Real Numbers 150 - 159

Based on new syllubus

P a g e 1 | 159 Available in ykoyyur.blogspot.com

Available in:ykoyyur.blogspot.com
SSLC Mathematics Solutions – Part -1 YK SSLC Mathematics Solutions – Part -1 YK
Infinite AP.:
Arithmetic progression In an AP there are infinite number of terms. Such an AP is called a infinite AP. Each of
these Arithmetic Progressions (APs) do not have last term.
1.2 Arithmetic Progressions: a) 3, 7, 11, . . . .
b) 1, 4, 7, 10, . . .
An arithmetic progression is a list of numbers in which c) -10, -15, -20, . . . .
each term is obtained by adding a fixed number to Note: You will If we know the first term a’ and the common difference d’ then we can
the preceding term except the first term. write an AP.
Example 1: , , − , − ........... write the first term a and the common difference d.
Here, = d= - = - = -1 - =− - = -1
i) 1, 2, 3, 4 ................. each term is 1 more than the term preceding it.
Example 2 : Which of the following list of numbers form an AP? If they form an AP,
ii) 100, 70, 40, 10 ........ each term is 30 less than the term preceding it. write the next two terms
i) 4, 10, 16, 22 ........
each term is obtained by adding 1 to the term preceding it.
iii) -3, -2, -1, 0 .............. ii) 1, -1, -3, -5 ........
iii) -2, 2, -2, 2 ........
all the terms in the list are 3 , i.e., each term is obtained by
iv) 3, 3, 3, 3 ................ iv) 1, 1, 1, 2, 2, 2, 3, 3, 3 ..........
adding(or subtracting) 0 to the term preceding it.
Solution :
-1, -1.5,-2.0,-2.5 each term is obtained by adding – 0.5 to (i.e., subtracting 0.5 i) 4, 10, 16, 22 ........
v) from) the term preceding it. - = 10 – 4 = 6
.......
This fixed number is called the common difference of the AP. Remember that it can be positive, - = 16 – 10 = 6
negative or zero. - = 22 – 16 = 6
i.e., ak + 1 – ak is the same every time.

Let us denote the first term of an AP by a1, second term by a2, So, the given list of numbers forms an AP with the common difference d = 6. The next two
. . ., nth term by an and the common difference by d. Then terms are: 22 + 6 = 28 and 28 + 6 = 34.
ii) 1, -1, -3, -5 ........
the AP becomes a1, a2, a3, . . ., an.
- = -1 – 1 = -2
So, a2 – a1 = a3 – a2 = . . . = an – an – 1 = d - = -3 – (-1) = -2
a, a + d, a + 2d, a + 3d, . . . - = -5 – (-3) = -2
i.e., ak + 1 – ak is the same every time.
Represents an arithmetic progression where a is the first term
and d the common difference. This is called the general form So, the given list of numbers forms an AP with the common difference d = -2. The next two
of an AP. terms are: – 5 + (– 2 ) = – 7 and – 7 + (– 2 ) = – 9
iii ) -2, 2, -2, 2 ........
Finite AP.: - = 2 – (-2) = 2 + 2 = 4
In an AP there are only a finite number of terms. Such an AP is called a finite AP. Each - = -2 – 2 = -4
of these Arithmetic Progressions (APs) has a last term. - = 2 – (-2) = 2 + 2 = 4
Here, ≠ So, the given list of numbers does not form an AP.
a) The heights ( in cm ) of some students of a school standing in a queue in the morning
iv) 1, 1, 1, 2, 2, 2, 3, 3, 3 ..........
assembly are 147 , 148, 149, . . ., 157.
b) The balance money ( in Rs ) after paying 5 % of the total loan of Rs 1000 every month - =1 – 1 =0
- =1 – 1 =0
is 950, 900, 850, 800, . . ., 50.
- = 2 – 1 =1
c) The total savings (in Rs ) after every month for 10 months when Rs50 are saved each
- = - ≠ - So, the given list of numbers does not form an AP.
month are 50, 100, 150, 200, 250, 300, 350, 400, 450, 500.

P a g e 2 | 159 Available in ykoyyur.blogspot.com P a g e 3 | 159 Available in ykoyyur.blogspot.com


SSLC Mathematics Solutions – Part -1 YK SSLC Mathematics Solutions – Part -1 YK
EXERCISE 1.1 i) a = 10, d = 10
1. In which of the following situations, does the list of numbers involved make an arithmetic = 10,
progression, and why? = + d = 10 + 10 = 20
= + d = 20 + 10 = 30
i). The taxi fare after each km when the fare is Rs15 for the first km and Rs 8 for each
= + d = 30 + 10 = 40
additional km Thus the first four terms of an AP are 10, 20, 30, 40
The first term = 15, = 15 + 8 = 23 , = 23 + 8 = 31 ......... ii) a = -2, d = 0
Here, each term is obtained by adding a common difference = 8, except first term. = -2,
= + d = -2 + 0 = -2
ii). The amount of air present in a cylinder when a vacuum removes of the air = + d = -2 + 0 = -2
remaining in the cylinder at a time. = + d = -2 + 0 = -2
Let the initial volume of the air present in the cyclinder be V. Thus the first four terms of an AP are -2, -2, -2, -2,
iii) a = 4, d = -3
The remaining air in the cylinder after using vacuum pump first time V - = ,
= 4,
Remaining air in the cylinder after using vacuum pump second time = +d=4-3= 1
− x = − = and so on. = + d = 1 - 3 = -2
= + d = -2 - 3 = -5
Here, the terms are V, , .. ….. Thus the first four terms of an AP are 4, 1, -2, -5
- = − V = − iv) a = -1, d =
- = − = − = − = -1,
∴ - ≠ - = + d = -1 + = −
Hence, it does not form an AP = +d=− + = 0
iii). The cost of digging a well after every metre of digging, when it costs Rs 150 for the first = +d= 0 + =
metre and rises by Rs 50 for each subsequent metre.
The cost of digging for the first meter = Rs 150 Thus the first four terms of an AP are -1, − , 0,
Cost of digging for the second meter = 150+50 = Rs 200 v) a = -1.25, d = -0.25
Cost of digging for the third meter = 200+50 = Rs 250 = -1.25
Cost of digging for the fourth meter = 250+50 = Rs 300 = + d = -1.25 - 0.25 = -1.50
Thus the list of numbers is 150, 200, 250, 300........ = + d = -1.50 - 0.25 = - 1.75
Here, we can find the common difference = 50 = + d = -1.75 + 0.25 = -2.00
So it forms an AP. Thus the first four terms of an AP are -1.25, -1.50, -1.75, -2.00
iv). The amount of money in the account every year, when Rs 10000 is deposited at 3. For the following APs, write the first term and the common difference:
compound interest at 8 % per annum. i) 3, 1, - 1, - 3.......
The first term a = 3,
We know that amount A = P 1 + Common difference d = - = 1 – 3 = -2
Here, P = 10,000; r = 8%, n = 1,2,3… ii) -5, -1, 3, 7.......
Amount in first year = 10000 1 + = 10000x = 100x108 = Rs 10800 The first term a = 5
Common difference d = - = -1 – (-5) = -1 + 5 = 4
Amount in second year = 10000 1 + = 10000x x = 108 x 108 = Rs 11664 iii) , , , , . . . . .
Thus the list of numbers is 10000, 10800, 11664 .. ….. The first term a =
- = 10800 − 10000 = 800
Common difference d = - = – =0
- = 11664 − 10800 = 864
iv) 0.6, 1.7, 2.8, 3.9, ........
There for - ≠ -
The first term a = 0.6
Hence it does not form an AP. Common difference d = - = 1.7 – 0.6 = 1.1
2. Write first four terms of the AP, when the first term a and the common difference d are 4. Which of the following are APs ? If they form an AP, find the common difference d and
given as follows: write three more terms

P a g e 4 | 159 Available in ykoyyur.blogspot.com P a g e 5 | 159 Available in ykoyyur.blogspot.com


SSLC Mathematics Solutions – Part -1 YK SSLC Mathematics Solutions – Part -1 YK
i) 2, 4, 8, 16 ......... a - a = -8 – (-4) = −8 + 4 = - 4
- =4–2=2 a - a = -12 – (-8) = −12 + 8 = -4
- =8–4=4 Here, a - a = a - a = a - a
Here, - ≠ - Therefore the given list of numbers forms an AP with common difference d = -4
There fore the given list of numbers does not form an AP. The next 3 terms of this AP are -12 – 4 = -16; -20; -24
ii) 2, , 3, ……… iv) − , − , − , − , . . . . .
a -a = –2= a -a =− – − =− + =0
a -a =3– = a -a =− – − =− + =0
a -a = –3= a -a =− – − =− + =0
Here, a - a = a - a = - Here, a - a = a - a = a - a
Therefore the given list of numbers forms an AP with common difference d = Therefore the given list of numbers forms an AP with common difference d=0
The next 3 terms of this AP are, + = 4; 4 + = ; + =5 The next 3 terms of this AP are − , − , − ,
iii) -1.2, -3.2, -5.2, -7.2 ..... v) 1, 3, 9, 27 ..........
a - a = -3.2 – (-1.2) = -3.2 + 1.2 = -2 a -a =3–1 =2
a - a = -5.2 – (-3.2) = -5.2 + 3.2 = -2 a -a =9–3 =6
a - a = -7.2 – (-5.2) = -7.2 + 5.2 = -2 Here, a - a ≠ a - a
Here, a - a = a - a = a - a There fore the given list of numbers does not form an AP.
Therefore the given list of numbers forms an AP with common difference d = −2 vi) a, 2a, 3a, 4a .........
The next 3 terms of this AP are, -7.2 – 2 = -9.2 ; -9.2 – 2 = −11.2; −11.2 – 2 = -13.2 a - a = 2a – a = a
iv) -10, -6, -2, 2 ...... a - a = 3a – 2a = a
a - a = -6 – (-10) = -6 + 10 = 4 a - a = 4a – 3a = a
a - a = -2 – (-6) = -2 + 6 = 4 Here,a - a = a - a = a - a
a - a = 2 – (-2) = 2 + 2 = 4 Therefore the given list of numbers forms an AP with common difference d=a
Here, a - a = a - a = a - a The next 3 terms of this AP are 5a, 6a, 7a
Therefore the given list of numbers forms an AP with common difference d = 4 vii) a, a2, a3, a4…….
The next 3 terms of this AP are 2 + 4 = 6 ; 6 + 4 = 10; 10 + 4 = 14 a - a = a2 – a = a(a − 1)
i) 3, 3+ √ , 3 + √ , 3 + 3√ , ……… a - a = a3 – a2 = a2(a – 1)
a - a = 3+ √2 – 3 = √2 Here,a - a ≠ a - a
a - a = 3 + 2√2 – 3+ √2= √2 There fore the given list of numbers does not form an AP.
a - a = 3 + 3√2 – 3 + 2√2 = √2 viii) √ , √ , √ , √ ……..
Here, a - a = a - a = a - a a - a = √8 – √2 = 2√2 - √2 = √2
Therefore the given list of numbers forms an AP with common difference d = √2 a - a = √18 – √8 = 3√2 - 2√2 = √2
The next 3 terms of this AP are 3 + 3√2 + √2 = 3 + 4√2; 3 + 5√2; 3 + 6√2 a - a = √32 – √18 = 4√2 - 3√2 = √2
ii) 0.2, 0.22, 0.222, 0.2222 ............. Here,a - a = a - a = a - a
a - a = 0.22 – 0.2 = 0.02 Therefore the given list of numbers forms an AP with common difference d = √2
a - a = 0.222 - 0.22 = 0. 002 The next 3 terms of this AP are √50 , √72 , √98
Here, a - a ≠ a - a ix) √ , √ , √ , √ ……….
There fore the given list of numbers does not form an AP. a - a = √6 – √3
iii) 0, -4, -8, -12 ............ a - a = √9 – √6 = 3 - √6
a - a = -4 – 0 = −4 Here,a - a ≠ a - a

P a g e 6 | 159 Available in ykoyyur.blogspot.com P a g e 7 | 159 Available in ykoyyur.blogspot.com


SSLC Mathematics Solutions – Part -1 YK SSLC Mathematics Solutions – Part -1 YK
There fore the given list of numbers does not form an AP. 0 = 21 − 3n + 3
x) 1 1 , 32, 52, 72, …… 3n = 24
n=8
a - a = 32 – 1 1 = 9 – 1 = 8 8th term is Zero
a - a = 52 – 3 2 = 25 – 9 = 16 Example 5 : Determine the AP whose 3rd term is 5 and the 7th term is 9.
Here,a - a ≠ a - a Solutin: + ( − 1) =
There fore the given list of numbers does not form an AP. + (3 − 1) = 5 Alternate Method:
xv) 11, 52, 72, 73, ………. + 2 = 5 ------- (1)
d=
+ (7 − 1) = 9
a - a = 52 – 1 1 = 25 - 1 = 24 + 6 = 9 --------(2) a =a ;a =a
a - a = 72 – 5 2 = 49 - 25 = 24 + = 5 d= = = =1
a - a = 73 – 72 = 73 - 49 = 24 + = 9 a= + ( − 1) a
Here, a - a = a - a = a - a a = + ( 7 − 1)1
-4d = - 4
Therefore the given list of numbers forms an AP with common difference d = 24 a = 9 + ( 7 − 1)1
⇒ d=1
The next 3 terms of this AP are 73 + 24 = 97, 97 + 24 = 121, 121 + 24 = 145 a=9+6 =3
⇒ + 2(1) = 5 ⇒ a + 2 = 5 ⇒ a = 5 – 2 = 3
1.3 nth Term of an AP ∴ AP: 3, 4, 5, 6, - - -
Example 6 : Check whether 301 is a term of the list of numbers 5, 11, 17, 23, . . .
′ Common difference is d' then the nth term
Solution: a = 5, d = 11 – 5 = 6
is
+ ( − 1) =
5 + ( − 1)6 = 301
= + − 5 + 6n − 6 = 301
6n − 1 = 301
6n = 301 + 1
nth term from the last n [ -last term , − 6n = 302
n = =
− − Here n is not an integer
There fore 301 is not a term of the list of numbers 5, 11, 17, 13 .........
Example 3 : Find the 10th term of the AP : 2, 7, 12, . . . Example 7 : How many two-digit numbers are divisible by 3?
Solution : a = 2, d = 7 - 2 = 5 and n = 10 Solution: 12, 15, 18 ......99
= + ( − 1) a = 12, d = 3, an = 99
= 2 + (10 − 1)5 + ( − 1) =
= 2 + (9)5 12 + ( − 1)3 = 99
= 2 + 45 12 + 3n − 3 = 99
= 47 3n + 9 = 99
Example 4 : Which term of the AP : 21, 18, 15, . . . is – 81? Also, is any term 0? Give 3n = 99 – 9
3n = 90 ⇒ n = 30
reason for your answer.
There for 30 two digit numbers are divisible by 3.
Solution: a = 21, d = 18 - 21 = -3 and = -81. Now we have to find n
Example 8 : Find the 11th term from the last term (towards the first term) of the
= + ( − 1) AP : 10, 7, 4, . . ., – 62.
−81 = 21 + (n − 1)(-3) Solution: a = 10, d = 7 - 10 = -3, l = -62
−81 = 21 − 3n + 3 = + ( − 1)
−81 = 24 − 3n nth term from the last = − ( − 1)
3n = 24 + 81 = 105 = −62 − (11 − 1)(−3)
n = 35 = −62 + 33 − 3
which term is Zero? = −62 + 30
0 = 21 + (n − 1)(−3) = −32

P a g e 8 | 159 Available in ykoyyur.blogspot.com P a g e 9 | 159 Available in ykoyyur.blogspot.com


SSLC Mathematics Solutions – Part -1 YK SSLC Mathematics Solutions – Part -1 YK
Example 9 : A sum of Rs 1000 is invested at 8% simple interest per year. Calculate the a = 28
interest at the end of each year. Do these interests form an AP? If so, find the interest at i) a = a + (n − 1)d
the end of 30 years making use of this fact. 0 = −18 + (10 − 1)d
Solution: The formula to calculate the simple intrest I = 0 = −18 + 9d
9d = 18
So, the interest at the end of the 1st year = = Rs 80 d = 2
the interest at the end of the 2nd year = = Rs 160 ii) = +( − )
−5 = a + (18 − 1)(−3)
the interest at the end of the 3rd year = = Rs 240 −5 = a − 17x3
There fore the terms are 80, 160, 240, - - - −5 = a – 51
Here - = - = d = 80 a = 46
It is an AP as the difference between the consecutive terms in the list is 80, iii) = +( − )
The interest at the end of 30 years a n; a = 80, d = 80, n = 30 3.6 = −18.9 + (n − 1)(2.5)
3.6 = −18.9 + 2.5n − 2.5
a = a + (n − 1)d
3.6 = −21.4 + 2.5n
a = 80 + (30 − 1)80
2.5n = 3.6 + 21.4
a = 80 + 29x80
a = 80 + 2320 n = = = 10
.
a = Rs 2400 iv) = +( − )
Example 10 : In a flower bed, there are 23 rose plants in the first row, 21 in the second, a = 3.5 + (105 − 1)(0)
19 in the third, and so on. There are 5 rose plants in the last row. How many rows are a = 3.5 + 104x0
there in the flower bed? a = 3.5
Solution: The number of rose plants in the 1st, 2nd, 3rd, . . ., rows are :23, 21, 19,- - - 2. Choose the correct choice in the following and justify :
Here - = - = -2 (i) 30th term of the AP: 10, 7, 4, . . . , is
There fore it is an AP. a = 23, d = -2, an = 5, n = ? = + ( − 1)
+ ( − 1) = d = a - a = 7 – 10 = -3
23 + ( − 1)(−2) = 5 a = 10 + (30 − 1)(−3)
23 − 2n + 2 = 5 a = 10 + (29)(−3)
−2n + 25 = 5 a = 10 − 87
−2n = 5 – 25 a = −77
−2n = −20 (A) 97 (B) 77 (C) -77 (D) -87
n = 10 (ii) 11 th term of an AP: -3, - , 2, ..... is
So, there are 10 rows in the flower bed. = 10. a = a + (n − 1)d
EXERCISE 1.2 d = a - a = - – (-3) = - + 3 =
5
1. Fill in the blanks in the following table, given that a is the first term, d the common a = −3 + (11 − 1)
difference and a the nth term
n of the AP: 2
5
a d n an a = −3 + (10)
2
(i) 7 3 8 28 a = −3 + 25
(ii) − 18 2 10 0 a = 22
(iii) 46 −3 18 −5 (A) 28 (B) 22 (C) -38 (D) -48
(iv) − 18.9 2.5 10 3.6 3. In the following APs, find the missing terms in the boxes :
(v) 3.5 0 105 3.5 i) 2, , 14 ,, 26

i) = +( − ) ii) ,
18 ,, 13, 8 ,, 3,
a = 7 + (8 − 1)3
a = 7 + 7x3 iii) ,, 5, ,, 6 , 8 ,,, 9 ,
a = 7 + 21
P a g e 10 | 159 Available in ykoyyur.blogspot.com P a g e 11 | 159 Available in ykoyyur.blogspot.com
SSLC Mathematics Solutions – Part -1 YK SSLC Mathematics Solutions – Part -1 YK
iv),, -4,,, -2 , , 0 , , 2 ,, 4 ,,, 6 164
n =
3
n is not an integer. So, -150 is not a term of the AP: 11, 8, 5, 2, ..
v) ,, 53,,,, 38,,, 23, , ,, 8, ,,, -7, , −22
7. Find the 31st term of an AP whose 11th term is 38 and the 16th term is 73.
4. Which term of the AP : 3, 8, 13, 18, . . . ,is 78? Solution:a = a + (n − 1)d Alternate Method:
Solution:a = a + (n − 1)d a = 38, a = 73, a = ?
d=
d = a - a = 8 – 3 = 5; a = 3; an = 78; n = ? a + (11 − 1)d = 38
78 = 3 + (n − 1)5 a + 10d = 38 ---------------(1) a =a ;a =a
78 = 3 + 5n − 5 a + (16 − 1)d = 73 d= = = =7
78 = 5n − 2 a + 15d = 73 ----------------(2)
5n = 78 + 2 = +( − ) a
from (1) and (2) = + ( 31 − 16)7
5n = 80
+ = = 73 + ( 15)7
n = 16
5. Find the number of terms in each of the following APs : + = = 73 + 105 = 178
Solution: i) 7, 13, 19 ....... 205 -5d = -35
a = a + (n − 1)d d = =7
d = a - a = 13 – 7 = 6; a = 7; an = 205; n = ? (1) ⇒ a + 10x7 = 38
205 = 7 + (n − 1)6 ⇒ a + 70 = 38
205 = 7 + 6n − 6 ⇒ a = 38 − 70
205 = 6n + 1
⇒ a = −32
6n = 205 − 1
a = −32 + (31 − 1)7
6n = 204
a = −32 + (30)7
n =
a = −32 + 210
n = 34 a = 178
(ii) 18, 15 , 13 ....... -47 8. An AP consists of 50 terms of which 3rd term is 12 and the last term is 106. Find the 29th
a = a + (n − 1)d term.50
d = a - a = 15 – 18 =- ; a = 18; an = -47; n = ?
5 Solution:a + (n − 1)d = a
−47 = 18 + (n − 1) − n = 50, a = 12, a = 106 a = ?
2
5 5 a + (50 − 1)d = 106
−47 = 18 − n + a + 49d = 106 -------- (1) Alternate Method:À
2 2
36 − 5n + 5 a + 2d = 12 -------- -(2) d=
−47 = + =
2 a =a ;a =a
41 − 5n + =
−47 = 47d = 94 d= = = =2
2
−94 = 41 − 5n ⇒d = 2 = +( − ) a
−5n = −94 − 41 Substitute d = 2 in eqn (2) = + ( 29 − 3)2
−5n = −135 a + 2(2) = 12 = 12 + ( 26)2
n = 27 a + 4 = 12 = 12 + 52 = 64
6. Check whether – 150 is a term of the AP : 11, 8, 5, 2 . . . a = 12 - 4
Solution:a = a + (n − 1)d
a = 8
d = a - a = -3; a = 11; an = 150; n = ?
a = 8 + (29 − 1)2
−150 = 11 + (n − 1)(−3)
a = 8 + (28)2
−150 = 11 − 3n + 3
a = 8 + 56
−150 = 14 − 3n
a = 64
−3n = −150 − 14
9. If the 3rd and the 9th terms of an AP are 4 and – 8 respectively, which term of this AP is
−3n = −164
zero?

P a g e 12 | 159 Available in ykoyyur.blogspot.com P a g e 13 | 159 Available in ykoyyur.blogspot.com


SSLC Mathematics Solutions – Part -1 YK SSLC Mathematics Solutions – Part -1 YK
Solution:a = 4, a = −8 Alternate method:À
12. Two APs have the same common difference. The difference between their 100th terms is
an = a + (n − 1) d 100, what is the difference between their 1000th terms?
a3 = a + (3 − 1) d d= Solution:Let the first terms of an AP’s be a’ and b’. Common difference – d
4 = a + 2d ------------(i) a =a ;a =a For the first AP,
a9 = a + (9 − 1) d
−8 = a + 8d ---------- (ii) d = = = = -2 a = a + (100 − 1) d
Substract (i) from (ii),we get = + ( − 1) a a = a + 99d
−12 = 6d ⇒ d = −2 = + ( 3 − 1)(−2) a = a + (1000 − 1) d
From equation (i), = 4 + ( 2)(−2) a = a + 999d
4 = a + 2 (−2) =4−4 =0 For 2 nd AP,
4=a−4 a = b + (100 − 1) d
a=8 a = b + 99d
If an = 0 , a = b + (1000 − 1) d
an = a + (n − 1) d a = b+ 999d
0 = 8 + (n − 1) (−2) = 8 − 2n + 2 The difference of 100th terms is 100
2n = 10
There for (a + 99d) − (b + 99d) = 100
n=5
a − b = 100 ------------------- (i)
So, the 5th term is 0
The difference of 1000th terms is ?
10. The 17th term of an AP exceeds its 10th term by 7. Find the common difference.
Solution:an = a + (n − 1) d (a + 999d) − (b + 999d) = a − b
a = a + (17 − 1) d From equation (i) ,
a = a + 16d a1 − a2 = 100
Similarlly, a = a + 9d So, the difference of 1000th terms is 100.
But , a − a = 7 13. How many three-digit numbers are divisible by 7?
(a + 16d) − (a + 9d) = 7 Solution:The first 3 digit number which is divisible by 7 is a = 105 and d = 7
7d = 7 The last 3 digit number which is divisible by 7 is = 994
d=1 There for AP: 105, 112, 119, …994
11. Which term of the AP: 3, 15, 27, 39, . . . will be 132 more than its 54th term? n=?
Solution:AP: 3, 15, 27, 39, … an = a + (n − 1) d
a = 3, 994 = 105 + (n − 1) 7
d = a − a = 15 − 3 = 12 889 = (n − 1) 7
a = a + (54 − 1) d (n − 1) = 127
a = 3 + (53) (12) n = 128
a = 3 + 636 = 639 There for 128 three digit numbers are divisible by 7.
132 + 639 = 771 Or
Now we find which term is 771 The 3-digit numbers which are divisible by 7 are 105, 112, 119, .... 994 .
a = 771. These numbers are in AP:
an = a + (n − 1) d a = 105 and d = 7, an = 994
771 = 3 + (n − 1) 12 ⇒ a + (n - 1) d = 994
768 = (n − 1) 12 ⇒ 105 + (n - 1) × 7 = 994
(n − 1) = 64 ⇒7(n - 1) = 889
n = 65 ⇒ n - 1 = 127
There fore 65th term is 132 more than 54th term. ⇒ n = 128
Or 14. How many multiples of 4 lie between 10 and 250?
n’th term is 132 more than 54th term. Solution:Multiples of 4 lie between 10 and 250 are 12, 16, 20, 24, …248
n = 54 + a = 12, d = 4, a = 248
= 54 + 11 = 65th term. an = a + (n - 1) d

P a g e 14 | 159 Available in ykoyyur.blogspot.com P a g e 15 | 159 Available in ykoyyur.blogspot.com


SSLC Mathematics Solutions – Part -1 YK SSLC Mathematics Solutions – Part -1 YK
248 = 12 + (n - 1) × 4 an = a + (n − 1) d
248 = 12 + 4n – 4 a = a + (4 − 1) d
248 = 8 + 4n a = a + 3d
4n = 248 - 8 Similarlly,
4n = 240 a = a + 7d
n = 60 a = a + 5d
Hence, ther are 60 multiples of 4 lie between 10 and 250. a = a + 9d
15. For what value of n, are the nth terms of two APs: 63, 65, 67 , . . . and 3, 10, 17, . . . equal? But , a + a = 24
Solution: a = 63, d = a − a = 65 − 63 = 2 a + 3d + a + 7d = 24
an = a + (n − 1) d 2a + 10d = 24
an = 63 + (n − 1) 2 = 63 + 2n − 2
a + 5d = 12 ------------(i)
an = 61 + 2n ----------- (i)
a + a = 44
3, 10, 17, …
a = 3, d = a − a = 10 − 3 = 7 a + 5d + a + 910d = 44
an = 3 + (n − 1) 7 2a + 14d = 44
an = 3 + 7n – 7 a + 7d = 22 -----------(ii)
an = 7n − 4 ------------ (ii) By substracting (ii) from (i),
According to question , nth term of both AP’s are equal. 2d = 22 − 12
⇒61 + 2n = 7n − 4 2d = 10
⇒61 + 4 = 5n ⇒5n = 65 d=5
⇒ n = 13 Substituting d = 5 in equation (i) ,
Hence, the 13th the two given AP’s are equal.
16. Determine the AP whose third term is 16 and the 7th term exceeds the 5th term by 12. a + 5d = 12
a = 16 a + 5 (5) = 12
a + (3 − 1) d = 16 a + 25 = 12
a + 2d = 16 ---------- (i) a = −13
a − a = 12 a2 = a + d = − 13 + 5 = −8
[a+ (7 − 1) d] − [a + (5 − 1) d]= 12 a3 = a2 + d = − 8 + 5 = −3
(a + 6d) − (a + 4d) = 12 Hence the first three terms are −13, −8, and −3.
2d = 12 19. Subbia Rao started work in 1995 at an annual salary of Rs 5000 and received an
d=6 increment of Rs 200 each year. In which year did his income reach Rs 7000?
From equation (i) , The annual salary received by Subba Rao in the years 1995 onwards are
a + 2 (6) = 16 5000, 5200, 5400,----7000
a + 12 = 16 Hence, these numbers forms an AP.
a=4 a = 5000, d = 200, an = 7000.
Then the required AP is 4, 10, 16, 22, … an = a + (n − 1) d
17. Find the 20th term from the last term of the AP: 3, 8, 13, . . ., 253. 7000 = 5000 + (n − 1) 200
Given AP: 3, 8, 13, …, 253 200(n − 1) = 2000
nth term from the last = l – ( n – 1)d (n − 1) = 10
l = 253, a = 3, d = 5 n = 11
nth term from the last = 253 – ( 20 – 1)5 Thus the 11th years of his service or in 2005, Subba Rao received an annual salary of
= 253 – ( 19)5 Rs 7000.
= 253 – 95 20. Ramkali saved Rs 5 in the first week of a year and then increased her weekly savings
= 253 – 95 = 158 by Rs 1.75. If in the nth week, her weekly savings become Rs 20.75, find n.
18. The sum of the 4th and 8th terms of an AP is 24 and the sum of the 6th and 10th terms is a = 5, d = 1.75, an = 20.75, n = ?
44. Find the first three terms of the AP. an = a + (n − 1) d
20.75 = 5 + (n - 1) × 1.75

P a g e 16 | 159 Available in ykoyyur.blogspot.com P a g e 17 | 159 Available in ykoyyur.blogspot.com


SSLC Mathematics Solutions – Part -1 YK SSLC Mathematics Solutions – Part -1 YK
15.75 = (n - 1) × 1.75 78 = [2x24 + (n - 1)(-3)]
15.75 = 1.75n – 1.75 78 = [48 - 3n + 3 ]
1.75n = 15.75 + 1.75 156 = [48 - 3n + 3 ]
1.75n = 17.50 156 = 51n - 3n2
.
n= = = 10 52 = 17n - n2
.
n2 – 17n + 52 = 0
n2 – 13n – 4n + 52 = 0
1.4 Sum of First n Terms of an AP
n(n – 13) – 4(n – 13) = 0
(n – 13) (n – 4) = 0
• First term - a Common n = 13 CxÀªÁ n = 4
difference - d S = [2a + (n - 1)d]
Example 14 : Find the sum of :
(i) the first 1000 positive integers (ii) the first n positive integers
• When the first and the last terms of an Solution: (i) Let S = 1 + 2 + 3 +...... + 1000
AP are given and the common S = [a + l] S = [2a + (n - 1)d]
difference is not given
S = 500[2 + 999]
S = 500[1001]
Example 11 : Find the sum of the first 22 terms of the AP : 8, 3, –2, . . .
S = 500500
Solution: Here a = 8, d = 3 - 8 = -5, n = 22.
(i) Let S = 1 + 2 + 3 +...... + n
S = [2a + (n - 1)d]
S = [2a + (n - 1)d]
S = [2x8 + (22 - 1)(-5)] S = [2x1 + (n - 1)1]
S = 11[16 + 21(-5)]
S = [2 + n - 1]
S = 11[16 - 105]
S = 11x-89 = -979 S = [n + 1]
Example 12 : If the sum of the first 14 terms of an AP is 1050 and its first term is 10, Example 15 : Find the sum of first 24 terms of the list of numbers whose nth term is
find the 20th term. given by an = 3 + 2n.
Solution: Here, S14 = 1050, n = 14, a = 10
Solution: an = 3 + 2n
S = [2a + (n - 1)d] a = 3 + 2x1 = 3 + 2 = 5
1050 = [2x10 + (14 - 1)d] a = 3 + 2x2 = 3 + 4 = 7
1050 = 7[20 + 13d] a = 3 + 2x3 = 3 + 6 = 9
1050 = 140 + 91d There for AP is : 5, 7, 9, - - -
91d = 1050 − 140 a = 5, d = 2, n = 24
91d = 910 S = [2a + (n - 1)d]
d= = 10 S= [2x5 + (24 - 1)2]
an = a + (n − 1) d S = 12[10 + 23x2]
a = 10 + (20 – 1)10 S = 12[10 + 46]
a = 10 + 19x10 S = 12x56
a = 10 + 190 S = 672
a = 200 Example 16 : A manufacturer of TV sets produced 600 sets in the third year and 700 sets
Example 13 : How many terms of the AP : 24, 21, 18, . . . must be taken so that their in the seventh year. Assuming that the production increases uniformly by a fixed number
sum is 78? every year, find :
Solution:a = 24, d = 21-24 = -3, Sn = 78 , We have to find ‘n’
(i) the production in the 1st year (ii) the production in the 10th year
S = [2a + (n - 1)d]
(iii) the total production in first 7 years

P a g e 18 | 159 Available in ykoyyur.blogspot.com P a g e 19 | 159 Available in ykoyyur.blogspot.com


SSLC Mathematics Solutions – Part -1 YK SSLC Mathematics Solutions – Part -1 YK
Solution:i) Since the production increases uniformly by a fixed number every year, the S100 = 50[1.2 + 108.9]
number of TV sets manufactured in 1st, 2nd, 3rd, . . ., years will form an AP. S100 = 50[110.1]
S100 = 5505
Let us denote the number of TV sets manufactured in the nth year by an
iv) , , ---------- to 11 terms
a = 600, a = 700,
a + 2d = 600 a=
a + 6d = 700 d = a2 − a1 = − = =
By solving the equation we get, n = 11
d = 25 and a = 550 Sn = [2a + (n - 1) d]
(i) Therefore, production of TV sets in the first year is = 550
S11 = [2x + (11 – 1 ) × ]
(ii) Production of TV sets in the 10th year is : a = a + 9d
S11 = [ + ]
a = 550 + 9x25
= 550 + 225 = 775 S11 = [ ]
(iii) The total production of TV sets in first 7 years is S11 = [ ]
S = [2a + (n - 1)d] S11 = [ ] =
S = [2x550 + (7 - 1)25] 2. Find the sums given below :
S = [1100 + 6x25] i) 7 + 10 + 14 + -------- + 84
a = 7, l = 84
S = [1100 + 150]
d = a2 − a1 = 10 - 7 = - 7 =
S = [1250] l = a + (n - 1)d
S = 7x625 = 4375 84 = 7 + (n - 1) ×
Exercise 1.3 77 = (n - 1) ×
1. Find the sum of the following APs: 154 = 7n − 7
i) 2, 7, 12 ...... to 10 terms 7n = 161
a = 2, d = − = 7 − 2 = 5, n = 10 n = 23
Sn = [2a + (n - 1) d] Sn = (a + l)
S10 = [2(2) + (10 - 1) × 5] S23 = (7 + 84)
S10 = 5[4 + (9) × (5)] = x 91 =
S10 = 5 × 49 = 245
ii) -37, -33, -29 ...... to 12 terms = 1046
a = −37 ii) 34 + 32 + 30 + ...... + 10
d= − = (−33) − (−37 = − 33 + 37 = 4 a = 34, d = a2 − a1 = 32 − 34 = −2, l = 10
n = 12 l = a + (n − 1) d
10 = 34 + (n − 1) (−2)
Sn = [2a + (n - 1) d]
−24 = (n − 1) (−2)
S12 = [2(-37) + (12 - 1) × 4] 12 = n − 1
S12 = 6[-74 + 11 × 4] n = 13
S12 = 6[-74 + 44] Sn = (a + l)
S12 = 6(-30) = -180 S13 = (34 + 10)
iii) 0.6, 1.7, 2.5 ..... to 100 terms
S13 = x44
a = 0.6
d = a2 − a1 = 1.7 − 0.6 = 1.1 S13 = 13 × 22
n = 100 S13 = 286
Sn = [2a + (n - 1) d] iii) -5 + (-8) + (-11)+ .... + (-230)
a = −5, l = −230, d = a2 − a1 = (−8) − (−5) = − 8 + 5 = −3
S100 = [1.2 + (99) × 1.1] l = a + (n − 1)d
P a g e 20 | 159 Available in ykoyyur.blogspot.com P a g e 21 | 159 Available in ykoyyur.blogspot.com
SSLC Mathematics Solutions – Part -1 YK SSLC Mathematics Solutions – Part -1 YK
−230 = − 5 + (n − 1) (−3) S10 = [2a + (10 - 1)d]
−225 = (n − 1) (−3)
125 = 5(2a + 9d)
(n − 1) = 75
25 = 2a + 9d ----------- (ii)
n = 76
Substract equation (i) from (2)
Sn = (a + l)
30 = 2a + 4d ----------- (iii)
S76 = [(-5) + (-230)] Substract (ii) fromÀ (iii)
S76 = 38(-235) −5 = 5d
S76 = -8930 d = −1
3. In an AP: From equation (i),
i) Given a = 5, d = 3, a n = 50 find n and Sn 15 = a + 2(−1)
a = 5, d = 3, an = 50 15 = a − 2
an = a + (n − 1)d, a = 17
⇒ 50 = 5 + (n - 1) × 3 = a + (10 − 1)d
⇒ 3(n - 1) = 45
= 17 + (9) (−1)
⇒ n - 1 = 15 = 17 − 9 = 8
⇒ n = 16
v) Given d = 5, S9 = 75 find a and
Sn = (a + an) d = 5, S9 = 75
S16 = (5 + 50) = 440 Sn = [2a + (n - 1)d]
S16 = 8 (55) = 440 75 = [2a + (9 - 1)5]
ii) Given a = 7, = 35 find d and S13
75 = (2a + 40)
a = 7, = 35
an = a + (n − 1)d, 75 = 9 (a + 20)
⇒ 35 = 7 + (13 - 1)d 75 = 9a + 180
⇒ 12d = 28 9a = 75 − 180
⇒ d = 28/12 = 2.33 a=
Sn = (a + an) an = a + (n − 1)d
S13 = (7 + 35) a9 = a + (9 − 1) (5)
= + 8(5)
S13 = (42) = 13x21
S13 = 273 = + 40
iii) Given = 37, d = 3 find É a and S12 = =
= 37, d = 3 vi) Given a = 2, d = 8, Sn = 90 findÀ n and
an = a + (n − 1)d, a = 2, d = 8, Sn = 90
⇒ = a + (12 − 1)3
Sn = [2a + (n - 1)d]
⇒ 37 = a + 33
⇒a=4 90 = [2a + (n - 1)d]
Sn = (a + an) ⇒ 180 = n(4 + 8n - 8)
⇒180= n(8n - 4)
S12 = (4 + 37)
⇒180= 8n2 - 4n
S12 = 6 (41) ⇒ 8n2 - 4n - 180 = 0
S12 = 246 ⇒ 2n2 - n - 45 = 0
iv) Given = 15, S10 = 125 findÉ d and ⇒ 2n2 - 10n + 9n - 45 = 0
a3 = 15, S10 = 125 ⇒ 2n(n -5) + 9(n - 5) = 0
an = a + (n − 1)d, ⇒ (2n - 9)(2n + 9) = 0
a3 = a + (3 − 1)d n = 5 (Positive number)
15 = a + 2d ------------ (i) There forÀ a5 = 8 + 5 × 4 = 34
Sn = [2a + (n - 1) d] vii) Given a = 8, an = 62, Sn = 210 find n and d

P a g e 22 | 159 Available in ykoyyur.blogspot.com P a g e 23 | 159 Available in ykoyyur.blogspot.com


SSLC Mathematics Solutions – Part -1 YK SSLC Mathematics Solutions – Part -1 YK
a = 8, an = 62, Sn = 210 144 = (a + 28)
Sn = (a + an) (16) × (2) = a + 28
210 = (8 + 62) 32 = a + 28
⇒ 35n = 210 a=4
4. How many terms of the AP : 9, 17, 25, . . . must be taken to give a sum of 636?
⇒ n= =6
a=9
= a + (n – 1)d d = a2 − a1 = 17 − 9 = 8
62 = 8 + 5d Sn = [2a + (n - 1)d]
⇒ 5d = 62 - 8 = 54
636 = [2 × a + (8 - 1) × 8]
⇒ d = = 10.8
viii) Given an = 4, d = 2, Sn = -14 findÉ n and a 636 = [18 + (n- 1) × 8]
636 = n [9 + 4n − 4]
an = 4, d = 2, Sn = −14
an = a + (n − 1)d 636 = n (4n + 5)
4n2 + 5n − 636 = 0
4 = a + (n − 1)2
4 = a + 2n − 2 4n2 + 53n − 48n − 636 = 0
n (4n + 53) − 12 (4n + 53) = 0
a + 2n = 6
a = 6 − 2n ---------- (i) (4n + 53) (n − 12) = 0
4n + 53 = 0 or n − 12 = 0
Sn = (a + an)
n = (-53/4) or n = 12
-14 = (a + 4) ⇒n = 12
−28 = n (a + 4) 5. The first term of an AP is 5, the last term is 45 and the sum is 400. Find the number of terms
−28 = n (6 − 2n + 4) {From equation (i)À} and the common difference.
a = 5, l = 45, Sn = 400
−28 = n (− 2n + 10)
−28 = − 2n2 + 10n Sn = (a + l)
2n2 − 10n − 28 = 0 400 = (5 + 45)
n2 − 5n −14 = 0
400 = (50)
n2 − 7n + 2n − 14 = 0
n (n − 7) + 2(n − 7) = 0 25n = 400
(n − 7) (n + 2) = 0 n = 16
Either n − 7 = 0 or n + 2 = 0 l = a + (n − 1) d
n = 7 or n = −2 45 = 5 + (16 − 1) d
From equation (i) , 40 = 15d
a = 6 − 2n d= =
a = 6 − 2(7) 6. The first and the last terms of an AP are 17 and 350 respectively. If the common difference
a = 6 − 14 is 9, how many terms are there and what is their sum?
a = −8 a = 17, l = 350, d = 9
ix) Given a = 3, n = 8, S = 192 find d l = a + (n − 1) d
a = 3, n = 8, S = 192 350 = 17 + (n − 1)9
Sn = [2a + (n - 1)d] 333 = (n − 1)9
(n − 1) = 37
192 = [2 × 3 + (8 - 1)d] Sn = (a + l)
192 = 4 [6 + 7d]
48 = 6 + 7d S38 = (17 + 350)
42 = 7d S38 = 19 × 367
d=6 S38 = 6973
x) Given l = 28, S = 144 and there are 9 terms. Find the value of a 7. Find the sum of first 22 terms of an AP in which d = 7 and 22nd term is 149.
l = 28, S = 144, n = 9 d = 7, = 149, S22 = ?
an = a + (n − 1)d
Sn = (a + l)
a22 = a + (22 − 1)d

P a g e 24 | 159 Available in ykoyyur.blogspot.com P a g e 25 | 159 Available in ykoyyur.blogspot.com


SSLC Mathematics Solutions – Part -1 YK SSLC Mathematics Solutions – Part -1 YK
149 = a + 21 × 7 10. Show that a1, a2, a3 ..... a n ,......form an AP where a n is defined as below
149 = a + 147 (i) = 3 + 4n (ii) = 9 - 5n
a=2 Also, find the sum of the first 15 terms in each case.
Sn = (a + an) (i) = 3 + 4n
S22 = (2 + 149) = 11 × 151 a1 = 3 + 4(1) = 7
S22 = 1661 a2 = 3 + 4(2) = 3 + 8 = 11
8. Find the sum of first 51 terms of an AP whose second and third terms are 14 and 18 a3 = 3 + 4(3) = 3 + 12 = 15
respectively. a4 = 3 + 4(4) = 3 + 16 = 19
a2 = 14, a3 = 18, d = a3 − a2 = 18 − 14 = 4 ⇒ a2 − a1 = 11 − 7 = 4
a2 = a + d a3 − a2 = 15 − 11 = 4
14 = a + 4 a4 − a3 = 19 − 15 = 4
a = 10 So, the given sequence forms an AP with first term =7 and common difference = 4
Sn = [2a + (n - 1)d] Sn = [2a + (n - 1)d]
S51 = [2 × 10 + (51 - 1) × 4] S15 = [2(7) + (15 - 1) × 4]

= [20 + (50) × 4] = [(14) + 56]


= [20 + 200] = (70) = 15 × 35 = 525
(ii) = 9 - 5n
= [220] = 51 × 110 = 5610
a1 = 9 − 5 × 1 = 9 − 5 = 4
9. If the sum of first 7 terms of an AP is 49 and that of 17 terms is 289, find the sum of
a2 = 9 − 5 × 2 = 9 − 10 = −1
first n terms.
a3 = 9 − 5 × 3 = 9 − 15 = −6
S7 = 49, S17 = 289
a4 = 9 − 5 × 4 = 9 − 20 = −11
Sn = [2a + (n - 1)d]
⇒ a2 − a1 = − 1 − 4 = −5
S7 = [2a + (n - 1)d] a3 − a2 = − 6 − (−1) = −5
a4 − a3 = − 11 − (−6) = −5
S7 = [2a + (7 - 1)d] So, the given sequence forms an AP with first term = 4 and common difference = -5
Sn = [2a + (n - 1)d]
49 = [2a + 6d]
7 = (a + 3d) S15 = [2(4) + (15 - 1) (-5)]
a + 3d = 7 ----------- (i) = [8 + 14(-5)]
Similarlly,
= (8 - 70) = (-62) = 15(-31) = -465
S17 = [2a + (17 - 1)d]
11. If the sum of the first n terms of an AP is 4n – n2, what is the first term (that is S )? What
289 = (2a + 16d) is the sum of first two terms? What is the second term? Similarly, find the 3rd, the 10th and
the nth terms.
17 = (a + 8d)
Sn = 4n − n2
a + 8d = 17 --------- (ii)
First term a = S1 = 4(1) − (1) 2 = 4 − 1 = 3
Substract equation (ii) from (i)
Sum of first two terms
5d = 10 ⇒ d = 2 S2= 4(2) − (2)2 = 8 − 4 = 4
From equation (i)
a2 = S2 − S1 = 4 − 3 = 1
a + 3(2) = 7 d = a2 − a = 1 − 3 = −2
a+6=7 ⇒a=1 nth term an = a + (n − 1)d
Sn = [2a + (n - 1)d] = 3 + (n − 1) (−2)
= [2(1) + (n - 1) × 2] = (2 + 2n - 2) = 3 − 2n + 2
= 5 − 2n
= (2n) = n2
So, the third term a3 = 5 − 2(3) = 5 − 6 = −1
10th term a10 = 5 − 2(10) = 5 − 20 = −15

P a g e 26 | 159 Available in ykoyyur.blogspot.com P a g e 27 | 159 Available in ykoyyur.blogspot.com


SSLC Mathematics Solutions – Part -1 YK SSLC Mathematics Solutions – Part -1 YK
12. Find the sum of the first 40 positive integers divisible by 6. 16. A sum of Rs 700 is to be used to give seven cash prizes to students of a school for their
6, 12, 18, 24 … overall academic performance. If each prize is Rs 20 less than its preceding prize, find the
This is an AP with common difference = 6 and the first term = 6 value of each of the prizes.
a = 6, d = 6, S40 = ? Let the first prize = a
Sn = [2a + (n - 1)d] The amount of 2nd prize = a − 20
S40 = [2(6) + (40 - 1) 6] The amount of 3rd prize = a − 40
This is an AP with common difference -20 and the first term a
= 20[12 + (39) (6)] d = −20, S7 = 700
= 20(12 + 234) = 20 × 246 = 4920
Sn = [2a + (n - 1)d]
13. Find the sum of the first 15 multiples of 8.
The numbers multiples of 8 are [2a + (7 - 1)d] = 700
8, 16, 24, 32… [2a + 6d] = 700
These numbers form an AP with common difference 8 and the first term 8 7 [a + 3d] = 700
a = 8, d = 8, S15 = ? a + 3d = 100
Sn = [2a + (n - 1)d] a + 3(−20) = 100
S15 = [2(8) + (15 - 1)8] a − 60 = 100
a = 160
= [6 + (14) (8)] So, the values of prizes Rs 160, Rs 140, Rs 120, Rs 100, Rs 80, Rs 60, and Rs 40.
= [16 + 112] = (128) = 15 × 64 = 960 17. In a school, students thought of planting trees in and around the school to reduce air
pollution. It was decided that the number of trees, that each section of each class will
14. Find the sum of the odd numbers between 0 and 50.
plant, will be the same as the class, in which they are studying, e.g., a section of Class I
The odd numbers between 0 and 50
will plant 1 tree, a section of Class II will plant 2 trees and so on till Class XII. There are
1, 3, 5, 7, 9 … 49
three sections of each class. How many trees will be planted by the students?
This is an AP with common difference 1 and the first term 2
1, 2, 3, 4, 5………………..12
a = 1, d = 2, l = 49
This is an AP with common difference 1 and the first term 1
l = a + (n − 1) d
a = 1, d = 2 − 1 = 1
49 = 1 + (n − 1)2
48 = 2(n − 1) Sn = [2a + (n - 1)d]
n − 1 = 24 S12 = [2(1) + (12 - 1)(1)]
n = 25 = 6 (2 + 11) = 6 (13) = 78
Sn = (a + l) Hence, the trees planted by the students of each section = 78
S25 = (1 + 49) There for the trees planted by the students of 3 sections = 78 x 3 = 234
18. A spiral is made up of successive semicircles, with centres alternately at A and B,
= (50) = (25)(25) = 625
starting with centre at A, of radii 0.5 cm, 1.0 cm, 1.5 cm, 2.0 cm, . . . as shown in Fig.
15. A contract on construction job specifies a penalty for delay of completion beyond a
1.4. What is the total length of such a spiral made up of thirteen consecutive semi
certain date as follows: Rs 200 for the first day, Rs 250 for the second day, Rs 300 for the
third day, etc., the penalty for each succeeding day being Rs 50 more than for the circles (Take π = )
preceding day. How much money the contractor has to pay as penalty, if he has delayed
the work by 30 days?
This is an AP with common difference 50 and the first term 200
a = 200, d = 50
The penalty payable for the delay of 30 days = S30
Sn = [2a + (n - 1)d]
S30 = [2(200) + (30 - 1) 50]
= 15 [400 + 1450] = 15 (1850)
= Rs 27750
[Hint: length of successive semi circles is l1, l2, l3, l4, .....with centers as A, B,A,B ..... l1, l2, l3, l4, .....]

P a g e 28 | 159 Available in ykoyyur.blogspot.com P a g e 29 | 159 Available in ykoyyur.blogspot.com


SSLC Mathematics Solutions – Part -1 YK SSLC Mathematics Solutions – Part -1 YK
The length of the semi circles = πr 20. In a potato race, a bucket is placed at the starting point, which is 5 m from the first potato,
l1 = π(0.5) = cm and the other potatoes are placed 3 m apart in a straight line. There are ten potatoes in the
l2 = π(1) = π cm line (see Fig. 1.6).
l3 = π(1.5) = cm
l1, l2, l3 . . . are the lengths of semicircles
, π, , 2π, ....
d = l2- l1 = π - =
A competitor starts from the bucket, picks up the nearest potato, runs back with it, drops
a = cm
it in the bucket, runs back to pick up the next potato, runs to the bucket to drop it in, and
Sn = [2a + (n - 1)d] she continues in the same way until all the potatoes are in the bucket. What is the total
There for the total length of such a spiral made up of thirteen consecutive semi circles distance the competitor has to run?
S13 = [2x + (13 - 1) ] = [π + 6π] [Hint : To pick up the first potato and the second potato, the total distance (in metres)
run by a competitor is 2 × 5 + 2 × (5 + 3)]
= (7π) = ×7× = 143 cm
The distances from the bucket to potatos 5, 8, 11, 14…
19. 200 logs are stacked in the following manner: 20 logs in the bottom row, 19 in the next row, They have to run twice, then the distacnes run by the competiror 10, 16, 22, 28, 34,……….
18 in the row next to it and so on (see Fig. 1.5). In how many rows are the 200 logs placed a = 10, d = 16 − 10 = 6, S10 =?
and how many logs are in the top row? Sn = [2a + (n - 1)d]
S10 = [2(10) + (10 - 1)(6)]
= 5[20 + 54] = 5 (74) = 370
Hence, the distance the competitor has to run is 370km

Summery:
 An arithmetic progression (AP) is a list of numbers in which each term is obtained
The logs are in an AP by adding a fixed number d to the preceding term, except the first term. The fixed
20, 19, 18… number d is called the common difference
a = 20, d = a2 − a1 = 19 − 20 = −1  The general form of an AP : a, a + d, a + 2d, a + 3d......
Sn = 200
Sn = [2a + (n - 1)d]
 In an AP if there are only a finite number of terms. Such an AP is called
a finite AP. Such AP has a last term.
200 = [2(20) + (n - 1)(-1)]
 The AP has infinite number of terms is called infinite Arithmetic
200 = [40 - n + 1] Progression. Such APs do not have a last term.
400 = n (40 − n + 1)
400 = n (41 − n)
 The first term – a and the common difference is d then the nth term of
an AP
400 = 41n − n2
= + ( − 1)
n2 − 41n + 400 = 0
n2 − 16n − 25n + 400 = 0  The nth term from the last [ last term – l , common difference – d ]
n (n − 16) −25 (n − 16) = 0 − ( − 1)
(n − 16) (n − 25) = 0
(n − 16) = 0 or n − 25 = 0  a is the first term, d is the common difference then sum to nth term
n = 16 or n = 25 S = [2a + (n - 1)d]
an = a + (n − 1)d
a16 = 20 + (16 − 1) (−1)⇒ a16 = 20 − 15 = 5
 If common difference is unknown then the sum to nth term
Similarly, S = [a + l] { l – the last term }
a25 = 20 + (25 − 1) (−1) = 20 − 24
= −4 (negetive number is not possible)
Hence the number of rows is 16 and the number of logs in the top row is 5
P a g e 30 | 159 Available in ykoyyur.blogspot.com P a g e 31 | 159 Available in ykoyyur.blogspot.com
SSLC Mathematics Solutions – Part -1 YK SSLC Mathematics Solutions – Part -1 YK
1. Give two different examples of pair of
Triangles (i) similar figures:
Pair of circles
2.2 Similar Figures Pair of squares
Two polygons of the same number of sides are similar, if (ii) non-similar figures.
A triangle and a square
All the corresponding angles are equal and
A rectangle and a Quadrilateral
4. State whether the following quadrilaterals are similar or not:

All the corresponding sides are in the same ratio (or


proportion).

EXERCISE 2.1
1. Fill in the blanks using the correct word given in brackets
i) All circles are _____________ (congruent, similar)
ii) All squares are ____________(similar, congruent) The corresponding angles are not equal. Hence, they are not similar
iii) All____________triangles are similar. (isosceles, equilateral) 2.3 Similarity of Triangles
iv) Two polygons of the same number of sides are similar, if (a) their corresponding angles
are___________and (b) their corresponding sides are____________(equal, proportional) Basic proportionality theorem[Thales theorem]
2. Give two different examples of pair of
(i) similar figures. (ii) non-similar figures.
If a line is drawn parallel to one side of a triangle to
3. State whether the following quadrilaterals are similar or not:
intersect the other two sides in distinct points, the other two
sides are divided in the same ratio

Solutions:
1. Fill in the blanks using the correct word given in brackets
v) All circles are similar (congruent, similar)
vi) All squares are similar (similar, congruent) Data: In ∆ABC, the line drawn parallel to BC intersects AB and AC at D and E .
vii) All similar triangles are equilateral. (isosceles, equilateral)
To prove: = ,
viii) Two polygons of the same number of sides are similar, if (a) their corresponding
angles are equal and (b) their corresponding sides are proportional (equal, Construction: Join BE and CD. Draw DM⟘AC and EN⟘AB.
proportional)

P a g e 32 | 159 Available in ykoyyur.blogspot.com P a g e 33 | 159 Available in ykoyyur.blogspot.com


SSLC Mathematics Solutions – Part -1 YK SSLC Mathematics Solutions – Part -1 YK
Proof : Now in ∆ADC,
(∆ ) × × EGǁDC (∵EFǁDC)
= = [ ∵Area of triangle = x Base x Height]
(∆ ) × × ∴ = (Theorem 2.1) ----(1)
(∆ ) × × Similarly, from ∆CAB,
= =
(∆ ) × ×
∆BDE and ∆DEC stand on the same base DE and in between BCǁDE
=
∴ Area (∆BDE) = Area (∆DEC) --- (3) i.e., = ----(2)
∴ From (1), (2) and (3),
∴ From (1), (2) axiom (1),
=
=
Example 3 : In Fig. 2.16, = and ∠PST = ∠PRQ prove that ∆PQR is an isosceles
triangle.
If a line divides any two sides of a triangle in the same Solution:
ratio, then the line is parallel to the third side. Given that, =
∴ STǁQR (∵Theorem 2.2)
⇒∠PST = ∠PQR (∵Corresponding angles) ----(1)
Example 1 :If a line intersects sides AB and AC of a ABC at D and E respectively and is parallel But, ∠PST = ∠PRQ (∵ Given ) -------(2)
So, ∠PRQ = ∠PQR [from (1), (2) and axiom (1) ]
to BC, prove that = ( See fig 2.13 )
Therefore, PQ = PR (∵Sides opposite the equal angles)
Solution: DEǁBC (Data) i.e., PQR is an isosceles triangle.
∴ = ( Theorem 2.1)

or =
Exercise 2.2
1. In Fig. 2.17, (i) and (ii), DE || BC. Find EC in (i) and AD in (ii).
+1 = +1
=
= (∵ Taken reciprocals)
Example 2 : ABCD is a trapezium with AB || DC. E and F are points on non parallel sides AD
a n d BC r es pect ively su ch tha t EF is parallel to (See fig 2.14 ) Show that =

(i) In triangle△ABC, DE∥BC (Given)


∴ = [Thales theorem]
.
⇒ = ⇒ EC = = = 2 cm.
.
(ii) In △ABC, DE∥BC (Given)
solution: Join AC to intersect EF at G. (See fig 2.15)
ABǁDC ªÀÄvÀÄÛ EFǁAB (Given) ∴ = [Thales theorem]
.
So, EF || DC (Lines parallel to the same line are parallel to each other ) ⇒ =
. .

P a g e 34 | 159 Available in ykoyyur.blogspot.com P a g e 35 | 159 Available in ykoyyur.blogspot.com


SSLC Mathematics Solutions – Part -1 YK SSLC Mathematics Solutions – Part -1 YK
. . .
⇒ AD = = = 2.4 cm. In ΔABC, DE || AC (Given)
.
2. E and F are poin ts on th e sides PQ an d PR respectively of a PQR. For each of ∴ = -------(1) [ Thales theorem]
the following cases, state whether EF || QR : In ΔABC, DF || AE (Given)
(i) PE = 3.9cm EQ = 3cm PF = 3.6cm FR = 2.4cm
(ii) PE = 4cm QE = 4.5cm PF = 8cm RF = 9cm ∴ = -----------(2) [ Thales theorem]
(iii) PQ = 1.28cm PR = 2.56cm PE = 0.18cm PF = 0.36cm From equation (i) and (ii)
Solution: =
(i) PE = 3.9 cm, EQ = 3 cm ,PF = 3.6 cm, FR = 2,4 cm (Given)
. 5. In Fig. 2.20, DE || OQ and DF || OR. Show that EF || QR
∴ = = = 1.3 [Thales theorem] In ΔPQO, DE || OQ (Given)
.
And =
.
= = 1.5 ∴ = -------------- (1) [ Thales theorem]
Therefore , ≠ In ΔPOR, DF || OR (Given)
Hence, EF is not parallel to QR ∴ = ---------------(2) [ Thales theorem]
(ii) PE = 4 cm, QE = 4.5 cm, PF = 8cm, RF = 9cm ⇒ = [From equation (1) and (2) ]
∴ = = = [Thales theorem]
. ∴ In ΔPQR, EF || QR. [Converse of BPT]
And, = 6. In Fig. 2.21, A, B and C are points on OP, OQ and OR respectively such that AB || PQ
and AC || PR. Show that BC || QR.
Therefore , =
In ΔOPQ, AB || PQ (Given)
Hence, EFǁQR ∴ = -------------------(1) [ Thales theorem]
(iii) PQ = 1.28 cm, PR = 2.56 cm, PE = 0.18 cm, PF = 0.36 cm
Here, EQ = PQ - PE = 1.28 - 0.18 = 1.10 cm In ΔOPR, AC || PR (Given)
And, FR = PR - PF = 2.56 - 0.36 = 2.20 cm ∴ = -------------------(2) [ Thales theorem]
.
∴ = = = ... (i) = [From equation (1) and (2) ]
.
and, =
.
= = ... (ii) ∴ ΔOQR £À°è, BC || QR. [Converse of BPT]
.
7. Using Theorem 2.1, prove that a line drawn through the mid-point of one side of a triangle
∴ = parallel to another side bisects the third side. (Recall that you have proved it in Class
There fore, EFǁQR IX).
3. In Fig. 2.18, if LM || CB and LN || CD, prove that Data: : In ΔABC, D is the mid-point of AB ⇒ AD=DB.
= The parallel line DE to BC drawn from D intersects AC at E
To prove: E is the mid-point of AC.
In the given fig, LM || CB, Proof: D is the mid-point of AB.
= ... (i) [corollary of BPT]
∴ AD = DB ⇒ = 1 ----------------------(1)
Similarlly, LN || CD,
In ΔABC, DE || BC,
∴ = ... (ii) [corollary of BPT]
∴ = [Thales theorem] ⇒1 = [From equation (1)]
From (i) and (ii) , ∴ AE =EC ⇒ E is the mid-point of AC
= 8. Using Theorem 2.2, prove that the line joining the mid-points of any two sides of a
4. In Fig. 2.19, DE || AC and DF || AE. Prove that triangle is parallel to the third side. (Recall that you have done it in Class IX).
Data: In ΔABC, D and E are the mid-points of AB and AC
=

P a g e 36 | 159 Available in ykoyyur.blogspot.com P a g e 37 | 159 Available in ykoyyur.blogspot.com


SSLC Mathematics Solutions – Part -1 YK SSLC Mathematics Solutions – Part -1 YK
⇒AD=BD and AE=EC.
To prove: DE || BC
Proof: D is the mid-point of AB (Given)
∴ AD = DB ⇒ = 1 ------------------------ (1) 2.4 Criteria for Similarity of Triangles
E is the mid-point of AC (Given)
∴ AE=EC ⇒ = 1 --------------------------(2)

= [From equation (1) and (2)]


∴ DE || BC [By BPT]
9. ABCD is a trapezium in which AB || DC and its diagonals intersect each other at the point O.
Show that =
Data: In trapezium ABCD, ABǁBC, and AC and BD intersects each other at O.
It must be noted that as done in the case of congruency of two
To Prove: = triangles, the similarity of two triangles should also be expressed
Construction: Draw EO from O such that EO || DC || AB symbolically, using correct correspondence of their vertices. For
ProofÉ: In ΔADC, OE || DC (Construction) example, for the triangles ABC and DEF of Fig. 2.22, we cannot write
ABC ~ EDF or ABC ~ FED. However, we can write BAC ~ EDF
= -------------------------(1) [By BPT]
In ΔABD, OE || AB (Construction)
If in two triangles, corresponding angles are equal, then their
= [By BPT] corresponding sides are in the same ratio (or proportion) and
hence the two triangles are similar.
⇒ = ----------------------(2) [ Taken reciprocals]

= [From equation (1) and (2)]


This criterion is referred to as the AAA (Angle–Angle–Angle) cr
⇒ = it er ion of similarity of two triangles.
10. The diagonals of a quadrilateral ABCD intersect each other at the point O such
that = Show that ABCD is a trapezium
Given: In ABCD, AC and BD intersects at O Data: In ∆ABC and ∆DEF,
∠A = ∠D, ∠B = ∠E and ∠C = ∠F
Such that =
To prove: = = (<1) and ∆ABC~ ∆DEF
To prove: ABCD is a trapezium
Construction: Draw EO through O such that EO || AB which Construction:Cut DP = AB from DE and DQ = AC
Intersects AD at E from DF and join PQ
Proof: In ΔDAB, EO || AB ProofÉ: In ∆ABC and ∆DPQ,
∴ = [ BPT] ⇒ = ------------------------- (1) [taken reciprocals] AB = DP [Construction]
AC = DQ [Construction]
Similarlly, = (Given) ∠A = ∠D [data]
∴∆ABC ≅ ∆DPQ [SAS Congruency rule]
⇒ = -----------------(2) ⇒BC = PQ ---------(1) and
⇒ ∠B = ∠P [CPCT] But ∠B = ∠E [Given]
∴ = [From equation (1) and (2)]
∴ ∠P = ∠E
EO || DC ªÀÄvÀÄÛ EO || AB [ converse of BPT] ∴PQǁEF [Since corresponding angles are equal]
⇒ AB || DC.
∴ABCD is a trapezium ∴ = = [by Corolary of BPT]

P a g e 38 | 159 Available in ykoyyur.blogspot.com P a g e 39 | 159 Available in ykoyyur.blogspot.com


SSLC Mathematics Solutions – Part -1 YK SSLC Mathematics Solutions – Part -1 YK

⇒ = = [By construction and (1) ]


∴∆ABC~ ∆DEF
Theorem 2.5 : If one angle of a triangle is equal to one
angle of the other triangle and the sides including these
If two angles of one triangle are respectively equal to two angles of another
angles are proportional, then the two triangles are similar
triangle, then the two triangles are similar.
This may be referred to as the AA similarity criterion for two triangles.
Given:In ∆ABC and ∆DEF, ∠A = ∠D and
If in two triangles, sides of one triangle are proportional to = (< 1) ------------ (1)
(i.e., in the same ratio of ) the sides of the other triangle, then To Prove: ∆ABC ≅ ∆DEF
their corresponding angles are equal and hence the two Construction: Cut DP = AB from DE
triangles are similiar. and DQ = AC from DF.Join PQ
Proof: In ∆ABC and ∆DPQ,
AB = PQ [By Construction]
This criterion is referred to as the SSS (Side–Side–Side) similarity criterion for two triangles. AC = DF [By Construction]
Data: In ∆ABC and ∆DEF, ∠A = ∠D [Given]
= = (<1) ---------(1) ∆ABC ≅ ∆DPQ [ By SAS Congruency rule]----------(2)
To Prove: ∠A = ∠D, ∠B = ∠E and ∠C = ∠F From eqn (1) we get,
And ∆ABC ≅ ∆ DEF = ⇒ = [AB = DP and AC = DQ]
Construction: Cut DP = AB from DE and ⇒ PQǁEF [By converse of corollary of BPT]
DQ = AC from DF.Join PQ ⇒∠P = ∠E , ∠Q = ∠F [Corresponding angles]
Proof : = [Given] ∴ ∆DPQ ~ ∆DEF [by AA similarity ctriteria] --------(3)
⇒ ∆ABC ≅ ∆DEF [From equation (2) and (3) ]
⇒ = [∵ DP = AB, DQ = AC] Example 4 : In Fig. 2.29, if PQ || RS, prove that ∆POQ ~ ∆SOR
∴ PQǁEF [corollary of Converse of BPT in ∆ DEF] Solution: PQǁRS [given]
⇒ ∠P = ∠E ªÀÄvÀÄÛ ∠Q = ∠F ∴ ∠P = ∠S [Alternate angles]
∴ ∆DPQ ~∆DEF [ AA Similarity criteria] ∠Q = ∠R [Alternate angles]
And ∠POQ = ∠SOR [vertically opposite angles]
∴ = [Corresponding sides of similar triangles]
∴∆POQ ~ ∆SOR [AAA similarity criteria]
⇒ = ------(1) [AB = DP Construction] Example 5 : Observe Fig. 2.30 and then find P.
Solution: In ∆ABC and ∆PQR,
But, = -------(2) [Given] .
= = , = =
⇒ = [ ∵ (1) ªÀÄvÀÄÛ (2) jAzÀ] .

⇒BC = PQ And = =

In ∆ABC and ∆DPQ ,
⇒ = =
BC = PQ [Proved]
AB = DP [Construction] ∴ ∆ABC ~ ∆RQP [SSS similarity criteria]
AC = DQ [Construction] ∠C = ∠P [Corresponding angles of similar triangles]
∴∆ABC ≅ ∆DPQ [SSS Congruency rule] But ∠C = 180 - ∠A - ∠B [The sum of interior angles of a triangle is 180 0]
Hence, ∠A = ∠D, ∠B = ∠P and ∠C = ∠Q =180 0 – 800 - 600 = 400 ⇒ ∠P = 400
⇒∠A = ∠D, ∠B = ∠E and ∠C = ∠F and ∆ABC ≅ ∆ DEF

P a g e 40 | 159 Available in ykoyyur.blogspot.com P a g e 41 | 159 Available in ykoyyur.blogspot.com


SSLC Mathematics Solutions – Part -1 YK SSLC Mathematics Solutions – Part -1 YK
From (3) and (4),
∆AMC ~ ∆PNR [SAS similarity criteria] --(5)
Example 6 : In Fig. 2.31, OA . OB = OC . OD. Show that ∠A = ∠C ªÀÄvÀÄÛ ∠B = ∠D.
ii) From (5) = -------(6)
Solution: OA.OB = OC.OD [Given]
⇒ = ------ (1)
But, = [From (1)] -------(7)

∠AOD = ∠COB [Vertically opposite angles] -----(2) ∴ = -------(8)


From equation (1) and (2),
∆AOD ~ ∆COB [SAS similarity criteria] (iii) = ------ (9)
∴ ∠A = ∠C and ∠D = ∠B [Corresponding angles of similar triangles]
⇒ = [From (8)]
Example 7 : A girl of height 90 cm is wa lking a wa y fr om t he ba se of a lamp-
post at a speed of 1.2 m/s. If the lamp is 3.6 m above the ground, find the length of her ⇒ = = [CM and RN are the medians]
shadow after 4 seconds.
Solution: ⇒ = ------- (10)
AB is a Lamp post. CD i the hs hight of the girl
DE is the length of the shadow of the girl. ⇒ = = [From (9) and (10)]
Let DE = x’ m ∴ ∆CMB ~ ∆RNQ [SSS similarity criteria]
Now, BD = 1.2m × 4 = 4.8m [ Note: you can solve (ii) and (iii) using same method as solved for (i)]
In ∆ABE and ∆CDE,
∠B = ∠D = 90 0
And ∠E = ∠E [Common angle] Exercise 2.3
∴ ∆ABE ~ ∆CDE [AA similarity criteria]
1) State which pairs of triangles in Fig. 6.34 are similar. Write the similarity criterion used by
∴ = you for answering the question and also write the pairs of similar triangles in the symbolic
. .
⇒ = ( ∵90cm = m = 0.9m) form :.
.
⇒ 4.8+x = 4x
⇒ 3x = 4.8
⇒x = 1.6
Hence, the length of her shadow after 4 seconds is 1.6m
Example 8 : In Fig. 2.33, CM and RN are r es pect ively th e media ns of ∆ABC
a nd ∆PQR. If ∆ABC ~ ∆PQR, prove that :
(i) ∆AMC ~ ∆PNR
(ii) =
iii) ∆CMB ~ ∆RNQ
Solution:
i) ∆ABC ~ ∆PQR
⇒ = = ---------(1)
and ∠A = ∠P , ∠B = ∠Q , ∠C = ∠R ------ (2)
But, AB = 2AM and PQ = 2PN [CM and RN are the medians]
⇒ =
⇒ = ------------(3)
But, ∠A = ∠P [ From (2) ] ---------(4)

P a g e 42 | 159 Available in ykoyyur.blogspot.com P a g e 43 | 159 Available in ykoyyur.blogspot.com


SSLC Mathematics Solutions – Part -1 YK SSLC Mathematics Solutions – Part -1 YK
(i) In ΔABC and ΔPQR, ∠F = ∠R = 30°
∠A = ∠P = 60° [Given] ; ∠B = ∠Q = 80° [Given]; ∠C = ∠R = 40° [Given] ⇒ ΔDEF ~ ΔPQR [AAA similarity criteria]
∴ ΔABC ~ ΔPQR [AAA similarity criteria] 2. In Fig. 2.35, ∆OBA ~ ∆ODC, ∠BOC = 125° and ∠CDO = 70°. Find ∠DOC , ∠DCO and
(ii) In ΔABC and ΔPQR , ∠OAB
Solution:
= =
DOB is a straight line
∴ ΔABC ~ ΔQRP [SSS similarity criteria] ∴ ∠DOC + ∠ COB = 180°
(iii)In ΔLMP and ΔDEF,
⇒ ∠DOC = 180° - 125° = 55°
LM = 2.7, MP = 2, LP = 3, EF = 5, DE = 4, DF = 6 In ΔDOC,
= = ∠DCO + ∠ CDO + ∠ DOC = 180°
⇒ ∠DCO + 70º + 55º = 180°
= = ⇒ ∠DCO = 180 0 - 125° = 550
. ΔODC ~ ΔOBA. [Given]
= =
∴ ∠OAB = ∠OCD [CPCT]
Here, = ≠ ⇒ ∠ OAB = 55°
∴ ΔLMP and ΔDEF are not similar 3. Diagonals AC and BD of a trapezium ABCD with AB || DC intersect each other at the
(iv) In ΔMNL and ΔQPR, point O. Using a similarity criterion for two triangles. Show that =
In ΔBOA and ΔDOC,
= =
∠ABO = ∠CDO [AB || CD, Alternate angles]
∠M = ∠Q = 70°
∠BAO =∠DCO [AB || CD, Alternate angles]
∴ ΔMNL ~ ΔQPR [SAS similarity criteria]
∠BOA = ∠DOC [Vertically opposite angles ]
(v) In ΔABC and ΔDEF,
∴ ΔBOA ~ ΔDOC [AAA Similarity criteria]
AB = 2.5, BC = 3, ∠A = 80°, EF = 6, DF = 5, ∠F = 80° OD
. ∴ = OB [By CPCT]
⇒ = =
⇒ = [Taken on reciprocals]
and, = =
4. In Fig. 2.36, = and ∠1 = ∠2 Show that ∆PQS ~ ∆TQR
⇒ ∠B ≠ ∠F
⇒ ΔABC and ΔDEF are not similar In ΔPQR,
(vi) In ΔDEF, ∠1 = ∠2 [Given]
∠D + ∠E + ∠F = 180° [Sum of the interior angles of a triangle] ∴ PQ = PR --------------(1)
⇒ 70° + 80° + ∠F = 180° But, = [Given]
⇒ ∠F = 180° - 70° - 80°
= [From (1)] ------------(2)
⇒ ∠F = 30°
In ∆PQR, In ΔPQS and ΔTQR,
∠P + ∠Q + ∠R = 180 [Sum of the interior angles of a triangle] = [From eqn (2)]
⇒ ∠P + 80° + 30° = 180°
⇒ ∠P = 180° - 80° -30° ∠Q = ∠Q [Common angle]
⇒ ∠P = 70° ∴ ΔPQS ~ ΔTQR [SAS similarity criteria]
In ΔDEF and ΔPQR , 5. S and T are points on sides PR and QR of ∆PQR such that ∠P = ∠RTS. Show that
∆RPQ ~ ∆RTS
∠D = ∠P = 70°
Solution:
∠F = ∠Q = 80°

P a g e 44 | 159 Available in ykoyyur.blogspot.com P a g e 45 | 159 Available in ykoyyur.blogspot.com


SSLC Mathematics Solutions – Part -1 YK SSLC Mathematics Solutions – Part -1 YK
In ΔRPQ and ΔRST, ∠AEB = ∠CBF [AE || BC,Alternate angles]
∠RTS = ∠QPS [Given] ∴ΔABE ~ ΔCFB [AA similarity criteria]
∠R = ∠R [Common angle]
∴ ΔRPQ ~ ΔRTS [AA similarity criteria]

9. In Fig. 2.39, ∆ABC and ∆AMP are two right tr ia n gl es, r ig h t an g led a t B a n d M
6. In Fig. 2.37, if ∆ABE ≅ ∆ACD, show that ∆ADE ~ ∆ABC
Solution: respectively. Prove that:
i) ∆ABC ~ ∆AMP
ΔABE ≅ ΔACD [Given]
∴ AB = AC -----------(1) [By CPCT] ii) =
and AD = AE -----------(2) [By CPCT] (i) In ΔABC and ΔAMP ,
ΔADE ªÀÄvÀÄÛ ΔABCUÀ¼À°è,
∠A = ∠A [Common angle]
Dividing (2) by (1)
∠ABC = ∠AMP = 90°
= ∴ ΔABC ~ ΔAMP [AA similarity criteria]
∠A = ∠A [Common angle] (ii) ΔABC ~ ΔAMP [Proved in(i)]
∴ ΔADE ~ ΔABC [SAS Similarity criteria]
7. In Fig. 2.38, altitudes AD and CE of ABC intersect each other at the point P. Show that ⇒ = [the corresponding sides of similar triangles are proportional]
i) ∆AEP ~ ∆CDP 10. CD and GH are respectively the bisectors of ∠ACB and ∠EGF such that D and H lie on
ii) ∆ABD ~ ∆CBE sides AB and FE of ∆ABC and ∆EFG respectively. If ∆ABC ~ ∆EFG, show that:
iii) ∆AEP ~ ∆ADB
iv) ∆PDC ~ ∆BEC
i) =
(i) In ΔAEP and ΔCDP, ii) ∆DCB ~ ∆HGE
iii) ∆DCA ~ ∆HGF
∠AEP = ∠CDP = 90°
∠APE = ∠CPD [Vertically opposite angles] (i) ΔABC ~ ΔFEG [Given]
∴ ΔAEP ~ ΔCDP[AA similarity criteria] ∴ ∠A = ∠F, ∠ACB = ∠FGE [Corresponding angles of similar triangles]
(ii) In ΔABD and ΔCBE, CD is the bisector of ∠ACB, GH is the bisector of ∠FGE
∠ADB = ∠CEB = 90° ∴ ∠ACD = ∠FGH
∠ABD = ∠CBE [Common angle] In ΔACD and ΔFGH,
∴ ΔABD ~ ΔCBE [AA similarity criteria] ∠A = ∠F
(iii) In ΔAEP and ΔADB, ∠ACD = ∠FGH
∠AEP = ∠ADB = 90 0 ∴ΔACD ~ ΔFGH [AA similarity criteria]
∠PAE = ∠DAB [Common angle] ⇒ =
∴ ΔAEP ~ ΔADB [AA similarity criteria] (ii) ΔABC ~ ΔFEG [Given]
(iv) In ΔPDC and ΔBEC, ∴ ∠B = ∠E, and ∠ACB = ∠FGE [Corresponding angles of similar triangles]
∠PDC = ∠BEC = 90° CD is the bisector of ∠ACB, GH is the bisector of ∠FGE
∠PCD = ∠BCE [Common angle] ∴∠DCB = ∠HGE
∴ ΔPDC ~ ΔBEC [AA similarity criteria] Now, In ΔDCB and ΔHGE,
8. E is a point on the side AD produced of a parallelogram ABCD and BE intersects CD at F. ∠DCB = ∠HGE
Show that ∆ABE ~ ∆CFB ∠B = ∠E
Solution: ∴ ΔDCB ~ ΔHGE [AA similarity criteria]
In ΔABE and ΔCFB, (iii) ΔABC ~ ΔFEG [Given]
∠A = ∠C [Opposite angles of a parallelogram] ∴ ∠A = ∠F, ∠ACB = ∠FGE [Corresponding angles of similar triangles]

P a g e 46 | 159 Available in ykoyyur.blogspot.com P a g e 47 | 159 Available in ykoyyur.blogspot.com


SSLC Mathematics Solutions – Part -1 YK SSLC Mathematics Solutions – Part -1 YK
CD is the bisector of ∠ACB, GH is the bisector of ∠FGE 14. Sides AB and AC and median AD of a t r i a n g l e ABC ar e r es p ect i vel y proportional
∴ ∠ACD = ∠FGH to sides PQ and PR and median PM of another triangle PQR. Show that ∆ABC ~ ∆PQR
Given: In ΔABC and ΔPQR,
In ΔDCA and ΔHGF,
AD and PM are the medians drawn to
∠ACD = ∠FGH
BC and QR respectively.
∠A = ∠F
∴ ΔDCA ~ ΔHGF [AA similarity criteria] and = =
11. In Fig. 2.40, E is a point on side CB produced of an isosceles triangle ABC with AB = To prove: ΔABC ~ ΔPQR
AC. If AD⟘BC and EF⟘AC , prove that ∆ABD ~ ∆ECF Construction: Produce AD to E such that
ABC is an isosceles triangle AD = DE, join CE
AB = AC and produce PM to N such that PM = MN, join RN
⇒∠B = ∠C [ Opposite sides of equal angles] Proof: In ΔABD and ΔCDE,
⇒ ∠ABD = ∠ECF ------ (1) AD = DE [Construction]
In ΔABD and ΔECF, BD = DC [AD is Median]
∠ADB = ∠EFC =90 0 [ AD⟘BC, EF⟘AC ] ∠ADB = ∠CDE [Vertically opposite angles]
⇒ ∠ABD = ∠ECF [From (1)] ∴ ΔABD ≅ ΔCDE [SAS Congruency rule]
∴ ΔABD ~ ΔECF [AA similarity criteria] ⇒ AB = CE [By CPCT] -------------(i)
12. Sides AB and BC and median AD of a triangle ABC are respectively propor- tional to Similarlly, In ΔPQM and ΔMNR,
sides PQ and QR and median PM of PQR (see Fig.2.41). Show that ∆ABC ~ ∆PQR ⇒ PQ = RN [By CPCT] -------------(ii)
Given: In ΔABC and ΔPQR,
= = But, = = [Given]

To prove: ΔABC ~ ΔPQR ⇒ = = [ From (i) and (ii) ]


Proof: = =
⇒ = =
= = [D and M are the mid-points of BC and QR respectively] ⇒ = = [∵ 2AD = AE ªÀÄvÀÄÛ 2PM = PN]

⇒ = = ∴ ΔACE ~ ΔPRN [SSS similarity criteria]


∴ ∠2 = ∠4
⇒ ΔABD ~ ΔPQM [SSS similarity criteria]
Similarlly, ∠1 = ∠3
∴ ∠ABD = ∠PQM [Corresponding angles of similar triangles]
⇒ ∠ABC = ∠PQR -----------(1) ∴ ∠1 + ∠2 = ∠3 + ∠4
In ΔABC and ΔPQR, ⇒ ∠A = ∠P --------------(iii)
In ΔABC and ΔPQR,
= [Given]
∠ABC = ∠PQR [From (1)] = [ Given ]
∴ ΔABC ~ ΔPQR [SAS similarity criteria] ∠A = ∠P [From (iii)]
13. D is a point on the side BC of a triangle ABC such that ∠ADC = ∠BAC . Show that ∴ ΔABC ~ ΔPQR [SAS similarity criteria]
CA2 = CB.CD. 15. A vertical pole of length 6 m casts a shadow 4 m long on the ground and at the same time
In ΔADC and ΔBAC, a tower casts a shadow 28 m long. Find the height of the tower.
∠ADC = ∠BAC [Given] Length of the vertical Pole = AB = 6m
∠ACD = ∠BCA [Common angle] Length of the shadow casts by the Pole = BC = 4 m
∴ ΔADC ~ ΔBAC [AA similarity criteria] Length of the shadow casts by the Tower = EF = 28 m
∴ = [The corresponding sides of the similar triangles are proportional] Let the height of the tower = DE = h ‘m
In ΔABC and ΔDEF,
⇒ CA2 = CB.CD.

P a g e 48 | 159 Available in ykoyyur.blogspot.com P a g e 49 | 159 Available in ykoyyur.blogspot.com


SSLC Mathematics Solutions – Part -1 YK SSLC Mathematics Solutions – Part -1 YK
∠C = ∠F [The angles make by sun at same time] ∠M = ∠N = 900 [Construction]
∠B = ∠E = 90° ∴ ∆ABM ~ ∆PQN [AA similarity criteria]
∴ ΔABC ~ ΔDEF [AA similarity criteria] ⇒ = - - - - - - (2)
∴ = [corresponding sides of the similar triangles] But, ∆ABC ~ ∆PQR [Given]
∴ = ∴ = = - - - - (3)

⇒ h = 6× = 6 × 7 ⇒ h = 42 m ⇒ = [ From (2) and (3)]


( )
∴ Height of the tower = 42 m. ∴ ( )
= x - ------- [From (1) and (3)]
16. If AD and PM ar e median s of tr ian gles ABC an d PQR, r espectively wher e ( )
⇒ =
∆ABC ~ ∆PQR prove that = ( )
( )
ΔABC ~ ΔPQR [Given] = = = [From (3)]
( )
∴ = = -------------- (1)
Example 9 : In Fig. 2.43, the line segment XY is parallel to side AC of ∆ABC and it
and ∠A = ∠P, ∠B = ∠Q, ∠C = ∠R --------- (2)
AD and PM are the Medians divides the triangle into two parts of equal areas. Find the ratio
Solution: XYǁAC [Given]
∴ BD = and QM = ----------(3)
∠ BXY = ∠A [Corresponding angles]
From equations (i) and (iii), we get
∠ BYX = ∠C [Corresponding angles]
∴ = = ----------(4) ∴ ∆ABC ~ ∆XBY [AA similarity criteria]
Area( )
= [Theorem 2.6] ------- (1)
In ΔABD and ΔPQM, Area( )

∠B = ∠Q [ From (2) ] But, Area(ABC) = 2 Area (XBY) [Given]


( )
= [ From (iv)À] ( )
= -------- (2)

∴ ΔABD ~ ΔPQM [SAS similarity criteria] = [ from (1) and (2)]


⇒ = =


Or = [Taken reciprocals]

2.5 Areas of Similar Triangles
⇒ 1 - = 1 -

The ratio of the areas of two similar triangles is equal to the √ √ √
= ⇒ = =
square of the ratio of their corresponding sides. √ √

Exercise 2.4
1. Let ∆ABC ~ ∆DEF and their areas be, respectively, 64 cm 2 and 121 cm2 . If EF =
Given: ∆ABC ~ ∆PQR 15.4 cm, find BC.
( ) ΔABC ~ ΔDEF [Given]
To Prove: = = =
( ) Area ΔABC = 64 cm2 and area ΔDEF = 121 cm2; EF = 15.4 cm
Construction: Draw AM⟘BC and PN⟘QR ( )
= = = [∵ ΔABC ~ ΔDEF] ------------(i)
( )
( )
ProofÉ: ( )
= = - - -(1) [Area of triangle= xbasexheight] =
In ∆ABM and ∆PQN, ⇒ = ⇒ = ⇒ = .
∠B = ∠Q [Corresponding angles of the similar triangle]

P a g e 50 | 159 Available in ykoyyur.blogspot.com P a g e 51 | 159 Available in ykoyyur.blogspot.com


SSLC Mathematics Solutions – Part -1 YK SSLC Mathematics Solutions – Part -1 YK
⇒ BC = x15.4 ⇒ BC = 8 × 1.4 ∴ ΔDEF ~ ΔCAB
Area( )
⇒ BC = 11.2 cm ∴ =
Area( )

2. Diagonals of a trapezium ABCD with AB || DC intersect each other at the point O. If ( )


⇒ =
AB = 2 CD, find the ratio of the areas of triangles ∆AOB and ∆COD ( )
Area( )
Solution: In trapezium ABCD, AB || DC, ⇒ = [ Area∆ABC = Area∆CAB]
Area( )
Diagonals AC and BD intersect each other at O ⇒ Area(DEF) : Area(ABC) = 1: 4
In ΔAOB and ΔCOD,
6. Prove that the ratio of the areas of two similar triangles is equal to the square of the ratio
∠1 = ∠2 [Alternate angles]
of their corresponding medians.
∠3 = ∠4 [Alternate angles]
Solution: ΔABC ~ ΔDEF [Given]
∠5 = ∠6 [Vertically opposite angles] Area( )
∴ ΔAOB ~ ΔCOD [AAAsimilarity criteria] ∴ Area( )
= and = =
Area(
⇒ Area(
) ( )
= = [∴ AB = 2CD]
) ⇒ = = ---------(1)
Area( )
⇒ = = In ΔABM and ΔDEN,
Area( )
∴ The ratio of the areas of triangles ΔAOB and ΔCOD is = 4:1 ∠B = ∠E [ΔABC ~ ΔDEF]
3. In Fig. 2.44, ABC and DBC are two triangles on the same base BC. If AD intersects BC at = [from (1)]
(
O, show that ( ) =
)
∴ ΔABM~ ΔDEN [SAS similarity criteria]
Construction: Draw AP⟘ BC and DM ⟘BC ⇒ =
Area( )
Area( ) ∴ = =
Proof: Area( = = -------- (1) Area( )
)
In ΔAPO and ΔDMO,
7. Prove that the area of an equilateral triangle described on one side of a square is equal
to half the area of the equilateral triangle described on one of its diagonals
∠APO = ∠DMO = 90 0
Solution: ΔAPB and ΔAQC are equilateral triangles
∠AOP = ∠DOM [Vertically opposite angles]
∴ ΔAPB ~ ΔAQC [AAA Similarity criteria]
∴ ΔAPO ~ ΔDMO [AA similarity criteria] ( )
⇒ = -------------- (2) ∴ ( )
=
Area( ) ( ) √
⇒ = [ From (1) and (2)] ⇒ = [Diagonal of a square = √2side]
Area( ) ( )
( )
4. If the areas of two similar triangles are equal, prove that they are congruent. ⇒ =
( )
Given : ΔABC ~ ΔPQR and Area ΔABC = AreaΔPQR
⇒ Area(APB) = × Area(AQC)
To prove: ΔABC ≅ ΔPQR
Proof : ΔABC ~ ΔPQR Tick the correct answer and justify :
Area( ) 8. ABC and BDE are two equilateral triangles such that D is the mid-point of BC. Ratio of
⇒ =
Area( ) the areas of triangles ABC and BDE is
⇒1= [Area(ABC)= Area(PQR)] A) 2 : 1 B) 1 : 2 C) 4 : 1 D) 1 : 4
⇒ BC2 = QR2 ⇒ BC = QR ΔABC and ΔBDE are equilateral triangle. D is the mid-point of BC
Similarlly, AB = PQ and AC = PR ∴ BD = DC = BC
∴ ΔABC ≅ ΔPQR [SSS congruency rule] Let the sides of ΔABC = 2a
5. D, E and F are respectively the mid-points of sides AB, BC and CA of ∆ABC. Find the ⇒ the sides of ΔBDE = a
ratio of the areas of ∆DEF and ∆ABC.
Solution: ΔABC ~ ΔBDE
In ΔABC, D,E and F are the mid-points of AB, BC and AC respectively Area( )
∴ =
Area( )
∴ DF = BC, DE = AC, ªÀÄvÀÄÛ EF = AB[Mid-point theorem] Area( ) ( )
In ΔDEF and ΔCAB, ⇒ = = =
Area( )
= = = ∴ Ans: (C) 4:1

P a g e 52 | 159 Available in ykoyyur.blogspot.com P a g e 53 | 159 Available in ykoyyur.blogspot.com


SSLC Mathematics Solutions – Part -1 YK SSLC Mathematics Solutions – Part -1 YK
9. Sides of two similar triangles are in the ratio 4 : 9. Areas of these triangles are in the ratio ⇒ CD.AC = BC2 --------- (2)
A) 2 : 3 B) 4 : 9 C) 81 : 16 D) 16 : 81 AD.AC + CD.AC = AB2 + BC2 [ By adding (1) and (2)]
ΔABC ~ ΔDEF and = ⇒ AC (AD+CD) = AB2 + BC2
Area( ) ⇒ AC × AC = AB2 + BC2
∴ =
Area( ) ⇒ AC2 = AB2 + BC2
Area( ) Theorem2 .9: In a triangle, if square of one side is equal to the sum of the squares of
⇒ = =
Area( ) the other two sides, then the angle opposite the first side is a right angle.
∴ Ans : (D) 16:81 Given: In ∆ABC, AC2 = AB2 + BC2
To prove: ∠B = 900
2.6 Pythagoras Theorem Construction: Draw ∆PQR such that
∠Q = 900 and PQ = AB, QR = BC
If a perpendicular is drawn from the vertex of the right angle
of a right triangle to the hypotenuse then triangles on both Proof :
sides of the perpendicular are similar to the whole triangle and In ∆PQR,
to each other. PR2 = PQ2 + QR2 [by Pythogoras theorem]
PR2 = AB2 + BC2 [Construction] ----------- (1)
But, AC2 = AB2 + BC2 [Given] -----------(2)
In a right triangle, the square of the hypotenuse is equal to the
∴ AC = PR [from (1) and (2)] -----------(3)
sum of the squares of the other two sides.
AB = PQ [Construction]
BC = QR [Construction]
AC = PR [from (3)]
In a triangle, if square of one side is equal to the sum of the ∴ ∆ABC ≅ ∆PQR [SSS congruency rule]
squares of the other two sides, then the angle opposite the first ∴ ∠B = ∠Q [By CPCT]
side is a right angle. But, ∠Q = 900 [Construction]
∴ ∠B = 900
Example10: In Fig. 2.48, ∠ACB = 90° and CD⟘AB prove that =
Theorem 2.8: In a right triangle, the square of the hypotenuse is equal to the sum of
Solution: ∆ACD ~ ∆ABC [Theorem 2.7]
the squares of the other two sides.
Given: In ⊿ABC, ∠B = 900 ∴ = ⇒ AC2 = AD.AB ----------------- (1)
2
To Prove: AC = AB + BC 2 2
∆BCD ~ ∆BAC [Theorem 2.7]
Consruction: Draw BD⟘AC ∴ = ⇒ BC2 = BA.BD ---------------(2)
Proof: In ∆ADB and ∆ABC dividing (2) by (1)
∠B = ∠D = 900 = x =
∠A = ∠A [Common angle] Example 11 : A ladder is placed against a wall such that its foot is at a distance of
∆ADB ~ ∆ABC [AA similarity criteria] 2.5 m from the wall and its top reaches a window 6 m above the ground. Find the length
⇒ = of the ladder.
⇒ AD.AC = AB2 --------- (1) Solution:
In ∆BDC and ∆ABC Let AB is a ladder, CA is a wall and A is a window
∠B = ∠D = 900 ∴BC = 2.5m ªÀÄvÀÄÛ CA = 6m
∠C = ∠C [Common angle] AB2 = BC2 + CA2 [by Pythagoras theorem]
∆BDC ~ ∆ABC [AA similarity criteria] AB2 = (2.5)2 + 62
⇒ = AB2 = 6.25 + 36
AB2 = 42.25

P a g e 54 | 159 Available in ykoyyur.blogspot.com P a g e 55 | 159 Available in ykoyyur.blogspot.com


SSLC Mathematics Solutions – Part -1 YK SSLC Mathematics Solutions – Part -1 YK
AB = 6.5 Adding (1) and (2),
The length of the ladder 6.5m OB2 + OD2 = BP2 + OP2 + OQ2 + DQ2
Example 12: In Fig. 2.50, if AD⟘BC, prove that AB2 + CD2 = BD2 + AC2 OB2 + OD2 = CQ2 + OP2 + OQ2 + AP2 [ BP=CQ and DQ=AP]
OB2 + OD2 = CQ2 + OQ2 + OP2 + AP2
OB2 + OD2 = OC2 + OA2 [from (3) and (4)]
Solution: In ∆ADC, ∠ ADC = 900
Exercise 2.5
∴ AC2 = AD2 + CD2 -------(1) [by Pythagoras theorem]
In ∆ADB, ∠ ADB = 900 1. Sides of triangles are given below. Determine which of them are right triangles.In
∴ AB2 = AD2 + BD2 -------(2) [by Pythagoras theorem] case of a right triangle, write the length of its hypotenuse.
Substract (2) from (1) i) 7cm, 24cm, 25cm
AB2 - AC2 = BD2 - CD2 ii) 3cm, 8cm, 6cm
AB2 + CD2 = BD2 + AC2 iii) 50cm, 80cm, 100cm
Example 13 : BL and CM are medians of a triangle ABC right angled at A. Prove that iv) 130cm, 12cm, 5cm
4 (BL2 + CM2 ) = 5 BC2 (i) 7 cm, 24 cm and 25 cm.
Solution: In ∆ABC, ∠A = 900 , BL and CM are the medians Squaring the numbers 49, 576, and 625.
In ∆ABC, 49 + 576 = 625
BC2 = AB2 + AC2 [Pythagoras Theorem] -----(1) (7)2 + (24)2 = (25)2
In ∆ABL, ∴ Converse of Pythagoras theorem, It is a right triangle.
BL2 = AL2 + AB2 [Pythagoras Theorem] Length of the hypotenuse = 25cm
(ii) 3 cm, 8 cm and 6 cm.
⇒ BL2 = + AB2 [L is the mid-point of AC] Squaring the numbers, 9, 64, and 36.
⇒4BL2 = AC2 + 4AB2 --------------------------(2) 9 + 36 ≠ 64
In ∆CMA,
⇒ 32 + 62 ≠ 82
CM2 = AC2 + AM2
It is not a right triangle
⇒ CM2 = AC2 + [M is the mid-point of AB] (iii) 50 cm, 80 cm and 100 cm.
Squaring the numbers 2500, 6400 and 10000.
⇒CM = AC +
2 2
2500 + 6400 ≠ 10000
⇒ 4CM 2 = 4AC2 + AB2 -----------------------(3) ⇒ 502 + 802 ≠ 1002
Adding (2) and (3)
It is not a right triangle
4(BL2 + CM 2) = 5(AC2 + AB2 ) (iv) 13 cm, 12 cm and 5 cm.
⇒ 4(BL2 + CM2 ) = 5 BC2 [from(1)]
Squaring the numbers 169, 144, and 25.
Example 14 : O is any point inside a rectangle ABCD (see Fig. 2.52). Prove that
144 +25 = 169
OB2 + OD2 = OA2 + OC2
⇒122 + 5 2 = 132
Solution: Draw PQ through the point O such that PQǁADǁ BC
∴ Converse of Pythagoras theorem, It is a right triangle.
∴ PQ⟘AB and PQ⟘DC [∠B = 90 0 and ∠C = 90 0]
Length of the hypotenuse = 13cm
In ∆OPB, ∠OPB = 90 0
2. PQR is a triangle right angled at P and M is a point on QR such that PM⟘QR. Show
OB2 = BP2 + OP2 -------------- (1)
In ∆OQD, that PM2 = QM . MR
Solution: In ΔPQM, ∠P = 900 and PM⟘QR
OD2 = OQ2 + DQ2 ---------------(2)
In ∆OQC, ∴ ∆PQM ~ ∆RPM [Theorem 2.7]
OC2 = OQ2 + CQ2 ---------------(3) ⇒ =
In ∆OAP, ⇒PM2 = QM. MR
OA2 = AP2 + OP2 --------------(4) 3. In Fig. 2.53, ABD is a triangle right angled at A and AC⟘BD. Show that

P a g e 56 | 159 Available in ykoyyur.blogspot.com P a g e 57 | 159 Available in ykoyyur.blogspot.com


SSLC Mathematics Solutions – Part -1 YK SSLC Mathematics Solutions – Part -1 YK
i) AB2 = BC.BD Solution:ABCD is a rhombus
ii) AC2 = BC.DC ∴ AC and BD bisects each other perpendicularly at O
iii) AD2 = BD.CD JAzÀÄ ¸Á¢ü¹
∴ AO = CO = and BO = DO =
(i) In ΔADB, ∠A = 900 , AC⟘BD
In ΔAOB, ∠AOB = 90°
⇒∆ABD~∆ CBA [Theorem 2.7]
AB2 = AO2 + BO2 [by Pythagoras theorem]
⇒ = ⇒ AB2 = BC.BD
(ii) In ΔADB, ∠A = 900 , AC⟘BD ⇒AB2 = +
⇒∆ABC~∆ DAC [Theorem 2.7] ⇒AB2 = +
⇒ = ⇒ AC2 = BC.DC ⇒4AB2 = AC2 + BD2
(iii) In ΔADB, ∠A = 90 0 , AC⟘BD AB2 + AD2 + DC2 + BC2 = AC2 + BD2 [ AB=BC=CD=DA]
⇒∆ABD~∆ CAD [Theorem 2.7] 8. In Fig. 2.54, O is a point in the interior of a triangle ABC, OD⟘BC, OE⟘AC and
⇒ = ⇒ AD2 = BD × CD OF⟘AB . Show that
i) OA2 + OB2 + OC2 - OD2- OE2 - OF2 = AF2 + BD2 + CE2
4. ABC is an isosceles triangle right angled at C. Prove that AB2 = 2AC2 ii) AF2 + BD2 + CE2 = AE2 + CD2 + BF2 JAzÀÄ ¸Á¢ü¹.
Solution:In ΔABC,∠C = 900 Solution:Join OA, OB and OC
AC = BC [Given] (i) In ΔAOF, ∠OFA = 90°
AB2 = AC2 + BC2 [By Pythagoras theorem] ∴ OA2 = OF2 + AF2 [ By Pythagoras theorem]
AB2= AC2 + AC2 [AC = BC] In ΔBOD, ∠ODB = 90°
∴ OB2 = OD2 + BD2 [ By Pythagoras theorem]
AB2 = 2AC2 In ΔCOE, ∠OEC = 90°
5. ABC is an isosceles triangle with AC = BC. If AB2 = 2 AC2, prove that ABC is a right ∴ OC2 = OE2 + EC2 [ By Pythagoras theorem]
triangle. Adding all three,
Solution:InΔABC, OA2 + OB2 + OC2 = OF2 + AF2 + OD2 + BD2 + OE2 + EC2
AC = BC and AB2 = 2AC2 [ Given] OA2 + OB2 + OC2 – OD2 – OE2 – OF2 = AF2 + BD2 + CE2.
AB2 = AC2 + AC2 (ii) AF2 + BD2 + EC2 = OA2 - OF2 + OB2 - OD2 + OC2 - OE2
AB2 = AC2 + BC2 [AC = BC] AF2 + BD2 + EC2 = OA2 - OE2 + OC2 - OD2 + OB2 - OF2
∴ΔABC is a right triangle ∴ AF2 + BD2 + CE2 = AE2 + CD2 + BF2.
9. A ladder 10 m long reaches a window 8 m above the ground.
6. ABC is an equilateral triangle of side 2a. Find each of its altitudes.
Find the distance of the foot of the ladder from base of the wall.?
Solution:ABC is an equilateral triangle
AB = BC = CA = 2a. Solution:Height of the wall CA = 8m, Height of the ladder AB = 10m
Draw height AM⊥ BC AB2 = AC2 + BC2 [By Pythagoras theorem]
In ΔABM, 102 = 82 + BC2
⇒BM = MC = a [∆ABC is an equilateral, AM⟘BC Bisects BC ] BC2 = 100 - 64
BC2 = 36
∠AMB = 900 [ AM⟘BC is the height]
AB2 = AD2 + BD2 BC = 6m
The foot of the ladder is 6m away from the base of the wall
⇒ AD2 = (2a)2 - a2
⇒ AD2 = 4a2 – a2 10. A guy wire attached to a vertical pole of height 18 m is 24 m long and has a stake attached
⇒ AD = √3a to the other end. How far from the base of the pole should the stake be driven so that
7. Prove that the sum of the squares of the sides of a rhombus is equal to the sum of the the wire will be taut?
squares of its diagonals.
Solution:Length of the Pole AB= 18m,
Length of the wire AC = 24m the wire.
AC2 = AB2 + BC2 [By Pythagoras theorem]
242 = 18 2 + BC2
BC2 = 576 - 324

P a g e 58 | 159 Available in ykoyyur.blogspot.com P a g e 59 | 159 Available in ykoyyur.blogspot.com


SSLC Mathematics Solutions – Part -1 YK SSLC Mathematics Solutions – Part -1 YK
BC2 = 252 14. The perpendicular from A on side BC of a ∆ABC intersects BC at D such that
BC = 6√7m DB = 3 CD (see Fig. 2.55). Prove that 2 AB2 = 2 AC2 + BC2
11. An aeroplane leaves an airport and flies due north at a speed of 1000 km per hour. At the Solution:In ΔABC, AD ⊥ BC and DB = 3CD
same time, another aeroplane leaves the same airport and flies due west at a speed of In right angle triangles ADB and ADC,
AB2 = AD2 + BD2 ... (i) [By Pythagoras theorem]
1200 km per hour. How far apart will be the two planes after 1 hours? AC2 = AD2 + DC2 ... (ii) [By Pythagoras theorem]
Solution:The speed of the aeroplane which flies due north = 1000 km/hr Substract (i) from (ii),
∴ The distance travelled in 1 hours = 1500 km AB2 - AC2 = DB2 - DC2
AB2 - AC2 = 9CD2 - CD2 [∴ BD = 3CD]
The speed of the aeroplane which flies due west = 1200km/hr
∴ The distance travelled in 1 hours = 1800 km AB2 - AC2 = 8 x [∵BC = DB + CD = 3CD + CD = 4CD]
In ΔAOB, ∠AOB = 90° ∴ AB2 - AC2 =
AB2 = AO2 + OB2 [by Pythagoas theorem] ⇒ 2(AB2 - AC2) = BC2
⇒ AB2 = (1500)2 + (1800)2 ⇒ 2AB2 - 2AC2 = BC2
⇒ AB = √2250000 + 3240000 ∴ 2AB2 = 2AC2 + BC2.
⇒ AB = √5490000 15. In an equilateral triangle ABC, D is a point on side BC such that BD = BC Prove that
⇒ AB = 300√6 km 9AD2 = 7AB2
12. Two poles of heights 6 m and 11 m stand on a plane ground. If the distance between the Solution:In an equilateral triangle ABC,
feet of the poles is 12 m, find the distance between their tops. Let AB = BC = AC = a BD = = , Draw AE⟘BC
Solution:Pole CD= 11m and Pole AB = 6m ⇒ BE = EC = =
∴ CP = 11 - 6 = 5 m
⇒AE = a -
2 2
= =
Distance between two poles BD = 12m = AP

ΔAPC, ∠CPC = 90° ⇒ AE =
AP2 + PC2 = AC2 DE = BE - BD = - =
AC2 = (12m)2 + (5m)2 In ΔADE, ∠AED = 900
AC2 = (144+25)m2 = 169 m2 AD2 = AE2 + DE2 [by Pythagoras Theorem]
AC = 13m √
AD2 = +
∴ The distance between the top = 13 m. 2
AD = + = = =
13. D and E are points on the sides CA and CB respectively of a triangle ABC right angled
⇒ AD2 = AB2
at C. Prove that AE2 + BD2 = AB2 + DE2
Solution:In ΔACE, ∠ACE = 900 ⇒ 9 AD2 = 7 AB2
∴ AC2 + CE2 = AE2 .... (i) 16. In an equilateral triangle, prove that three times the square of one side is equal to four
In ΔBCD, ∠ BCD = 90 0 times the square of one of its altitudes.
∴ BC2 + CD2 = BD2 .... (ii) Solution: In an equilateral triangle ABC,
Let AB = BC = AC = a .AE⟘BC .
Adding (1) and (2),
AC2 + CE2 + BC2 + CD2 = AE2 + BD2 ... (iii) ⇒ BE = EC = =
In ΔCDE, ∠DCE = 900 In ΔABE, ∠AEB = 900
DE2 = CD2 + CE2 ------------ (iv) AB2 = AE2 + BE2 [by Pythagoras Theorem]
In ΔABC, ∠ACB = 900 = AE2 +
AB2 = AC2 + CB2 ----------- (v)
Substitute (iv) and (v) in (iii) = AE2 +
DE2 + AB2 = AE2 + BD2.

P a g e 60 | 159 Available in ykoyyur.blogspot.com P a g e 61 | 159 Available in ykoyyur.blogspot.com


SSLC Mathematics Solutions – Part -1 YK SSLC Mathematics Solutions – Part -1 YK

⇒ AE2 = = Pair of Linear Equations in


4AE2 = 3a2
⇒ 4 × (height)2 = 3 × (Side)2 two Variables
(v) Tick the correct answer and justify : Linear equation with one variable: The algebraic equation of the type + =
In ∆ABC, AB = 6√ cm, AC = 12 cm and BC = 6 cm∆ABC AiÀÄ°è AB = 6√ cm, AC = 12cm
( ≠ and are real numbers, − variable is called linear equation of one variable.
the angle B is,
These type of equations having only one solution.
A) 120 0 B) 600 C) 900 D) 450
AB = 6√3cm, AC = 12 cm, and BC = 6 cm Example : + = ⟹ =− ⟹ =
AB2 = 108, AC2 = 144 and BC2 = 36 3.2 Pair of Linear Equations in Two Variables
AB2 + BC2 = AC2 2x + 3y = 5 ;
108 + 36 = 144 x - 2y - 3 = 0 and
∴ ∠B = 90° x - 0y = 2, ⇒ x = 2
The ans is C).900
An equation which can be put in the form ax + by + c = 0, where a, b and c are real numbers,
and a and b are not both zero, is called a linear equation in two variables x and y. A solution
of such an equation is a pair of values, one for x and the other for y, which makes the two
2.7 Summary sides of the equation equal.
1. Two figures having the same shape but not necessarily the same size are called similar figures. In fact, this is true for any linear equation, that is, each solution (x, y) of a linear equation
2. All the congruent figures are similar but the converse is not true in two variables, ax + by + c = 0, corresponds to a point on the line representing the
3. Two polygons of the same number of sides are similar, if (i) their corresponding angles are equation, and vice versa.
equaland (ii) their corresponding sides are in the same ratio (i.e., proportion). 2x + 3y=5; x - 2y - 3 = 0
4. If a line is drawn parallel to one side of a triangle to intersect the other two sides in distinct
points, then the other two sides are divided in the same ratio. These two linear equations are in the same two variables x and y. Equations like these are
5. If a line divides any two sides of a triangle in the same ratio, then the line is parallel to the third called a pair of linear equations in two variables.
side. The general form for a pair of linear equations in two variables x and y is,
6. If in two triangles, corresponding angles are equal, then their corresponding sides are in the + + = 0 and + + =0
same ratio and hence the two triangles are similar (AAA similarity criterion). Here, , , , , , are real numbers
7. If in two triangles, two angles of one triangle are respectively equal to the two angles of the Some examples of pair of linear equations in two variables are:
other triangle, then the two triangles are similar (AA similarity criterion). (i) 2x + 3y - 7 = 0 ; 9x - 2y + 8 = 0
8. If in two triangles, corresponding sides are in the same ratio, then their corresponding angles (ii) 5x = y ; -7x + 2y + 3 = 0
are equal and hence the triangles are similar (SSS similarity criterion). (iii) x + y = 7 ; 17 = y
9. If one angle of a triangle is equal to one angle of another triangle and the sides including these Two lines in a plane, only one of the following three possibilities can happen:
angles are in the same ratio (proportional), then the triangles are similar (SAS similarity (i) The two lines will intersect at one point.
criterion). (ii) The two lines will not intersect, i.e., they are parallel.
10. The ratio of the areas of two similar triangles is equal to the square of the ratio of their (iii) The two lines will be coincident.
corresponding sides.
11. If a perpendicular is drawn from the vertex of the right angle of a right triangle to the
hypotenuse, then the triangles on both sides of the perpendicular are similar to the whole
triangle and also to each other.
12. In a right triangle, the square of the hypotenuse is equal to the sum of the squares of the other
two sides (Pythagoras Theorem).
13. If in a triangle, square of one side is equal to the sum of the squares of the other two sides,
then the angle opposite the first side is a right angle.
Example 1: Akhila goes to a fair with Rs 20 and wants to have rides on the Giant Wheel
and play Hoopla. Represent this situation algebraically and graphically (geometrically).

P a g e 62 | 159 Available in ykoyyur.blogspot.com P a g e 63 | 159 Available in ykoyyur.blogspot.com


SSLC Mathematics Solutions – Part -1 YK SSLC Mathematics Solutions – Part -1 YK
Solution: The pair of equations formed is : Solution : Two solutions of each of the equations :
y = x ⇒ 2y = x + 2 = 4 and 2 + 4 = 12
⇒ x – 2y = 0 (1) and 3x + 4y = 20 (2) +2 =4 ⟹2 =4− ⟹ =
Let us represent these equations graphically. 0 4
For this, we need at least two solutions for = 2 0
each equation.
x 0 2
2 + 4 = 12 ⟹ =
y= 2 1 0 6
= 3 0
x 0 4 8
y= 5 2 -1 Exercise 3.1
1. Aftab tells his daughter, “Seven years ago, I was seven times as old as you were
then.Also, three years from now, I shall be three times as old as you will be.” (Isn’t
Example 2 : Romila went to a stationery shop and purchased 2 pencils and 3 erasers for this interesting?) Represent this situation algebraically and graphically.
Rs 9. Her friend Sonali saw the new variety of pencils and erasers with Romila, and she Let the present age of Aftab = years and the present age of his daughter = years.
also bought 4 pencils and 6 erasers of the same kind for Rs 18. Represent this situation Age of Aftab before7years = ( − 7) and the age of his daughter before 7 years = ( − 7)
Then the algebraic representation is,
algebraically and graphically. − 7 = 7( − 7) ⇒ − 7 = 7 − 49 ⇒ − 7 = −42
Solution : Let us denote the cost of 1 pencil by x’ and one eraser by y’. Then the algebraic Age of Aftab after 3 years= ( + 3) years and age of his daughter after 3 years= + 3
Then the algebraic representation is,
representation is given by the following equations: + 3 = 3( + 3) ⇒ + 3 = 3 + 9 ⇒ − 3 = 9 − 3 ⇒ − 3 = 6
2x + 3y = 9 -------------(1) Solutions of each of the equations :
4x + 6y = 18 -----------(2) − 7 = −42 ⟹ 7 = + 42 ⟹ =
(1) ⇒ 3y = 9 - 2x -7 0 7
y=
= 5 6 7
x 0 3 6
y= 3 1 -1 = ⟹ = = =
(2) ⇒ 6y = 18 – 4x = ⟹ = = =
y=
−3 =6⟹ =
x 0 3 6
6 3 0
y= 3 1 -1
= 0 -1 -2
E xa mple 3 : Two r a ils a r e r ep r es ent ed b y t h e equa t ions x + 2y – 4 = 0
and 2x + 4y – 12 = 0 Represent this situation geometrically. =6⟹y= = =0
x=3⟹y= = = −1
The two lines are intersecting each other. There fore there
x=0⟹y= = = −2 is a unique solution. The coordinates of intersecting
point are (42, 12)

P a g e 64 | 159 Available in ykoyyur.blogspot.com P a g e 65 | 159 Available in ykoyyur.blogspot.com


SSLC Mathematics Solutions – Part -1 YK SSLC Mathematics Solutions – Part -1 YK
2. The coach of a cricket team buys 3 bats and 6 balls for Rs 3900. Later, she buys another bat and 3.3. Graphical Method of Solution of a Pair of Linear Equations
3 more balls of the same kind for Rs1300. Represent this situation algebraically and
geometrically.
Let no.of bats = , Let no.of balls = . Then the algebraic representation is, Consistent pair : A pair of linear equations in two variables, which has
3 + 6 = 3900 and + 3 = 1300
a solution, is called a consistent pair of linear equations.
3 + 6 = 3900 ⟹ 6 = 3900 − 3 ⟹ =
Dependent pair : A pair of linear equations which are equivalent has
( )
300 100 -100 = 300 ⟹ = = = 500 infinitely many distinct common solutions. Such a pair is called a
= 100 ⟹ =
( )
= = 600 dependent pair of linear equations in two variables.
= 500 600 700 ( )
= −100 ⟹ = = = 700
Inconsistent pair : A pair of linear equations which has no solution, is
called an inconsistent pair of linear equations.
+ 3 = 1300 ⟹ =
a x + b y + c = 0
x = 400 ⟹ y = = = 300
a x + b y + c = 0 JA§ JgÀqÀÄ ¸À«ÄÃPÀgÀtUÀ½UÉ,
x = 700 ⟹ y = = = 200
x = 1000 ⟹ y = = = 100
Comparing Representing Algebraic
Consistency
the ratios on graph solution
400 700 1000

= 300 200 100 a b


≠ Intersecting Unique
Consistant
a b each other solution
The two lines are intersecting each other.
There fore there is a unique solution. The
coordinates of intersecting point are (1300, 0) a b c
3. The cost of 2 kg of apples and 1kg of grapes on a day was found to be Rs160. After a = = coincident Infinite
a b c lines. dependent
month, the cost of 4 kg of apples and 2 kg of grapes is Rs 300. Represent the situation sulutions
algebraically and geometrically.
Let the cost of apples/ kg = Rs x, the cost of grapes/kg = Rs y a b c
Then the algebraic representation is, = ≠ Parallel lines No solutions Inconsistent
2 + = 160; 4 + 2 = 300 a b c
2 + = 160 ⟹ = 160 − 2
50 60 70 Example 4 : Check graphically whether the pair of equations
1) + = ( ) and
= − 60 40 20 − = ( )
is consistent. If so, solve them graphically.
4 + 2 = 300 ⟹ = +3 =6⟹3 =6− ⟹ =
= ⟹ = = =
70 80 75
= ⟹ = = =
= 10 -10 0
0 6
= 2 0
Both lines are parallel to each other. There
fore there are no solutions for thes equations.

P a g e 66 | 159 Available in ykoyyur.blogspot.com P a g e 67 | 159 Available in ykoyyur.blogspot.com


SSLC Mathematics Solutions – Part -1 YK SSLC Mathematics Solutions – Part -1 YK
2 − 3 = 12 ⟹ 3 = 2 − 12 ⟹ = Exercise 3.2
( ) 0 3
= ⟹ = = =− 1. Form the pair of linear equations in the following problems, and find their solutions
( ) -4 -2 graphically.
= ⟹ = = =− = (i) 10 students of Class X took part in a Mathematics quiz. If the number of girls is 4 more
than the number of boys, find the number of boys and girls who took part in the quiz.
Both lines are intersecting at the point (6,0). Therefore the solution of the equation is =6 (ii) 5 pencils and 7 pens together cost Rs 50, whereas 7 pencils and 5 pens together
and = 0 ⇒ The equations are consistant pair. cost Rs 46. Find the cost of one pencil and that of one pen
Example 5 : Graphically, find whether the following pair of equations has no solution, (i) Let the number of girls be number of boys be
unique solution or infinitely many solutions: + = 10 (1) ; − =4 (2)
− + = ( ) (1) + = 10 ⟹ = 10 −
− + = ( ) =5⟹ = 10 − 5 = 5
Multiplying equation (2) by =4⟹ = 10 − 4 = 6
=6⟹ = 10 − 6 = 4
3 − + =0 5 4 6
5 −8 +1 = 0 5 6 4
= −
But, this is the same as Equation (1). Hence the lines represented by Equations (1)and (2)
(2)⇒ − =4⟹ = −4
are coincident. Therefore, Equations (1) and (2) have infinitely many solutions.
Example 6 : Champa went to a ‘Sale’ to purchase some pants and skirts. When her =5⟹ = 5−4 = 1
=4⟹ = 4−4 = 0
friends asked her how many of each she had bought, she answered, “The number of
=3⟹ = 3 − 4 = −1
skirts is two less than twice the number of pants purchased. Also, the number of skirts is
four less than four times the number of pants purchased”. Help her friends to find how 5 4 3
many pants and skirts Champa bought. = − 1 0 -1
Let us denote the number of pants by x and the number of skirts by y.
Then the equations are: Two lines are intersecting at the point (7, 3)
= 2 −2 (1) ∴ The solution is: x = 7, y = 3
= 4 −4 (2) ⇒Number of Girs = 7, Number of boys = 3
y = 2x − 2 (ii) Let the cost of a pencil be Rs , and the cost of a pen is Rs Then the equation are:
x = 2 ⟹ y = 2(2) − 2 = 4 − 2 = 2 5 + 7 = 50 and 7 + 5 = 46
x = 1 ⟹ y = 2(1) − 2 = 2 − 2 = 0 5 + 7 = 50 ⟹ =
x = 0 ⟹ y = 2(0) − 2 = 0 − 2 = −2 ( )
x=3⟹y= = = =5
2 1 0 50 − 5(10) 50 − 50
x = 10 ⟹ y = = =0
= − 2 0 -2 7 7
50 − 5(−4) 70
x = −4 ⟹ y = = = 10
7 7
y = 4x − 4
3 10 -4
x = 0 ⟹ y = 4(0) − 4 = 0 − 4 = −4
x = 1 ⟹ y = 4(1) − 4 = 4 − 4 = 0 = 5 0 10
The two lines intersect at the point (1, 0). 7x + 5y = 46
0 1
So, x = 1, y = 0 is the required solution of the
⟹ 5y = 46 − 7x ⟹ y =
= − -4 0 pair of linear equations, i.e., the number of pants
she purchased is 1 and she did not buy any skirt. ( )
x=8⟹y= = = = −2 8 3 -2
( )
x=3⟹y= = = =5
( ) = -2 5 12
x = −2 ⟹ y = = = = 12

Two lines are intersecting at the point (3, 5). ∴ The solution is: x = 3, y = 5

P a g e 68 | 159 Available in ykoyyur.blogspot.com P a g e 69 | 159 Available in ykoyyur.blogspot.com


SSLC Mathematics Solutions – Part -1 YK SSLC Mathematics Solutions – Part -1 YK
Therefore the cost of pencil = Rs 3; The cost of pen = Rs 5 (iv) − = ; − + =−
1) On comparing the ratios , , find out whether the lines representing the following = =− , = =− , = =− ∴ = =
pair of linear equations intersect at a point, parallel or coincident. ⇒This pair of linear equations are consistent and have infinite solutions.
(i) 5 − 4 + 8 = 0 (ii) 9 + 3 + 12 = 0 (iii) 6 − 3 + 10 = 0 (v) + = ; + =
7 + 6 − 9 = 0 18 + 6 + 24 = 0 2 − + 9 = 0
(i) − + = = = × = , = , = = ∴ = =
+ − =
⇒This pair of linear equations are consistent and have infinite solutions.
Comparing these with + + = 0 and + + = 0 we get
4. Which of the following pairs of linear equations are consistent/inconsistent? If
= 5, = −4, = 8 and = 7, = 6, = −9
consistent, obtain the solution graphically:
= , = = ∴ ≠ (i) + = 5, 2 + 2 = 10
∴ The pair of lines intersect at a point and have unique solution. (ii) – = 8, 3 − 3 = 16
(ii) + + = (iii) 2 + − 6 = 0, 4 − 2 − 4 = 0
+ + = (iv) 2 − 2 − 2 = 0, 4 − 3 − 5 = 0
Comparing these with a x + b y + c = 0 and a x + b y + c = 0 we get (i) + = ; + =
a = 9, b = 3, c = 12 and a = 18, b = 6, c = 24 = , = , = = ∴ = =
= = , = = , = = ∴ = = ∴ These are coincident lines and have infinite solutions. Hence the pair is consistent.
+ = 5⟹ =5−
∴ The pair of lines are coincident.and have infinite solution.
x= 2⟹ y =5−2=3
(iii) − + =
x= 3⟹ y =5−3=2
− + = x= 4⟹ y =5−4=1
Comparing these with + + = 0 and + + = 0 we get
= 6, = −3, = 10 and = 2, = −1, =9 2 3 4
= =3, = =3, = ∴ = ≠ = − 3 2 1

∴ The pair of lines are parallel and have no solution. 2x + 2y = 10 ⟹ =


( )
2) On comparing the ratios , and find out whether the lines representing the x=2⟹y= = =3
10 − 2(3) 10 − 6
following pair of linear equations are consistent or inconsistent? x=3⟹y= = =2
2 2
(i) 3 + 2 = 5 ; 2 − 3 = 7 x=4⟹y=
( )
= =1
(ii) 2 − 3 = 8 ; 4 − 6 = 9
(iii) + = 7 ; 9 − 10 = 14 2 3 4
(iv) 5 − 3 = 11 ; −10 + 6 = −22 = 3 2 1
(v) + 2 = 8; 2 + 3 = 12
(i) + = ; − = (ii) – = ; − =
= , = ∴ ≠ = , = = , = = ∴ = ≠

⇒This pair of linear equations are consistent. ∴ These are parallel lines and have no solutions.Hence the pair is inconsistent.
(ii) − = ; − = (iii) + – = ; − − =
= = , = = , = ∴ = ≠ = = , = =− ∴ ≠
⇒This pair of equations are inconsistent ∴these lines are consistent and intersect each other. These lines have unique solution
(2.2). 2 + – 6 = 0 ⟹ = 6 − 2
(iii) + = ; − =

= = × = , = = × =− ∴ ≠
⇒This pair of linear equations are consistent.

P a g e 70 | 159 Available in ykoyyur.blogspot.com P a g e 71 | 159 Available in ykoyyur.blogspot.com


SSLC Mathematics Solutions – Part -1 YK SSLC Mathematics Solutions – Part -1 YK
=0⟹ = 6 − 2(0) = 6 − 0 = 6 Half the perimeter: = 36 ⟹ + = 36
=1⟹ = 6 − 2(1) = 6 − 2 = 4 + = 36 ⟹ = 36 −
=2⟹ = 6 − 2(2) = 6 − 4 = 2 = ⟹ = − =
0 16 36
0 1 2 = ⟹ = − =
6 4 2 = − 36 20 0 = ⟹ = − =
= −
4 − 2 − 4 = 0⟹ = ∴ these lines are consistent and intersect each other. These lines have unique solution (16,20)
( ) ⇒Width = 16m Length = 20m
x=1⟹y= = = =0 6. Given the linear equation 2x + 3y – 8 = 0, write another linear equation in two variables
( ) such that the geometrical representation of the pair so formed is:
x=2⟹y= = = =2
( )
(i) Intersecting lines (ii) Parallel lines (iii) Coinsident lines
x=3⟹y= = = =4 (i) Given equation is 2x + 3y - 8 = 0
1 2 3 If the lines are intersecting then ≠
= 0 2 4 Therefor the second equation is 2 + 4 − 6 = 0
= =1, = ⇒ ≠
(iv) – – = ; − − =
= = , = = ∴ ≠ (ii) If the lines are parallel then = ≠
∴ these lines are consistent and intersect each other. These lines have unique solution. Therefor the the second line is 4 + 6 − 8 = 0
2x – 2y – 2 = 0 ⟹ y = = = , = = , = = ⇒ = ≠
( )
x=1⟹y= = = =0 (iii) If the lines are coincident then = =
Therfor the second line is 6 + 9 − 24 = 0
( )
x=2⟹y= = = =1
( )
=3⟹ = = = =2 = = , = = , = = ⇒ = =
1 2 3 6. Draw the graphs of the equations x – y + 1 = 0 and 3x + 2y – 12 = 0. Determine the
0 1 2 coordinates of the vertices of the triangle formed by these lines and the x-axis, and
=
shade the triangular region.
− +1 = 0 ⟹ = +1
4 – 3 –5 = 0 ⟹ =
( )− 0 1 2
= ⟹ = = = =
( )− 2 5 -1 = + 1 2 3
= ⟹ = = = =
(− )− − 1 5 -3
=− ⟹ = = =− == x= 0⟹ y =0+1=1
x= 1⟹ y =1+1=2
x= 2⟹ y =2+1=3
5. Half the perimeter of a rectangular garden, whose length is 4 m more than its width, is
36 m. Find the dimensions of the garden.
3x + 2y − 12 = 0 ⟹ y =
Let the width of the Garden = , Length = .
0 2 4
Length: = + 4
6 3 0
=
0 8 16 = ⟹ = + =
The coordinates of the vertices of the triangle are(2,3) , (−1,0) , (4,0)
4 12 20 = ⟹ = + =
= + = ⟹ = + = 3.4 Algebraic Methods of Solving a Pair of Linear Equations
In the previous section, we discussed how to solve a pair of linear equations graphically. The
graphical method is not convenient in cases when the point representing the solution of
the linear equations has non-integral coordinates

P a g e 72 | 159 Available in ykoyyur.blogspot.com P a g e 73 | 159 Available in ykoyyur.blogspot.com


SSLC Mathematics Solutions – Part -1 YK SSLC Mathematics Solutions – Part -1 YK
3.4.1 Substitution Method : 4 + 6y = 18
Step 1 : Find the value of one variable, say y in terms of the other variable, i.e., x from either
equation, whichever is convenient. 18 − 6 + 6 = 18
Step 2 : Substitute this value of y in the other equation, and reduce it to an equation in one 18 = 18
variable, i.e., in terms of x, which can be solved. This statement is true for all values of y. However, we do not get a specific value of y as a
solution. Therefore, we cannot obtain a specific value of x. This situation has arisen because
Step 3 : Substitute the value of x (or y) obtained in Step 2 in the equation used in Step 1 both the given equations are the same. Therefore, Equations (1) and (2) have infinitely many
to obtain the value of the other variable. solutions.
We have substituted the value of one variable by expressing it in terms of the other Example:10 Let us consider the Example 3 of Section 3.2. Will the rails cross each other?
variable to solve the pair of linear equations. That is why the method is known as the
+ 2 − 4 = 0 (1)
substitution method. 2 + 4 − 12 = 0 (2)
Example 7: Solve the following pair of equations by substitution method:. Equation (1) ⟹ = 4 − 2 (3)
− = ( ) Substituting in equation (2) we get,
+ = ( )
Equation (2) ⟹ + 2 = 3 2(4 − 2 ) + 4y − 12 = 0
8 − 4 + 4 − 12 = 0
⟹ = 3−2 (3) 8 − 12 = 0
Substitute the value of x in equation (1) we get, −4 = 0
7(3 − 2 ) − 15 = 2 ⇒ 21 − 14 − 15 = 2
which is a false statement. Therefore, the equations do not have a common solution. So, the
−29 = 2 − 21 ⇒ = = two rails will not cross each other.
Substitute = in equation (3),
=3−2 = 3− = = Exercise 3.3
∴The solution is, = , = 1) Solve the following pair of linear equations by substitution method.
(i) + = ( )
Example 8: Aftab tells his daughter, “Seven years ago, I was seven times as old as you − = ( )
were then.Also, three years from now, I shall be three times as old as you will be.” Equation (1) ⟹ = 14 − (3)
(Isn’t this interesting?) Represent this situation algebraically and graphically.
Substituting in equation (2) we get,
Let the present age of Aftab be Years, The present age of his daughter be y’ years
The age of Aftab before 7 Years = − 7 Years 14 − − = 4 ⇒ 14 − 2 = 4
The age of his daughter before 7 years = − 7 yeares −2 = 4 − 14 ⇒ −2 = −10 ⇒ = =5
− 7 = 7( − 7) ⟹ − 7 + 42 = 0 (1) Substituting = 5 in equation (3)
After 3 years, his age = + 3 years and his daughter’s age = y +3 years = 14 − = 14 − 5 ⇒ = 9
+ 3 = 3( + 3) ⟹ −3 = 6 (2) ∴ = 9, = 5
Equation (2) ⟹ = 3 + 6 (3) (ii) − = ( )
Substituting the value of in (1) we get, + = ( )
3 + 6 − 7 + 42 = 0 Equation (1) ⟹ s = 3 + t (3)
4 = 48 ⇒ = 12
Substituting in equation (2) we get,
Substituting the value of in equation we get,
= 3(12) + 6 = 36 + 6 = 42 + =6 ⇒ =6
Therfor the age of Aftab and his daughter is 42 and 12 respectively.
⇒ 6 + 5t = 36 ⇒ 5t = 36 − 6 ⇒ t =
Example 9 : Let us consider Example 2 in Section 3.3, i.e., the cost of 2 pencils and3
erasers is Rs 9 and the cost of 4 pencils and 6 erasers is Rs 18. Find the cost of each Substituting = 6 in equation (3)
s= 3+t ⇒ s= 3+6 ⇒ = 9
pencil and each eraser.
∴ = 9, =6
Let the cost of pencil be Rs x and the cost of rubber be Rs y , the the equations are (iii) − = ( )
2 +3 = 9 (1) − = ( )
4 + 6 = 18 (2) Equation (1) ⟹ = 3 − 3 Substituting in equation (2) we get,
Equation (1) ⟹ 2 = 9 − 3 ⟹ = (3) 9 − 3(3 − 3 ) = 9 ⇒ 9 − 9 + 9 = 9
Substituting in equation (2) we get, 9=9

P a g e 74 | 159 Available in ykoyyur.blogspot.com P a g e 75 | 159 Available in ykoyyur.blogspot.com


SSLC Mathematics Solutions – Part -1 YK SSLC Mathematics Solutions – Part -1 YK
This statement is true for all values of y. However, we do not get a specific value of y as a 2 + 3 = 11 (1)
solution. Therefore, we cannot obtain a specific value of x. This situation has arisen because 2 − 4 = −24 (2)
both the given equations are the same. Therefore, Equations (1) and (2) have infinitely many Equation (2) ⟹ 2 = 4 − 24 ⟹ = 2 − 12 (3)
solutions. Substituting in equation (1) we get,
(iv) . + . = . 2(2 − 12 ) + 3 = 11
. + . = . 4 − 24 + 3 = 11 ⇒ 7 = 11 + 24 ⇒ 7 = 35 ⇒ = 5
0.2 + 0.3 = 1.3 (1) × 10 substituting = 5 in equation (3)
0.4 + 0.5 = 2.3 (2) × 10 = 2x5 − 12 = 10 − 12 = −2
2 + 3 = 13 (3) ∴ = −2, =5
4 + 5 = 23 (4) = + 3
Equation (3) ⟹ 2 = 13 − 3 ⟹ = (5) 5 = (−2) + 3
Substituting in equation (4) we get, 5 − 3 = −2 ⇒ −2 = 2 ⇒ = = −1
4 + 5 = 23 3. Form the pair of linear equations for the following problems and find their solution by
26 − 6 + 5 = 23 substitution method
26 − 23 = ⇒ = 3 , Substituting = 3 in equation (5) (i) The difference between two numbers is 26 and one number is three times the other.
= =
( )
= =2 Find them.
Let the first number be , the second number be , then > . By the given condition the
∴ = 2, =3
equations are
(v) √ +√ = ( )
− = 26 (1)
√ −√ = ( ) =3 (2)

Equation (1) ⟹ √2 = −√3 ⟹ =− (3) Substituting the value of in equation (1) we get,

Substituting in equation (2) we get, 3 − = 26 ⇒ 2 = 26
= 13 , Substitute = 13 in equation (2)

√3 − − √8 = 0 ⇒ − − √4 × 2 =0 = 3(13) = 39
√ √
− − 2√2 =0 ⇒ − − 2√2 = 0 ∴ = 13, = 39
√ √
= 0 , Substituting = 0 in equation (3) (ii)The larger of two supplementary angles exceeds the smaller by 18 degrees. Find them.
√ ( )
Let the larger angle be , the smaller angle be . If the angles are supplementary then sum
=− =0 of two angles is 180 , By the given conditions the equations are,

∴ = 0, =0 + = 180 (1)
(vi) − =− ( ) = + 18 (2)
+ = ( ) Substituting the value of in equation (1) we get,
+ 18 + = 180 ⇒ 2 = 162
Equation (1) × 2 ⟹ 3 − 5 = −4 (3) = 81 Substitute = 81 in equation (2)
Equation (2) × 6 ⟹ 2 + 3 = 13 (4) = 81 + 18 = 99
Equation (3) ⟹ 3 = 5 − 4 ⟹ = (5) ∴ = 99 , = 81
(iii) The coach of a cricket team buys 7 bats and 6 balls for Rs 3800. Later, she buys 3
Substituting in equation (4) we get, bats and 5 balls for Rs 1750. Find the cost of each bat and each ball.
2 + 3 = 13 ⇒ = 13 Let the cost of a batÉ = Rs , the cost of a ball = Rs y. the equations are
19 − 8 = 39 ⇒ 19 = 39 + 8 ⇒ 19 = 47 ⇒ = 7 + 6 = 3800 (1)
3 + 5 = 1750 (2)
Substituting = in equation (5)À Equation (1) ⟹ 7 = 3800 − 6 ⟹ = (3)
= = × ⇒ = × ⇒ = Substituting the value of in equation (2)we get
3 + 5 = 1750
2. Solve 2x + 3y = 11 and 2x – 4y = – 24 and hence find the value of ‘m’ for which
y = mx + 3. = 1750 ⇒ 11400 + 17 = 12250

P a g e 76 | 159 Available in ykoyyur.blogspot.com P a g e 77 | 159 Available in ykoyyur.blogspot.com


SSLC Mathematics Solutions – Part -1 YK SSLC Mathematics Solutions – Part -1 YK
⇒ 17 = 12250 − 11400 ⇒ 17 = 850 ⇒ = = 50 Substituting the value of in equation (2)we get
10 + 3 − 7 = −30 ⇒ 10 − 4 = −30
Substitute = 50 in equation (3)
( ) ⇒ −4 = −30 − 10 ⇒ −4 = −40 ⇒ = = 10
= = = = 500
Substitute = 10 in equation (1),
∴ The cost of a bat = Rs 500 , The cost of a ball = Rs 50
= 10 + 3(10) = 10 + 30 = 40
(iv) The taxi charges in a city consist of a fixed charge together with the charge for the ∴ Age of Jocob = 40 and age of his son = 10
distance covered. For a distance of 10 km, the charge paid is Rs 105 and for a journey 3.4.2 Elimination Method
of 15 km, the charge paid is Rs 155. What are the fixed charges and the charge per
Example 11 : The ratio of incomes of two persons is 9 : 7 and the ratio of their
km? How much does a person have to pay for travelling a distance of 25km?
expenditures is 4 : 3. If each of them manages to save ` 2000 per month, find their
Let the fixed charges be Rs x, the charges/km be Rs . Then the equations are, monthly incomes
+ 10 = 105 (1) Solution: Let the incomes of two persons be Rs 9x and Rs 7x respectively.The expenditure
+ 15 = 155 (2)
Equation (1) ⟹ = 105 − 10 (3) be Rs 4y and Rs 3y, then we get the equations,
9x – 4y = 2000 (1)
Substituting the value of in equation (2)we get 7x – 3y = 2000 (2)
105 − 10 + 15 = 155
Multiply Equation (1) by 3 and Equation (2) by 4 to make the coefficients of y equal.
105 + 5 = 155 ⇒ 5 = 155 − 105 ⇒ = = 10 9x – 4y = 2000 (1) x 3
Substitute = 10 in equation (3) 7x – 3y = 2000 (2) x 4
= 105 − 10(10) = 105 − 100 = 5 27x – 12y = 6000 (3)
∴ the fixed charges is Rs 5 and charges per km is Rs 10 28x – 12y = 8000 (4)
The total charges to travel 25km is + 25 = 5 + 25(10) = 5 + 250 = Rs 255 -x = -2000
(v) A fraction becomes if 2 is added to both the numerator and the denominator If, 3 ⇒ x = 2000
Substitute x = 2000 in equation (1), we get
is added to both the numerator and the denominator it becomes Find the fraction. 9(2000) – 4y = 2000 ⇒18000 – 2000 = 4y ⇒4y = 16000 ⇒ y = 4000
Let the fraction be . By the given condition, ∴ The monthly incomes of two persons are = Rs 18000 and Rs 14000
+2 9 The method used in solving the example above is called the elimination method, because
= ⟹ 11 + 22 = 9 + 18
+ 2 11 we eliminate one variable first, to get a linear equation in one variable. In the example
⟹ 11 − 9 = 18 − 22 ⟹ 11 − 9 = −4 (1) above, we eliminated y. We could also have eliminated x. Try doing it that way.
+3 5 You could also have used the substitution, or graphical method, to solve this problem.
= ⟹ 6 + 18 = 5 + 15
+3 6 Try doing so, and see which method is more convenient.
⟹ 6 − 5 = 15 − 18 ⟹ 6 − 5 = −3 (2) Example 12 : Use elimination method to find all possible solutions of the following pair
Equation (1) ⟹ 11 = −4 + 9 ⟹ = (3) of linear equations
Substituting the value of in equation (2)we get
6 − 5 = −3 ⇒ = −3 2x + 3y = 8 (1)
4x + 6y = 7 (2)
⇒ −24 − = −33 ⇒ − = −33 + 24 ⇒ − = −9
Subtitute = 9 in equation (3) Multiply Equation (1) by 2 to make the coefficients of x equal.
( ) 2x + 3y = 8 (1) x 2
= = = =7 4x + 6y = 16 (3)
The fraction = Substracting (2) from (1) , we get
4x + 6y = 16 (3)
(vi) Five years hence, the age of Jacob will be three times that of his son. Five years ago, 4x + 6y = 7 (2)
Jacob’s age was seven times that of his son. What are their present ages? 0 = 9
Let age of Jocob = , and age of his son = By the given condition the equations are 0 = 9, which is a false statement. Therefore, the pair of equations has no solution
After 5 years + 5 = 3( + 5)
⟹ + 5 = 3 + 15 ⟹ − 3 = 10 (1) Example 13 : The sum of a two-digit number and the number obtained by reversing the
Before 5 years − 5 = 7( − 5) digits is 66. If the digits of the number differ by 2, find the number. How many such
⟹ − 5 = 7 − 35 ⟹ − 7 = −30 (2) numbers are there?
Equation (1) ⟹ = 10 + 3 (3) Let the two digits number = 10x + y

P a g e 78 | 159 Available in ykoyyur.blogspot.com P a g e 79 | 159 Available in ykoyyur.blogspot.com


SSLC Mathematics Solutions – Part -1 YK SSLC Mathematics Solutions – Part -1 YK
Number after reversing the digits = 10y + x (ii) 3x + 4y = 10 and 2x - 2y = 2
∴ 10x + y + 10y + x = 66 ⇒ 11x + 11y = 66 ⇒ x + y = 6 (1) Eliminating Method:
We are also given that the digits differ by 2, therefore, 3x + 4y = 10 (1)
x– y= 2 (2) 2x – 2y = 2 (2)
substract (2) from (1) Multiply Equation (2) by 2 to make the coefficients of y equal.
x+ y= 6 (1)
2x – 2y = 2 (2) x 2
x– y= 2 (2)
2y = 4 4x - 4y = 4 (3)
Adding equation (1) and (2)
⇒ y= 2
Substitute y = 2 in equation (1) 3x + 4y = 10 (1)
x+2=6 ⇒ x=4 4x - 4y = 4 (3)
Therfor the number is 10x + y = 10 x 4 + 2 = 42 7x = 14
⇒Two numbers are 42 and 24 ⇒ x=2
Substitute x = 2 in (1)
Exercise 3.4 3(2) + 4y = 10 ⇒ 6 + 4y = 10 ⇒ 4y = 10 – 6 ⇒ 4y = 4⇒ y = 1
∴ x = 2, y = 1
1. Solve the following pair of linear equations by the elimination method and the substitution Substituting Method:
method : 3x + 4y = 10 (1)
(i) x + y = 5 ªÀÄvÀÄÛ 2x - 3y = 4 2x – 2y = 2 (2)
Eliminating method: (2) ⇒ -2y = -2x +2 ⇒ y = x - 1
x + y= 5 (1) Substitute y = x – 1in equation (1)
2x –3y = 4 (2) 3x + 4(x – 1) = 10 ⇒ 3x + 4x - 4 = 10⇒ 7x = 10 + 4⇒ 7x = 14 ⇒x = 2
Multiply Equation (1) by 2 to make the coefficients of x equal. Substitute x = 2 in (1)
2x + 2y = 10 (3) 2(2) – 2y = 2 ⇒ 4 - 2y = 2⇒ -2y = 2 - 4⇒ -2y = -2 ⇒ y = 1
∴ x = 2, y = 1
Substracting (2) from (1), (iii) 3x - 5y - 4 = 0 ªÀÄvÀÄÛ 9x = 2y + 7
2x + 2y = 10 (3)
Eliminating method:
2x – 3y = 4 (2)
5y = 6 3x – 5y – 4 = 0 ⇒3x – 5y = 4 (1)
9x = 2y + 7 ⇒9x – 2y = 7 (2)
⇒ y=
Multiply Equation (1) by 3 to make the coefficients of x equal.
Substitute y = in equation (1),
9x – 15 y = 12 (3)
x+ = 5 ⇒ 5x + 6 = 25 ⇒ 5x = 19 ⇒x = Substracting (2) from (3)
∴ x = and y = 9x – 15 y = 12 (3)
Substituting Method: 9x – 2y = 7 (2)
x + y = 5 (1) - 13y = 5
2x – 3y = 4 (2)
-13y = 5 ⇒ y = −
(1) ⇒ y = 5 - x
Substitute y = 5 – x in (2) Substitute y = − in (1)
⇒2x – 3(5 – x) = 4 (3) 3x – 5 − = 4 ⇒ 3x + = 4 ⇒ 39x + 25 = 52 ⇒ 39x = 27⇒ x = =
⇒2x – 15 + 3x = 4 ⇒5x = 19 ⇒x=
∴x= and y = −
Substitute x = in (1)
Substituting Method:
+ y = 5 ⇒19 + 5y = 25 ⇒5y = 25 – 19 ⇒y = 3x – 5y – 4 = 0 ⇒3x – 5y = 4 (1)
∴ x= and y = 9x = 2y + 7 ⇒9x – 2y = 7 (2)

P a g e 80 | 159 Available in ykoyyur.blogspot.com P a g e 81 | 159 Available in ykoyyur.blogspot.com


SSLC Mathematics Solutions – Part -1 YK SSLC Mathematics Solutions – Part -1 YK
(1) ⇒-5y = 4 – 3x ⇒ 5y = 3x – 4 ⇒ y = (3) ⇒ x= 3
Substitute x = 3 in (1)
Substitute y = in (2)
3 – y = -2 ⇒ -y = - 2 – 3 ⇒ y = 5
9x – 2 = 7 ⇒9x – = 7 ⇒45x – 6x + 8 = 35 ⇒39x = 27 ⇒x = = The fraction =
Substitute x = in (1), (ii) Five years ago, Nuri was thrice as old as Sonu. Ten years later, Nuri will be twice as old
as Sonu. How old are Nuri and Sonu?
3 – 5y = 4 ⇒27 – 65y = 52 ⇒ – 65y = 52 - 27 ⇒ y = − ⇒y=− Let the age of Nuri = x and that of Sonu = y .According to question
∴À x = and y = − (x - 5) = 3(y - 5)
x - 3y = -10 (1) and
(iv) + = -1 and x - =3
(x + 10y) = 2(y + 10)
Eliminating Method: x - 2y = 10 (2)
+ = -1 ⇒ 3x + 4y = -6 (1) Substract (1) from (2)
x - 3y = -10 (1)
x- =3 ⇒ 3x - = 9 (2)
x - 2y = 10 (2)
Substracting (1) from (2) - y = -20
3x + 4y = -6 (1) ⇒y = 20
3x - = 9 (2) Substitute y = 20 in (1)
+5y = - 15 x - 60 = -10 ⇒ x = 50
⇒ y= - 3 ∴ The age of Nuri = 50 years and the age of Sonu = 20years
Substitute y = - 3 in (1) (iii) The sum of the digits of a two-digit number is 9. Also, nine times this number is
3x + 4(- 3) = -6 ⇒3x - 12 = -6 ⇒3x = 6 ⇒x = 2 twice the number obtained by reversing the order of the digits. Find the number.
Let the two digits number = xy , According to question,
∴ x = 2 and y =−3
x+y =9 (1)
Substituting Method:
2(10y + x) = 9(10x + y)
3x + 4y = -6 (1) 20y + 2x = 90x + 9y
3x - = 9 (2) 88x – 11y = 0
(2) ⇒ -y = 9 – 3x⇒ y = 3x – 9 (3) ⇒ 8x – y = 0 (2)
Substitute y = 3x – 9 in (1) By adding (1) and (2)
3x + 4(3x – 9) = -6 ⇒3x + 12x - 36 = -6 ⇒ 15x = 30 ⇒ x = 2 x+ y= 9 (1)
Substitute x = 2 in (3) 8x – y = 0 (2)
9x = 9
y = 3(2) – 9 ⇒y = 6 – 9 ⇒ y = -3
∴ x = 2 and y =−3 ⇒ x=1
2. Form the pair of linear equations in the following problems, and find their solutions Substitute x = 1 in (1)
(if they exist) by the elimination method : 1 + y= 9 ⇒ y= 8
(i) If we add 1 to the numerator and subtract 1 from the denominator, a fraction reduces to Therefor the number is xy = 18
1. It becomes if we only add 1 to the denominator. What is the fraction? (iv) Meena went to a bank to withdraw Rs 2000. She asked the cashier to give her
Let the fraction = Rs 50 and Rs 100 notes only. Meena got 25 notes in all. Find how many notes of Rs
50 and Rs 100 she received
According to question , = 1 ⇒ x + 1 = y – 1⇒ x – y = -2 (1) Let the number of Rs 50 notes = x and the number of Rs 100 notes = y. According to qn
and = ⇒ 2x = y + 1 ⇒ 2x – y = 1 (2) x + y = 25 (1) and
50x + 100y = 2000
Substract (1) from (2)
⇒ x + 2y = 40 (2)
x – y = -2 (1)
2x – y = 1 (2)
-x = -3

P a g e 82 | 159 Available in ykoyyur.blogspot.com P a g e 83 | 159 Available in ykoyyur.blogspot.com


SSLC Mathematics Solutions – Part -1 YK SSLC Mathematics Solutions – Part -1 YK
Solution: Let Rs x be the fare from the bus stand in Bangalore to Malleswaram, and Rs y to
Substract (1) from (2) Yeshwanthpur. From the given information, we have
x + 2y = 40 (2) 2x + 3y = 46, ⇒ 2x + 3y – 46 = 0 (1)
x + y = 25 (1) 3x + 5y = 74 ⇒ 3x + 5y – 74 = 0 (2)
y = 15
Substitute y = 15 in (1)
x + 15 = 25 ⇒ x = 25 – 15 ⇒ x = 10
Therefor the number of Rs 50 notes = 10 and the number of Rs 100 notes = 15
(v) A lending library has a fixed charge for the first three days and an additional charge
for each day thereafter. Saritha paid Rs 27 for a book kept for seven days, while Susy paid
Rs 21 for the book she kept for five days. Find the fixed charge and the charge for each
extra day . = =
Let the fixed charges for first 3 days Ì = Rs x =( =
( ) ( ) ) ( ) ( ) ( )
The additional charges for remaing days = Rs y According to question
⇒ = =
x + 4y = 27 (1)
x + 2y = 21 (2) ⇒ = = 1⇒ =1⇒x=8
Substract (1) from (2) ⇒ =1⇒ = 10
x + 2y = 21 (2)
Hence, the fare from the bus stand in Bangalore to Malleswaram is Rs 8 and the fare to
x + 4y = 27 (1)
Yeshwanthpur is Rs 10.
- 2y = -6
Example 15 : For which values of p does the pair of equations given below has unique
⇒y = 3
solution?
Substitute y = 3 in (1)
4x + py + 8 = 0
x + 4x3 = 27 ⇒ x + 12 = 27
2x + 2y + 2 = 0
⇒ x = 27 – 12 ⇒ x = 15 Here, a = 4, b = p, c = 8 and a = 2, b = 2, c = 2
The fixed charges = Rs 15 and the additional charges = Rs 3
Now for the given pair to have a unique solution : ≠
3.4.3 Cross - Multiplication Method
Equations are: ⇒ ≠ ⇒p ≠ 4
a x + b y + c = 0 Therefore, for all values of p, except 4, the given pair of equations will have a unique solution
a x + b y + c = 0 Example 16 : For what values of k will the following pair of linear equations have
infinitely many solutions?
x= y= [a b − a b ≠ 0] kx + 3y - (k - 3) = 0
= = 12x + ky - k = 0
For a pair of linear equations to have infinitely many solutions:
Here, = k, = 3, = -(k-3) and = 12, = k, = -k
= =
( )
⇒ = ⇒ = = ⇒ = ⇒ k2 = 36 ⇒ k = ±6
( )
= ⇒ =
⇒ 3k = k2 – 3k ⇒ 6k = k2 ⇒ (6k - k2 ) = 0 ⇒ k(6 - k ) = 0 ⇒ k = 0 CxÀªÁ 6 - k = 0 ⇒ k = 6
Example14:From a bus stand in Bangalore , if we buy 2 tickets to Malleswaram and 3 Therefore, the value of k, that satisfies both the conditions, is k = 6. For this value, the pair of
tickets to Yeshwanthpur, the total cost is Rs 46; but if we buy 3 tickets to Malleswaram linear equations has infinitely many solutions.
and 5 tickets to Yeshwanthpur the total cost is Rs 74. Find the fares from the bus stand
to Malleswaram, and to Yeshwanthpur.

P a g e 84 | 159 Available in ykoyyur.blogspot.com P a g e 85 | 159 Available in ykoyyur.blogspot.com


SSLC Mathematics Solutions – Part -1 YK SSLC Mathematics Solutions – Part -1 YK
Exercise 3.5 Here, a = 1, b = -3, c = -7 and a = 3, b = -3, c = -15

1. Which of the following pairs of linear equations has unique solution, no solution, or = ; = = 1; = =
infinitely many solutions. In case there is a unique solution, find it by using cross
multiplication method ⇒ ≠
(i) x - 3y - 3 = 0 Therefore the pair has unique solution.
3x – 9y – 2 =0
Here, a = 1, b = -3, c = -3 and a = 3, b = -9, c = -2

= ; = = ; = =
⇒ = ≠
Therefore the given pair of linear equations are parallel and not intersecting each other.
Hence the pair has no solution. = =
(ii) 2x + y = 5 ⇒ 2x + y – 5 = 0
3x + 2y = 8⇒ 3x +2y – 8 = 0 ⇒ =( ) ( )
=
( )( ) ( )( ) ( ) ( )
Here a = 2, b = 1, c = -5 and a = 3, b = 2, c = -8 ⇒ = =
= ; = ; = = ⇒ ≠ ⇒ = = ⇒ = ⇒ 6x = 24 ⇒x = 4
Therefore the pair of linear equations has unique solution = ⇒6y = -6 ⇒ y = -1
There fore x = 4 and y = -1
2. (i) For which values of a and b does the following pair of linear equations have an
infinite number of solutions?
2x + 3y = 7 ⇒ 2x + 3y – 7 = 0
(a - b) x + (a + b)y = 3a + b - 2 ⇒ x + (a + b)y – (3a + b – 2) = 0
For a pair of linear equations to have infinitely many solutions: = =
Here, a = 2, b = 3, c = -7 and a = (a - b), b = (a + b), c = -(3a +b – 2)
= = = ; = ; = =
( ) ( ) ( ) ( )
⇒ =( ) ( )
= = ⇒ = ⇒ 2(a + b) = 3(a – b )
( ) ( ) ( ) ( ) ( ) ( )
⇒ = = ⇒ = = ⇒ 2a + 2b = 3a – 3b
⇒ = ⇒x=2 ⇒a = 5b (1)
= ⇒( =(
= ⇒y = 1 ) )
⇒ 3(3a + b -2) = 7(a+b) ⇒9a + 3b – 6 = 7a + 7b
Therefore x = 2 and y = 1
⇒2a - 4b = 6 ⇒ a – 2b = 3 (2)
iii) 3x - 5y = 20 ⇒ 3x – 5y – 20 = 0
From (1) and (2)
6x - 10y = 40⇒6x – 10y – 40 = 0
Here, a = 3, b = -5, c = -20 and a = 6, b = -10, c = -40 (2)⇒ 5b – 2b = 3 ⇒ 3b = 3 ⇒ b = 1[∵a = 5b]
a = 5b ⇒ a = 5x1 ⇒ a = 5
= = ; = = ; = = ⇒ If a = 5 and b = 1 the pair of linear equations has infinite solutions.
⇒ = = ii) For which value of k will the following pair of linear equations have no solution?
3x + y = 1 ⇒ 3x + y – 1= 0
Therefore the given pair of linear equations are coincident and the pair has infinite solutions.
(2k - 1)x + (k - 1)y = 2k + 1 ⇒ (2k - 1)x + (k - 1)y - (2k + 1) = 0
(iv) x - 3y - 7 = 0
3x - 3y - 15 = 0 For a pair of linear equations to have no solutions: = ≠

P a g e 86 | 159 Available in ykoyyur.blogspot.com P a g e 87 | 159 Available in ykoyyur.blogspot.com


SSLC Mathematics Solutions – Part -1 YK SSLC Mathematics Solutions – Part -1 YK
E°è a = 3, b = 1, c = -1 ªÀÄvÀÄÛ a = (2k - 1), b = (k - 1), c = -(2k +1) Let the fixed charge = x and daily food charges = y. According to question
=( ; =( ; = =( x + 20y = 1000 (1)
) ) ( ) )
x + 26y = 1180 (2)
= ⇒( =
) ( ) Substracting equation (1) from (2)
⇒ 3(k – 1) = (2k – 1 ) ⇒ 3k – 3 = 2k – 1 ⇒k = 2 x + 26y = 1180 (2)
Therefore, if k = 2 then the pair of linear equations have no solution. x + 20y = 1000 (1)
3. Solve the following pair of linear equations by the substitution and cross-multiplication methods : 6y = 180
8x + 5y = 9 (1)
3x + 2y = 4 (2) ⇒ y = 30
Substitution Method: Substitute y = 30 in equation (1)

8x + 5y = 9 ⇒ 5y = 9 – 8x ⇒ y = x + 20 × 30 = 1000 ⇒ x = 1000 – 600 ⇒ x = 400
Equation (2) ⇒ 3x + 2y = 4 ⇒ 3x + 2

=4 Therefore the fixed charges = Rs 400 and daily food charges = Rs 30
– (ii) A fraction becomes when 1 is substracted from the numerator and it becomes
⇒ 3x + = 4 Multiplying by 5
15x + 18 – 16x = 20 when 8 is added to its denominator. Find the fraction.
⇒-x = 20 – 18 ⇒ - x = 2 ⇒ x = -2 Let the given fraction = According to questions,
Substitute x = -2 in (1)
= ⇒ 3x - y = 3 (1)
3(-2) + 2y = 4 ⇒ -6 + 2y = 4
⇒2y = 4 + 6 ⇒2y = 10 ⇒ y = 5 = ⇒ 4x - y = 8 (2)
Therefore the solution is: x = -2 ªÀÄvÀÄÛ y = 5
By substracting equation(1) from (2)
Cross multiplication Method: 4x - y = 8 (2)
8x + 5y = 9 ⇒ 8x + 5y – 9 = 0 (1)
3x - y = 3 (1)
3x + 2y = 4 ⇒ 3x + 2y – 4 = 0 (2)
Here, a = 8, b = 5, c = -9 and a = 3, b = 2, c = -4 x =5
Substituting x = 5 in equation (1)
15 - y = 3 ⇒ y = 12
Therefore the fraction =
iii) Yash scored 40 marks in a test, getting 3 marks for each right answer and losing 1
mark for each wrong answer. Had 4 marks been awarded for each correct answer and 2
marks been deducted for each incorrect answer, then Yash would have scored 50 marks.
= = How many questions were there in the test?
Let the number of right answer = x and the number of wrong answer = y
⇒ =( =
( )( ) ( )( ) ) ( ) ( ) ( ) According to question,
⇒ = = 3x - y = 40 (1) and
4x - 2y = 50 ⇒ 2x - y = 25 (2)
⇒ = =1⇒ = 1 ⇒ x = -2
By substracting equation(1) from (2)
= 1⇒ y = 5
2x - y = 25 (2)
Therefore the solution: x = -2 ªÀÄvÀÄÛ y = 5
3x - y = 40 (1)
4. Form the pair of linear equations in the following problems and find their solutions (if -x = -15
they exists) by any algebraic method. ⇒ x = 15
i) A part of monthly hostel charges is fixed and the remaining depends on the number Substituting x = 15 in equation (1)
of days one has taken food in the mess. When a student A takes food for 20 days she has 3(15) - y = 40 ⇒-y = 40 – 45 ⇒-y = -5 ⇒ y = 5
to pay Rs 1000 as hostel charges whereas a student B, who takes food for 26 days, pays Therefore the right answers = 15 ; Wrong answers = 5; Total questions = 20
Rs 1180 as hostel charges. Find the fixed charges and the cost of food per day.

P a g e 88 | 159 Available in ykoyyur.blogspot.com P a g e 89 | 159 Available in ykoyyur.blogspot.com


SSLC Mathematics Solutions – Part -1 YK SSLC Mathematics Solutions – Part -1 YK
iv) Places A and B are 100 km apart on a highway. One car starts from A and another (1) ⇒ 2p + 3q = 13 (3)
from B at the same time. If the cars travel in the same direction at different speeds, they (2) ⇒ 5p – 4q = -2 (4)
meet in 5 hours. If they travel towards each other, they meet in 1 hour. What are the
From (3) 2p + 3q = 13 ⇒ 3q = 13 – 2p ⇒ q =
speeds of the two cars?
Let the speed of car A = x km/h and speed of car B = y km/h Substitute q = in equation (4) we get,
The speed of both car travel in same direction = (x - y) km/h 5p – 4 = -2 ⇒ 5p – = -2 Multiplying by 3,
The speed of both car travel in opposite direction = (u + v) km/h ⇒ 15p– 52 + 8 = -6 ⇒ 23p = 46 ⇒ p = 2
According to question, Substitute p = 2 in (1) we get,
5(x - y) = 100 ⇒ x - y = 20 (1) 2(2) + 3q = 13 ⇒4 + 3q = 13 ⇒3q = 9 ⇒q = 3
1(x + y) = 100 ⇒ x + y = 100 (2) ⇒ substituting the value of p and q ,
By adding equation (1) and (2)
= p⇒ = 2⇒x=
x - y = 20 (2)
x + y = 100 (1) =q ⇒ =3 ⇒ y=
2x = 120 Example 18 : Solve the following pair of equations by reducing them to a pair of
⇒ x = 60 linear equations
Substituting x = 60 in equation (1) + =2
60 - y = 20 ⇒-y = -40 ⇒y = 40
Therefore the speed of car A and B = 60 km/h and 40 km/h + =1
v) The area of a rectangle gets reduced by 9 square units, if its length is reduced by 5 Solution: + = 2 ⇒5 + = 2
units and breadth is increased by 3 units. If we increase the length by 3 units and the
breadth by 2 units, the area increases by 67 square units. Find the dimensions of the - = 1 ⇒6 - 3 = 1
rectangle.
Let the area = xy According to question, Let = p; =q
(x - 5) (y + 3) = xy – 9 5p + q = 2 (1)
⇒xy + 3x – 5y – 15 = xy – 9 ⇒ 3x - 5y = 6 (1)
6p - 3q = 1 (2)
(x + 3) (y + 2) = xy + 67
From (1) ⇒ q = 2 – 5p Substituting in (2) we get
⇒xy + 2x +3y + 6 = xy + 67 ⇒ 2x + 3y = 61 (2) 6p - 3(2 – 5p) = 1
3x - 5y = 6 ⇒ 3x = 6 + 5y ⇒x = ⇒6p - 6 + 15p = 1 ⇒21p = 7 ⇒p =
Substituting the value of x in equation (1), we get Substitute p = in (1),
2 + 3y = 61 ⇒ + 3y = 61 Multiplying by 3,
5 + q = 2 ⇒q = 2 - ⇒ q =
⇒12 + 10y + 9y = 183 ⇒ 19y = 183 - 12 ⇒19y = 171 ⇒ y = 9
Substituting y = 9 in equation (1), = p⇒ = ⇒3=x–1⇒x=4
3x – 5(9) = 6 ⇒ 3x – 45 = 6 ⇒ 3x = 51 ⇒ x = 17 =q⇒ = ⇒ 3 = y– 2 ⇒ y= 5
Therefore length of the rectangle = 17 units and bredth = 15 units
Example 19:A boat goes 30 km upstream and 44 km downstream in10 hours. In 13
3.5 Equations Reducible to a Pair of Linear Equations in Two Variables
hours, it can go4 0 km u ps t r ea m a nd 5 5 km down-stream. Determine the speed
Example 17:Solve the pair of equations
of the stream and that of the boat in still water.
+ = 13; - = -2 Solution: Let speed of the boat in still water = x km/h and the speed of the stream = y km/h
Then the speed of the boat down stream = (x + y) km/h ªÀÄvÀÄÛ
Solution: + = 13 ⇒ 2 + 3 = 13 (1)
Speed of the boat in upstram = (x - y) km/h
- = -2⇒ 5 - 4 = -2 (2)
time =
Let = p and =q
In the first case, when the boat goes 30 km upstream, let the time taken, in hour be T 1

P a g e 90 | 159 Available in ykoyyur.blogspot.com P a g e 91 | 159 Available in ykoyyur.blogspot.com


SSLC Mathematics Solutions – Part -1 YK SSLC Mathematics Solutions – Part -1 YK
Then T1 = + = 2⇒ + = 2 ⇒ 3p + 2q - 12 = 0 (1)
Let T2 be the time, in hours, taken by the boat to go 44 km downstream + = ⇒ + = ⇒2p + 3q – 13 = 0 (2)
Then T2 = Here, a = 3 , b = 2, c = -12 and a = 2, b = 3, c = -13
The total time taken (T1 + T2 ) ⇒ + = 10 (1) = =
In the second case, in 13 hours it can go 40 km upstream and 55 km downstream we get the equation,
⇒( )( ) ( )( )
=( ) ( )
= ( ) ( )
+ = 13 (2)
⇒ = =
Let = p; = q,
⇒ = =
(1) ⇒ 30p + 44q = 10 ⇒ 30P + 44q – 10 = 0 (3)
⇒ = ⇒5p = 10 ⇒p = 2
(2) ⇒ 40p + 55q = 13⇒40p + 55q – 13 = 0 (4)
Here, a = 30 , b = 44, c = -10 and a = 40, b = 55, c = -13 = ⇒ 5q = 15 ⇒ q = 3
= p ⇒ = 2 ⇒ x = ªÀÄvÀÄÛ = q⇒ = 3 ⇒ y=
(ii) + = ; - = −
√ √

Let = p and = q

+ = 2 ⇒ 2p + 3q = 2 ⇒ 2p + 3q - 2 = 0 (1)

- = −1⇒ 4p – 9q = -1 ⇒ 4p – 9q + 1 = 0 (2)
= = √
Here, a = 2 , b = 3, c = -2 and a = 4, b = -9, c = 1
⇒ =( ) ( )
=
( )( ) ( )( ) ( ) ( )
= =
⇒ = =
⇒( =( =
⇒ = = )( ) ( )( ) ) ( ) ( ) ( )

⇒ = =
⇒ =
⇒ = =
p = ⇒p=
⇒ = ⇒ -30p = -15 ⇒ p =
=
= ⇒ -30q = -10 ⇒ q =
⇒q = ⇒q=
=p⇒ = ⇒ √ = 2 ⇒ x = 4ªÀÄvÀÄÛ
⇒ = p⇒ = ⇒ x– y= 5 √ √
= q⇒ = ⇒ = 3⇒ y=9
=q⇒ = ⇒ x + y = 11
Adding the equations we get, (iii) + 3y = 14; - 4y = 23
2x = 16 ⇒ x = 8 Let =p
8 – y = 5⇒ y = 3 4p + 3y = 14 (1) x 3
Therefore the speed of the boat = km/h and speed of the stream = 3km/h 3p – 4y = 23 (2) x 4
12p + 9y = 42 (3)
Exercise 3.6 12p - 16y = 92 (4)
1. Solve the following pairs of equations by reducing them to a pair of linear equations Substracting equation(3) from equation(4) we get,
(i) + = 2; + = -25y = 50 ⇒ y = -2
Substitute y = 2 in equation (1)
Let = p and = q 4p + 3(-2) = 14
⇒4p = 20⇒p = 5

P a g e 92 | 159 Available in ykoyyur.blogspot.com P a g e 93 | 159 Available in ykoyyur.blogspot.com


SSLC Mathematics Solutions – Part -1 YK SSLC Mathematics Solutions – Part -1 YK
=p⇒ =5⇒x= ⇒ + = 6; + =5

Therefore x = and y = -2 Let =p; =q


(iv) + = ; - = 6p + 3q = 6 (1)
2p + 4q = 5 (2)
Let =p; =q Multply equation (2) by 3,
5p + q = 2 (1) 6p + 12q = 15 (3)
6p - 3q = 1 (2) Substract (1) from (3),
(1)x3 = 15p + 3q = 6 (3) 6p + 12q = 15
Adding (2) and (3) we get, 6p + 3q = 6
6p - 3q = 1 9q = 9
15p + 3q = 6 ⇒ q=1
21p = 7 ⇒p = Substitute q= 1 in equation (2) we get,
Substituting p = in (1), 2p + 4(1) = 5⇒ 2p = 1
+ q = 2 ⇒q = 2 - = ⇒p =

=p⇒ = ⇒ 3 = x -1 ⇒ x = 4 =p⇒ = ⇒ y= 2

=q⇒ = ⇒ 3= y-2 ⇒ y = 5 =q⇒ =1⇒x=1


(vii) + = ; - = −
(v) = ; =
Let =p; =q
- = 5; + = 15
10p + 2q = 4 (1)
⇒ - = 5; + = 15 15p - 5q = -2 (2)
(1) ⇒ 5p + q = 2 (3)
Let = p ; = q, (3) ⇒ q = 2 – 5p (4)
7p – 2q = 5 (1) Substitute (4) in (2) we get,
8p + 7q = 15 (2) 15p - 5(2-5p) = -2
(1)⇒ 7p = 5 + 2q ⇒ p = ⇒15p – 10 + 25p = -2
Substituting in (2) we get, ⇒40p = 8
8
5 + 2q
+ 7q = 15 ⇒p= ⇒
7
+ 7q = 15 Multiplying by 7 , Substitute p = in (3) we get,
40 + 16q + 49q = 105 5 +q=2⇒1 +q=2
65q = 65 ⇒ q = 1 ⇒q = 1
Substituting q = 1 in equation (1) , =p⇒ =
7p – 2(1) = 5
⇒x+y=5 (5)
⇒7p = 7 ⇒ p = 1
= q⇒ =1
=p⇒ = 1⇒ y= 1
⇒x – y = 1 (6)
=q ⇒ =1⇒ x=1 By adding (5) and (6)
(vi) 6x + 3y = 6xy; 2x + 4y = 5xy x+ y=5
6x + 3y = 6xy; 2x + 4y = 5xy x– y=1
Divide both equations by xy we get 2x = 6
+ = ; + = ⇒ x= 3
Substituting x = 3 in equation (5), we get

P a g e 94 | 159 Available in ykoyyur.blogspot.com P a g e 95 | 159 Available in ykoyyur.blogspot.com


SSLC Mathematics Solutions – Part -1 YK SSLC Mathematics Solutions – Part -1 YK
3 + y = 5 ⇒ y = 5 -3 ⇒ y = 2 (ii) 2 women and 5 men can together finish an embroidery work in 4 days, while 3
Therefore the solutions are x = 3, y = 2 women and 6 men can finish it in 3 days. Find the time taken by 1 woman alone to finish
(viii) + = ; - = the work, and also that taken by 1 man alone.
( ) ( )
Let the time taken by women to finish embroidery work = x days and
Let =p; =q the time taken by men = y days
P+q= ⇒ 4p + 4q = 3 (1) Therefore the work finished by one women in a day = and
- = ⇒4p – 4q = -1 (2) the work finished by a man in day = . According to question,
Substract (2) from (1) + = ; + =
4p + 4q = 3
4p – 4q = -1 Let = p; =q
8q = 4 2p + 5q = ⇒ 8p + 20q = 1 (1)
⇒q = ⇒q = 3p + 6q =
Substitute q = in (1) we get, ⇒9p + 18q = 1 (2)

4p + 4 = 3 ⇒ 4p + 2 = 3 ⇒4p = 1 ⇒p = (1) ⇒ 8p = 1 – 20q ⇒ p =

=p⇒ = ⇒ 3x + y = 4 (3) Substituting p = in (2)
– –
=q⇒ = ⇒ 3x – y = 2 (4) 9 + 18q = 1 ⇒ + 18q = 1
By adding (3) and (4), we get ⇒9 – 180q + 144q = 8 Multiplying by 8
3x + y = 4 ⇒-36q = - 1 ⇒ q =
3x – y = 2 Substituting q = in (1),
6x =6
⇒ x=1 8p + 20 = 1 ⇒ 8p + = 1 ⇒ 8p + = 1
Substituting x = 1in equation (3), ⇒ 72p + 5 = 9 ⇒ 72p = 4 ⇒ p = ⇒ p=
3(1) + y = 4 ⇒ y = 4 – 3 ⇒ y = 1 = p⇒ = ⇒x = 18
Therefore the solutions are x = 1, y = 1
2. Formulate the following problems as a pair of equations, and hence find their solutions. =q ⇒ = ⇒y = 36
(i) Ritu can row downstream 20 km in 2 hours, and upstream 4 km in 2 hours. Find her Therefore the time taken by a women to finish the work = 18 days and a man = 36 days
speed of rowing in still water and the speed of the current. (iii) Roohi travels 300 km to her home partly by train and partly by bus. She takes 4
hours if she travels 60 km by train and the remaining by bus. If she travels 100 km by
Let the speed of Ritu in still water = x km/h the speed of the stream = y km/h
train and the remaining by bus, she takes 10 minutes longer. Find the speed of the train
The speed of Ritu in upstream = (x - y) km/h
and the bus separately.
The speed of Ritu in downstream = (x + y) km/h, According to question Let the speed of the train = x km/h and the speed of the bus y km/h
2(x + y) = 20
According to question,
⇒ x + y = 10 (1)
2(x - y) = 4 + = 4 (1)
⇒ x-y=2 (2) + = (2)
By adding equation (1) and (2) ,
x + y = 10 Let = p and = q
x-y= 2 60p + 240q = 4 ⇒15p + 60q = 1 (3)
2x = 12 100p + 200q = ⇒ 600p + 1200q = 25
⇒ x= 6 ⇒24p + 48q = 1 (4)
Substituting x = 6 in (1) (3) ⇒ 15p = 1 - 60q ⇒ P =
6 + y = 10 ⇒ y = 10 – 6 ⇒y = 4 Substitute P = in (4) ,
Therefore the speed of Ritu in still water = 6 km/h and the speed of the water = 4 km/h.

P a g e 96 | 159 Available in ykoyyur.blogspot.com P a g e 97 | 159 Available in ykoyyur.blogspot.com


SSLC Mathematics Solutions – Part -1 YK SSLC Mathematics Solutions – Part -1 YK
24 + 48q = 1
⇒ + 48q = 1 Multiplying by 15 , Circles
24 – 1440q + 720q = 15 ⇒ -720q = - 9 ⇒ q =
Substitute q = in (3),
15p + 60 =1
15p + = 1⇒ 15p = 1 - ⇒ 15p = ⇒p =
=p⇒ = ⇒ x = 60
= q⇒ = ⇒ y = 80
Therefore the speed of the train = 60 km/h and the speed of the bus = 80 km/h.

Non-intersecting Line: The line PQ and the circle have no common


Summery: point. In this case, PQ is called a non-intersecting line.PQ is non-
1. Two linear equations in the same two variables are called a pair of linear equations in two intersecting line for the circle of center A
variables. The most general form of a pair of linear equations is
a x + b y + c = 0 and a x + b y + c = 0
2. A pair of linear equations in two variables can be represented, and solved, by the: Secant: There are two common points M and N that the line PQ and
(i) graphical method (ii) algebraic method the circle have. In this case, we call the line PQ a secant of the circle
3. Graphical Method :
of center B
The graph of a pair of linear equations in two variables is represented by
two lines. Tangent: There is only one point O which is common to the line PQ
(i) If the lines intersect at a point, then that point gives the unique solution of the two equations. and the circle. In this case, the line is called a tangent to the circle of
In this case, the pair of equations is consistent. center C
(ii) If the lines coincide, then there are infinitely many solutions — each point
on the line being a solution. In this case, the pair of equations is dependent
(consistent). 4.2 Tangent to a Circle
(iii)If the lines are parallel, then the pair of equations has no solution. In this case, the pair of Tangent to a circle is a line that intersects the circle at only one point. There is only one
equations is inconsistent tangent to a circle at a point. The common point of the tangent and the circle is called the
4. Algebraic Methods : We have discussed the following methods for finding the solution(s) point of contact.
of a pair of linear equations :
(i) Substitution Method The tangent at any point of a circle is perpendicular to the radius
(ii) Elimination Method through the point of contact.
(iii) Cross-multiplication Method
5. If a pair of linear equations is given by a x + b y + c = 0 and a x + b y + c = 0, the the
following situations can arise :
≠ In this case, the pair of linear equations is consistent Given:A circle with centre O and tangent XY at a point P.
To Prove: OP⟘XY
= ≠ In this case, the pair of linear equations is inconsistent
Consruction: Take any point Q, other than P on the tangent XY
= = In this case, the pair of linear equations is dependent and consistent and join OQ
6. There are several situations which can be mathematically represented by two equations that Proof: Hence, Q is a point on the tangent XY, other than the point of contact P. So Q lies
are not linear to start with. But we alter them so that they are reduced to a pair of linear outside the circle..
equations [∵There is only one point of contact to a tangent]

P a g e 98 | 159 Available in ykoyyur.blogspot.com P a g e 99 | 159 Available in ykoyyur.blogspot.com


SSLC Mathematics Solutions – Part -1 YK SSLC Mathematics Solutions – Part -1 YK
Let OQ intersect the circle at R 4. Draw a circle and two lines parallel to a given line such that one is a tangent and the
∴ OP = OR [∵ Radius of the same circle ] other, a secant to the circle.
Now, OQ = OR + RQ AB – A line
⇒OQ > OR PQ – A secant
⇒OQ> OP [∵OP = OR] XY – A tangent
Therefore, OP is the shortest distance to the tangent from the center O
∴ OP⟘XY [∵ Perpendicular distance is always the shortest distance]
Remarks :
1. By theorem above, we can also conclude that at any point on a circle there
can be one and only one tangent. 4.3 Number of Tangents from a Point on a Circle
2. The line containing the radius through the point of contact is also sometimes called the Case 1 : There is no tangent to a circle passing through a point lying inside the circle.
‘normal’ to the circle at the point. Case 2 : There is one and only one tangent to a circle passing through a point lying on the circle.
Case 3 : There are exactly two tangents to a circle through a point lying outside the circle.
Exercise 4.1
1. How many tangents can a circle have? The lengths of tangents drawn from an external point to a
circle are equal.
Answer: Infinite
2. Fill in the blanks :
i) A tangent to a circle intersects it in point (s).
Answer: One
(ii) A line intersecting a circle in two points is called a . zÀvÀÛ: PQ and PR are the two tangents drawn from
Answer: Secant an external point P to a circle of center O. JoinOP,
iii) A circle can have parallel tangents at the most. OQ, OR
Answer: Two [Note: we can draw only two(pair) parallel tangents each other. But we can T Prove: PQ = PR
draw infinite parallel pair of tangents] Proof: In right angle triangle OQP and ORP,
iv) The common point of a tangent to a circle and the circle is called . OQ = OR [Radius of the same circle]
Answer: Point of cantact OP = OP [Common side]
3. A tangent PQ at a point P of a circle of radius 5 cm meets a line through the centre O at ∴ , ∆OQ P ≅∆ ORP [RHS]
a point Q so that OQ = 12 cm. Length PQ is : ∴ , PQ = PR [CPCT]
a) 12 cm b) 13 cm c) 8.5 cm d) √ cm Example 1 : Prove that in two concentric circles, the chord of the larger circle, which
Answer: touches the smaller circle, is bisected at the point of contact.
The line drawn from the point of contact to the center of the circle is perpendicular to the We are given two concentric circles C1 and C2 with
tangent. centre O and a chord AB of the larger circle C1 which
⇒OP ⊥ PQ touches the smaller circle C2 at the point P (see Fig. 4.8).
In ΔOPQ,
We need to prove that AP = BP.
OQ2 = OP2 + PQ2[Pythagoras Theorem]
⇒ (12)2 = 52 + PQ2 Let us join OP. Then, AB is a tangent to C2 at Pand OP
is its radius. Therefore, by Theorem 4.1,
⇒PQ2 = 144 - 25
Therefore OP⟘AB [From theorem 4.1]
⇒PQ2 = 119 Now AB is a chord of the circle C1 and OP⟘AB
⇒PQ = √119 cm
Therefore, OP is the bisector of the chord AB, as the perpendicular from the centre bisects
(d) √ cm the chord,
⇒ AP = BP

P a g e 100 | 159 Available in ykoyyur.blogspot.com P a g e 101 | 159 Available in ykoyyur.blogspot.com


SSLC Mathematics Solutions – Part -1 YK SSLC Mathematics Solutions – Part -1 YK
Example 2 : Two tangents TP and TQ are drawn to a circle with centre O from an 2. In Fig. 4.11, if TP and TQ are the two tangents to a circle with centre O so that POQ =
external point T. Prove that PTQ = 2 OPQ. 110°, then PTQ is equal to
Solution:TP and TQ are the two tangents drawn from TP and TQ are the tangents to a circle at P and Q OP and
OQ are radius of the circle at point of contacts P and Q
an external point T to the circle with centre O
∴ OP ⊥ TP and OQ ⊥ TQ
To Prove: ∠PTQ = 2∠OPQ
∠OPT = ∠OQT = 90°
Let ∠PTQ = θ (1) In Quadrilateral POQT,
TP = TQ [ ∵Theorem 4.2] ∠PTQ + ∠OPT + ∠POQ + ∠OQT = 360°
Therefore TPQ is an isosceles triangle. ⇒ ∠PTQ + 90° + 110° + 90° = 360°
∠TPQ = ∠TQP = [180 − θ] ⇒ ∠PTQ = 70°
⇒Ans (B) 70°.
⇒ ∠TPQ = ∠TQP = 900 - θ (2) A) 60 B) 70 C) 80 D) 90
∠OPT = 900 (3) 3. If tangents PA and PB from a point P to a circle with centre O are inclined to each other
∠OPQ = ∠OPT - ∠TPQ
at angle of 80°, then POA is equal to
⇒ ∠OPQ = 900 - 90 − θ [∵ from(2)and (3)À] OA and OB are the radius drawn at the point of contact of the tangents BP and BQ
⇒ ∠OPQ = θ ∴ OA ⊥ PA and OB ⊥ PB
⇒∠PTQ = 2∠OPQ [∵From (1)] ∠OBP = ∠OAP = 90°
Example 3 : PQ is a chord of length 8 cm of a circle of radius 5 cm. The tangents at P In Quadrilateral AOBP,
and Q intersect at a point T (see Fig. 4.10). Find the length TP. ∠AOB + ∠OBP + ∠OAP + ∠APB = 360°
Solution: JoinOT Let it intersect PQ at the point R
⇒ ∠AOB + 90° + 90° + 80° = 360°
Then ∆TPQ is isosceles and TO is the a ngle bisect
or of ∠P T Q . ⇒ ∠AOB = 100°
∴ OT⟘PQ therefore OT bisects PQ Now, In ΔOPB and ΔOPA,
⇒ PR = RQ = 4 cm. AP = BP [Tangents drawn from an external point]
∴ RO = √5 − 4 ⇒RO = √25 − 16 ⇒ RO = √9 OA = OB [Radius of the same circle]
⇒ RO = 3cm OP = OP [Common]
∠ OPR + ∠ TPR = 90 0 (1) [∵In ∆PRO , ∠ PRO = 900 ] ∴ ΔOPB ≅ ΔOPA [SSS congruence rule]
∠ PTR + ∠ TPR = 90 0
(2) [∵In ∆PTR , ∠ PRT = 900 ] ⇒∠POB = ∠POA
From (1) and (2), ∠AOB = ∠POB + ∠POA
∠ OPR = ∠ PTR (3) ⇒ 2 ∠POA = ∠AOB
∴ ∆PRO and ∆PTR right triangles are similar [AA similarity criteria]
⇒ ∠POA = 50°
⇒ = ⇒ = ⇒PT = =
⇒ (A) 50°
A) 500 B) 600 C) 700 D) 800
4. Prove that the tangents drawn at the ends of a diameter of a circle are parallel.
EXERCISE 4.2 AB is a diameter. PQ and RS are the tangents drawn
In Q.1 to 3, choose the correct option and give justification.
to the circle at point A and B
1. From a point Q, the length of the tangent to a circle is 24 cm and the distance of Q from
OA and OB are the radius drawn at point of contact.
the centre is 25 cm. The radius of the circle is
∴ OA ⊥ PQ and OB ⊥ RS
OP ⊥ PQ and ΔOPQ is a right angle triangle.
∠OAP = ∠OAQ = ∠OBR = ∠OBS = 90º
OQ = 25 cm and PQ = 24 cm
In the fig,
In ΔOPQ, By Pythagoras theorem,
∠OBR = ∠OAQ [Alternate angles ]
⇒ (25)2 = OP2 + (24)2
∠OBS = ∠OAP [Alternate angles ]
⇒OP2 = 625 - 576⇒OP2 = 49 ⇒OP = 7 cm
GvÀÛgÀ: (A) 7 cm. ⇒ PQǁ RS
5. Prove that the perpendicular at the point of contact to the tangent to a circle passes
A) 7 cm B) 12 cm C) 15 cm D) 24.5 cm
through the centre.

P a g e 102 | 159 Available in ykoyyur.blogspot.com P a g e 103 | 159 Available in ykoyyur.blogspot.com


SSLC Mathematics Solutions – Part -1 YK SSLC Mathematics Solutions – Part -1 YK
AB is the tangent drawn to the circle with radius O 9. In Fig. 4.13, XY and X1Y1 are two parallel tangents to a circle with centre O and
another tangent AB with point of contact C intersecting XY at A and X1Y1at B. Prove
To Prove:The perpendicular at P passes through the center O.
that ∠AOB = 90°.
If possible,let the perpendicular passing through some other Let the tangent AB touches the circle at C. Join OC.
Point say Q In ΔOPA and ΔOCA,
Join QP and OP OP = OC [radius of the same circle]
OP is the radius at point of contact AB is the tangent AP = AC [The tangents from an external point A]
∴ OP⟘AB ⇒ ∠OPA = 90 0 AO = AO [Common]
But, ∠RPA = 90° (PQ ⊥ AB) ∴ ΔOPA ≅ ΔOCA [SSS congruence rule]
⇒ Which is possible only when points P and Q coinside. ⇒ ∠POA = ∠COA (1)
∴ the perpendicular at the point of contact to the tangent to a circle passes through the centre. Similarlly,
6. The length of a tangent from a point A at distance 5 cm from the centre of the circle is 4 ΔOQB ≅ ΔOCB
cm. Find the radius of the circle ∠QOB = ∠COB (2)
AB is the tangent to the circle at point B. OB is the radius at point of contact POQ is the diameter ∴ ∠POQ = 180 0
∴ OB ⊥ AB
⇒∠POA + ∠COA + ∠COB + ∠QOB = 180 º
OA = 5cm and AB = 4 cm [Given]
In ΔABO, From (1) and (2) we get,
OA2 = AB2 + BO2 [Pythagoras theorem] 2∠COA + 2∠COB = 180º
⇒ 52 = 4 2 + BO2 ⇒ BO2 = 25 - 16 ⇒ ∠COA + ∠COB = 90º ⇒ ∠AOB = 90°
⇒ BO2 = 9 ⇒ BO = 3
∴ Radius = 3 cm.
10. Prove that the angle between the two tangents drawn from an external point to a circle
is supplementary to the angle subtended by the line-segment joining the points of
7. Two concentric circles are of radii 5 cm and 3 cm. Find the length of the chord of the contact at the centre
larger circle which touches the smaller circle. Let PA and PB are the tangents drawn from an external point to the circle with center O
Two concentric circles of radius 5cm and 3cm drawn with Join OA and OB
common center O To Prove: ∠APB + ∠BOA = 180º
AB is the chord of circle with radius 5cm such that it touches
the circle of radius 3cm at P Proof: OA ⊥ PA [Radius at point of contact to the circle]
∴ AB is the tangent to the smaller circle at P ∴ ∠OAP = 90°
⇒ OP ⊥ AB
Similarlly,
∴ AP = PB [The perpendicular drawn from the center to the chord bisects the chord]
OA2 = AP2 + OP2 [By Pythagoras theorem] OB ⊥ PB ∴ ∠OBP = 90°
⇒ 52 = AP2 + 3 2 ⇒ AP2 = 25 – 9 ⇒ AP = 4, In quadrilateral OAPB,
AB = 2AP = 2 × 4 = 8 cm ∠OAP +∠APB +∠PBO +∠BOA = 360º[Sum of interior agles]
∴ The length of the chord = 8 cm. ⇒ 90º + ∠APB + 90º + ∠BOA = 360º
8. A quadrilateral ABCD is drawn to circumscribe a circle (see Fig. 4.12). Prove that
⇒ ∠APB + ∠BOA = 180º
AB + CD = AD + BC
From the figure, 11. Prove that the parallelogram circumscribing a circle is a rhombus.
Given: ABCD is a parallelogram circumscribing a circle.
DR = DS [Tangents from the external point D ] (1) To prove: AB = BC = CD = DA
AP = AS [Tangents from the external point A ] (2) Proof: ABCD MAzÀÄ ¸ÀªÀiÁAvÀgÀ ZÀvÄÀ ¨sÄÀ ðd.
BP = BQ [Tangents from the external point B ] (3)
∴ AB = CD (1)
CR = CQ [Tangents from the external point C ] (4)
∴ BC = AD (2)
(1) + (2) + (3) + (4)
We know that the tangent drawn from an external point
DR + AP + BP + CR = DS + AS + BQ + CQ
to the circles are equal
⇒ (BP + AP) + (DR + CR) = (DS + AS) + (CQ + BQ)
Therefore, DR = DS, AP = AS, BP = BQ, and CR = CQ
⇒AB + CD = AD + BC

P a g e 104 | 159 Available in ykoyyur.blogspot.com P a g e 105 | 159 Available in ykoyyur.blogspot.com


SSLC Mathematics Solutions – Part -1 YK SSLC Mathematics Solutions – Part -1 YK
Adding all these, we get Given: ABCD is a quadrilateral circumscribing a circle
DR + CR + BP + AP = DS + CQ + BQ + AS with center O. Let the circle touches the quadrilateral at
⇒ (BP + AP) + (DR + CR) = (DS + AS) + (CQ + BQ) points P,Q,R and S
⇒ AB + CD = AD + BC (3) To Prove:∠AOB + ∠COD = 1800 and ∠AOD + ∠BOC = 1800
Substituting (1) and (2) in (3), Construction: Join OP, OQ, OR and OS.
2AB = 2BC ⇒ AB = BC (4) Proof: The tangents drawn from an external point to the
From equation (1), (2) and (4), circle substend equal angle at the center.
AB = BC = CD = DA ⇒∠1 = ∠2; ∠3 = ∠4; ∠5 = ∠6; ∠7 = ∠8
∴ ABCD is a Rhombus But,
12. A triangle ABC is drawn to circumscribe a circle of radius 4 cm such that the segments ∠1 + ∠2 + ∠3 + ∠4 + ∠5 + ∠6 + ∠7 +∠8 = 360º
BD and DC into which BC is divided by the point of con tact D ar e of len gth s 8 cm (∠1 + ∠2) + (∠3 + ∠4) + (∠5 + ∠6) + (∠7 +∠8) = 360º
and 6 cm respectively (see Fig. 4.14). Find the sides AB and AC. ⇒ 2(∠2 + ∠3) + 2(∠6 + ∠7) = 360º
In ΔABC, ⇒ (∠2 + ∠3) + (∠6 + ∠7) = 180º
CF = CD = 6cm [Tangents from an external point] ⇒ ∠AOB + ∠COD = 1800
BE = BD = 8cm [Tangents from an external point] Similarlly, ∠AOD + ∠BOC = 1800
AE = AF = x [Tangents from an external point]
∴ Opposite sides of a quadrilateral circumscribing a circle subtend supplementary
⇒a = AB = AE + EB = x + 8
angles at the centre of the circle.
b = BC = BD + DC = 8 + 6 = 14
Summary
c= CA = CF + FA = 6 + x
1. The tangent to a circle is perpendicular to the radius through the point of contact.
S= = = ⇒ S = 14 + x 2. The length of the tangents from an external point to the circle are equal.
Area of ΔABC = ( − )( − )( − )
= 14 + [14 + − ( + 8)](14 + − 14)[14 + − (6 + )]
= (14 + ) [14 + − − 8](14 + − 14)[14 + − 6 − ]
= (14 + ) (6)( )(8)
= (14 + ) 48 cm2 (1)
Similarlly, Area of ΔABC= Area of ΔOCB+ Area of ΔOBA+ Area of ΔOAC
= BC.OD + AB.OE + AC.OF
= (14x4) + (8+x)4 + (6+x)4 = 28 + 16 + 2 + 12 + 2
= (56 + 4 )cm2 (2)
From (1) and (2),
(14 + ) 48 = 56 + 4
48x (14 + x) = (56 + 4x)2 [ Squaring on both sides]
[ ( )]
⇒ 48x =
⇒ 48x = 16 (14 + x) ⇒ 48x = 224 + 16x ⇒ 32x = 224 ⇒ x = 7 cm
Therefore, AB = x + 8 = 7 + 8 = 15 cm
CA = 6 + x = 6 + 7 = 13 cm
13. Prove that opposite sides of a quadrilateral
circumscribing a circle subtend supplementary angles
at the centre of the circle.

P a g e 106 | 159 Available in ykoyyur.blogspot.com P a g e 107 | 159 Available in ykoyyur.blogspot.com


SSLC Mathematics Solutions – Part -1 YK SSLC Mathematics Solutions – Part -1 YK
Let the required radius = R. Therefore the area = πR2
Area Related to The area of the circle of radius 8 cm = π × 82 = 64π cm2
The area of the circle of radius 6 cm = π × 62 = 36π cm2

circles The sum of the areas of two circles = 64π cm2 + 36π cm2 = 100π cm2
According to question,
πR2 = 100π cm2⇒ R2 = 100cm2 ⇒ R2 = 100 cm2 ⇒ R = 10 cm
5.2 Perimeter and Area of a Circle — A Review 3. Fig. 5.3 depicts an archery target marked with its five scoring regions from the centre
The distance covered by travelling once around a circle is its perimeter, usually called its outwards as Gold, Red, Blue, Black and White. The diameter of the region representing
circumference. You also know from your earlier classes, that circumference of a circle bears Gold score is 21 cm and each of the other bands is 10.5 cm wide.Find the area of each of
the five scoring regions.
a constant ratio with its diameter. This constant ratio is denoted by the Greek letter (read
as ‘pi’). In other words, The diameter of the Golden colour Circle = 21 cm
1st circle 2 nd circle 3rd circle 4th circle 5th circle
r1 = 10.5 cm r2 = 21 cm r3 = 31.5 r4 = 42 r5 = 52.5
A1 = π r 1 2 A2 = π r22 A3 = π r 3 2 A4 = π r 4 2 A5 = π r 5 2
π (10.5)2 π(21)2 π(31.5)2 π(42)2 π(52.5)2
346.5 cm2 1386 cm2 3118.5 cm2 5544 cm2 8662.5 cm2
Area of Golden colour = π r12 = π (10.5)2 = 346.5 cm2
Area of Red colour = [Area of 2 nd – Area of 1 st ]
= 1386 - 346.5 cm2 = 1039.5 cm2
The cost of fencing a circular field at the rate of R s 24 per metre is Rs 5280. The field
Area of blue colour = [Area of 3rd – Area of 2 nd ]
is to be ploughed at the rate of R s 0.50 per m2. Find the cost of ploughing the field. (Take
= 3118.5 - 1386 cm2 = 1732.5 cm2
= )
Area of black colour = [Area of 4th – Area of 3 rd ]
Solution:Length of the fence (in metres) = = = 220 «ÄÃ. = 5544 - 3118.5 cm2 = 2425.5 cm2
So, circumference of the field = 220 m. Therefore, if r metres is the radius of the field, Area of white colour = [Area of 5th – Area of 4th]
= 8662.5 cm2 - 5544 cm2 = 3118.5cm2
then, 2 r = 220 Or 2 x x r = 220 ⇒ r = = 35m 4. The wheels of a car are of diameter 80 cm each. How many complete revolutions does
Therefore, area of the field = πr = x 352 = (22 x 5 x 35)m2 each wheel make in 10 minutes when the car is travelling at a speed of 66 km per hour?
Total cost of ploughing the field at the rate of 0.50/sqm = (22 x 5 x 35) x 0.5 = gÀÆ 1925 The diameter of the wheels of a car = 80 cm
Circumference of the wheel C = 2πr = 2r × π = 80 π cm
The Car traveled a distance in 10 minutes = (66 × 1000 × 100 × 10)/60 = 110000 cm
Exercise 5.1 ಚ ದ ದೂರ
Therefore no of revolution = = = = 4375
[Unless stated otherwise, use = ]
5. Tick the correct answer in the following and justify your choice : If the perimeter and the
1. The radii of two circles are 19 cm and 9 cm respectively.Find the radius of the circle area of a circle are numerically equal, then the radius of the circle is
which has circumference equal to the sum of the circumferences of the two circles. A) 2 Units B) Units C) 4 Units D) 7 Units
Let the required radius = R. Therefore the circumference C = 2πR Radius of the circle = r
The circumference of the circle of radius 19 cm = 2π × 19 = 38π cm ∴ Circumference(Perimeter) = 2πr ∴ Area = π r2
The circumference of the circle of radius = 2π × 9 = 18π cm According to question,
The sum of the circumference of two circles = 38π + 18π = 56π cm Perimeter = Area
⇒ 2πR = 56π cm [According to question] ⇒ 2R = 56 cm ⇒ R = 28 cm 2πr = π r2⇒ 2 = r
2. The radii of two circles are 8 cm and 6 cm respectively. Find the radius of the circle ∴ A) 2 Units
having area equal to the sum of the areas of the two circles.

P a g e 108 | 159 Available in ykoyyur.blogspot.com P a g e 109 | 159 Available in ykoyyur.blogspot.com


SSLC Mathematics Solutions – Part -1 YK SSLC Mathematics Solutions – Part -1 YK
5.3 Areas of Sector and Segment of a Circle Example 3 : Find the area of the segment AYB shown in Fig. 5.9, if radius of the
The portion (or part) of the circular region enclosed by two radii and the corresponding arc is circle is 21 cm and ∠AOB = 120 0 ( = )
called a sector of the circle and the portion (or part) of the circular region enclosed between Solution: Area of the segment
a chord and the corresponding arc is called a segment of the circle. =Area of sector OAYB – Area of ∆OAB ------- (1)
θ
Area of the sector = x
120
= x x 21 x 21 = 462cm2 (2)
To find the area of ∆OAB, draw OM⟘AB as shown in fig.5.10
Note that OA = OB
Therefore, by RHS congruence ∆AMO ≅ ∆BMO
So, M is the mid-point of AB and ∠AOM = ∠BOM = 600
In ∆OAM, = Cos600 ⇒ = ⇒ OM = cm
√ √
In ∆OAM, = Sin60 ⇒0
= ⇒ AM = cm
some relations (or formulae) to calculate their areas. ⇒AB = 2AM ⇒ 21√3cm
Let OAPB be a sector of a circle with centre O and radius r √
(see Fig. 5.6). Let the degree measure of ∠AOB be θ , ∴ Area of ∆OAB = x AB x OM = x 21√3 x = cm2 (3)
If the angle at the center is 3600, then the area of the sector = √ √
Are of the segment = 462 - = = (88 - 21√3)cm2
So, when the degr ee measure of the a ngle at t he
Centre is 1, area of the sector = Exercise 5.2
Therefore, when the degree measure of the angle at the centre is θ, [Unless stated, otherwise use = ]
θ
Area of the sector = xθ⇒ x 1. Find the area of a sector of a circle with radius 6 cm if angle
Area of the sector of angle θ = x of the sector is 60°.
Area of the sector of angle θ = ×π r2
Length of the arc of a sector of angle θ = x °
Area of the sector of angle 600 = °
×π r2 cm2
Areaof SegmentAPB Area of OAPB Sector – Area of ∆OAB = ×6x6 x = cm2
2. Find the area of a quadrant of a circle whose circumference is 22 cm
Area of OAQB Major sector r2 - Area of Minor sectorOAPB Quadrant of a circle = Angle of sector 900
Circumference C = 2πr = 22 cm
Area of AQB Major segment r2 – Area of APB Minor segment Radius r = cm = = cm
Area of the sector of angle θ = °
×π r2
Example 2 : Find the area of the sector of a circle with radius 4 cm and of angle 30°.
Also, find the area of the corresponding major sector (Use = 3.14) Area of the sector of angle 90° = ×π r2 = × x x = cm2
°
Solution: Given sector is OAPB. 3. The length of the minute hand of a clock is 14 cm.
Area of the sector OAPB = x Find the area swept by the minute hand in 5 minutes.
. The minute hand is the radius of the circle. ⇒ Radius(r) = 14 cm
⇒ x 3.14 x 4 x 4 = ≈ 4.19 cm2
The angle of rotation formed by minute hand in 1 hour = 360°
Area of the corresponding major sector °
∴ The angle of rotation in 5 minutes = × 5 = 30°
= r2 - Area of sector OAPB = (3.14 x 16 - 4.19) cm2≈ 46.1 cm2
Area of the sector of angle θ = °
×π r2
Alternate Method:
360−θ ∴ Area of the sector of angle 30 = 0
°
× x 14 x 14= × x 14 x 14
Area of the corresponding major sector = x
360−30 = ×22 x 7 = cm2
= x 3.14 x 4 x 4 = 46.05 ≈ 46.1 cm 2

P a g e 110 | 159 Available in ykoyyur.blogspot.com P a g e 111 | 159 Available in ykoyyur.blogspot.com


SSLC Mathematics Solutions – Part -1 YK SSLC Mathematics Solutions – Part -1 YK
4. A chord of a circle of radius 10 cm subtends a right angle at the centre. Find the area of = π r2 - 20.4 cm2 = 3.14 x 15x15 - 20.4 = 3.14 x 225 – 20.4
the corresponding : (i) minor segment (ii) major sector. (Use = 3.14) = 706.5 – 20.4 = 686.1cm2
Radius of the circle = 10 cm 7. A chord of a circle of radius 12 cm subtends an angle of 120° at the centre. Find the
The angle of the Major sector = 360° - 90° = 270° area of the corresponding segment of the circle. (Use = 3.14 and √ = 1.73).
Area of the major sector = ×π r2 cm2 Radius of the circle(r) = 12 cm
° Draw AB ⟘ OD ⇒ OD bisects AB
= × 3.14 x10 x 10 = 75 x 3.14 cm2 = 235.5 cm2 ⇒∠A = 180° - (90° + 60°) = 30°
In right angle ΔAOB , OA = 10 cm, OB = 10 cm cos 30° = ⇒

= ⇒ AD = 6√3 cm
Area of ΔAOB = × OA × OB= ×10 × 10 = 50 cm2 --- (1)
⇒ AB = 2 × AD = 12√3 cm
The angle of the Minor sector = 90°
sin 30° = ⇒ = ⇒ OD = 6 cm
Area of the minor sector = °
×π r2 cm2
The area of ΔAOB = × AB × OD
= × 3.14 x10 x 10 = 25 x 3.14 cm2 = 25 × 3.14 cm2 = 78.5 cm2 -------- (2)
Area of minor segment = (2) - (1) = 78.5 cm2 - 50 cm2 = 28.5 cm2 = x 12√3 x 6 cm2 = 36√3 cm = 36 × 1.73 = 62.28 cm2

5. In a circle of radius 21 cm, an arc subtends an angle of 60° at the centre. Find: The angle of minor sector = 120°
°
(i) the length of the arc (ii) area of the sector formed by the arc ∴ Area of the minor sector = ×π r2 cm2
°
(iii) area of the segment formed by the corresponding chord °
= x 3.14 x 12 x 12 cm2 = x 3.14 x 12 x 12 cm2
°
Radius of the circle = 21 cm = 3.14 x 4 x 12 cm2 = 3.14 x 48 cm2 = 150.72 cm2
(i) The length of the Arc AB = × 2πr ∴ Area of the minor segment = Area of the minor sector – Area of ΔAOB
°
Arc AB = ×2x x 21 = × 2 x 22 x 3 = 22cm = 150.72 cm2 - 62.28 cm2 = 88.44 cm2
°
(ii) The angle formed by arc AB = 60° 8. A horse is tied to a peg at one corner of a square shaped grass field of side 15 m by
Area of the sector of angle 60 0 = ° ×π r2 cm2 means of a 5 m long rope (see Fig. 5.11). Find
(i) The area of that part of the field in which the horse can graze
= °
x x 21x21 cm2= x 22x 3 x21 cm2= x 22x21 cm2
(ii) The increase in the grazing area if the rope were10 m long instead of 5 m. (Use = 3.14)
= 11x21 cm2 = 231 cm2 Given, the side of the square = 15 m
√ √ √
(iii) The area of the equilateral ΔAOB = x (OA)2 = x (21)2 = cm2 The length of the rope [Radius of the arc(r)] = 5 m
Hence the required area = Area of the sector formed by the Arc - area of ΔAOB The radius of the field in which the horse can graze = 5 m.
√ (i) Area of the field graze by the horse
= − cm2 [Horse is tied at the corner of the square.So, it graze only
6. A chord of a circle of radius 15 cm subtends an angle of 60° at the centre. Find the areas quadrant of the circle of radius 5m]
of the corresponding minor and major segments of the circle. (Use = √ = 1.73) = =
.
=
.
= 19.625 m2
Radius of the circle = 15 cm (ii) The length of the rope is 10m then, the area graze
In triangle ΔAOB, ∠AOB and ∠A = ∠B = 600 [∵OA = OB = 15cm] .
by the horse = = = = 78.5 m2
∴ ΔAOB is an equilateral triangle.
Therefore increase in grazing area
√ √ √
The area of ΔAOB = x (OA)2 = x (15)2 = cm2 = 78.5 m2 - 19.625 m2 = 58.875 m2
=
. 2
cm = 97.3 cm 2 9. A brooch is made with silver wire in the form of a circle with diameter 35 mm. The wire
is also used in making 5 diameters which divide the circle into 10 equal sectors as shown
The angle formed by the arc AB = 60° in Fig. 5.12. Find :
∴ The area of the sector formed by the arc AB = °
×π r2 cm2 (i) the total length of the silver wire required
= ° x (3.14)x 15x15 cm = x 3.14 x 5x15 cm = 1.57 x 75 cm2 = 117.75 cm2
2 2 (ii) the area of each sector of the brooch.
Area of the minor segment = Area of the sector formed by the arc AB – Area of ΔAOB Number of diameters = 5; Legnth of the diameter = 35 mm
∴ Radius (r) = 35/2 mm
= 117.75 - 97.3 = 20.4 cm2
(i) The total lenth of wire required
Area of the major segment = Area of the circle – Area of minor segment

P a g e 112 | 159 Available in ykoyyur.blogspot.com P a g e 113 | 159 Available in ykoyyur.blogspot.com


SSLC Mathematics Solutions – Part -1 YK SSLC Mathematics Solutions – Part -1 YK
= Perimeter of the brooch + length of 5 diameter 13. A round table cover has six equal designs as shown in Fig. 12.14. If the radius of the cover
= 2π r + (5×35) mm is 28 cm, find the cost of making the designs at th e rate of Rs 0.35 per cm2 0.35 per
= (2 × × ) + 175 mm = 110 + 175 mm = 285 mm cm2 (Use √ = 1.7).
(ii) Number of sectors = 10 The number of equal designs = 6; The radius of the cover = 28 cm
Making Cost of the design = gÀÆ 0.35 / cm2
Therefore area of each sector =
°
The angle of each sector = = 60°
= = = = mm2 ΔAOB AiÀÄ°è OA = OB [Radius of the same circle]
10. An umbrella has 8 ribs which are equally spaced (see Fig. 5.13). Assuming umbrella to ∴ ∠A = ∠B = 600
be a flat circle of radius 45 cm, find the area between the two consecutive ribs of the √
umbrella. ∴Area of the equilateral ΔAOB = x (OA)2

= x (28)2 = 1.7x 7 x 28 = 333.2 cm2
°
Area of the sector OACB = °
×π r2 cm2
= × x 28 cm = ×22x 4x28 cm2
2 2

= ×22x 2x28 cm2 = 410.67cm2


Area of the design = Area of the sector OACB - Area of the ΔAOB
Total ribs in the umbrella = 8
= 410.67 cm2 - 333.2 cm2 = 77.47 cm2
The radius of the umbrella when it to be flat = 45 cm
∴ The total area of 6 designs = 6 × 77.47 cm2 = 464.82 cm2
The area between the two consecutive ribs = ∴ Total cost of making designs = 464.76 cm2 × gÀÆ 0.35 /cm2 = Rs 162.68
= = = = cm2 = 795.5 cm2 14. Tick the correct answer in the following :
Area of a sector of angle p (in degrees) of a circle with radius R is
11. A car has two wipers which do not overlap. Each wiper has a blade of length 25 cm A) x 2πr B) x 2πr C) x 2πR D) x 2πR
sweeping through an angle of 115°. Find the total area cleaned at each sweep of the blades. °
The area of the sector of angle p = °
×π R 2 cm2 = °
×π R 2 x = x 2πR
The angle of the sector formed by the wiper = 115°
Answer (D) x 2
Radius of the sector = length of the wiper = 25 cm
°
Area of the sector formed by the wiper = ×π r2 cm2
°
=
°
× x 25 x 25 cm2
5.4 Areas of Combinations of Plane Figures
°
In Fig. 5.15, two circular flower beds have been shown on two sides of a square lawn
= × x 625 cm2 = × x 625 cm2 = cm2 ABCD of side 56 m. If the centre of each circular flower bed is the point of intersection
The total area coveed by blades of two wipers O of the diagonals of the square lawn, find the sum of the areas of the lawn and the
=2× cm2 = = 1254.96 cm2 flower beds.
Solution: Area of the square lawn = 56 x 56 m2 (1)
12. To warn ships for underwater rocks, a lighthouse spreads
Let The radius = OA = OB = x m
a red coloured light over a sector of angle 80° to a distance
Therefore, x2 + x2 = 562 [By Pythagoras theorem OA2 + OB2 = AB2]
of 16.5 km. Find the area of the sea over which the ships are
warned. (Use = 3.14) ⇒2x2 = 56 x 56
⇒x2 = 56 x 28 (2)
Let the lighthouse be at O
Now, Area of the sector OAB
Radius of the sector = length of the beam r = 16.5 km
Angle of the sector formed by the beam = 80° = x πx = x x 28 x 56 [ From eqn (2)] (3)
° Total Area = [Area of sector OAB + Area of sector ODC + Area of ∆OAD + Area of ∆OBC]
The area of the sector which light spreads = Area of the sector = °
×π r2 km2
= ×3.14 x 16.5x16.5km2 = ×3.14 x 272.25km2 = 189.97 km2 = x x28x56+ x x28x56 + x56x56+ x56x56
= 22x56+ 22x56 + 14x56+14x56
= 56(22+ 22 + 14+14) = 56(22+ 22 + 14+14) =56 x72 = 4032m2

P a g e 114 | 159 Available in ykoyyur.blogspot.com P a g e 115 | 159 Available in ykoyyur.blogspot.com


SSLC Mathematics Solutions – Part -1 YK SSLC Mathematics Solutions – Part -1 YK
Example 5: Find the area of the shaded region in Fig. 5.16, where ABCD is a square 2. Find the area of the shaded region in Fig. 5.20, if radii of the two concentric circles with
of side 14 cm centre O are 7 cm and 14 cm respectively and AOC = 40°.
Solution : Area of square ABCD = 14 x 14 cm2 = 196 cm Radius of the inner circle = 7 cm
Radius of the outer circle = 14 cm
Diaameter of each circle = = 7cm
The angle of the sector = 40°
So, radius of the circle = cm °
Area of the sector OAC = ° ×π r2 cm2
So, area of each circle = r2 = x x = cm2
= x x 142 cm2 = x 22 x 2x14 cm2 = cm2
Therefore area of four circles = 4 x = 154cm2 °
Area OBD = Area of the sector = °
×π r2 cm2
Therefore area of shaded region = (196 - 154)= 42cm2
GzÁºÀgÀuÉ 6: ABCD AiÀÄÄ 10 cm = x x 72 cm2 = x 22 x 7 cm2= cm2
¨ÁºÀĪÀÅ ¼Àî ZËPÀªÁVzÉ ªÀÄvÀÄÛ ¥Àæw ZËPÀzÀ ∴ Area of shaded region
¨ÁºÀĪÀÅ ªÁå¸ÀªÁVgÀĪÀAvÉ CzsÀðªÀÈvÀÛªÀ£ÀÄß = Area of the sector OAC - Area of sector OBD
J¼É¢zÉ. avÀæ 5.17 gÀ°è bÁAiÉÄUÉƽ¹zÀ = − cm2 = cm2 = cm2
«£Áå¸ÀzÀ «¹ÛÃtðªÀ£ÀÄß PÀAqÀÄ»r¬Äj.
3. Find the area of the shaded region in Fig. 5.21, if ABCD is a square of side 14 cm and
( = 3.14 JAzÀÄ §¼À¹)
APD and BPC are semicircles.
Side of the square = 14 cm
Area I + Area II = Area ABCD – Area of the two semicircles circle of radius 5 cm
Diameter of the semi circle = 14 cm
⇒Area of ABCD – Area of the circle of radius5 cm = a2 - ∴ Radius of the semi circle = 7 cm
⇒10 x 10 - 3.14x 5 2 = 100 – 3.14 x 25 = 100 – 78.5 = 21.5cm2 Area of the square = 14 × 14 = 196 cm2
Area III + Area IV = 21.5cm2 Area of the semi circle = = = = 77 cm2
Therefore, Area of shaded region = Area ABCD – Area [ I + II + III + IV] Area of two semicircle = 2 × 77 cm2 = 154 cm2
= 100 – 2x(21.5) = 100 – 43 = 57cm2 ∴ Area of shaded region = 196 cm2 - 154 cm2 = 42 cm2
4. Find the area of the shaded region in Fig. 5.22, where a circular arc of radius 6 cm has
been drawn with vertex O of an equilateral triangle OAB of side 12 cm as centre.
Exercise 5.3 OAB is a equilateral triangle, Therefore each angle = 60°.
[ Unless stated otherwise, use = ] Radius of the circle = 6 cm.
1. Find the area of the shaded region in Fig. 5.19, if PQ = 24 cm, PR = 7 cm and O is the centre Side of the triangle = 12 cm.
of the circle. √
Area of the equilateral triangle = (OA)2
PQ = 24 cm and PR = 7 cm √
= (12) = √3 x 3x12 =
2
√ cm2
∠P = 90° [Angle of semi circle ]
∴ Hypotenuse QR = Diameter of the circle Area of the circle = π R =2
×6 = 2
cm2 = cm2
°
QR2 = PR2 + PQ2 [Pythagoras theorem in ∆ PRQ] The area of the sector of angle 60° = ×π r cm
2 2
°
⇒ QR2 = 72 + 24 2 ⇒ QR2 = 49 + 576 = × x 62 cm2 = cm2 = cm2
⇒ QR2 = 625 ⇒ QR = 25 cm
∴ Area of the shaded region
∴ Radius of the circle = cm
= Area of equilateral triangle + Area of the circle – Area of the sector
Area of semi circle = = = cm2 = cm2 = 245.54 cm2 = √ + − cm2 = √ + cm2
Area of ΔPQR = × PR × PQ 5. From each corner of a square of side 4 cm a quadrant of a circle of radius 1 cm is cut and
also a circle of diameter 2 cm is cut as shown in Fig. 5.23. Find the area of the remaining
= × 7 × 24 cm2 = 84 cm2
portion of the square.
∴ Area of shaded region = 245.54 cm2 - 84 cm2 = 161.54 cm2 The side of the square = 4 cm; Radius of the circle = 1 cm
[ Or − 84 = = cm2 ]

P a g e 116 | 159 Available in ykoyyur.blogspot.com P a g e 117 | 159 Available in ykoyyur.blogspot.com


SSLC Mathematics Solutions – Part -1 YK SSLC Mathematics Solutions – Part -1 YK
Area of the square = (Side)2 = 42 = 16 cm2 8. Fig. 5.26 depicts a racing track whose left and right ends are semicircular.
The distance between the two inner
Area of each quadrant = cm2 = = cm2
parallel line segments is 60 m and they are
∴ Area of four quadrant = 4 × cm2 = cm2 each 106 m long. If the track is 10 m wide,
Area of the circle = π R2 cm2 = × 12 = cm2 find :
Area of the square = Side2 = 4 2 = 16cm2 (i) The distance around the track along its
inner edge
+ cm2 = cm2
(ii) The area of the track.
∴ Area of shaded region = Area of square – [Area of four quadrants+ area of circle]
= 16 − + cm2 = cm2 = cm2
6. In a circular table cover of radius 32 cm, a design is formed leaving an equilateral
triangle ABC in the middle as shown in Fig. 5.24. Find the area of the design.
Radius of the circle = 32 cm
AD is the median drawn through the center O ⇒ BD =
∴ Radius of the circle AO = AD[ AD is the median]
⇒ AD = 32 cm ⇒ AD = 48 cm
In triangle ΔADB,
AB2 = AD2 + BD2 [By Pythagoras theorem]
Width of the track = 10 m
⇒ AB2 = 48 2 + 2
⇒ AB2 = 2304 +
Distance between parallel lines DE = CF = 60 m, Length of each parallel line = 106 m
⇒ = 2304 ⇒ AB2 = 3072 ⇒ AB = 32√ cm Radius of inner circle r = OD = O'C = = 30 m
√ √
Area of ΔABC = (AB)2 = (32√3)2 = 768√ cm2 Radius of outer circle R = OA = O'B = 30 + 10 m = 40 m
Area of the circle = π R =2
× 32 × 32 = cm2 AB = CD = EF = GH = 106 m
(i) The distance around the track along its inner edge
∴Area of the design = Area of the circle - Area of ΔABC
= CD + EF + 2 × (Circumference of inner semi-circle)
= − √ cm2 = 106 + 106 + (2 × πr) m = 212 + (2 × × 30) m = 212 + m= m
7. In Fig. 5.25, ABCD is a square of side 14 cm. With centres A, B, C and D, four circles are (ii) Area of the running track
drawn such that each circle touch externally two of the remaining three circles. Find the
area of the shaded region. = Area ABCD + Area EFGH + 2 xArea of outer semi-circle - 2 x Inner semi-circle
Side of the Square = 14 cm = (AB × CD) + (EF × GH) + 2 × -2× m2
∴ Radius of each circle = = 7 cm = (106 × 10) + (106 × 10) + 2 × (R2 -r2) m2 = 1060 + 1060 + × 700 m2
2 2
Area of square ABCD = 14 = 196 cm 2 2
= [1060 + 1060 +( 22 × 100)] m = [2120 +2200] m = 4320 m2
Area of the quadrant = cm =2
= 2
cm = 2
cm 9. In Fig. 5.27, AB and CD are two diameters of a circle (with
centre O) perpendicular to each other and OD i s t h e d i am
∴ Area of four quadrant = 4× cm2 = 154 cm2 et er of t h e s m a l l er ci r cl e. I f OA = 7 cm, find the area
∴ Area of shaded region of the shaded region.
= Area of the square ABCD - 4 Area of four quadrant Radius of the greater circle R = 7 cm
= 196 cm2 - 154 cm2= 42 cm2 Radius of the smaller circle r = cm
Height of ΔBCA = OC = 7 cm; Base of ΔBCA = AB = 14 cm
Area of ΔBCA = × AB × OC = × 7 × 14 = 49 cm2
Area of greater circle = πR2 = × 72 = 154 cm2

P a g e 118 | 159 Available in ykoyyur.blogspot.com P a g e 119 | 159 Available in ykoyyur.blogspot.com


SSLC Mathematics Solutions – Part -1 YK SSLC Mathematics Solutions – Part -1 YK
Area of greater semi-circle = cm2 = 77 cm2 Radius of the quadrant of the circle = OB
Area of smaller circle = πr = 2
× × = cm2 OAB is a right angle triangle
∴ in right angle ΔOAB,
Area of shaded aregion
OB2 = AB2 + OA2 [By Pythagoras theorem ]
= Area of greater semi-circle - Area of ΔBCA + Area of smaller circle
⇒ OB2 = 20 2 + 202⇒ OB2 = 400 + 400
= 77 – 49 + cm2 = cm2 = cm2 = 66.5 cm2
⇒ OB2 = 800⇒ OB = 20√2 cm
10. The area of an equilateral triangleABC is 17320.5 cm2.With each vertex of the triangle as The area of the quadrant of circle = cm2
centre, a circle is drawn with radius equal to half the length of the side of the triangle . √ .
(see Fig. 5.28). Find the area of t h e s h a d ed r eg i on . ( Use = 3 . 1 4 a n d = 3.14 and = cm2 = cm2 = 3.14 x 200 cm2 = 628 cm2
√ = 1.73205) ZËPÀzÀ «¹ÛÃtð = 20 × 20 = 400 cm2
ABC is an equilateral triangle ∴ ∠A = ∠B = ∠C = 60° Area of shaded region
Area of ΔABC = 17320.5 cm2 ⇒

× (AB)2 = 17320.5 = Area of quadrant of circle - Area of the square = 628 - 400 cm2 = 228 cm2
14. AB and CD are respectively arcs of two concentric circles of radii 21 cm and 7 cm and
⇒ AB2 = 17320.5 × ⇒ AB2 = 4 × 10 4 ⇒ AB = 200 cm
. centre O (see Fig. 5.32). If AOB = 30°, find the area of the shaded region
Radius of the circle = cm = 100 cm Radius of the greater circle R = 21 cm and radius of smaller circle r = 7 cm
°
Area of three equal sector of angle 600 =3x ×π r2 cm2 The angle formed by two concentric are = 30°
°
°
2 2
=3x ×3.14 x 100 cm = ×3.14 x 100 cm = 2 2
cm2 = 15700 cm2 Area of outer sector = ° ×π r2 cm2
Area of Shaded region =Area of ΔABC - Area of three equal sector = x x 21x21 cm2 = x 22 x 3 x7 cm2
= (17320.5 – 15700) cm2 = 1620.5 cm2 = x 11 x 3 x7 cm2 = cm2
11. On a square handkerchief, nine circular designs each of radius 7 cm are made °
Area of inner sector = ×π r2 cm2
(see Fig. 5.29). Find the area of the remaining portion of the handkerchief. °

Number of circles = 9; Radius of each circle = 7 cm = x x7x7 cm2 = x 11 x7 cm2 = cm2


There are three circles touch each other Area of shaded region
∴ Side of the square = 3 × diameter of the circle = 3 × 14 = 42 cm = Area of outer sector - Area of inner sector
Area of the square = 42 × 42 cm2 = 1764 cm2 = − cm2 = − cm2 = cm2 = cm2
Area of 9 equal circle = 9π r2 = 9x × 7 × 7 = 1386 cm2 15. In Fig. 5.33, ABC is a quadrant of a circle of radius 14 cm and a semicircle is drawn with
The area of remaing part of the handkerchief BC as diameter. Find the area of the shaded region.
= Area of the square - Area of 9 equal circle = 1764 - 1386 = 378 cm2 The radius of quadrant ABC of circle = 14 cm
12. In Fig. 5.30, OACB is a quadrant of a circle with centre O and radius 3.5 cm. If OD = 2 cm, AB = AC = 14 cm
find the area of the BC is the diameter of semi circle
i) quadrant OACB ii) shaded region. Now, ABC is a right angle triangle
Radius of the quadrant of the circle = 3.5 cm = cm ∴BC2 = AB2 + AC2 [Pythgoras theorem]
(i) Area of OACB quadrant = cm2 = cm2= cm2 ⇒ BC2 = 14 2 + 142 ⇒ BC = 14√2 cm

(ii) Area of ∆BOD = × × 2 cm = cm 2 2 Radius semi circle = cm = 7√2 cm
Area of ΔABC = × 14 × 14 cm2 = 7 × 14 × 14 = 98 cm2
Area of shaded region
= Area of OACB – Area of ∆ BOD The area of the quadrant of circle = cm2 = cm2 = 154 cm2
= − cm2= − cm2 = cm2 = 6.125 cm2 √ √
Area of semi circle = = = 154 cm2
13. In Fig. 5.31, a square OABC is inscribed in a quadrant OPBQ. If OA = 20 cm, find the area
of the shaded region. (Use = 3.14) Area of shaded region
Side of the square = OA = AB = 20 cm = Area of semi circle + Area of ΔABC- Area of quadrant of circle

P a g e 120 | 159 Available in ykoyyur.blogspot.com P a g e 121 | 159 Available in ykoyyur.blogspot.com


SSLC Mathematics Solutions – Part -1 YK SSLC Mathematics Solutions – Part -1 YK
= 154 + 98 - 154 cm2 = 98 cm2
16. Calculate the area of the designed region in Fig. 5.34 common between the two
quadrants of circles of radius 8 cm each.
AB = BC = CD = AD = 8 cm
Area of ΔABC = Area of ΔADC = x 8 x 8 = 32 cm2
6 Constructions
Area of Quadrant AECB = Area of quadrant AFCD 6.2 Division of a Line Segment
= cm2 = = cm2 Construction 6.1: To divide a line segment in a given ratio.
Area of shaded region Divede a line segment AB in the ratio m:n
Example: Devide the line segment AB in the ratio 3:2
= (Area of quadrant AECB – Area ofΔABC)
+ (Area of quadrant AFCD – Area of ΔADC)
= − 32 + − 32 cm2
=2x − 32 cm2 = 2 x cm2
2 2
=2x cm = cm

Summary:
1. Circumference of the circle = 2πr
Step-1: Draw any ray AX, making an acute angle with AB
2. Area of the circle = πr
3. The radius of the circle r the angle measure with θ (Can draw above or below the given line)
Then the Length of the Arc of the sector = x 2πr Step-2: Locate 5 (= m + n) points A1, A2, A3, A4 and A5 on AX so that AA1 = A1 A2 =
4. The radius of the circle r the angle measure with θ A2 A3 = A3 A4 = A4 A5
Then the area of the sector = x πr Step-3: Join BA5
5. Area of segment of a circle = Area of the corresponding sector - Area of the Step-4: Through the point A3 (m = 3), draw a line parallel to A5B (by making an angle
corresponding triangle. equal to AA5 B) at A3 intersecting AB at the point C (see Fig.). Then, AC : CB = 3 : 2
Justification:
A3CǁA5B ⇒ = [Basic proportionality theorem]⇒ = = ⇒ 3:2
Now AC : CB = 3 : 2

Alternate Method:
Step 1: Draw any ray AX making an
acute angle with AB .
Step 2: Draw a ray BY parallel to AX by
making ∠ABY equal to ∠BAX
Step 3: Locate the points A1, A2, A3 (m
= 3) on AX and B1, B2 (n = 2) on BY
such that AA1 = A1A2 = A2 A3 = BB1
= B1 B2
Step 4: Join A3B2 . Let it intersect AB at
a point C

P a g e 122 | 159 Available in ykoyyur.blogspot.com P a g e 123 | 159 Available in ykoyyur.blogspot.com


SSLC Mathematics Solutions – Part -1 YK SSLC Mathematics Solutions – Part -1 YK
Step 4: Dr a w a line t hr oug h C1 pa r a llel
Justification: to the line CA to intersect BA at A 1 [Note:
Extended BA]
In ∆ AA3C and ∆ BB2C ∠ AC A3 = BC B2 ( Vertically opposite angles)
Justification:
∠ CAA3 = CBB2 (Alternate angles) ∆ ~∆ ′ ⇒ = =
∆ AA3C ~ ∆ BB2C ( AA similarity criteria)
But, = = ∴ = ⇒ = = =
⇒ = [BPT] ⇒ = ⇒ = ⇒ AC : BC = 3:2
Exercise 6.1
Construction 6.2:
In each of the following, give the justification of the construction also:
To construct a triangle similar to a given triangle as per given scale factor.
1. Draw a line segment of length 7.6 cm and divide it in the ratio 5 : 8. Measure the two
Example1:Construct a triangle similar to a given triangle ABC with its sides equal to
parts.
of the corresponding side of the triangle ABC [i,e. of scale factor ]
Solution: Given a triangle ABC, we are required to construct another triangle whose sides are
of the corresponding sides of the triangle ABC.
Step-1: Draw any ray BX making an acute angle
with BC on the side opposite to the vertex A
Step-2: Locate 4 (the greater of 3 and 4 in )
points B1, B2, B3 and B4 on BX so that BB1
= B1 B2 = B2 B3 = B3B4. Step-1: Draw any ray AX, making an acute angle with AB
Step-3: Join B4C and draw a line through B3 the (Can draw above or below the given line)
3rd point, (3 being smaller of 3 and 4in )parallel Step-2:Locate 13(5+8) points A1, A2, ---A12, A13, on AX so that AA1 = A1 A2 = A2 A3---A12 A13
toB4C to intersect BC at C1 Step-3:Join BA13
Step-3: Dr a w a line t hr oug h C1 pa r a llel Step-4: Through the point A5 (m = 5), draw a line parallel to A13 B (by making an angle
to the line CA to intersect BA at A 1 equal to AA13 B) at A5 intersecting AB at the point C (see Fig.). Then, AC : CB = 5 : 8
Justification:
Justification:
= ∴ = = ⇒ =
A5CǁA13B ⇒ = [BPT] ⇒ = = ⇒ 5:8
C1A1 ǁ CA ∴ ∆ A1BC1 ~ ∆ ABC ⇒ = = = 2. Construct a triangle of sides 4 cm,
Example 2 : Construct a triangle similar to a 5 cm and 6 cm and then a triangle
given triangle ABC with its sides equal to of similar to it whose sides are of
the corresponding side of the triangle ABC the corresponding sides of the
[i,e. of scale factor ] first triangle.
Step1: Construct any ∆ABC. Draw any ray BX Step-1: Construct any ∆ABC.Draw
making an acute angle with BC on the side any ray BX making an acute angle
opposite to the vertex A. with BC on the side opposite to the
Step 2: Locate 5 points (the greater of 5 and 3 vertex A
in ) B1, B2, B3, B4 and B5 on BX such that Step-2: Locate 3 (the greater of 3
BB1= B1B2 = B2B3 = B3B4 = B4B5 and 4 in ) points B1, B2, B3 on
Step 3: Join B3( the 3rd point, 3 being smaller of BX so that BB1 = B1 B2 = B2 B3
3 and 5 in ) to C and draw a through B5 parallel Step-3:Join B3C and draw line through B2 (The 2nd point, being smaller of 2 and 3 in ) parallel
to B3C intersect BC at C1 to B3C to intersect BC at C1

P a g e 124 | 159 Available in ykoyyur.blogspot.com P a g e 125 | 159 Available in ykoyyur.blogspot.com


SSLC Mathematics Solutions – Part -1 YK SSLC Mathematics Solutions – Part -1 YK
Step – 4: Dr a w a line t hr oug h C1 pa r a llel to the line CA to intersect BA at A 1 Justification:
Justification:
∆ ~∆ ′ ⇒ = =
= ∴ = = ⇒ =
But, = = ∴ = ⇒ = = =
⇒C1A1 ǁCA ∴ ∆ A1BC1 ~ ∆ ABC
3. Construct a triangle with sides 5 cm, 6 cm and 7 cm and then another triangle whose 5. Draw a triangle ABC with side BC = 6cm, AB = 5cm, and ∠ABC = 600 . Then construct
a triangle whose sides are of the corresponding sides of the triangle ABC.
ssides are of the corresponding sides of first.
Step-1: Construct a ∆ABC with side BC =
Step-1: Construct a triangle BC= 7cm,
6cm, AB = 5cm and ∠ABC = 600
AB=5cm and AC =6cm
Step-2: Draw any ray BX making an acute
Step-2: Draw any ray BX making an acute
angle with BC on the side opposite to the
angle with BC on the side opposite to the
vertex A.
vertex A.
Step-3: Locate 4 points (the greater of 4 and 3
Step-3: Locate 7 points (the greater of 7 and 5
in ) B1, B2, B3 , B4 on BX such thatBB1=
in ) B1, B2, B3, B4 ---- B7 on BX such that
B1B2 = B2B3 = B3B4
B1B2 = B2B3 = B3B4 = ---- = B6B7
Step-4: Join B4( the 3 th point, 3 being smaller
Step-4: Join B5C( the 5th point, 5 being
of 3 and 4 in ) to C and draw a line through
smaller of 5 and 7 in ) to C and draw a line
B3 parallel to B4C intersect BC at C1
through B7 parallel to B5C intersect BC at C1 Justification:
[Note:Extended BC]
= ∴ = = ⇒ =
Step-5: Dr a w a line t hr oug h C1 pa r a llel
to the line CA to intersect BA at A1 [Note: C1A1 ǁ CA ∴ ∆ A1BC1 ~ ∆ ABC
Extended BA] ⇒ = = =
Justification: 6. Draw a triangle ABC with side BC = 7cm,
∆ ~∆ ′ ′ ⇒ = = ∠B =45 0 , ∠A =1050 then construct a
But, = = triangle whose sides are times the
corresponding sides of ∆ABC.
∴ = ⇒ = = =
Step-1: Construct a ∆ABC with sideb BC=
4. Construct an isosceles triangle whose base is 4cm and then another triangle whose sides
7cm, ∠B =450 , ∠A =1050
are 1 times the corresponding sides of the isosceles triangle. Step -2: Draw any ray BX making an acute
Step-1: Construct an Isoceles triangle with base angle with BC on the side opposite to the
8cm and altitude 4cm., vertex A.
Step-2: Draw any ray BX making an acute Step-3: Locate 4 points (the greater of 4 and
angle with BC on the side opposite to the 3 in ) B1, B2, B3 , B4 on BX such that BB1
vertex A. = B1B2 = B2B3 = B3B4
Step-3: Locate 3 points (the greater of 3 and 2 Step-4: Join B4( the 4th point, 4 being greater
in ) B1, B2, B3 on BX such that BB1= B1B2 = of 3 and 4 in ) to C and draw a line through
B2B3 B3 parallel to B4C intersect BC at C1
Step-4: Join B2( the 5 th point, 5 being smaller Step-5: Dr a w a line t hr oug h C1 pa r a llel to the line CA to intersect BA at A1
of 5 and 7 in ) to C and draw a line through Justification:
B3 parallel to B2C intersect BC at C1 = ∴ = = ⇒ =
[Note:Extended BC] C1A1 ǁ CA ∴ ∆ A1BC1 ~ ∆ ABC
Step-5: Dr a w a line t hr oug h C1 pa r a llel to the line CA to intersect BA at A 1 [Note:
Extended BA] ⇒ = = =

P a g e 126 | 159 Available in ykoyyur.blogspot.com P a g e 127 | 159 Available in ykoyyur.blogspot.com


SSLC Mathematics Solutions – Part -1 YK SSLC Mathematics Solutions – Part -1 YK
7. Draw a right triangle in which the sides(other than hypotenuse) are lengths 4cm and Exercise 6.2
3cm. Then construct another triangle whose sides are times corresponding sides of the
In each of the following, give also the justification of the construction:
given triangle.
Step-1: Construct right with sides 4cm and 3cm 1. Draw a circle of radius 6 cm. From a point 10 cm away from its centre, construct the pair
of tangents to the circle and measure their lengths.
(exept hypotenuse)
Step-2: Draw any ray BX making an acute Step-1: Draw PO = 10cm. Join PO and bisect
it. Let M be the mid- point of PO
angle with BC on the side opposite to the
Step-2: Taking M as centre and MO as radius,
vertex A.
draw a circle. Let it intersect the given circle
Step-3: Locate 4 points (the greater of 3 and 4
at the points Q and R
in ) B1, B2, B3,B4 on BX such thatBB1= B1B2 Step-3: Join PQ and PR measure the length
= B2B3 = B3B4 Then PQ and PR are the required two
Step-4: Join B4( the 4 th point, 4 being greater tangents
of 4 and 3 in ) to C and draw a line through Justfication:
B3 parallel to B4C intersect BC at C1 Join O, Q ∠PQO is an angle in semi circle
Step-4: Dr a w a line t hr oug h C1 pa r a llel ∴ ∠ PQO = 90 0 ⇒ PQ ⟘OQ, OQ is the radius
to the line CA to intersect BA at A 1 of given circle.Therefore PQ is the tangent to
Justification: the circle. Similarlly PR also the tangent to the
circle.
= ∴ = = ⇒ =
∆ PQO is a right angle triangle
C1A1 ǁ CA ∴ ∆ A1BC1 ~ ∆ ABC OP2 = OQ2 + PQ2 [ Pythagoras theorem]
⇒ = = = ⇒ PQ2 = 102 – 6 2
6.3 Construction of Tangents to a Circle ⇒ PQ2 = 64 ⇒ PQ = 8 cm
To construct the tangents to a circle from a point outside it 2. Construct a tangent to a circle of radius 4 cm from a point on the concentric circle of
We are given a circle with centre O and a point P outside it. We have to construct the two radius 6 cm and measure its length. Also verify the measurement by actual calculation.
tangents from P to the circle. Step-1: Draw two concentric circles at O with
Step 1: Join PO and bisect it. Let M be radius 4cm and 6cm. Locate a point P on
the mid- point of PO greater circle and join PO. Bisect it. Let M be
Step 2: Taking M as centre and MO as the mid- point of PO
radius, draw a circle. Let it intersect Step-2: Taking M as centre and MO as radius,
the given circle at the points Q and R. draw a circle. Let it intersect the given circle
Step 3: Join PQ and PR at the points Q
Then PQ and PR are the required Step-3: Join PQ and mesure the length. It is
two tangents ≈4.5cm
Justification: Then PQ a≈4.5cm is the required tangent
Join OQ, ∠PQO is an angle in semi Justification:
circle Join O, Q ∠PQO is an angle in semi circle
∴ ∠ PQO = 90 0 ⇒ PQ ⟘OQ, OQ is ∴ ∠ PQO = 90 0 ⇒ PQ ⟘OQ, OQ is the
the radius of given circle. Therefore radius of given circle.Therefore PQ is the
PQ is the tangent to the circle. tangent to the circle.
Similarlly PR also the tangent to the ∆ PQO is a right angle triangle
circle. OP2 = OQ2 + PQ2 [ Pythagoras theorem]
⇒ PQ2 = 6 2 – 42
⇒ PQ2 = 20 ⇒ PQ = 4.47 cm

P a g e 128 | 159 Available in ykoyyur.blogspot.com P a g e 129 | 159 Available in ykoyyur.blogspot.com


SSLC Mathematics Solutions – Part -1 YK SSLC Mathematics Solutions – Part -1 YK
3. Draw a circle of radius 3 cm. Take two points P and Q on one of its extended diameter
each at a distance of 7 cm from its centre. Draw tangents to the circle from these two
points P and Q.

Step-1:Draw AB = 8cm. Draw circles of radii 4cm and 3cm at A and B.


Step-2:Draw perpendicular bisector to AB, it intersect AB at M
Step-1:Construct a circle of radius 3cm at O. Draw a diameter and extend the diameter 7cm Step-3:Draw a circle with center M and passes through A and B. It intersects a circle of radius
on both sides. Name the end point as p and q. Draw perpendicular bisector of Op and Oq and 4cm at P,Q and the circles of radius 3cm at S,R.
they intersect at M and N. Step-4:Join BP, BQ,AS and AR. These are required tangents.
Step-2:From centers M and N draw circles of radius MO and NO. These circles intersect the Justification:
given circle at A,B and C,D. Join AP and BS
Step-3: Join p to A and B and q to C and D. ∠APB and ∠ASB are the angles on semi-circles
Now, pA,pB, qC and qD are the required tangents. ∴ ∠APB = 900 and ∠ASB = 90 0
Justification: Since, AP and BS are the radius, BP and AS have to be the tangents to the circle.
Join p, A ∠pAO is angle on semi circle. Similarlly BQ and AR are also the tangents.
∴ ∠ pAO = 90 0 ⇒ pA ⟘OA 6. Let ABC be a right triangle in which AB = 6 cm, BC = 8 cm and B = 90°. BD is the
OA is the radius of the given circle. So, pA has to be a tangent. perpendicular from B on AC. The circle through B, C, D is drawn. Construct the tangents
Similarlly pB, qC, qD are the tangents. from A to this circle.
Step-1:Draw the line segments BC= 8cm.and AB =6cm
4. Draw a pair of tangents to a circle of radius 5 cm which are inclined to each other at an
perpendicular to each other. Join AC. Thus, ∆ABC is a
angle of 60°.
right angle triangle.
Step-1:Construct angle 1200 [180 0- 600]
Step-2:Draw perpendicular to BC which meets BC at O
at the center.For this draw a diameter and
Step-3:With O as center and OB as radius draw a circle
construct ∠ROQ = 600. Then we get
which intersects AC at D then ∠BDC = 90 0 . Thus BD
∠ROP = 120 0 at the center.
is perpendicular to AC.
Step-2:Draw two perpendiculars at P and
Step-4: With a as center and AB as radius draw an arc
R and the intersect at A.
cutting the circle at M
Now, AP and AR are the required
Step-5: Join AM. Thus AB and AM are required
tangents
tangents.
Justification:
Justification:
AR⟘OR [Construction] ⇒∠ARO = 900
In ∆ABC , ∠ABC = 900
And OR is the radius at point of contact.
∴ AR is the tangent. Similarly AP is also ⇒∆ABC is right angle triangle.
the tangent to the circle. ⇒AB⟘BO and BO is the radius. So , AB has to be a tangent to the circle.
AB = AM [Construction ]
5. Draw a line segment AB of length 8 cm. Taking A as centre, draw a circle of radius 4 cm
∴ AM is also a tangent.
and taking B as centre, draw another circle of radius 3 cm. Construct tangents to each
circle from the centre of the other circle. 7. Draw a circle with the help of a bangle. Take a point outside the circle. Construct the pair
of tangents from this point to the circle.
P a g e 130 | 159 Available in ykoyyur.blogspot.com P a g e 131 | 159 Available in ykoyyur.blogspot.com
SSLC Mathematics Solutions – Part -1 YK SSLC Mathematics Solutions – Part -1 YK

7
Step-1:Draw a circle by using a bangle.
Step-2: Draw two chords MN and NL
Step-3:Draw perpendicular bisector of
Coordinate Geometry
MN and NL they intersect at point O Coordinate axes:
which is the center of the circle. | |
A set of a pair of perpendicular axes and
Step-4: Take a point P outside the
circle join OP and bisect it.
Step-5: Let D be the midpoint of OP. Horizontal line
Taking D as center and OD as radius,
draw a circle which intersects the given
circle at A and B. X1OX X - Axis
Step-6: Join AP and BP. Thus AP and
BP are the required tangents from P
Vertical line
Justification:
Join OA,OB. ∠OAP is the angle lying in the semi-circle.
∴ ∠OAP = 900 ⇒ AP⟘OA,
YOY1 Y - Axis
Since, OA is the radius of the circle with center O. So AP has to be a tangent to the circle.
Similarlly BP is also a tangent.
The intersection point of X and Y axes is called the Orgin ′O
Summary
In this chapter, you have learnt how to do the following constructions:
1. To divide a line segment in a given ratio The distance of a point from the y-axis is called its x-coordinate, or abscissa. The distance
2. To construct a triangle similar to a given triangle as per a given scale factor of a point from the x-axis is called its y-coordinate, or ordinate. The coordinates of a
which may be less than 1 or greater than 1. point on the x-axis are of the form (x, 0), and of a point on the y-axis are of the form (0, y).
3. To construct the pair of tangents from an external point to a circle. The Coordinate axes divides the plane in to four parts. They are called quadrants.
The coordinaes of the orgin is ( , )
7.2 Distance Formula
The distance between two points on X-axis or on
the straight line paralle to X-axis is
Distance = −
The distance between two points on Y-axis or on
the straight line paralle to Y-axis isÀ
Distance = −
AB = AC + BC
The distance between two points which are
neither on X or Y axis nor on the line paralle to
X orY axis
= ( − ) +( − )
The distance between the point P(x,y) and the orgin
= +
Example 1:Do the points (3, 2), (–2, –3) and (2, 3) form a triangle? If so, name the type
of triangle formed.
P(3, 2), Q(−2, −3), R(2, 3)
Formula = ( − ) +( − )
P a g e 132 | 159 Available in ykoyyur.blogspot.com P a g e 133 | 159 Available in ykoyyur.blogspot.com
SSLC Mathematics Solutions – Part -1 YK SSLC Mathematics Solutions – Part -1 YK
AB + BC = 3√2 + 2√2 = 5√2
PQ = 3 − (−2) + 2 − (−3) Since,, AB + BC = AC we can say that the points A, B and C are collinear.
= (3 + 2) +(2 + 3) Therefore, they are seated in a line
= (5) +(5) = √25 + 25 = √50 = 7.07 Example 4 : Find a relation between x and y such that the point (x , y) is equidistant
from the points (7, 1) and (3, 5).
QR = (−2 − 2) +(−3 − 3)
Let the point P (x, y) is equi distance from the
= (−4) +(−6) = √16 + 36 = √52 = 7.21 points A (7, 1) and B (3, 5)
PR = (3 − 2) +(2 − 3) = (1) +(−1) PA = PB ⇒ PA2 = PB2
= √1 + 1 = √2 = 1.41 PA = (x − 7) +(y − 1)
Since the sum of any two of these distances is PB = (x − 3) +(y − 5)
greater than the third distance, therefore the point
P,Q and R form a triangle. AP2 = BP2 ⟹ (x – 7) +(y − 1)
Also, PQ2 + PR2 = QR2 by the converse of = (x − 3) +(y − 5)
Pythagoras theorem ∠P = 90O we have
Therefore, PQR is a right triangle. (x − 7)2 + (y − 1)2 = (x − 3)2 + (y − 5)2
Example2: Show that the points (1, 7), (4, 2), (–1, –1) and (– 4, 4) are the vertices of a x + 7 − 2(x)(7) + y + 1 − 2(y)(1) = x + 3 − 2(x)(3) + y + 5 − 2(y)(5)
square. x + 49 − 14x + y + 1 − 2y = x + 9 − 6x + y + 25 − 10y
Solution: A (1, 7), B (4, 2), C (-1, -1) and D (-4, 4) x − x − 14x + 6x + y − y − 2y + 10y = 34 − 50
AB = (4 − 1) + (2 − 7) = (3) +(−5) = √9 + 25 = √34 −8x + 8y = −16 ÷ −8 ⇒ − = 2
Which is the require relation.
BC = (−1 − 4) +(−1 − 2) = (−5) +(−3) = √25 + 9 = √34
Remark :Note that the graph of the equation x – y = 2 is a line. From your earlier studies, you
CD = −4 − (−1) + 4 − (−1) know that a point which is equidistant from A and B lies on the perpendicular bisector of
AB. Therefore, the graph of x – y = 2 is the perpendicular bisector of AB
= (−4 + 1) +(4 + 1)
Example 5 : Find a point on the y-axis which is equidistant from the points A(6, 5) and
= (−3) +(5) = √9 + 25 = √34 B(– 4, 3).
DA = 1 − (−4) +(7 − 4) We know that a point on the y − axis is of the form (0, y). P (0, y) So, let the point P(0, y)
be equidistant from A and B. Then PA = PB
= (1 + 4) +(3) = (5) +(3) = √25 + 9 = √34 (6 − 0)2 + (5 − )2 = (−4 − 0)2 + (3 − )2
AC = (−1 − 1) +(−1 − 7) 36 + 5 + − 2(5)( ) = 16 + 3 + − 2(3)( )
= (−2) +(−8) = √4 + 64 = √68 36 + 25 + − 10 = 16 + 9 + −6
BD = (−4 − 4) +(4 − 2) − − 10 + 6 = 25 − 61 ⇒ −4 = −36
= (−8) +(2) = √64 + 4 = √68 = = 9 Therefore the required point is ( , )
Since, AB = BC = CD = DA and AC = BD, all the PA = (6 − 0) +(5 − 9) = (6) +(−4) = √36 + 16 = √52
four sides of the quadrilateral ABCD are equal and its
diagonals AC and BD are also equal. Thereore, ABCD PB = (−4 − 0) +(3 − 9) = (−4) +(−6) = √16 + 36 = √52
is a square
Example 3 : Fig. 7.6 shows the a r r a n gement o f Exercise 7.1
des ks in a classroom. Ashima, Bharti and
Camella are seated at A (3, 1), B(6, 4) and C(8, 6) 1. Find the distance between the following pairs of points :
respectively. Do you think they are seated in a line? i) (2, 3), (4, 1) ii) (-5, 7), (-1, 3) iii) (a, b), (-a, -b)
G ive r ea s on s for your answer. i) (x , y ) = (2, 3) , (x , y ) = (4, 1)
AB = (6 − 3) +(4 − 1) = (3) +(3) Formula = ( − ) +( − )
2 3 4 1
= √9 + 9 = √18 = √9 × 2 = √ d = (4 − 2) +(1 − 3) = (2) +(−2)
BC = (8 − 6) +(6 − 4) = (2) +(2) d = √4 + 4 = √2 × 4 = 2√2 Units
= √4 + 4 = √8 = √4 × 2 = √ ii) (x , y ) = (−5, 7) , (x , y ) = (−1, 3) -5 7 -1 3
AC = (8 − 3) +(6 − 1) = (5) +(5) d = (−1 − [−5) +(3 − 7) = (4) +(−4)
= √25 + 25 = √50 = √25 × 2 = √ d = √16 + 16 = √2 × 16 = 4√2 Units
iii) (x , y ) = (a, b) , (x , y ) = (−a, −b)

P a g e 134 | 159 Available in ykoyyur.blogspot.com P a g e 135 | 159 Available in ykoyyur.blogspot.com


SSLC Mathematics Solutions – Part -1 YK SSLC Mathematics Solutions – Part -1 YK
d= (−a − a) +(−b − b) = (−2a) +(−2b) = √9 + 9 = √18 = 3√2 ( )
d = √4a + 4b = 4(a + b ) = 2√a + b Units a b -a -b CD = (6 − 9) +(1 − 4) = (−3) +(−3)
2. Find the distance between the points (0, 0) and (36, 15). Can you now find the distance = √9 + 9 = √18 = 3√2 ( )
between the two towns A and B discussed in Section 7.2. DA = (6 − 3) +(1 − 4) = (3) +(−3)
(x, y) =(36, 15) = √9 + 9 = √18 = 3√2 ( )
d = x +y = √36 +15 = √1296 + 225 = √1521 = Units AB = BC = CD = DA
We can find the distance between the two towns A and B.Suppose town A is at the Orgin, Diagonal AC = (9 − 3) +(4 − 4) = (6) +(0) = √36 = 6 ( )
then the town has to be at (36,15). The distance between these two towns is 39km (1, 5), Diagonal BD = (6 − 6]) +(7 − 1) = (0) +(6) = √36 = 6 ( )
3. Determine if the points (1, 5), (2, 3) and (– 2, – 11) are AC = BD
collinear. Thus, AB = BC = CD = DA , diagonals: AC = DB
A (1, 5), B (2, 3) ªÀÄvÀÄÛ C (-2, -11) Since all the four sides and diagonals are equal.
Hence, ABCD is a square. So, Champa is correct.
AB = (2 − 1) +(3 − 5) = (1) +(−2)
6. Name the type of quadrilateral formed, if any, by the following points, and give reasons
= √1 + 4 = √5
for your answer:
BC = (−2 − 2) +(−11 − 3) i) (-1, -2), (1, 0), (-1, 2), (-3, 0) ii) (-3, 5), (3, 1), (0, 3), (-1, -4) iii) (4, 5), (7, 6), (4, 3), (1, 2)
= (−4) +(−14) ) (− , − ), ( , ), (− , ), (− , )
= √16 + 196 = √212 AB = 1 − (−1) + 0 − (−2)
AC = (−2 − 1) +(−11 − 5)
= (1 + 1) +(0 + 2) = (2) +(2)
= (−3) +(−16) = √9 + 256 = √265
= √4 + 4 = √8 = √4 × 2 = 2√2
AB + BC ≠ AC
BC = (−1 − 1) +(2 − 0) = (−2) +(2)
∴ These are non-collinear points
= √4 + 4 = √8 = √4 × 2 = 2√2
4. Check whether (5, – 2), (6, 4) and (7, – 2) are
the vertices of an isosceles triangle. CD = −3 − (−1) +(0 − 2) = (−3 + 1) +(−2)
Formula d = (x − x ) +(y − y ) (−2) +(−2) = √4 + 4 = √8 = √4 × 2 = 2√2
=
PQ = (6 − 5) + 4 − (−2)
DA = −3 − (−1) + 0 − (−2)
= (1) +(6) = √1 + 36 = √37 (i)
= (−3 + 1) +(2) = √4 + 4 = √8 = √4 × 2 = 2√2
QR = (7 − 6) +(−2 − 4) AB = BC = CD = DA
= (1) +(−6) = √1 + 36 = √37 (ii)
AC = −1 − (−1) + 2 − (−2) = (−1 + 1) +(2 + 2) = (0) +(4) = √16 = 4
PR = (7 − 5) +(−2 − [−2])
= (2) +(0) = √4 = 2 (iii) BD = (−3 − 1) +(0 − 0) = (−4) +(0) = √16 = 4
(i), (ii), (iii)  PQ = QR, Thus, AC = BD
Since,Two sides of the tringle are equal. Since, the four sides AB, BC, CD and DA are equal and the diagonals AC = DB are equal.
Hence, ∆PQR is an isosceles triangle. So the quadrilateral ABCD is a square.
5. In a classroom, 4 friends are seated at the ii) (-3, 5), (3, 1), (0, 3), (-1, -4)
points A, B, C and D as shown in Fig. 7.8.
Champa and Chameli walk into the class and AB = 3 − (−3) + 1 − (−3)
after observing for a few minutes Champa asks
= (3 + 3) +(1 + 3) = (6) +(4)
Chameli, “Don’t you think ABCD is a
square?” Chameli disagrees. Using distance = √36 + 16 = √52
formula, find which of them is correct. BC = (0 − 3) +(3 − 1) = (−3) +(2)
The coordinates of the points A,B,C and D are, = √9 + 4 = √13
A(3,4) , B(6, 7) , C(9,4), D(6,1) CD = (−1 − 0) +(−4 − 3)
AB = (6 − 3) +(7 − 4) = (3) +(3) = (−1) +(−7) = √1 + 49
= √9 + 8 = √18 = 3√2 () = √50 = √25 × 2 = 5√2
BC = (9 − 6) +(4 − 7) = (3) +(−3)

P a g e 136 | 159 Available in ykoyyur.blogspot.com P a g e 137 | 159 Available in ykoyyur.blogspot.com


SSLC Mathematics Solutions – Part -1 YK SSLC Mathematics Solutions – Part -1 YK
9. If Q(0, 1) is equidistant from P(5, –3) and R(x, 6), find the values of x. Also find the
DA = −3 − (−1) +(−4 − 5) distances QR and PR.
= (−3 + 1) +(−9) = (−2) +(−9) The point Q (0, 1) is equi distance from the points P (5, -3) and R (x, 6)
= √4 + 81 = √85 PQ = QR ⇒ PQ2 = PR2
AB ≠ BC ≠ CD ≠ DA PQ = (5 − 0) +(−3 − 1) = (5 ) +(−4) = √25 + 16 = √41
Since, the four sides AB, BC, CD and DA are not equal.Hence these poist does not form a QR = (x − 0) +(6 − 1) = (x) +(5) = √x + 25
quadrilateral. PQ2 = PR2 ⟹ √x + 25 = √41
iii) (4, 5), (7, 6), (4, 3), (1, 2)
x + 25 = 41⇒ x = 41 − 25 ⇒ x = 16 ⇒ x = ±√16 ⇒ x = ±4
Formula: d = (x − x ) +(y − y ) The coordinate of the point R is (4,6) or (−4,6)
AB = (7 − 4) +(6 − 5) If the coordinates of R is (4,6) then,
= (3) +(1) = √9 + 1 = √10 QR = (4 − 0) +(6 − 1) = (4) +(5) = √16 + 25 = √41
BC = (4 − 7) +(3 − 6) PR = (4 − 5) +(6 − (−3)) = (−1) +(6 + 3) = √1 + 81 = √82
= (−3) +(−3) = √9 + 9 If the coordinates of R is (−4,6) then,
= √18 = √9 × 2 = 3√2 QR = (−4 − 0) +(6 − 1) = (−4) +(5) = √16 + 25 = √41
CD = (1 − 4) +(2 − 3) PR = (−4 − 5) +(6 − (−3)) = (−9) +(6 + 3) = √81 + 81 = √81 × 2 = 9√2
10. Find a relation between x and y such that the point (x, y) is equidistant from the point
= (−3) +(−1) = √9 + 1 = √10
(3, 6) and (– 3, 4).
DA = (1 − 4) +(2 − 5) The point P (x, y) is equidistance from the points A (3, 6) and B (-3, 4).
= (−3) +(−3) = √9 + 9 PA = PB ⇒ PA2 = PB2
= √18 = √9 × 2 = 3√2 PA = (x − 3) +(y − 6)
AB = CD , BC = DA
PB = (x − (−3)) +(y − 4)
AC = (4 − 4) +(3 − 5) = (0) +(−2) = √0 + 4 = √4 = 2
BD = (1 − 7) +(2 − 6) = (−6) +(−4) = √36 + 16 = √52 = √4 × 13 = 2√13 AP2 = BP2 ⟹ (x − 3) +(y − 6) = (x − (−3)) +(y − 4)
(x − 3)2 + (y − 6)2 = (x + 3)2 + (y − 4)2
AC ≠ DB
x + 3 − 2(x)(3) + y + 6 − 2(y)(6) = x + 3 + 2(x)(3) + y + 4 − 2(y)(4)
Thus opposite sides are equal. AB = CD , & BC = DA
x + 9 − 6x + y + 36 − 12y = x + 9 + 6x + y + 16 − 8y
But diagonals are not equal. AC ≠ DB ∴ The given points are forming a parallelogram. x − x − 6x − 6x + y − y − 12y + 8y = 25 − 45
7. Find the point on the x-axis which is equidistant from (2, –5) and (–2, 9). −12x − 4y = −20 ÷ −4
We know that a point on the X − axis is of the form (x, 0). 3 + − 5 = 0 This is the required relation
Let the point P (x, 0) is equi distance from the points A(2, -5) and B(-2, 9) 3 + − 5 = 0 is representing a straight line
AP = BP Thus the point equidistance from the point A and B on the perpendicular bisector of AB
( − 2)2 + (0 − (−5))2 = ( − (−2) )2 + (0 − 9)2 7.3 Section Formula
( − 2)2 + 52 = ( + 2 )2 + (−9)2
+ 2 − 2( )(2) + 25 = + 2 + 2( )(2) + 81 The coordinates of the point P(x, y) which divides the line segment joining points ,
−4 + 25 = 4 + 81 and ( , ), internally, in the ratio m1 : m2 are
−4 − 4 = 81 − 25 ⇒ −8 = 56 ⇒ = = −7 + +
( , )= ,
Thus, the required point is (−7, 0) + +
8. Find the values of y for which the distance between the points P(2, – 3) and Q(10, y) is
10 units. The mid-point of a line segment divides the line segment
(x , y ) = (2, −3) , (x , y ) = (10, y), d = 10 in the ratio 1: 1. Then the coordinates of the midpoint
Formula: d = (x − x ) +(y − y ) of the line segment,
10 = (10 − 2) +( − (−3)) = (8) +( + 3) 2 -3 10 y + +
10 = 64 + ( + 3) ⇒ 100 − 64 = ( + 3) ( , )= ,
⇒ ( + 3) = 36 ⇒ + 3 = ±√36 ⇒ + 3 = ±6
⇒ y = 6 − 3 = 3 or x = −6 − 3 = −9

P a g e 138 | 159 Available in ykoyyur.blogspot.com P a g e 139 | 159 Available in ykoyyur.blogspot.com


SSLC Mathematics Solutions – Part -1 YK SSLC Mathematics Solutions – Part -1 YK
Example 6 : Find the coordinates of the point which divides the line segment joining the Example 9 : Find the ratio in which the y-axis divides the line segment joining the
points (4, – 3) and (8, 5) in the ratio 3 : 1 internally. points (5, – 6) and (–1, – 4). Also find the point of intersection.
(x , y ) = (4, −3), (x , y ) = (8,5), m : m = 3: 1 We know that a point on the Y − axis is of the form (0, y). Let the ratio be k : 1
( ) ( )
x= = = = =7
4 -3 8 5 A(x , y ) = (5, −6) , B(x , y ) = (−1, −4), m = k, m = 1
( ) ( )
y= = = = =3 (x, y) = ,
Therefore the required point is (7,3) ( ) ( ) ( ) ( )
(0, y) = ,
Example 7 : In what ratio does the point (– 4, 6) divide the line segment joining the
points A(– 6, 10) and B(3, – 8)?
⇒0= ⇒ −k + 5 = 0 ⇒ k = 5 ⇒ The ratio is 5: 1
P(x, y) = (−4, 6), A(x , y ) = (−6, 10) , B(x , y ) = (3, −8), m =? , m =?
( ) ( )
y= = = =
(x, y) = ,
∴ The coordinates of the point of intersection 0,
( ) ( ) ( ) ( )
(−4,6) = , Example 10 : If the points A(6, 1), B(8, 2), C(9, 4) and D(p, 3) are the vertices of a
parallelogram, taken in order, find the value of p.
−4 = Or 6=
Solution: We know that diagonals of a parallelogram bisect each other.
−4m − 4m = 3m − 6 m So, the coordinates of the mid-point of AC = coordinates of the mid-point of BD
+ +
−4m − 3m = −6 m + 4m The coordinates of the Midpoint = ,
−7m = −2m +
,
+
=
+
,
+
⇒ =
+

= = ⇒ m : m = 2: 7
30 = 2p + 16 ⇒ 2p = 30 − 16 ⇒ p = ⇒ p=7
We should verify that the ratio satisfies the y-coordinate also.
( ) ( )
= = = =6
Exercise 7.2
Therefore, the point (– 4, 6) divides the line segment joining the points A(– 6, 10) and
B(3, – 8) in the ratio 2 : 7 1. Find the coordinates of the point which divides the join of (–1, 7) and (4, –3) in the
Example: Find the coordinates of the points of trisection (i.e., points dividing in three equal ratio 2 : 3.
parts) of the line segment joining the points A(2, – 2) and B(– 7, 4). Let the Coordinates of the Points be(x,y)
Let P and Q be the trisection points of AB. ⇒ AP = PQ = QB m : m = 2: 3 (x , y ) = (−1, 7), (x , y ) =
(4, −3) , -1 7 4 -3
Therefore, P divides AB internally in the ratio 1 : 2. Therefore, the coordinates of P, by applying
the section formula, (x, y) = ,
A(x , y ) = (2, −2) , B(x , y ) = (−7, 4) =
( ) ( )
,
( ) ( )
= , = , ⇒ (x, y)= (1 , 3)
m = 1, m = 2
m x +m x m y +m y 2. Find the coordinates of the poin ts of trisection of the line segment join ing (4, –1) and
P(x, y) = 1 2 2 1 , 1 2 2 1
m1 m2 m1 m2 (–2, –3).
( ) ( ) ( ) ( ) Let P and Q are the trisection points of AB
= , = , -1 7 4 -3
⇒ AP = PQ = QB
∴ The point P divides AB internally in the ratio 1 : 2
= , = (−1, 0)
A(x , y ) = (4, −1) , B(x , y ) = (−2, −3),
m = 1, m = 2
Now, Q also divides AB internally in the ratio 2 : 1. So, the coordinates of Q are
∴ The coordinates of P is,
A(x , y ) = (2, −2) , B(x , y ) = (−7, 4)
m = 2, m = 1 P(x, y) = ,
m x +m x m y +m y
Q(x, y) = 1 2 2 1 , 1 2 2 1 =
( ) ( )
,
( ) ( )
m1 m2 m1 m2

( ) ( ) ( ) ( ) = , = , = 2,
= , = , = , = (−4, 2)
The point Q divides AB internally in the ratio 2 : 1
Therefore, the coordinates of the points of trisection of the line segment joining A and B are A(x , y ) = (4, −1) , B(x , y ) = (−2, −3) ; m1 = 2, m2 = 1
(–1, 0) and (– 4, 2).

P a g e 140 | 159 Available in ykoyyur.blogspot.com P a g e 141 | 159 Available in ykoyyur.blogspot.com


SSLC Mathematics Solutions – Part -1 YK SSLC Mathematics Solutions – Part -1 YK
Q(x, y) = , [Using section formula] B(6, – 8) in the ratio 2 : 7
5. Find the ratio in which the line segment joining A(1, – 5) and B(– 4, 5) is divided by thex-
( ) ( ) ( ) ( )
= , = , = , = 0, axis. Also find the coordinates of the point of division.
We know that a point on the X − axis is of the form (x, 0) Let the ratio be k ∶ 1
3. To conduct Sports day activities in your A(x , y ) = (1, −5) , B(x , y ) = (−4, 5) m1 = k, m2 = 1
rectangular shaped school groun ABCD,
(x, y) = ,
lines have been drawn with chalk powder at
( ) ( ) ( ) ( )
a distance of 1m each. 100 flower pots have (x, 0) = ,
been placed at a distance of 1m from each
other along AD, as shown in fig 7.12. 0=
Niharika runs th the distance AD on the 5k − 5 = 0 ⇒5k = 5
k = 1, the ratio is 1: 1
2nd line and posts a green flag. Preet runs ( ) ( )
x= = =
th the distance AD on the eight line and
−3
posts a red flag. What is the distance ∴ The coordinates of the point of division = , 0
2
between both the flags? If Rashmi has post 6. If (1, 2), (4, y), (x, 6) and (3, 5) are the vertices of a parallelogram taken in order, find
a blue flag exactly halfway between the line x and y.
segment joining the two flags, where should Solution:Let A(1,2), B(4,y), C(x,6) and D(3,5) are the vertices of the parallelogram.
she post her flag? Since ABCD is a parallelogram
Solution: The distance of green flag Therefore diagonals AC and BD bisects each other.
posted by Niharika on the 2 nd line So, the coordinates of both AC and BD are same.
= × AD = × 100 = 25 m ∴ Mid point of AC = Mid point of BD = ,
The distance of red flag posted by Preet on the 8th line
1 1 , = ,
= × AD = × 100 = 20 m
5 5 , = ,
Coordinates of Green flag = (2,25) = (x , y )
Coordinates of red flag = (8,20) = (x , y ) 2 25 8 20 = , =
The distance between flags d = x+ 1 = 7, 5+ y = 8
(x − x ) +(y − y ) x = 7−1, y= 8−5
d = (8 − 2) +(20 − 25) = (6) +(−5) = √36 + 25 = √61 m x = 6, y = 3
The coordinates of blue flag, if Rashmi post in between these two flags be 7. Find the coordinates of a point A, where AB is the diameter of a circle whose centre is
(2, – 3) and B is (1, 4).
(x, y) = , = , = , = (5 , 22.5)
The center of the Circle is the mid-point of the
4. Find the ratio in which the line segment joining the points (– 3, 10) and (6, – 8) is divided diameter
by (– 1, 6). ∴ (x, y) = (2, −3), A(x , y ) =? , B(x , y ) = (1, 4)
P(x, y) = (−1, 6), A(x , y ) = (−3, 10) , B(x , y ) = (6, −8) , m1 =?, m2 =?
(x, y) = ,
(x, y) = ,
) ( ) ( ) ( )
(2, −3) = ,
(−1,6) = , -3 10 6 -8 1 + x1 4 + y1
= 2, = −3
−1 = Or 6 = 1 + x = 4, 4 + y = −6
−m − m = 6m − 3 m x = 4 − 1, y = −6 − 4
x = 3, y = −10
−m − 6m = −3 m + m
∴ The coordinates of a point A is (3, −10)
−7m = −2m
= = 8. If A and B are (-2, -2) and (2, -4) respectively, find the coordinates of P such that AP
m : m = 2: 7 We should verify that the ratio satisfies the y-coordinate also = AB and P lies on the line segment AB
( ) ( )
= = = =6
Given AP = AB
Therefore, the point (– 1, 6) divides the line segment joining the points A(– 3, 10) and

P a g e 142 | 159 Available in ykoyyur.blogspot.com P a g e 143 | 159 Available in ykoyyur.blogspot.com


SSLC Mathematics Solutions – Part -1 YK SSLC Mathematics Solutions – Part -1 YK
P divides AB in the ratio 3:4 Example 11:Find the area of a triangle whose vertices
⇒ AP: PB = 3: 4 are (1, –1),(– 4, 6) and (–3, –5).
Q(x, y) = , -2 -2 2 -4 A (1, -1), B(-4, 6) ªÀÄvÀÄÛ C (-3, -5)

( ) ( ) ( ) ( ) Area = [ ( − )+ ( − )+ ( − )]
= , = , = ,
= 1(6 − (−5)) + (−4) −5 − (−1) + (−3)(−1 − 6)
9. Find the coordinates of the points which divide the line segment joining A(– 2, 2) and = [1(6 + 5) + (−4)(−5 + 1) + (−3)(−7)]
B(2, 8) into four equal parts
The point X divides AB in the ratio 1: 3 = [11 + 16 + 21]
The coordinates of X is, = (48) = 24
(x, y) = , -2 2 2 8
Area of the triangle is = 24 Square units
( ) ( ) ( ) ( )
Example 12 : Find the area of a triangle formed by
= , = , = , = −1, the points A(5, 2), B(4, 7) and C (7, – 4).
A (5, 2), B (4, 7) and C (7, -4)
The point Y is the mid-point of AB
The coordinates of Y Area = [ ( − ) + ( − ) + ( − )]
(x, y) = , = , = , = (0, 5) = [5(7 − (−4)) + 4(−4 − 2) + 7(2 − 7)]
= [5(7 + 4) + 4(−6) + 7(−5)]
The point Z divides AB in the ratio 3: 1
The coordinates of Z is, = [55 − 24 − 35]
(x, y) = , = (55 − 59)
( ) ( ) ( ) ( ) = (−4) = −2
= , = , = , = 1,
Since area is a measure, which cannot be negative, we
10. Find the area of a rhombus if its vertices are (3, will take the numerical value of – 2, i.e., 2. Therefore,
0), (4, 5), (– 1, 4) and (– 2, – 1) taken in the area of the triangle = 2 square units.
order[Hint: Area of rhombus = (product of its Example 13 : Find the area of the triangle formed by the points P(–1.5, 3), Q(6, –2)
diagonals)] and R(–3, 4).
AC = (−1 − 3) +(4 − 0) = (−4) +(4) Area of the triangle= [ ( − ) + ( − ) + ( − )]
= √16 + 16 = √16 × 2 = 4√2 = [(−1.5)(−2 − 4) + 6(4 − 3) + (−3)(3 − (−2))]
BD = (−2 − 4) +(−1 − 5) = (−6) +(−6) = [(−1.5)(−6) + 6(1) + (−3)(3 + 2)]
= √36 + 36 = √36 × 2 = 6√2 = [9 + 6 − 15] = (15 − 15)
The area of the rhombus = × 4√2 × 6√2
= (0) = 0

= = 12(2) = 24 square units. If the area of a triangle is 0 square units, then its vertices
7.4 Area of a Triangle will be collinear.
Area of triangle = × base × height
Example 14 : Find the value of k if the points A(2, 3), B(4, k) and C(6, –3) are
By Heron’s Formula Area of the triangle = s(s − a)(s − b)(s − c) , Here, s = collinear.
a, b and c are the sides of the triangle. Since the given points are collinear, the area of the triangle formed by them must be 0, i.e.,
We could find the lengths of the three sides of the triangle using distance formula. But this [ ( − ) + ( − ) + ( − )] = 0
could be tedious, particularly if the lengths of the sides are irrational number. Then we can
[2( − (−3)) + 4(−3 − 3) + 6(3 − )] = 0
use the following formula to find the area of the triangle.
[2( + 3) + 4(−6) + 6(3 − )] = 0
Area of the triangle = [ ( − )+ ( − )+ ( − )] [2 + 6 − 24 + 18 − 6 ] = 0
(−4 ) = 0 ⇒ k = 0

P a g e 144 | 159 Available in ykoyyur.blogspot.com P a g e 145 | 159 Available in ykoyyur.blogspot.com


SSLC Mathematics Solutions – Part -1 YK SSLC Mathematics Solutions – Part -1 YK
Example 15 : If A(–5, 7), B(– 4, –5), C(–1, –6) and D(4, 5) are the vertices of a [8(−4 − (−5)) + k(−5 − 1) + 2(1 − (−4))] = 0
quadrilateral, find the area of the quadrilateral ABCD.
[8(−4 + 5) + k(−6) + 2(1 + 4)] = 0
By joining B to D, we will get two triangles ABD and BCD
∴ Area ∆ABD = [x (y − y ) + x (y − y ) + x (y − y )] [8(1) + k(−6) + 2(5)] = 0
= [(−5)(−5 − 5) + (−4)(5 − 7) + 4(7 − (−5))] [8 − 6k + 10] = 0
= [(−5)(−10) + (−4)(−2) + 4(7 + 5)] = [50 + 8 + 48] (−6k + 18) = 0
= (106) = 53 Sq.units −6k = −18 ⇒ k = =3
1 3. Find the area of the triangle formed by joining the mid-points of the sides of the triangle
∴ Area ∆BCD = [x (y − y ) + x (y − y ) + x (y − y )]
2 whose vertices are (0, –1), (2, 1) and (0, 3). Find the ratio of this area to the area of the
= [(−4)(−6 − 5) + (−1)(5 − (−5)) + 4(−5 − (−6))] given triangle.
= [(−4)(−11) + (−1)(5 + 5) + 4(−5 + 6)] Let A(0, -1), B(2, 1) and C(0, 3) be the
vertices of the triangle ABC
= [44 − 10 + 4] = (38) = 19 Sq.units
D, E and F are the mid-point of AB,BC and AC
Therefore the area of quadrilateral ABCD = 53 + 19 = 72 Sq.units The coordinates of D
Exercise 7.3 (x, y) = , = ,
1. Find the area of the triangle whose vertices are : = , = (1, 0)
i) (2, 3), (-1, 0), (2, -4) ii) (-5, -1), (3, -5) (5, 2) The coordinates of E
i) (2, 3), (-1, 0), (2, -4) x + x y + y
(x, y) = 2 1 , 2 1 =
0+2
,
3+ 1

Area = [ ( − ) + ( − ) + ( − )] 2 4
= , = (1, 2)
= [2(0 − (−4)) + (−1)(−4 − 3) + 2(3 − 0)]
The coordinates of F
= [2(4) + (−1)(−7) + 2(3)] = [8 + 7 + 6] = (21) = Sq.units (x, y) = , = ,
ii) (-5, -1), (3, -5) (5, 2)
= 0 , = (0, 1)
Area = [x (y − y ) + x (y − y ) + x (y − y )]
The area of ∆DEF with vertices D(1, 0) , E(1, 2) and F(0, 1)
= [(−5)(−5 − 2) + 3(2 − (−1)) + 5(−1 − (−5))] 1
= [ ( − ) + ( − ) + ( − )]
= [(−5)(−7) + 3(2 + 1) + 5(−1 + 5)] = [35 + 9 + 20] = (64) 2
= [1(2 − 1) + 1(1 − 0) + 0(0 − 2)] = [1(1) + 1 + 0]
= 32 Sq.units.
2. In each of the following find the value of ‘k’, for which the points are collinear. = [1 + 1] = (2) = 1 Sq.units
i) (7, -2), (5, 1), (3, k) ii) (8, 1), (k, -4) (2, -5) The area of given triangle = [ ( − )+ ( − )+ ( − )]
i) (7, -2), (5, 1), (3, k)
= [0(1 − 3) + 2(3 − (−1)) + 0(−1 − 1)] = [0 + 2(3 + 1)) + 0]
Since the given points are collinear, the area of the triangle formed by them must be 0, i.e.,
[x (y − y ) + x (y − y ) + x (y − y )] = 0 = [0 + 8 + 0] = (8) = 4 Sq.units
The ratio of the ∆ABC and ∆DEF = 4: 1
[7(1 − k) + 5(k − (−2)) + 3(−2 − 1)] = 0
4. Find the area of the quadrilateral whose vertices, taken in order, are (– 4, – 2), (– 3, – 5),
[7(1 − k) + 5(k + 2) + 3(−3)] = 0 (3, – 2) and (2, 3).
[7 − 7k + 5k + 10 − 9] = 0 A(-4, -2), B(-3, -5), C(3, -2) and D(2, 3)
(−2k + 8) = 0 By joining B to D, we will get two triangles ABD
and BCD
−2k = −8 ⇒ k = =4 ∴ Area ABD
ii) (8, 1), (k, -4) (2, -5)
Since the given points are collinear, the area of the triangle formed by them must be 0, i.e.,
[x (y − y ) + x (y − y ) + x (y − y )] = 0

P a g e 146 | 159 Available in ykoyyur.blogspot.com P a g e 147 | 159 Available in ykoyyur.blogspot.com


SSLC Mathematics Solutions – Part -1 YK SSLC Mathematics Solutions – Part -1 YK
1 7.5 Summary
= [x (y − y ) + x (y − y ) + x (y − y )]
2 1. The distance between two given points d = (x − x ) +(y − y )
= [(−4)(−5 − 3) + (−3)(3 − (−2)) + 2(−2 − (−5))]
2. The distance from the orgin to the given points d = x +y
= [(−4)(−8) + (−3)(3 + 2) + 2(−2 + 5)]
= [32 − 15 + 6] = (23) = Sq.units 3. Section formula :P is the point which divides the line segment joining the points
∴ AreaBCD A(x , y ) and B(x , y )
1 If the point P divides the line in the ratio m: n then the coordinates of P
= [x (y − y ) + x (y − y ) + x (y − y )]
2 P(x, y) = ,
= [(−3)(−2 − 3) + 3(3 − (−5)) + 2(−5 − (−2))] 4. If P is the midpoint of AB, it divides in the ratio 1:1
= [(−3)(−5) + 3(3 + 5) + 2(−5 + 2)] P £À ¤zÉÃð±ÁAPÀUÀ¼ÀÄ P(x, y) = ,
= [15 + 24 − 6] = (33) = Sq.units 5. Area of triangle = [x (y − y ) + x (y − y ) + x (y − y )]
∴ Area of ABCD = + = = 28 Sq.units

5. You have studied in Class IX, (Chapter 9, Example


3), that a median of a triangle divides it into two
triangles of equal areas. Verify this result for
ABC whose vertices area (4 – 6), B(3, –2) and
C(5, 2).
Coordinates of D, the midpoint of BC
+ + + −
( , )= , = ,
= , =( , )
Area∆ABD
1
= [x (y − y ) + x (y − y ) + x (y − y )]
2
= [4(−2 − 0) + 3(0 − (−6)) + 4(−6 − (−2))]
= [4(−2) + 3(6) + 4(−6 + 2)]
= [−8 + 18 − 16] = (18 − 24)
= (−6) = −3 Sq.units
Since area is a measure, which cannot be negative, we will take the numerical value of – 3,
i.e., 3. Therefore, the area of the triangle = 3square units.
Area ∆ADC = [x (y − y ) + x (y − y ) + x (y − y )]
= [4(0 − 2) + 4(2 − (−6)) + 5(−6 − 0)]
= [4(−2) + 4(2 + 6) + 5(−6)] = [−8 + 32 − 30] = (−6) = −3 ZÀz g À ª
À iÀ Á£ÀU¼
À ÀÄ
Since area is a measure, which cannot be negative, we will take the numerical value of – 3,
i.e., 3. Therefore, the area of the triangle = 3square units.
Hence, the mid-point of a triangle divides it into two triangles of equal areas.

P a g e 148 | 159 Available in ykoyyur.blogspot.com P a g e 149 | 159 Available in ykoyyur.blogspot.com


SSLC Mathematics Solutions – Part -1 YK SSLC Mathematics Solutions – Part -1 YK
iii)If r = 2 ,
Real Numbers a = 4q + 2 ⇒ a = 2(2q + 1) This is divisible by 2. Therefore this is an even number.
iv) If r = 3 ,
Euclid’s division algorithm, as the name suggests, has to do with divisibility of integers. Stated a = 4q + 3 ⇒ a = 2(2q+1) + 1 this is not divisible by 2. Therefore this is an odd number.
simply, it says any positive integer a can be divided by another positive integer b in such a way ∴ Any positive integer is of the form 4q + 1 Or 4q + 3 where q is some integer.
that it leaves a remainder r that is smaller than b. Example 4 : A sweetseller has 420 kaju barfis and 130 badam barfis. She wants to
8.2 Euclid’s Division Lemma stack them in such a way that each stack has the same number, and they take up the
Theorem 8.1 least area of the tray. What is the number of that can be placed in each stack for this
purpose?
(Euclid’s Division Lemma) : Given positive integers a and b, there 420 = 130 x 3 + 30
exist unique integers q and r satisfying a = bq + r, 0 r < b. 130 = 30 x 4 + 10
30 = 10 x 3 + 0
A lemma is a proven statement used for proving another statement So, the HCF of 420 and 130 is 10.
Example 1 : Use Euclid’s algorithm to find the HCF of 4052 and 12576. Therefore the sweetseller can make stacks of 10 for both kinds of burfi
4052 12576 3 420 4052 9 272 420 1 Exercise 8.1
12156 3780 272 1. Use Euclid’s division algorithm to find the HCF of :
420 272 148 (i) 135 and 225 (ii) 196 and 38220 (iii) 867 and 255
12576 = 4052 x 3 + 420 4052 = 420 x 9 + 272 420 = 272 x 1 + 148 (i) 135 ªÀÄvÀÄÛ 225

148 272 1 135 225 1 90 135 1 45 90 2


124 148 1 24 124 5
135 90 90
148 124 120 90 45 0
124 24 4
225 = 135 x 1 + 90 135 = 90 x 1 + 45 90 = 45 x 2 + 0
272 = 148 x 1 + 124 148 = 124 x 1 + 24 124 = 24 x 5 + 4 ∴ HCF = 45
(ii) 196 ªÀÄvÀÄÛ 38220
4 24 6
196 38220 195
24 24 = 4 x 6 + 0 ∴ The HCF of 4052 and 12576 is 4 38220 38220 = 196 x 195 + 0
0 0 ∴ HCF = 196
Example 2 : Show that every positive even integer is of the form 2q, and that every (iii) 867 ªÀÄvÀÄÛ 255
positive odd integer is of the form 2q + 1, where q is some integer. 255 867 3 102 255 2 51 102 2
Solution :Let ‘a’ be a possitive even number, 765 204 102
(i) a = 2q + r here, 0≤ r < 2 ⇒ r must be 0 or 1 102 51 0
But ‘a’ is a positive even number. So r = 0
∴ a = 2q + 0 ⇒ a = 2q 867 = 255 x 3 + 102 255 = 102 x 2 + 51 102 = 51 x 2 + 0
(ii)Let ‘a’be a positive odd integer , r≠ ⇒r = 1 ∴ a = 2q + 1 ∴ HCF = 51
Example 3 : Show that any positive odd integer is of the form 4q + 1 or 4q + 3, where q is 2. Show that any positive odd integer is of the form 6q + 1, or 6q + 3, or 6q + 5, where q is
some integer. some integer.
Let a' and b are be a positive integer and a > b Let a be a positive integer and let b = 6
By Division algorithm,
Therefore by division algorithm
a = bq + r ; 0≤ r < b
If b = 4, a = bq + r [ 0 ≤ r < b ]
a = (4x2) + r, 0≤ r < 4 ∴ r = 0, 1, 2, 3 ⇒ a = 6q + r [ r = 0,1,2,3,4,5 ]
i) If r = 0, (i) if r = 0 , a = 6q ⇒ divisible by 2. ∴ 6q is even
a = 4q ⇒a = 2(2q) this is divisible by 2. Therefore this is an even number. (ii) if r = 1 , a = 6q + 1 ⇒ not divisible by 2 ∴ 6q + 1 is odd
ii) If r = 1, (iii) if r = 2 , a = 6q + 2 ⇒ divisible by 2. ∴ 6q + 2 is even
a = 4q + 1 ⇒ a = 2(2q) + 1 this is not divisible by 2. Therefore this is an odd number.

P a g e 150 | 159 Available in ykoyyur.blogspot.com P a g e 151 | 159 Available in ykoyyur.blogspot.com


SSLC Mathematics Solutions – Part -1 YK SSLC Mathematics Solutions – Part -1 YK
(iv) if r = 3, a = 6q + 3 ⇒ not divisible by 2 . ∴ 6q + 3 is odd 8.3 The Fundamental Theorem of Arithmetic
(v) if r = 4 , a = 6q + 4 ⇒ divisible by 2. ∴ 6q + 4 is even
Theorem 8.2 (Fundamental Theorem of Arithmetic) : Every composite number can
(iv) if r = 5, a = 6q + 5 ⇒ not divisible by 2 . ∴ 6q + 5 is odd be expressed ( factorised ) as a product of primes, and this factorisation is unique,
any positive odd integer is of the form 6q + 1, or 6q + 3, or 6q + 5, where q is some integer. apart from the order in which the prime t h e p r i m e f a c t o r s o c c u r
3. An army contingent of 616 members is to march behind an army band of 32 members in
a parade. The two groups are to march in the same number of columns. What is the The Fundamental Theorem of Arithmetic says that every composite number can be
maximum number of columns in which they can march? factorised as a product of primes. Actually it says more. It says that given any composite
(iii) 867 ªÀÄvÀÄÛ 255 number it can be factorised as a product of prime numbers in a‘unique’ way, except for
the order in which the primes occur. That is, given any composite number there is one and
32 616 19 8 32 4 only one way to write it as a product of primes, as long as we are not particular about the
608 32 order in which the primes occur. So, for example, we regard 2 × 3 × 5 × 7 as the same
8 0 as 3 × 5 × 7 × 2, or any other possible order in which these primes are written.
616= 32 x 19 + 8 32 = 8 x 4 + 0 Example 5 : Consider the numbers 4n, where n is a natural number. Check whether
∴ HCF = 8 there is any value of n for which 4n ends with the digit zero.
They can march maximum 8 columns. Solution: If the number 4 n, for any n, were to end with the digit zero, then it would be divisible
4. Use Euclid’s division lemma to show that the square of any positive integer is either of by 5. That is, the prime factorisation of 4 n would contain the prime 5. This is not possible
the form 3m or 3m + 1 for some integer m.
[Hint : Let x be any positive integer then it is of the form 3q, 3q + 1 or 3q + 2. Now square because 4n = (2)2n; so the only prime in the factorisation of 4 n is 2.
each of these and show that they can be rewritten in the form 3m or 3m + 1.] So, the uniqueness of the Fundamental Theorem of Arithmetic guarantees that there are no
Any positive integer divisible by 3, we get the remainder 0,1 or 2 other primes in the factorisation of 4n.
⇒ a is of the form 3q, 3q + 1 or 3q + 2 So, there is no natural number n for which 4 n ends with the digit zero.
i) if a = 3q, Example 6 : Find the LCM and HCF of 6 and 20 by the prime factorisation method.
a2 = (3q)2 = 9q2 = 3(3q2) = 3m ( m = 3q2) Solution: 6 = 2 1 x 31
ii) if a = 3q + 1, 20 = 2 x 2 x 5 = 2 2 x 51
a2 = (3q + 1)2 = 9q 2 + 6q + 1 = 3(3q 2 + 2) + 1 = 3m+ 1 ( m = 3q2 + 2) HCF (6,20) = 2 and LCM (6, 20) = 2 x 2 x 3 x 5 = 60
iii)if a = 3q + 2 , Any two positive integers a and b, HCF (a, b) × LCM (a, b) = a × b.
a2 = (3q + 2)2 = 9q 2 + 12q + 4 ⇒ a2 = 9q2 + 12q + 3 + 1
⇒ 3(3q2 + 4q +1) + 1 = 3m + 1 (m = 3q2 + 4q +1) We can use this result to find the LCM of two positive integers, if we have already found the
From (i) (ii) and (iii) HCF of the two positive integers.
We say, square of any positive integer is either of the form 3m or 3m + 1 for some integer m. Example 7 : Find the HCF of 96 and 404 by the prime factorisation method. Hence, find
5. Use Euclid’s division lemma to show that the cube of any positive integer is of the form their LCM.
9m, 9m + 1 or 9m + 8 Solution: We can write the prime factors of 96 and 404 are as follows
Let a be a positive inteer and let b = 3 96 = 25 x 3 ; 404 = 2 2 x 101
By Euclid’s division algorithm, HCF(96,404) = 2 2 = 4
a = bq + r [ 0 ≤ r < b ] ⇒ a = 3q + r [ r = 0,1,2 ] ∴ LCM (96, 404) = = 9696
(i)if r = 0 , a = 3q
Example 8 : Find the HCF and LCM of 6, 72 and 120, using the prime factorisation
⇒a3 = (3q)3 ⇒a3 = 9q 3 ⇒ 9m [ ∵ m = q3 ] method.
(ii)if r = 1 , a = 3q + 1 6 = 2 x 3 ; 72 = 23 x 3 2; 120 = 23 x 3 x 5
a3 = (3q + 1)3 ⇒ a3 = 27q3 + 3x(3q)2x1 + 3x3qx1 + 1 ⇒ a3 = 27q 3 + 27q2 + 9q + 1 ∴ HCF (6, 72, 120) = 21 x 31 = 2x3 = 6
⇒ a3 = 9(3q 3 + 9q + q) + 1 ⇒a3 = 9m + 1 [ ∵ m = 3q3 + 9q + q] ∴ LCM (6, 72, 120) = 2 3 x 32 x 5 1 = 8 x 9 x 5 = 360
(iii)if r = 2, a = 3q + 2
a3 = (3q + 2)3 ⇒ a3 = 27q3 + 54q2 + 18q + 8
⇒ a3 = 9(3q 3+ 6q2 + 2q) + 8 ⇒ a3 = 9m + 8 [ ∵ m = 3q3+ 6q 2 + 2q]
∴ We say, the cube of any positive integer is of the form 9m, 9m + 1 or 9m + 8

P a g e 152 | 159 Available in ykoyyur.blogspot.com P a g e 153 | 159 Available in ykoyyur.blogspot.com


SSLC Mathematics Solutions – Part -1 YK SSLC Mathematics Solutions – Part -1 YK

Exercise 8.2 If the number 6n, for any n, were to end with the digit zero, then it would be divisible by 5. That
is, the prime factorisation of 4n would contain the prime 5. This is not possible because the
1. Express each number as a product of its prime factors:
prime factors of 6 are 2 and 3.
(i) 140 (ii) 156 (iii) 3825 (iv) 5005 (v) 7429 Therefore 5 is not a factor of 6. ⇒6n = (2 x 3)n
(i) 140 = 2 × 2 × 5 × 7 = 22 × 5 × 7
(ii) 156 = 2 × 2 × 3 × 13 = 22 × 3 × 13 So, there is no natural number n for which 6 n ends with the digit zero.
(iii) 3825 = 3 × 3 × 5 × 5 × 17 = 32 × 52 × 17 Explain why 7 × 11 × 13 + 13 and 7 × 6 × 5 × 4 × 3 × 2 × 1 + 5 are composite numbers
(iv) 5005 = 5 × 7 × 11 × 13 6. 7 × 11 × 13 + 13 =13 (7x11 + 1) =13(77 + 1 ) =13 (78) =13x2x3x13
The product of two or more than two prime numbers is a composite number.
(v) 7429 = 17 × 19 × 23
Therefore 7 x 11 x 13 + 13 is a composite number.
2. Find the LCM and HCF of the following pairs of integers and verify that LCM × HCF = 7×6×5×4×3×2×1+5
product of the two numbers. =5(7 × 6 × 4 × 3 × 2 × 1 +1) =5(1008 + 1) =5(1009)
(i) 26 and 91 (ii) 510 and 92 (iii) 336 and 54. The product of two or more than two prime numbers is a composite number.
(i) 26 = 2 × 13 Therefore 7 × 6 × 5 × 4 × 3 × 2 × 1 + 5 is a composite number
91 =7 × 13 7. There is a circular path around a sports field. Sonia takes 18 minutes to drive one round
HCF = 13; LCM = 2 × 7 × 13 =182 of the field, while Ravi takes 12 minutes for the same. Suppose they both start at the
Product of two numbers = 26 × 91 = 2366 same point and at the same time, and go in the same direction. After how many minutes
LCM x HCF = 13 × 182 = 2366 will they meet again at the starting point?
∴ LCM x HCF = Product of two numbers To find the time they meet again in the same point, we have to find the LCM of time
18 = 2 × 3 × 3; 12 = 2 × 2 × 3
(ii) 510 = 2 × 3 × 5 × 17 LCM = 2 × 2 × 3 × 3 = 36
92 =2 × 2 × 23 Therefore after 36 minutes they meet again at the starting point.
HCF = 2; LCM = 2 × 2 × 3 × 5 × 17 × 23 = 23460
Product of two numbers = 510 × 92 = 46920
8.4 Revisiting Irrational Numbers
LCM x HCF = 2 × 23460 = 46920
A number which can not be expressed in the form of is called irrational number.Here,
∴ LCM x HCF = Product of two numbers
p, q Z, q ≠ 0
(iii) 336 = 2 × 2 × 2 × 2 × 3 × 7
Theorem 8.3: Let p be a prime number. If p divides a 2, then p divides a, where a is
54 = 2 × 3 × 3 × 3
a positive integer.
HCF = 2 × 3 = 6; LCM = 2 × 2 × 2 × 2 × 3 × 3 × 3 × 7 =3024
Theorm 8.4: √ is irrational.
Product of two numbers = 336 × 54 =18144
LCM x HCF = 6 × 3024 = 18144 Proof: Let us assume, to the contrary, that √2 is rational.
∴ LCM x HCF = Product of two numbers ⇒ √2 = [ p,q∈ Z, q ≠ 0 and (p,q)=1 ]
So, there is no other common factors for p and q other than 1
3. Find the LCM and HCF of the following integers by applying the prime factorisation
NowÀ, √2 = ⇒ √2q = p Squaring on both sides we get,
method (i) 12, 15 and 21 (ii) 17, 23 and 29 (iii) 8, 9 and 25
(i) 12 = 2 × 2 × 3; 15 =3 × 5; 21 =3 × 7 √2 = p2 ⇒ 2q2 = p 2 (1)
HCF = 3; LCM = 2 × 2 × 3 × 5 × 7 = 420 ⇒ 2 divides p2 ⇒ 2, divides p . [ By theorem]
(ii) 17 = 1 × 17; 23 = 1 × 23; 29 = 1 × 29
∴ Let p = 2m,
HCF = 1; LCM = 1 × 17 × 19 × 23 = 11339
(iii) 8 =1 × 2 × 2 × 2; 9 =1 × 3 × 3; 25 =1 × 5 × 5 (1) ⇒ 2q 2 = (2m) ⇒q2 = 2m2
HCF = 1; LCM = 1 × 2 × 2 × 2 × 3 × 3 × 5 × 5 = 1800 ⇒2,divides q 2 ⇒ 2, divides q [By theorem]
4. Given that HCF (306, 657) = 9, find LCM (306, 657). ∴ 2 is the common factor for both p and q
LCM x HCF = Product of two numbers
This contradicts that there is no common factor of p and q .
×
∴ LCM (306, 657) = = 22338 Therefore our assumption is wrong. So, √2 is a an irrational number.
5. Check whether 6n can end with the digit 0 for any natural number n. Example 9 : Prove that √ is irrational.
Here, n is a natural number.
Proof: Let us assume, to the contrary, that √3 is rational.

P a g e 154 | 159 Available in ykoyyur.blogspot.com P a g e 155 | 159 Available in ykoyyur.blogspot.com


SSLC Mathematics Solutions – Part -1 YK SSLC Mathematics Solutions – Part -1 YK
⇒ √3 = [ p,q∈ Z, q ≠ 0 and (p,q)=1 ] 2. Prove that 3 + 2√ is irrational.
So, there is no other common factors for p and q other than 1 Proof: Assume that 3 +2√5 is a rational number.
NowÀ, √3 = ⇒ √3q = p Squaring on both sides we get, ⇒3 + 2√5 = [ p,q∈ Z, q ≠ 0 and (p,q)=1 ]
√3 = p2 ⇒ 3q2 = p2 (1) ⇒2√5 = - 3 ⇒ √5 =
⇒ 3 divides p2 ⇒ 3 divides p [ By theorem]
∴ Let p = 3m, Here, is a rational number but √5 is an irrational number. This is not possible
(1) ⇒ 3q 2 = (3m) ⇒q 2 = 3m2 So, our assumption is wrong. Therefore 3 + 2√5 is an irrational number.
⇒3 divides q 2 ⇒ 3 divides q [ By theorem] 1. Prove that the following are irrationals:
∴ 3 is the common factor for both p and q , This is not possible. (i ) (ii) 7√5 (iii) 6 + √2

Therefore our assumption is wrong. So, √3 is a an irrational number. (i)

 The sum or difference of a rational and an irrational number is irrational and
Proof: Assume that is a rational number.
 The product and quotient of a non-zero rational and irrational number is √
irrational. ⇒ = [ p,q∈ Z, q ≠ 0 and (p,q)=1 ]

Example 10 : Show that 5 - √ is irrational √
⇒ = ⇒ √2 =
Proof: Assume that 5 -√3 is a rational number.
⇒5 - √3 = [ p,q∈ Z, q ≠ 0 and (p,q)=1 ] Here, is a rational number, but √2 is an irrational. This is impossible.

⇒5- = √3 ⇒ = √3 Therefore our assumption is wrong. ∴ is an irrational number.



(ii) 7√
Here, is a rational number but √3 is an irrational number. This is not possible
Proof: Assume that 7√5 is a rational number.
So, our assumption is wrong. Therefore 5 - √3 is an irrational number. 7√5 = [ p,q∈ Z, q ≠ 0 and (p,q)=1 ]
Example 11 : Show that 3√ is irrational.
⇒√5 =
Proof: Assume that 3√2 is a rational number.
Here, is a rational number, but √5 is an irrational. This is impossible.
⇒3√2 = [ p,q∈ Z, q ≠ 0 and (p,q)=1 ]
Therefore our assumption is wrong. ∴ 7√5 is an irrational number.
⇒ √2 =
(iii) 6 + √
Here, s a rational number but √2 is an irrational number. This is not possible Proof: Assume that 6 + √2 is a rational number
So, our assumption is wrong. Therefore 3√2 is an irrational number DzÀÝjAzÀ 3√2 MAzÀÄ ⇒6 + √2 = [ p,q∈ Z, q ≠ 0 and (p,q)=1 ]
⇒ √2 = – 6 ⇒√2 =
Exercise 8.3 Here, is a rational number, but √2 is an irrational. This is impossible.
1. Prove that √ is irrational. Therefore our assumption is wrong. ∴ 6 + √2 is an irrational number.
Proof: Let us assume, to the contrary, that √5 is rational.
⇒ √5 = [ p,q∈ Z, q ≠ 0 and (p,q)=1 ] 8.5 Revisiting Rational Numbers and Their Decimal Expansion:
So, there is no other common factors for p and q other than 1 Theorem 8.5: Let x be a rational number whose decimal expansion
NowÀ, √5 = ⇒ √5q = p , squaring on both sides we get, terminates.Then x can be expressed in the form where p and q are
coprime, and the prime factorisation of q is of the form 2 n.5 m, where n, m are
√5 = p2 ⇒ 5q2 = p2 (1) non-negative integers.
⇒ 5 divides p2 ⇒ 5 divide s p [ By theorem]
∴ Let p = 5m,
(1) ⇒ 5q 2 = (3m) ⇒q 2 = 5m2 Theorem 8.6 : Let x = be a rational number, such that the prime
⇒5 divides q 2 ⇒ 5 divides q [ By theorem] factorisation of q is of the form 2n5m, where n, m are non-negative integers.
∴ 5 is the common factor for both p and q ; this is not possible Then x has a decimal expansion which terminates.
Therefore our assumption is wrong. So, √5 is a an irrational number.

P a g e 156 | 159 Available in ykoyyur.blogspot.com P a g e 157 | 159 Available in ykoyyur.blogspot.com


SSLC Mathematics Solutions – Part -1 YK SSLC Mathematics Solutions – Part -1 YK
(iii) = = = = 0.009375
Theorem 8.7 : Let x = be a rational number, such that the prime factorisation of q
(iv) ⇒ = = 0.115
is not of the form 2n5m, where n, m are non-negative integers. Then, x has a decimal
expansion which is non-terminating repeating (recurring). (v) ⇒ = x = = 0.4
(vi) ⇒ = = 0.7
Exercise 8.4 3. The following real numbers have decimal expansions as given below. In each case,
1. Without actually performing the long division, state whether the following rational decide whether they are rational or not. If they are rational, and of the form what can
numbers will have a terminating decimal expansion or a non-terminating repeating decimal you say about the prime factors of q?
expansion (i) . (ii) 0.120120012000120000... (iii) .
(i) (ii) (iii) (iv) (v) (i) 43.123456789 - has a terminating decimal expansion.
Therefore this in a rational number and is of the form
(vi) (vii) (viii) (ix) (x)
and q is of the form 2 nx5 m ⇒ =
(i) - Factorising the denominator 3125 =5 × 5 × 5 × 5 × 5 = 20 x 5 5
Here, The factors of 3125 is of the form 2 5 So, n. m.
has a terminating decimal expansion. (ii) 0.120120012000120000...
this has non-terminating and non- repeating decimal expansion.
(ii) - Factorising the denominator 8 =2 × 2 × 2 = 23x50 Therefore this is an irrational number
Here, The factors of 8 is of the form 2n.5m . So, has a terminating decimal expansion. (iii) 43. 123456789 - this has non-terminating repeating decimal expansion.
Therefore this in a rational number and is of the form
(iii) - Factorising the denominator 455 =5 × 7 × 13
Here, The factors of 455 is 5x7x13 is not in the form 2n x 5 m Let = 43. 123456789 (1)
So, this has non-terminating repeating decimal expansion. ⇒ 1000000000x = 43123456789123456789….. (2)
(iv) - Factorising the denominator 1600 =2 × 2 × 2 ×2 × 2 × 2 × 5 × 5 = 26 × 52 (1) – (2) = 999999999x = 43123456746
Here, The factors of 1600 is of the form 2n.5m. x= this is of the form . i.e., the factors of 999999999 is not in the form 2 n x 5 m
So, has a terminating decimal expansion.
(v) - Factorising the denominator 343 = 7 × 7 × 7 = 73 Summery:
Here, The factors of 343 is not in the form 2 n × 5m  Euclid’s Division Lemma: Given positive integers a and b, there exist whole numbers q
So, this has non-terminating repeating decimal expansion. and r satisfying a = bq + r, 0 ≤ r < b
(vi) - denominator is of the form 2n x 5 m  Euclid’s division algorithm: This is based on Euclid’s division lemma. According to this,
So, has a terminating decimal expansion. the HCF of any two positive integers a and b, with a > b, is obtained as follows:
Step 1 : Apply the division lemma to find q and r where a = bq + r, 0 ≤ r < b
(vii) denominator is not in the form 2n x 5 m
Step 2 : If r = 0, the HCF is b. If r ≠ 0 apply Euclid’s lemma to b and r.
So, this has non-terminating repeating decimal expansion. Step 3 : Continue the process till the remainder is zero. The divisor at this stage will be
(viii) ⇒ = dinominator 20 x 5 1 is of the form 2 n x 5m HCF (a, b). Also, HCF(a, b) = HCF(b, r).
So, has a terminating decimal expansion.  The Fundamental Theorem of Arithmetic :
(ix) ⇒ = = dinominator 21 x 51 is of the form 2n x 5 m  Every composite number can be expressed (factorised) as a product of primes, and this
factorisation is unique, apart from the order in which the prime factors occur.
So, has a terminating decimal expansion.  If p is a prime and p divides a2, then p divides a, where a is a positive integer.
x) ⇒ = = dinominator not in the form 2 n x 5m  Let x = be a rational number, such that the prime factorisation of q is of the form 2n5 m,
So, this has non-terminating repeating decimal expansion. where n, m are non-negative integers. Then x has a decimal expansion which terminates.
2. Write down the decimal expansions of those rational numbers in Question 1 above  Let x = be a rational number, such that the prime factorisation of q is not of the form 2n5 m,
which have terminating decimal expansions.
where n, m are non-negative integers. Then, x has a decimal expansion which is non-
(i) ⇒ x = = = 0.00416
terminating repeating (recurring).
(ii) ⇒ = x = = 2.125

P a g e 158 | 159 Available in ykoyyur.blogspot.com P a g e 159 | 159 Available in ykoyyur.blogspot.com


SSLC Mathematics Solutions in English YK

Content
Part -2

Sl.No. Chapters Page No.

1 Polynomials 2 - 13

2 Quadratic Equations 14 - 31

3 Introduction to Trigonometry 32 - 47

4 Some applications of Trigonometry 48 - 57

5 Statistics 58 - 76

6 Probability 77 - 86

7 Surface area and Volume 87 - 101

P a g e 1 | 101 Available in ykoyyur.blogspot.com


SSLC Mathematics Solutions in English YK SSLC Mathematics Solutions in English YK

The graph of y = 2x + 3 is a straight line passing

Polynomials through the points (– 2, –1) and (2, 7).


The graph of y = 2x + 3 intersects X – axis at the
points , 0 ,Thus the zero of the polynomial
Degree of the polynomial: 2x+3 is the x-coordinate of the point where the
graph of y= 2x + 3 intersects the axis
p(x) is a polynomial in x, the highest power of x in p(x) is called the degree of the ∴ is the zero of the linear polynomial
polynomial p(x). y = 2x + 3
Examples: ∴The linear polynomial ax + b (a ≠ 0) has
4x + 2 is a polynomial in the variable x ofdegree 1. exactly one zero,namely the x-coordinate of the
A polynomial of degree 1 is called a linear polynomial. point where the graph of y = ax + b intersects the
2y2 – 3y + 4 is a polynomial in the variable y of degree axis
A polynomial of degree 2 is called a quadratic polynomial. (i) Quadratic Polynomials:
quadratic polynomial in x is of the form ax2 + bx + c, where a, b, c are real numbers a ≠ 0 . Example: y = x2 – 3x – 4
is a polynomial in the variable x of degree 3 x -2 -1 0 1 2 3 4 5
5x3 – 4x2 + 2 - √2 is a polynomial in the variable x of degree 3 y 6 0 -4 -6 -6 -4 0 6
A polynomial of degree 3 is called a cubic polynomial. General form of a cubic polynomial is
ax3 + bx 2 + cx + d If we locate the points listed above on a graph paper
Where a, b, c, d are real numbers and a ≠ 0 and draw the graph, it will actually look like the one
[7u6 - u4 + 4u 2 + u - 8 is a polynomial of variable x and the degree of this polynomial is 6] given in Fig. In fact, for any quadratic polynomial

Example:: √ + 1, , ax2 + bx + c, a 0, the gr a ph of t he cor r esp o


nding equation y = ax2 + bx + c has one of the
If p(x) is a polynomial in x, and if k is any real number, then the value obtained by replacing x by two shapes either op enu pwa r ds or open
k in p(x), is called the value of p(x) at x = k, and is denoted by p(k). downwards depending on whether a > 0 or a < 0.
What is the value of p(x) = x 2 – 3x – 4 when x = - 1 ? (Thesecurves are called parabolas.) –1 and 4 are
p(-1) = (-1)2 - 3(-1) - 4 = 0 the x-coordinates of the points where the graph of y
Similarlly, p(4) = (4)2 - 3(4) - 4 = 0 = x2 – 3x – 4 intersects the x - axis. Thus, the
As p(-1) = 0 and p(4) = 0 -1 and 4 are called the zeros of the polynomial x2 – 3x – 4 zeroes of the quadratic polynomial x2 – 3x –
If k is a real number such that p(k) = 0 then k is called the Zeros of the polynomial p(x) 4 are x-coordinates of the points wher e the gr

If k is the zero of the polynomial p(x) = ax + b then p(k) = ak + b = 0 ⇒ k = − aph of y = x2 – 3x – 4 intersect s the x-axis.
This fact is true for any quadratic polynomial, i.e.,
The zero of the lenear equation ax + b is − the zeroes of a quadratic polynomial ax2 + bx + c,
a 0, are precisely the x-coordinates of the points
9.2 Geometrical Meaning of the Zeroes of a Polynomial where the parabola representing y = ax2 + bx + c
intersects the x-axis
(i) Linear Polynomial
Example y = 2x + 3
x -2 2
y -1 7

P a g e 2 | 101 Available in ykoyyur.blogspot.com P a g e 3 | 101 Available in ykoyyur.blogspot.com


SSLC Mathematics Solutions in English YK SSLC Mathematics Solutions in English YK

Case (i) : Here, the graph cuts x-axis Let u s t a ke a few mor e exa mp les . C onsider t he c u bic polynomials x3 and x3 – x2.
at two distinct points A and A1. The x-
We draw the graphs of y = x3 and y = x3 – x2 in Fig. 9.7 and Fig. 9.8 respectively. Note that 0
coordinates of A and A1 are the two
zeroes of the quadratic polynomial x2 is the only zero of the polynomial x3. Also, from Fig. 9.7, you can see that 0 is the x - coordinate
+ bx + c in this case (see Fig. 9.3). of the only point where the graph of y = x3 intersects the x - axis. Similarly, since x3 – x2 = x2
(x – 1), 0 and 1 are the only zeroes of the polynomial x3 – x2 . Also, from Fig. 9.8, these values
Case (ii) : Here, the graph cuts the are the x - coordinates of the only points where the graph of y = x3 – x2 intersects the x-axis
x-axis at exactly one point, i.e., at two
coincident points. So, the two points A
and A1 of Case (i) coincide here to
become one point A (see Fig. 9.4).
The x –coordinate of A is the only zero
for the quadratic polynomial
ax2 + bx + c in this case.

Case (iii) : Here, the graph is either


completely above the x-axis or
completely below the x-axis. So, it does
not cut the x - axis at any point (see Fig.
9.5).
So, the quadratic polynomial
ax2 + bx + c has no zero in this case.

So, you can see geometrically that a quadratic polynomial can have either two distinct zeroes
or two equal zeroes (i.e., one zero), or no zero. This also means that a polynomial of degree 2
has atmost two zeroes. From the examples above, we see that there are at most 3 zeroes for any cubic polynomial. In
Cubic Polynomials: other words, any polynomial of degree 3 can have at most three zeroes
Example: y = x3 - 4x Example 1 : Look at the graphs in Fig. 9.9 given below. Each is the graph of y = p(x), where
x -2 -1 0 1 2 p(x) is a polynomial. For each of the graphs, find the number of zeroes of p(x).
y 0 3 0 -3 0
Locating the points of the table on a graph
paper and drawing the graph, we see that the
graph of y = x3 – 4x actually looks like the one
given in fig 9.6
We see from the table above that – 2, 0 and 2
re zeroes of the cubic polynomial x3
– 4x. Observe that – 2, 0 and 2 are, in fact,
the x- coordinates of the only points where
the graph of y = x3 – 4x intersects the x -
axis. Since the curve meets the x - axis in only
these 3 points, their x - coordinates are the
only zeroes of the polynomial
Solution :

P a g e 4 | 101 Available in ykoyyur.blogspot.com P a g e 5 | 101 Available in ykoyyur.blogspot.com


SSLC Mathematics Solutions in English YK SSLC Mathematics Solutions in English YK

(i) The number of zeroes is 1 as the graph intersects the x-axis at one point only. Solution: x2 + 7x + 10 = x2 + 5x + 2x + 10
(ii) The number of zeroes is 2 as the graph intersects the x - axis at two points. = x (x + 5) +2 (x + 5) = (x + 2) (x + 5)
(iii) The number of zeroes is 3 as the graph intersects the x-axis at three points
∴ The value of x2 + 7x + 10 is zero when x = -2 or x = -5
(iv) The number of zeroes is 1 as the graph intersects the x-axis at one point only.
(v) The number of zeroes is 1. as the graph intersects the x-axis at one point only. ∴ -2 and -5 are the zeros of x2 + 7x + 10
(vi) The number of zeroes is 4. as the graph intersects the x-axis at four points Sum of the zeros = (-2) + (-5) = -7 = =

Exercise 9.1 Product of the zeros = (-2) x (-5) = 10 = =

1. The graphs of y = p(x) are given in Fig. 9.10 below, for some polynomials p(x). Find the Example 3 : Find the zeroes of the polynomial x2 – 3 and verify the relationship
between the zeroes and the coefficients
number of zeroes of p(x), in each case.
Solution: a2 – b2 = (a - b) (a + b)
∴ x2- 3 = (x - √3 ) (x + √3 )
∴ √3 and -√3 are the zeros of x2 - 3
Sum of the zeros = √3 + -√3 = 0 =
Product of the zeros = (√3 )(- √3 ) = -3 = =
Example 4 : Find a quadratic polynomial, the sum and product of whose zeroes are – 3
and 2, respectively.
Solution: Let the required polynomial be ax2 + bx + c and its zeros are α and .
∴ α + β = -3 = and αβ = 2 =
⇒ If a = 1 then b = 3 and c = 2
∴ Quadratic polynomial = x2 + 3x + 2
The relation between the zeros and the coefficients of Cubic polynomials:
If , β, γ are the zeros of the cubic polynomia ax3 + bx2 + cx + d then

(i) The number of zeroes is 0 as the graph not intersects the x-axis
+ + = ; + + = ; =
(ii) The number of zeroes is 1 as the graph intersects the x-axis at one point only.
(iii)The number of zeroes is 3 as the graph intersects the x-axis at three points . Exercise 9.2
(iv) The number of zeroes is 2 as the graph intersects the x-axis at two points. 1. Find the zeroes of the following quadratic polynomials and verify the relationship between
(v) The number of zeroes is 4 as the graph intersects the x-axis at four points. the zeroes and the coefficients.
(vi) The number of zeroes is 3 as the graph intersects the x-axis at three points. (i) x2 - 2x – 8 (ii) 4s2 – 4s - 1 (iii) 6x2 - 3 – 7x
(iv) 4u2 - 8u (v) t2 – 15 (vi) 3x 2 – x – 4
9.3 Relationship between Zeroes and Coefficients of a Polynomial (i) x2 – 2x – 8 = x2 – 4x + 2x – 8 = (x– 4)+ 2(x – 4) = (x - 4) (x + 2)
⇒ x = 4 and x = -2 are the zeros of polynomial x2 – 2x – 8
α and β are the zeros of the polynomial p(x) = ax2 + bx + c, a≠0 ( )
(x - α ) and (x - β) are the factors of p(x). Sum of the zeros = 4 + (-2) = 2 = =
Product of the zeros = (4) )(- 2) = -8 = =
(ii) 4s2 – 4s + 1 = 4s2 – 2s – 2s + 1 = 2s(s – 1) – 1 (2s – 1) = (2s – 1) (2s – 1)
Sum of Zeros +β = Product of Zeros β=
⇒ s = and s = are the zeros of the polynomial 4s2 – 4s + 1
( )
Sum of the zeros = + =1 = =
Example 2 : Find the zeroes of the quadratic polynomial x2 + 7x + 10, and verify the
Product of the zeros = x = =
relationship between the zeroes and the coefficients
(iii) 6x2– 3 – 7x = 6x2 –7x – 3 = 6x2 – 9x +2x –3 = 3x(2x –3) +1(2x –3) = (3x+1)(2x-3)
⇒ x = - and x = are the zeros of the polynomial 6x2 – 3 – 7x

P a g e 6 | 101 Available in ykoyyur.blogspot.com P a g e 7 | 101 Available in ykoyyur.blogspot.com


SSLC Mathematics Solutions in English YK SSLC Mathematics Solutions in English YK

Sum of the zeros =- + =1 = = =


( )
= ⇒ a = 4, b = 1 and c = 1
∴ The required polynomial is 4x2 + x + 1
Product of the zeros = - x = =
(vi) 4,1 - Let the required polynomial be ax2 + bx + c and its zeros are α and
(iv) 4u2 + 8u = 4u 2 + 8u + 0 = 4u(u + 2) ( )
⇒ u = 0 and u = −2 are the zeros of the polynomial 4u 2 + 8u α+ß= 4 = = and αß = 1 = =
Sum of the zeros = 0 + (-2) = -2 =
( )
= ⇒ a = 1, b = -4 and c = 1
∴ The required polynomial is x2 - 4x + 1
Product of the zeros = 0 x −2 = 0 = =
9.4 Division Algorithm for Polynomials:
(v) t2 – 15 = t2 - 0.t – 15 = (t - √15) (t + √15)
Let the zero of x3 - 3x2 – x + 3 is 1, then the factor is (x – 1)
⇒ t = √15 and t = - √15 are the zeros of the polynomial t2 – 15
Now, divide x3 - 3x2 – x + 3 by the factor (x – 1) then the quotient is x2 – 2x - 3.
Sum of the zeros = √15 + (-√15) = 0 = =
By factorising x3 - 3x2 – x + 3 we get the factors = (x -1) (x +1) (x -3)
Product of the zeros = √15 x (-√15)= -15 = = ∴ the zeros of the polynomial x3 - 3x2 – x + 3 is 1, -1 and 3
(vi) 3x2 – x – 4 = 3x2 – 4x + 3x – 4 = x(3x – 4)+1(3x – 4) = (3x - 4) (x + 1)
Example 6 : Divide 2x2 + 3x + 1 by x + 2.
⇒ x = and x = - 1 are the zeros of the polynomial 3x2 – x – 4
( ) x + 2 2x2 + 3x + 1 2x - 1 Solution: Note that we stop the division process when
Sum of the zeros = + (-1) = = =
2x2 + 4x either the remainder is zero or its degree is less than the
Product of the zeros = + (-1) = - = = - x+1 degree of the divisor. So, here the quotient is 2x – 1 and the
1. Find a quadratic polynomial each with the given numbers as the sum and product of its - x–2 remainder is 3. Also, (2x - 1) (x + 2) = 2x2 + 3x - 2 + 3 =
zeroes respectively. (i) , - 1 (ii) √ , (iii) 0, √ (iv) 1 , 1 (v) − , (vi) 4 , 1 2x2 + 3x + 1
+3
(i) , -1 - Let the required polynomial be ax2 + bx + c and its zeros are α and .
( )
⇒Dividend= Divisor x Quotient + Remainder
α+ß= = = and αß = −1 = = Example: Divide 7: 3x3 + x 2 + 2x + 5 by 1 + 2x + x 2
⇒ a = 4, b = -1 and c = -4 1 + 2x + x2 3x3 + x2 + 2x + 5 3x - 5 We first arrange the terms of the dividend and
∴ The required polynomial is 4x2 - x – 4 the divisor in the decreasing order of their
3x3 + 6x2 + 3x
(ii) √ , - Let the required polynomial be ax2 + bx + c and its zeros are α and . degrees. Recall that arranging the terms in this
- 5x2 - x + 5
( )
-5x2 - 10x - 5 order is called writing the polynomials in
α + ß = √2 = = and αß = = standard form. In this example, the dividend is
⇒ a = 3, b = −3√2 and c = 1 9x +10 already in standard form, and the divisor, in
∴ The required polynomial is 3x2 -3√2 +1 standard form, is x2 + 2x + 1
(iii) 0, √ − Let the required polynomial be ax2 + bx + c and its zeros are α and .

(x2 +2x +1) (3x -5) + (9x + 10) =3x3+x2+2x+5
α+ß= 0 = = and αß = √5 = = ⇒Dividend = Divisor x Quotient + Remainder
⇒ a = 1, b = 0 and c = √5 If p(x) and g(x) are any two polynomials and g(x) ≠ 0 then,
∴ The required polynomial is x2 + √5 p(x) = g(x) . q(x) + r(x)
(iv) 1, 1 - Let the required polynomial be ax2 + bx + c and its zeros are α and .
α+ß= 1 =
( )
= and αß = 1 = = q(x) - Quotient and r(x) - remainder
Here, r(x) = 0 or the degree of r(x) < the degree of g(x)
⇒ a = 1, b = -1 and c = 1
∴ The required polynomial is x2 - x + 1 This is known as The Division Algorithm for polynomials
Example 8: divide 3x 2 - x 3 - 3x + 5 by x
(v) − , - Let the required polynomial be ax + bx + c and its zeros are α and .
2 -x2 + x - 1 -x3 + 3x2 - 3x + 5 x-2
- 1 - x2 and verify the division algorithm. -x3 + x2 - x
α+ß= − = = and αß = = = Note that To car r y out division, we first write 2x2 - 2x + 5
both the dividend and divisor in decreasing 2x2 - 2x + 2
orders of their degrees. S o, dividend = – x3 +
3
3 x 2 – 3 x + 5 and divisor = –x2 + x – 1.
∴ Quotient = x - 2, Remainder = 3
P a g e 8 | 101 Available in ykoyyur.blogspot.com P a g e 9 | 101 Available in ykoyyur.blogspot.com
SSLC Mathematics Solutions in English YK SSLC Mathematics Solutions in English YK

Divisor x Quotient + Remainder = (-x2 + x - 1)(x -2) + 3 = -x3 + x2 -x + 2x2 - 2x + 2 + 3


= -x3 + 3x2 - 3x + 5 = Dividend. Hence, the division algorithm is verifyied. - x2 + 2 x4 + 0x3 + 0x2 - 5x + 6 -x2 - 2

Example 9 : Find all the zeroes of 2x4 – 3x3 – 3x2 + 6x – 2, if you know that two of its x4 + 0
- 2x2
zeroes are √ and - √ 2x2 - 5x + 6
Solution: Since two zeroes are √2 and - √2 2x2 + 0 - 4
- 5x + 10
(x - √2 ) (x + √2 ) = x2 - 2 are the factors of the polynomial
Quotient =-x2 – 2 ;remainder = -5x + 10
x2 - 2 2x4 – 3x3 – 3x2 + 6x – 2 2x2 - 3x + 1 Now, divide the polynomial by x2 – 2
-2x4 + 4x2 Factorise the Quotient = 2x2 - 3x + 1 2. Check whether the first polynomial is a factor of the second polynomial by dividing the
2x2 - 2x – x + 1 = 2x(x – 1) – 1(2x – 1) second polynomial by the first polynomial:
-3x3 + x2 + 6x - 2
(i) t2 - 3, 2 t4 + 3t3 - 2t2 - 9 t – 12 (ii) x2 + 3x + 1, 3x4 + 5x 3 - 7x2 + 2x + 2
-3x3 + 6x = (2x -1)(x - 1)
(iii) x3 - 3x + 1, x5 - 4x3 + x 2 + 3x + 1
+ x2 -2 ⇒ x = , x = 1 are the zeros (i) t2 - 3 2t4 + 3t3 - 2t2 - 9 t – 12
+ x2 -2 ∴ All 4 zeros are √2 ,- √2, , and 1
0 t2 - 3 2t4 + 3t3 - 2t2 - 9 t – 12 2t2 +3t + 4
2t4 + 0 - 6t2
Exercise 9.3 + 3t3 +4t2 – 9t
1. Divide the polynomial p(x) by the polynomial g(x) and find the quotient and remainder in + 3t3 + 0 - 9t
each of the following : +4t2 +0 - 12
3 2 2
(i) p(x) = x - 3x + 5x - 3 g(x) = x – 2 (ii) p(x) = x - 3x + 4x + 5 g(x) = x + 1 - x
4 2 2
+4t2 +0 - 12
4
(iii) p(x) = x - 5x + 6 g(x) = 2 - x
2 0
3 2 2 Remainder is Zero. Therefore first polynomial is the factor of the second polynomial.
(i) p(x) = x - 3x + 5x - 3 g(x) = x - 2
(ii) x 2 + 3x + 1 3x4 + 5x3 - 7x2 + 2x + 2
x2 - 2 x3 - 3x2 + 5x - 3 x -3
x2 +3x+1 3x4 +5x3 - 7x2 + 2x + 2 3x2 - 4x + 2
x3 - 0 - 2x
4 3 2
- 3x2 + 7x - 3 3x +9x + 3x
- 3x2 + 0 + 6 - 4x3 -10x2 + 2x
+ 7x - 9 - 4x3 –12x2 - 4x
+2x2 + 6x + 2
Quotient = x – 3 ; remainder = 7x - 9 +2x2 + 6x + 2
4
(ii) p(x) = x - 3x + 4x + 5
2 2
g(x) = x + 1 – x 0
Remainder is Zero. Therefore first polynomial is the factor of the second polynomial.
x2 - x + 1 x4 + 0.x3 - 3x2 + 4x + 5 x2 + x - 3
(iii) x 3 - 3x + 1 x5 - 4x3 + x2 + 3x + 1
4 3 2
x - x + x
x3 - 4x2 + 4x x3 -3x + 1 x5 - 4x3 + x2 + 3x + 1 x2 - 1
x3 - x2 + x x5 - 3x3 + x2
- 3x2 + 3x + 5 - x3 + 0 + 3x + 1
- 3x2 + 3x - 3 - x3 + 0 + 3x - 1
8 2

Quotient = x2 + x - 3 ;remainder = 8 Remainder is 2 Therefore first polynomial is not the factor of the second polynomial.
3. Obtain all other zeroes of 3x4 + 6x3 – 2x2 – 10x – 5, if two of its zeroes are and −
4 2
(iii) p(x) = x - 5x + 6 g(x) = 2 - x
and − are the zeros of 3x4 + 6x3 - 2x2 - 10x – 5

P a g e 10 | 101 Available in ykoyyur.blogspot.com P a g e 11 | 101 Available in ykoyyur.blogspot.com


SSLC Mathematics Solutions in English YK SSLC Mathematics Solutions in English YK

Verifying Division Algorithm,


∴ − and + are the factors of the polynomial. g(x) × q(x) + r(x) = (x2 ) × x + x
g(x) × q(x) + r(x) = x3 + x = p(x) ⇒p(x) = g(x) × q(x) + r(x)
⇒ − + = x2 - . Dividing the polynomial by x2 - ∴ It is satisfying divison algorithm is verified.
(iii) p(x) = x3 + 1
3x2 + 6x + 3 = 3(x2 + 2x + 1) g(x) = x2 ; q(x) = x and r(x) = 1; deg r(x) = 0
x2 - 3x4 + 6x3 - 2x2 - 10x – 5 3x2 + 6x+3 By Factorising (x2 + 2x + 1)
Verifying Division Algorithm
3x4 + 0 - 5x2 ⇒ x(x + 1) + 1(x + 1) g(x) × q(x) + r(x) = (x2 ) × x + 1
+ 6x3 +3x2 - 10x = (x + 1)(x + 1) ⇒g(x) × q(x) + r(x) = x3 + 1 = P(x) ⇒ p(x) = g(x) × q(x) + r(x)
+ 6x3 +0 -10x Therefore The factors of ∴ It is satisfying divison algorithm is verified.
3x4+6x3-2x2-10x–5 are
+3x2 +0 - 5
+3x2 +0 - 5 3 − + (x + 1)(x + 1) Summary:
0 1. Polynomials of degrees 1, 2 and 3 are called linear, quadratic and cubic polynomials
respectively.
Therefore All the Zeros of the polynomials are ,− , - 1 and - 1 2. A quadratic polynomial in x with real coefficients is of the form ax2 + bx + c, where a, b, c are real
numbers with a 0
4. On dividing x3 – 3x2 + x + 2 by a polynomial g(x), the quotient and remainder were x – 2 3. The zeroes of a polynomial p(x) are precisely the x - coordinates of the points, where the graph of
and –2x + 4, respectively. Find g(x). y = p( x) intersects the x - axis
Dividend P(x) = x3 - 3x2 + x + 2; Divisor g(x) = ? ; 4. A quadratic polynomial can have at most 2 zeroes and a cubic polynomial can have at most 3
Quotient q(x) = x – 2; Remainder r(x) = -2x + 4 zeroes.
P(x) = g(x).q(x) + r(x) 5. If and β are the zeros of polynomial ax2 + bx + c then, α + β = and αβ =
x3 - 3x2 + x + 2 = g(x).(x – 2) + (-2x + 4) 6. If , β and γ are the zeros of ax3 + bx2 + cx + d then,
⇒ g(x).(x – 2) = x3 - 3x2 + x + 2 – (-2x + 4) ⇒ g(x).(x – 2) = x3 – 3x2 + x + 2 + 2x - 4 −b
α+ β+ γ=
⇒ g(x).(x – 2) = x3 – 3x2 +3 x - 2 ⇒ g(x) = a
– c
αβ + βγ + γα =
x– 2 x − 3x + 3x − 2 x2 – x + 1 a
αβγ =
x3 - 2x2
∴ g(x) = x2 - x + 1 6. The division algorithm states that given any polynomial p(x) and any non-zero polynomial
- x2 + 3x g(x), there are polynomials q(x) and r(x) such that
2
-x + 2x p(x) = g(x). q(x) + r(x) Here, r(x) = 0 or deg r(x) < deg g(x)
+ x - 2
+ x - 2
0

5. Give examples of polynomials p(x), g(x), q(x) and r(x), which satisfy the division algorithm
and (i) degp(x) = degq(x) (ii) deg(x) = degr(x) (iii) deg r(x) = 0
(i) p(x) = 6x2 + 2x + 2
g(x) = 2; q(x) = 3x2 + x + 1
r(x) = 0 ⇒ deg p(x) = deg q(x) = 2
Verifying Division Algorithm,
g(x) × q(x) + r(x) = 2 (3x2 + x + 1) + 0
g(x) × q(x) + r(x) = 6x2 + 2x + 2 = P(x)
⇒ p(x) = g(x) × q(x) + r(x)
∴ It is satisfying divison algorithm is verified.
(ii) p(x) = x3 + x
g(x) = x2; q(x) = x and r(x) = x; deg g(x) = deg r(x) = 1
P a g e 12 | 101 Available in ykoyyur.blogspot.com P a g e 13 | 101 Available in ykoyyur.blogspot.com
SSLC Mathematics Solutions in English YK SSLC Mathematics Solutions in English YK

Example 2 : Check whether the following are quadratic equations:


Quadratic Equations (i) (x – 2)2 +1=2x – 3
(iii) x(2x + 3)=x 2 + 1
(ii) x(x + 1) + 8=(x + 2) (x – 2)
(iv) (x + 2)3 = x3 – 4
When we equate this polynomial to zero, we get a quadratic equation. (i) ( − ) + = −
x − 4x + 4 + 1 = 2x − 3 ⇒ x − 4x − 2x + 5 + 3 = 0 ⇒ x − 6x + 8 = 0
Any equation of the form p(x) = 0, where p(x) is a polynomial of degree 2, is This is in the form of ax + bx + c = 0
a quadratic equation. Therefore the given equation is quadratic equation.
(ii) ( + ) + = ( + )( − )
Standard form of quadratic equations:
x + x + 8 = x − 4 ⇒ x − x + x + 8 + 4 = 0 ⇒ x + 12 = 0
+ + = , Where a ≠ 0 This is not in the form of ax + bx + c = 0
Therefore the given equation is not a quadratic equation.
The features of quadratic equations:
(iii) ( + ) = +
 The quadratic equations has one variable
 The hieghest power of the variable is 2 2x + 3x = x + 1 ⇒ 2x − x + 3x − 1 = 0 ⇒ x + 3x − 1 = 0
 Standard form of quadratic equation: ax + bx + c = 0, This is in the form of ax + bx + c = 0
Therefore the given equation is quadratic equation.
Adfected quadratic equations : In a quadratic equation ax + bx + c = 0 , a ≠ 0,
(iv) ( + ) = −
b ≠ 0 then it is called adfected quadratic equations.
x + 2 + 3(x)(2) + 3x (2) = x − 4
Then, x − 3x − 5 = 0 , x + 5x + 6 = 0 , x + = 5 , (2x − 5) = 81 x + 8 + 12x + 6x = x − 4 ⇒ x −x + 6x + 12x + 8 + 4 = 0
Pure Quadratic equations : The quadrtic equations where a ≠ 0, b = 0 is called pure ⇒ 6x + 12x + 12 = 0 ÷ 6 ⇒ x + 2x + 2 = 0
quadratic equations. This is in the form of ax + bx + c = 0
The standard form of pure quadratic equation: ax + c = 0 [a ≠ 0] Therefore the given equation is quadratic equation.
Example 1 : Represent the following situations mathematically:
(i) John and Jivanti together have 45 marbles. Both of them lost 5 marbles each, and the Exercise 10.1
product of the number of marbles they now have is 124. We would like to find out how many
marbles they had to start with. 1. Check whether the following are quadratic equations :
(ii) A cottage industry produces a certain number of toys in a day. The cost of production (i) (x + 1)2 = 2(x – 3) (ii) x2 - 2x = (-2)(3 – x) (iii) (x – 2) (x + 1) =(x – 1) (x + 3)
of each toy (in rupees) was found to be 55 minus the number of toys produced in a day. (iv) (x – 3) (2x + 1) = x(x + 5) (v) (2x – 1) (x – 3) = (x + 5) (x – 1) (vi) x2 + 3x + 1=(x – 2)2
On a particular day, the total cost of production was Rs 750. We would like to find (vii) (x+2)3 = 2x(x 2 - 1) (viii) x 3 - 4x2 - x + 1 = (x – 2)3
out the number of toys produced on that day. (i) ( + ) = ( − )
(i) Let the number of marbles with Jhon be ‘x’ x + 2x + 1 = 2x − 6 ⇒ x + 2x − 2x + 1 + 6 = 0⇒ x + 7 = 0
Then the number of marbles with Jivanthi = 45 – x [∵Total number of marbles 45]
This is in the form of ax + bx + c = 0
The number of marbles left with John, when he lost 5 marbles = x – 5
Therefore the given equation is quadratic equation.
The number of marbles left with Jivanti, when she lost 5 marbles = 45 – x – 5 = 40 – x
∴ Their products = 124 (ii) − = (− )( − )
(x – 5) (40 – x) = 124 x − 2x = −6 + 2x ⇒ x − 2x − 2x + 6 = 0 ⇒ x − 4x + 6 = 0
⇒ 40x – x2 – 200 + 5x =124 ⇒ - x2 + 45x – 200 = 124 This is in the form of ax + bx + c = 0
⇒-x2 + 45x – 324 = 0 ⇒ x2 – 45x +324 = 0 Therefore the given equation is quadratic equation.
Therefore, the number of marbles John had, satisfies the quadratic equation x2 -45x + 324 = 0 (iii) ( − )( + ) = ( − )( + )
which is the required representation of the problem mathematically. x + x − 2x − 2 = x + 3x − x − 3 ⇒ x − x − 2 = x + 2x − 3
(ii) Let the number of toys produced on that day be x. ⇒ x − x − x − 2x − 2 + 3 = 0 ⇒ −3x + 3 = 0 × −1 ⇒ 3x − 1 = 0
Therefore, the cost of production (in rupees) of each toy that day = 55 – x This is not in the form of ax + bx + c = 0
So, the total cost of production (in rupees) that day = x (55 – x) Therefore the given equation is not a quadratic equation.
∴ x (55 – x ) = 750 (iv) ( − )( + ) = ( + )
⇒55x – x2 = 750 ⇒-x2 + 55x – 750 = 0 ⇒ x2 -55x + 750= 0 2x + x − 6x − 3 = x + 5x ⇒ 2x − 5x − 3 = x + 5x
∴ the number of toys produced that day satisfies the quadratic equation x2 - 55x – 750 = 0 ⇒ 2x − x − 5x − 5x − 3 = 0 ⇒ x − 10x − 3 = 0
which is the required representation of the problem mathematically. This is in the form of ax + bx + c = 0

P a g e 14 | 101 Available in ykoyyur.blogspot.com P a g e 15 | 101 Available in ykoyyur.blogspot.com


SSLC Mathematics Solutions in English YK SSLC Mathematics Solutions in English YK

Therefore the given equation is quadratic equation. Therefore the time taken to travel 480 = ℎ
(v) ( − )( − ) = ( + )( − )
⇒ +3= ⇒ 480( − 8) + 3 ( – 8 ) = 480
2x − 6x − x + 3 = x − x + 5x − 5 ⇒ 2x − 7x + 3 = x + 4x − 5
⇒2x − x − 7x − 4x + 3 + 5 = 0 ⇒ x − 11x + 8 = 0 ⇒480 − 3840 + 3 2 – 24 = 480 ⇒3840 + 3 2 – 24 = 0
This is in the form of ax + bx + c = 0 ⇒ 3 2 – 24 + 3840= 0
Therefore the given equation is quadratic equation. ⇒ 2 – 8 + 1280= 0
10.3 Solution of a Quadratic Equation by Factorisation
(vi) + + =( − ) Note: The zeros of the quadratic polynomial ax2 + bx + c and the roots of the quadratic
x + 3x + 1 = x − 2(x)(2) + 2 ⇒ x − x + 3x + 4x + 1 − 4 = 0 equation are the same.
7x − 3 = 0
This is not in the form of ax + bx + c = 0 Example 3 : Find the roots of the equation 2x2 – 5x + 3 = 0, by factorization.
2 −5 +3= 0
Therefore the given equation is not a quadratic equation. First term = , Last term =+
⇒ 2 −2 −3 +3 = 0
(vii) ( + ) = ( − )
⇒2 ( − 1) − 3( − 1) = Their product = +
x + 2 + 3(x)(2) + 3x (2) = 2x − 2x ⇒ x + 8 + 12x + 6x = 2x − 2x The middle term = −
⇒ ( − 1)(2 − 3) = 0
⇒ x −2x + 6x + 12x + 2x + 8 = 0 ⇒ −x + 6x + 14x + 8 = 0 × −1
⇒ − 1 = 0, 2 −3=0 Divide middle term such that product =
⇒ x − 6x − 14x − 8 = 0
This is not in the form of ax + bx + c = 0 ⇒ = 1, 2 = 3 + and their sum
3
Therefore the given equation is not a quadratic equation. = 1, = − ⇒ -5x = -2x – 3x
2
(viii) − − + =( − ) Example 4: Find the roots the equation 6x 2 – x – 2 = 0
x − 4x − x + 1 = x − 2 + 3(x)(2) − 3x (2) 6 − −2=0 First term = 6 , Last term =−2
⇒ x − 4x − x + 1 = x − 8 + 12x − 6x 6 −4 +3 −2 = 0 Their product = -12
⇒x − x − 4x + 6x − x − 12x + 1 + 8 = 0 ⇒ 2x − 13x + 9 = 0 2 (3 − 2) + 1(3 − 2) = 0
The middle term = −
This is in the form of ax + bx + c = 0 (2 + 1)(3 − 2) = 0
Divide middle term such that product
Therefore the given equation is quadratic equation. 2 + 1 = 0, 3 − 2 =
= -12 and sum −
2 = −1, 3 = 2
2. Represent the following situations in the form of quadratic equations :
⇒ = , =
⇒- x = - 4x + 3x
(i) The area of a rectangular plot is 528 m2 . The length of the plot (in metres) is one more
than twice its breadth. We need to find the length and breadth of the plot. Example 5:Find the roots the equation − √ + =
Let breadth b = m ⇒ Length = (2 + 1)m − √ + =
First term = 3 , Last term =+2
Area of the rectangle = × ⇒ 528 = (2 + 1) ⇒ 528 = 2 + 3 − √6 − √6 + 2 = 0
⇒ 2 + − 528 = 0 Their product = 6
√3 − √2. √3 − √2. √3 + √2 = 0
(ii) The product of two consecutive positive integers is 306. We need to find the The middle term = −2√6
integers. √3 √3 − √2 − √2 √3 − √2 = 0
Divide middle term such that product
Let two consecutive integers be x and (x + 1); Their products = 306 √3 − √2 √3 − √2 = 0
= 6 and sum − 2√6
⇒ x(x + 1) = 306 ⇒ x + x − 306 = 0 √3 − √2 = 0 , √3 − √2 = 0
(iii)Rohan’s mother is 26 years older than him. The product of their ages (in years) 3 ⇒ −2√6 = √6 − √6
years from now will be 360. We would like to find Rohan’s present age. √3 = √2, √3 = √2 ⇒ = , =
Let the present age of Rohan = ; The present age of his mother = + 26 Example 6 : Find the dimensions of the prayer hall discussed in Section 10.1.
After 3 Rohan’s age = + 3
After 3 years his mothers age = + 26 + 3 = + 29 2 + − 300 = 0 First term = 2 , Last term =−300
Product of their ages after 3 years = 360 2 − 24 + 25 − 300 = 0 Their produ= - 600
∴ ( + 3)( + 29) = 360 ⇒ + 29 + 3 + 87 = 360 2 ( − 12) + 25( − 12) = 0
The middle term = +
⇒ + 32 − 273 = 0 ( − 12)(2 + 25) =
− 12 = 0, 2 + 25 = 0 Divide middle term such that product
(iv) A train travels a distance of 480 km at a uniform speed. If the speed had been 8
= -600 and sum x
km/h less, then it would have taken 3 hours more to cover the same distance. We need to = 12, 2 = −25 ⇒ = = −12.5
find the speed of the train. ⇒+x = −24 + 25
Breadth = = 12
Let the speed of the train = /ℎ Length = 2 + 1 = 2(12) + 1 = 24 + 1 = 25
The time taken to travel 480 = ℎ
Reducing speed by 8 km/h, the speed of the train = ( − 8) /ℎ
P a g e 16 | 101 Available in ykoyyur.blogspot.com P a g e 17 | 101 Available in ykoyyur.blogspot.com
SSLC Mathematics Solutions in English YK SSLC Mathematics Solutions in English YK

Exercise 10.2 ∴ (27 − ) = 182 ⇒27 − = 182


− + 27 − 182 = 0 × −1
1. Find the roots of the following quadratic equations by factorisation: − + =
(i) x2 - 3x – 10 = 0 (ii) 2x 2 + x – 6 = 0 (iii) √ + + √ = − 13 − 14 + 182 = 0
(iv) − + = (v) − + = ⇒ ( − 13) − 14( − 13) = 0 ⇒( − 13)( − 14) = 0
(i) − − = ⇒( − 13) = 0, ( − 14) = 0 ⇒ = 13, = 14
− 5 + 2 − 10 = 0 ⇒ ( − 5) + 2( − 5) = 0 If first number = 13 then the second number = 27 − 13 = 14
⇒( − 5)( + 2) = 0 ⇒( − 5) = 0, ( + 2) = 0 If first number = 14 then the second number= 27 − 14 = 13
⇒ = 5, = −2 Therefore the numbers are 13 and 14
4. Find two consecutive positive integers, sum of whose squares is 365.
(ii) + − =
Let the positive numbsr be and the concecutiveinteger = + 1
2 + − 6 = 0 ⇒2 + 4 − 3 − 6 = 0
+ ( + 1) = 365 ⇒ + + 2 + 1 = 35
⇒2 ( + 2) − 3( + 2) = 0 ⇒( + 2)(2 − 3) = 0
⇒2 + 2 + 1 − 365 = 0⇒ 2 + 2 − 364 = 0 ÷ 2
⇒ + 2 = 0, 2 − 3 = 0
+ − =
⇒ = −2, 2 = 3⇒ = −2, =
+ 14 − 13 − 182 = 0 ⇒ ( + 14) − 13( + 14) = 0
(iii) √ + + √ = ⇒( + 14)( − 13) = 0 ⇒( + 14) = 0, ( − 13) = 0
√2 + 2 + 5 + 5√2 = 0 ⇒ = −14 , = 13
⇒√2 + √2 + 5( + √2) =0 ⇒ √2 + 5 + √2 =0 ∴ + 1 = 13 + 1 = 14
Two consecutive positive integers13, 14 DVªÉ.
⇒√2 + 5 = 0 , + √2 =0 ⇒√2 = −5 , = −√2 ⇒ = , = −√2

5. The altitude of a right triangle is 7 cm less than its base. If the hypotenuse is 13 cm, find the
(iv) − + = other two sides.
16 − 8 + 1 = 0 Let the base of the right angle triangle = ;Height = ( − 7)
⇒16 − 4 − 4 + 1=0 ⇒ 4 (4 − 1) − 1(4 − 1) = 0 According to Pythagoras theorem,
⇒(4 − 1)(4 − 1) = 0 ⇒ 4 − 1 = 0, 4 − 1 = 0
+ =
⇒4 = 1, 4 = 1⇒ = , = + ( − 7) = 13
(v) − + = + + 7 − 2( )(7) = 169
100 − 20 + 1 = 0 2 − 14 + 49 − 169 = 0
⇒100 − 10 − 10 + 1 = 0 ⇒10 (10 − 1) − 1(10 − 1) = 0 2 − 14 − 120 = 0 ÷ 2
⇒(10 − 1)(10 − 1) = 0 ⇒10 − 1 = 0, 10 − 1 = 0 − − =
⇒10 = 1, 10 = 1⇒ = , = − 12 + 5 − 60 = 0 ⇒ ( − 12) + 5( − 12) = 0
2. Solve the problems given in Example 1. ⇒( − 12)( + 5) = 0 ⇒( − 12) = 0, ( + 5) = 0
⇒ = 12, = −5
In example 1 we got the equations: (i) − + = and (ii) − + =
The base of = = 12
(i) − + =
− 36 − 9 + 324 = 0⇒ ( − 36) − 9( − 36) = 0 Height = ( − 7) = 12 − 7 = 5
⇒( − 36)( − 9) = 0⇒( − 36) = 0, ( − 9) = 0 6. A cottage industry produces a certain number of pottery articles in a day. It was observed
⇒ = 36, = 9 on a particular day that the cost of production of each article (in rupees) was 3 more than
The marbles with John = = 36 and the marbles with Jevan = 45 − = 45 − 36 = 9 twice the number of articles produced on that day. If the total cost of production on that
Or The marbles with John = 9 and the marbles with Jevan: 45 − = 45 − 9 = 36 day was ` 90, find the number of articles produced and the cost of each article.
Let the number of pots = ; The cost = (2 + 3)
(ii) − + =
− 25 − 30 + 750 = 0 ⇒ ( − 25) − 30( − 25) = 0 ∴ (2 + 3) = 90
⇒( − 25)( − 30) = 0 ⇒( − 25) = 0, ( − 30) = 0 + − =
⇒ = 25, = 30 2 + 15 − 12 − 90 = 0 ⇒ (2 + 15) − 6(2 + 15) = 0
The number of toys are 25 or 30 ⇒(2 + 15)( − 6) = 0⇒(2 + 15) = 0, ( − 6) = 0
3. Find two numbers whose sum is 27 and product is 182. ⇒2 = −15, = 6 ⇒ = , =6
Let the first number = then second number = 27 − Therefore number of pots = = 6; Cost = (2 + 3) = 2(6) + 3 = 12 + 3 = 15
Their product = 182
P a g e 18 | 101 Available in ykoyyur.blogspot.com P a g e 19 | 101 Available in ykoyyur.blogspot.com
SSLC Mathematics Solutions in English YK SSLC Mathematics Solutions in English YK

⇒ 2 − = < 0 ⇒There is no roots for this quadratic equation . The roots are
10.4 Solution of a Quadratic Equation by Completing the Square imaginary.
Example: Solve the equation − + = by completing the square. Solving the quadratic equations using formula:
3 −5 +2= 0 ×3 2ab = 15x Find the roots of the quadratic equation ax + bx + c = 0 by completing the square.
⇒9 − 15 + 6 = 0 ⇒ 9 − 15 = −6 2(3 )b = 15x ax + bx = −c [ multiply the equation by 4a ]
Add to both the sides b= = 4a x + 4abx = −4ac [Add to both the sides]
(3 ) − 2(3 ) + = −6 + 4a x + 4abx + b = −4ac + b
b =
3 − = −6 + ⇒ 3 − =
⇒(2ax) + 2(2ax)(b) + b = b − 4ac ⇒ (2ax + b) = b − 4ac
±√
⇒2ax + b = ±√b − 4ac ⇒ 2ax = −b ± √b − 4ac ⇒ x =
⇒ 3 − = ⇒ 3 − =± √ √
Roots are: x = , x=
±
⇒ 3 − =± ⇒3 = ± + ⇒3 = ⇒ = , = Example 10 : Solve Q. 2(i) of Exercise 10.1 by using the quadratic formula
⇒ = , = ⇒ = 1, = In Q.No.2(i) of exercise 10.i we got the equation 2 + − 528 = 0
This is in the form of ax + bx + c = 0
Example 7 : Solve the equation given in Example 3 by the method of completing the the a = 2, b = 1, c = −528
square: − + = ±√
Roots are x =
2 −5 +3= 0 ×2 2ab = 10x
4 − 10 + 6 = 0 ⇒4 − 10 = −6 x=
( )± ( ) ( )( )
⇒x=
±√
2(2 )b = 15x ( )
Add to both the sides ±√ ±
b= = ⇒x = ⇒x =
(2 ) − 2(2 ) + = −6 +
x= or x =
⇒ 2 − = −6 + ⇒ 2 − = b = x= or x =
⇒ 2 − = ⇒ 2 − =± x = 16 or x = −
± ⇒Breadth of the site = 16m and the length = 2x16 + 1 = 32 + 1 = 33m
⇒ 2 − =± ⇒2 = ± + ⇒2 = ⇒ = , =
Example 11:Find two consecutive odd positive integers, sum of whose squares is 290.
⇒ = , = ⇒ = , =1 Let the consecutive odd numbers be and + 2
x + (x + 2) = 290
Example 8 : Find the roots of the equation 5x2 – 6x – 2 = 0 by the method of completing the
x + x + 2 + 2(x)(2) = 290
square.
2ab = 30x 2x + 4x + 4 − 290 = 0
5x2 - 6x - 2 = 0 x5 2x + 4x − 286 = 0 ÷2
25 − 30 − 10 = 0 ⇒ 25 − 30 = 10 2(5 )b = 30x x + 2x − 143 = 0 this is in the form of ax + bx + c = 0
Add to both the sides a = 1, b = 2, c = −143
25 − 30 + (3) = 10 + (3)
2 2 b= =3
±√
Roots are x =
⇒(5 ) − 2(5 )(3) + (3)2 = 10 + 9 ⇒(5 − 3) = 19 b = (3)
( )± ( ) ( )( ) ±√
x= ⇒x=
⇒(5 − 3) = ±√19 ⇒5 = 3 ± √19 ( )
±√ ±
3±√ 3 √ 3 √
⇒x = ⇒x =
⇒ = ⇒ = , =
x= ,x =
Example 9 : Find the roots of 4x2 + 3x + 5 = 0 by the method of completing the
x= , x=
square.
4 + 3 + 5 = 0 ⇒4 + 3 = −5 2ab = 3x x = 11, x = −13
Add to both the sides 2(2 )b = 3x Therefore the consecutive odd numbers are 11 and 13
Example 12 : A rectangular park is to be designed whose breadth is 3 m less than its
(2 ) − 2(2 ) + = −5 + b= =
length. Its area is to be 4 square metres more than the area of a park that has already
⇒ 2 − = −5 + ⇒ 2 − = b = been made in the shape of an isosceles triangle with its base as the breadth of the
rectangular park and of altitude 12 m (see Fig. 10.3). Find its length and
P a g e 20 | 101 Available in ykoyyur.blogspot.com P a g e 21 | 101 Available in ykoyyur.blogspot.com
SSLC Mathematics Solutions in English YK SSLC Mathematics Solutions in English YK

Solution: The breadth of the rectangular park be x m and the length = (x+3) m + = 3 – Multiply both sides bu x
Area = x (x + 3)m2 = (x2 + 3x)m2. + 1 = 3 ⇒ − 3 + 1 = 0 this is in the form of ax + bx + c = 0
Now, the base of the isosceles triangle = x m = 1, = −3, =1
±√
Therefore the area = × x × 12 = 6x m Roots are, =
According to question, x2 + 3x = 6x + 4 =
( )± ( ) ( )( )
⇒ =
±√
⇒ =
±√
⇒ =

, =

∴ x2 - 3x - 4 = 0 this is in the form of ax + bx + c = 0 ( )

= 1, = −3, = −4 (ii) - =3 ⇒ =3
( )
±√
The roots are = ⇒ = 3 ⇒ −2 = 3x − 6x
x=
( )± ( ) ( )( )
⇒x =
±√ 3x − 6x + 2 = 0 this is in the form of ax + bx + c = 0
( ) a = 3, b = −6, c=2
±√ ±
⇒x = ⇒x = roots are, x =
±√

x= , x= ( )± ( ) ( )( )
x= ( )
x= ,x= ±√ ±√ ±√ × ± √
x = 4 , x = −1 ⇒x = ⇒x = ⇒x = ⇒x =
∴ The breadth = x = 4m and the length = x + 3 = 4 + 3 = 7m. ⇒x =
±√
⇒x = ⇒x =
±√
, x=
√ √

Example 13 : Find the roots of the following quadratic equations, if they exist, using the Example 15 : A motor boat whose speed is 18 km/h in still water takes 1 hour more to go
quadratic formula .(i) 3x2 - 5x + 2 = 0 (ii) x2 + 4x + 5 = 0 (iii) 2x 2 – 2√ x + 1 24 km upstream than to return downstream to the same spot. Find the speed of the
(i) 3x2 - 5x + 2 = 0 this is in the form of ax + bx + c = 0 stream.
a = 3, b = −5, c = +2 Speed of the stream: = km/h
±√ Speed of the motor boat upstream = (18 − )km/h
Roots are:x = Speed of the motor boat downstram = (18 + )km/h
( )± ( ) ( )( ) ±√
x= ( )
⇒x = Time taken to travel upsteram = hour

⇒x =
±√
⇒x =
± Time taken to travel down steram = hour

x = or x = ⇒ x = 1 or x = − = 1, .
(ii) x2 + 4x + 5 = 0 this is in the form of ax + bx + c = 0 24(18 + ) − 24(18 − ) = 1(18 − )(18 + )
a = 1, b = 4, c = +5 432 + 24x – 432 + 24x = 324 – x2 ⇒ 48x = 324 – x2
±√ - x2 + 324 - 48 = 0 x (-1)
Roots are x = + 48 − 324 = 0 this is in the form of ax + bx + c = 0
( )± ( ) ( )( ) ±√ = 1, = 48, = −324
x= ⇒x =
( ) ±√
±√ ± √ Roots are, =
⇒x = ⇒x = ⇒Roots are not Real numbers. ( )± ( ) ( )( )
x=
(iii) 2x2 – 2√ x + 1 this is in the form of ax + bx + c = 0 ( )
a = 2, b = −2√2, c = +1 ⇒x =
±√
⇒x =
±√
⇒x =
±

±√
roots are, x = ⇒x = , x=
( √ )± ( √ ) ( )( ) ⇒x = , x =
x= ( ) ⇒x = 6, = −54
√ ±√ √ ±√ √ √ ⇒Speed of the stream=
⇒x = ⇒x = ⇒x = ⇒x = ⇒ =6 /ℎ

Example 14:Find the roots of the following equations Exercise 10.3
(i) x + = 3, x ≠ 0 (ii) + = 3 x ≠ 0, x ≠ 2 1. Find the roots of the following quadratic equations, if they exist, by the method of
(i) x + = 3, x ≠ 0 completing the square:.
(i) 2x2 - 7x + 3 = 0 (ii) 2x2 + x - 4 = 0(iii) 4x2 + 4√ x + 3 = 0 (iv) 2x2 + x + 4 = 0
P a g e 22 | 101 Available in ykoyyur.blogspot.com P a g e 23 | 101 Available in ykoyyur.blogspot.com
SSLC Mathematics Solutions in English YK SSLC Mathematics Solutions in English YK

(i) − + = 2ab = 14x ( )± ( ) ( )( ) ±√


x= ⇒x =
2 − 7 = −3 × 2 2(2 )b = 14x ( )
±√ ±
4 − 14 = −6 ⇒x = ⇒x =
b= =
4 − 14 + = −6 + [Add to both the sides] ⇒x= , x= ⇒x= , x= ⇒x = 3 , x =
b = + − = this is in the form of ax + bx + c = 0
(2 ) − 2(2 )( ) + = −6 + (ii) a = 2, b = 1, c = −4
±√
2 − = ⇒ 2 − =± Roots are, x =
± ± ( )± ( ) ( )( ) ±√ ±√
⇒2 − = ± ⇒2 = ± + ⇒2 = ⇒ = , 2 =1 x= ⇒x = ⇒x =
( )
⇒ = , = ⇒ = , = x=

, x=

⇒ =3 , = (iii) + √ + = this is in the form of ax + bx + c = 0


(ii) 2x2 + x - 4 = 0 a = 4, b = 4√3, c = +3
2 + =4 ×2 2ab = 2x ±√
2(2 )b = 2x Roots are, x =
4 +2 = 8
b= = ( √ )± ( √ ) ( )( )
4 +2 + =8+ [Add to both the sides] x= ( )
b = √ ±√ × √ ±√
(2 ) + 2(2 ) + =8+ x= ⇒x =
√ ± √ √ √ √
2 + = ⇒ 2 + =± ⇒x = , ⇒x = ,x = ⇒x = , x=
√ √ ±√ ±√ (iv) + + = this is in the form of ax + bx + c = 0
⇒2 + = ± ⇒2 = ± + ⇒2 = ⇒ =
= 2, = 1, =4
√ √
⇒ = , = Roots are, =
±√

(iii) 4x 2 + 4√ x + 3 = 0 ( )± ( ) ( )( ) ±√ ±√
= ⇒ = ⇒ =
4 + 4√3 = −3 ( )

2ab = √3 There is no real root for this equation.


4 + 4√3 + √3 = −3 + √3 [Add √3 to both the sides]
3. Find the roots of the following equations:
2(2 )b = √3
(2 ) + 2(2 )(√3) + √3 = −3 + 3 (i) + = , ≠ (ii) − = , ≠− ,

b= = √3
2 + √3 = 0 (i) − = , ≠
2 + √3 = 0, 2 + √3 = 0 b = √3
√ √
x − = 3 Multiply the equation by x
2 = −√3, 2 = −√3 ⇒ = , = x − 1 = 3x ⇒x − 3x − 1 = 0 this is in the form of ax + bx + c = 0
(iv) 2x2 + x + 4 = 0 a = 1, b = −3, c = −1
2 + = −4 ×2 2ab = 2 ±√
2(2 )b = Roots are, x =
4 + 2 = −8
b= = x=
( )± ( ) ( )( )
(2 ) − 2(2 ) + = −8 + [Add to both the sides] ( )
b = ±√
⇒x =
±√
⇒x =
√ √
2 − = −8 + ⇒ 2 − = x= , x=
(ii) − = , ≠− ,
⇒ 2 − = < 0 There are no roots.The roots are imaginary
30(x − 7) − 30(x + 4) = 11(x + 4)(x − 7)
2. Find the roots of the quadratic equations given in Q.1 above by applying the quadratic 30x − 210 − 30x − 120 = 11(x + 4x − 7x − 28)
formula
-330 = 11x − 11(3x) − 11(28) ⇒-330 = 11x − 33x − 308
(i) − + = this is in the form of ax + bx + c = 0
a = 2, b = −7, c=3 ⇒−330 = 11x − 33x − 308 ⇒11x2 – 33x + 22 = 0
±√ ⇒x − 3x + 2 = 0 this is in the form of ax + bx + c = 0
Roots are, x = a = 1, b = −3, c=2

P a g e 24 | 101 Available in ykoyyur.blogspot.com P a g e 25 | 101 Available in ykoyyur.blogspot.com


SSLC Mathematics Solutions in English YK SSLC Mathematics Solutions in English YK

Roots are, x =
±√ 7. The difference of squares of two numbers is 180. The square of the smaller number is 8
times the larger number. Find the two numbers.
( )± ( ) ( )( ) ±√
x= ( )
⇒x = Let the larger and smaller numbers be x and respectively . According to question,
±√ ± x − y = 180 ªÀÄvÀÄÛ y = 8x⇒x − 8x − 180 = 0
⇒x = ⇒x = ⇒x = , x= ⇒x = , x =
⇒x − 18x + 10x − 180 = 0 ⇒x(x − 18) + 10(x − 18) = 0
4. The sum of the reciprocals of Rehman’s ages, (in years) 3 years ago and 5 years from now ⇒(x − 18)(x + 10) = 0 ⇒x − 18 = 0, x + 10 = 0
it is Find the present age. ⇒x = 18, x = −10
Let the present age of Rehman = Years ⇒Larger number x = 18
Age of Rehman before 3 years = ( − 3) years ∴ y = 8x = 8 × 18 = 144 ⇒y = ±√144 = ±12
His age after five years from now = ( + 5) Years. ∴ smaller number = ±12
Sum of the reciprocals of the age = The numbers are 18 and 12
+ = , . 8. A train travels 360 km at a uniform speed. If the speed had been 5 km/h more, it would
have taken 1 hour less for the same journey. Find the speed of the train.
3( + 5) + 3( − 3) = ( − 3)( + 5)
Speed of the train = km/h
3 + 15 + 3 − 9 = + 2 − 15 ⇒6 + 15 − 9 = + 2 − 15
⇒ + 2 − 15 = 6 + 6 ⇒ − 4 − 21 = 0 The time taken to travel 360 km = h
⇒ − 7 + 3 − 21 = 0 ⇒ ( − 7) + 3( − 7) = 0 If speed is 5 km/h more, then the required time is = h
⇒( − 7)( + 3) = 0 ⇒ − 7 = 0 , + 3 = 0
⇒ = + 1 ⇒360(x+5) = 360x + x(x+5)
⇒ = 7, = −3
⇒The present age of Rehman = 7 Years. ⇒360x+1800 = 360x + x2 + 5x ⇒360x+1800 = 360x + x2 + 5x
5. In a class test, the sum of Shefali’s marks in Mathematics and English is 30. Had she got 2 ⇒x + 5x − 1800 = 0 ⇒x + 45x − 40x − 1800 = 0
⇒x(x + 45) − 40(x + 45) = 0 ⇒(x + 45)(x − 40) = 0
marks more in Mathematics and 3 marks less in English, the product of their marks
would have been 210. Find her marks in the two subjects. ⇒x + 45 = 0, x − 40 = 0 ⇒x = −45, x = 40
Let the marks in Mathematics = x then the marks in English = 30 − x By question, Speed of the train = 40 km/h
(x + 2)(30 − x − 3) = 210 ⇒(x + 2)(27 − x) = 210 9. Two water taps together can fill a tank in 9 hours. The tap of larger diameter takes 10
⇒27x − x + 54 − 2x − 210 = 10 hours less than the smaller one to fill the tank separately. Find the time in which each tap
⇒−x + 25x − 156 = 0 × −1 can separately fill the tank.
⇒x − 25x + 156 = 0 ⇒x − 12x − 13x − 156 = 0 The time taken by the smaller tap to fill the tank = x hour
⇒x(x − 12) − 13(x − 12) = 0 ⇒(x − 12)(x − 13) = 0 The time taken by the larger tap to fill the tank = (x − 10) hour
⇒x − 12 = 0 , x − 13 = 0 ⇒x = 12, x = 13 The part of the tank filled by the smaller tap in one hourThe smaller tap =
If the marks in Mathematics = 12 then the marks in English = 30 − 12 = 18
The part of the tank filled by the smaller tap in one hourThe larger tap =
If the marks in Mathematics = 13 hen the marks in English= 30 − =
6. The diagonal of a rectangular field is 60 metres more than the shorter side. If the longer The time taken by both the tap to fill the tank = 9 =
side is 30 metres more than the shorter side, find the sides of the field. The part of the tank filled by both the tap in one hourThe smaller tap = =
Let the length of the shorter side = x m; Length of the longer side = (x + 30)m
Length of the diagonals = (x + 60)m + =
By Pythagoras theorem length of the diagonals = x + (x + 30) ⇒75(x − 10) + 75x = 8(x)(x – 10)⇒75x − 750 + 75x = 8x2 – 80x
x + (x + 30) = x + 60 ⇒x + (x + 30) = (x + 60) ⇒8x − 80x − 150x + 750 = 0 ⇒8x − 230x + 750 = 0
⇒x + x + 2(x)(30) + (30) = x + 2(x)(60) + (60) ⇒8x − 200x − 30x + 750 = 0 ⇒8x(x − 25) − 30(x − 25) = 0
⇒(x − 25)(8x − 30) = 0 ⇒x − 25 = 0, 8x − 30 = 0
⇒2x + 60x + 900 = x + 120x + 3600
⇒2x −x + 60x − 120x + 900 − 3600 = 0 ⇒x = 25, x = = = 3.75
⇒x − 60x − 2700 = 0 ⇒x − 90x + 30x − 2700 = 0 If the time taken by the smaller tap to fill the tank = 3.75 hr, then The time taken by the
⇒x(x − 90) + 30(x − 90) = 0⇒(x − 90)(x + 30) = 0 larger tap to fill the tank can not be negative.Therefore The time taken by the smaller tap to
⇒(x − 90) = 0, (x + 30) = 0 ⇒x = 90, x = −30 fill the tank = 25 hour. The time taken by the larger tap to fill the tank 25-10=15 Hours
⇒DAiÀÄvÀzÀ zÉÆqÀØ ¨ÁºÀÄ«£À GzÀÝ = (x + 30) = 90 + 30 = 120 m

P a g e 26 | 101 Available in ykoyyur.blogspot.com P a g e 27 | 101 Available in ykoyyur.blogspot.com


SSLC Mathematics Solutions in English YK SSLC Mathematics Solutions in English YK

10.An express train takes 1 hour less than a passenger train to travel 132 km between ∆= b − 4ac ⇒∆= (−4) − 4(2)(3)
Mysore and Bangalore (without taking into consideration the time they stop at ⇒∆= 16 − 24 ⇒∆= −8 < 0 Roots are imaginary
intermediate stations). If the average speed of the express train is 11km/h more than that Example 17 : A pole has to be erected at a point on the boundary of a circular park of
of the passenger train, find the average speed of the two trains. diameter 13 metres in such a way that the differences of its distances from two
Let the average speed of the passenger train = x km/h; diametrically opposite fixed gates A and B on the boundary is 7 metres. Is it possible to do
The average speed of express train = (x + 11)km/h so? If yes, at what distances from the two gates should the pole be erected?
The total distance to travel = 132 km BP = x m ;AP = (x + 7)m ; Diameter AB = 13 m
The time taken by passenger train = h ∠APB = 90 ⇒AP + PB = AB
⇒x + (x + 7) = 13 ⇒x + x + 7 + 2(x)(7) = 169
The time taken by express train =
⇒2x + 14x + 49 − 169 = 0 ⇒ 2x + 14x − 120 = 0 ÷ 2
When the difference between the time taken by two trains is 1 hour ⇒x + 7x − 60 = 0 ⇒x + 12x − 5x − 60 = 0
∴ − =1 ⇒x(x + 12) − 5(x + 12) = 0 ⇒(x + 12)(x − 5) = 0
132(x + 11) – 132 x = x(x + 11) ⇒132x + 1452 – 132x = x2 + 11x ⇒x + 12 = 0, x − 5 = 0
⇒x + 11x − 1452 = 0 ⇒x + 44x − 33x − 1452 = 0 ⇒x = −12, x = 5
⇒x(x + 44) − 33(x + 44) = 0 ⇒(x + 44)(x − 33) = 0 ⇒ x= -12 is not possible. Therefore BP = x m = 5m
⇒x + 44 = 0, x − 33 = 0 ⇒x = −44, x = 33 AP = (x + 7) = 5 + 7 = 12 m
The speed can not be negative
Find the discriminant of the equation 3x2 – 2x + = 0 and hence find the nature of the
Therefore the average speed of passenger train = 33 km/h
The average speed of the express train = (33 + 11) = 44 km/h roots. Find them if they are real.
a = 3, b = −2, c =
11. Sum of the areas of two squares is 468 m2. If the difference of their perimeters is 24 m,
find the sides of the two squares. b − 4ac = (−2) − 4(3) = 4−4 = 0
Let the side of first square be ‘x’ m and the side of the second square be y b − 4ac = 0 Roots are real and equal
According to question, 4x – 4y = 24m ⇒4x = 4y + 24 ⇒ x = (y + 6)m Roots are: ,
( )
= ( ) ,
( )
= , = ,
And x2 + y2 = 468 ⇒ (y + 6)2 + y2 = 468 ⇒ y2 + 12y + 36 + y2 = 468 ( )

⇒ 2y2 + 12y – 432 = 0 ⇒ y 2 + 6y – 216 = 0


⇒ y2 +18y – 12y -216 = 0 Exercise: 10.4
⇒y(y + 18)-12(y + 18) = 0 1. Find the nature of the roots of the following quadratic equations. If the real roots exist,
⇒(y – 12)(y + 18) = 0 find them:.(i) 2x 2 - 3x + 5 = 0 (ii) 3x2 - 4√ x + 4 = 0 (iii) 2x2 - 6x + 3 = 0
⇒y = 12 and y = -18 [ y = -18 is not possible] (i) 2x2 - 3x + 5 = 0
Therefore the side of the second square is 12m and a = 2, b = −3, c=5
the side of the first square is (12 + 6) = 18 m ∆ = b − 4ac
10.5 Nature of Roots ∆ = (−3) − 4(2)(5) ⇒∆ = 9 − 40
The value of b2 - 4ac decides the roots of quadratic equation ax2 + bx + c = 0 has real ⇒∆ = −31 ⇒∆ < 0 ⇒ Roots are imaginary
or not, therefore (ii) 3x2 - 4√ x + 4 = 0
b2 - 4ac is called the discriminant of this quadratic a = 3, b = − 4√3, c=4
equation.and denoted by ∆ [delta] ∆ = b − 4ac
So, the quadratic equation ax2 + bx + c = 0 has ∆ = (−4√3) − 4(3)(4) ⇒∆ = 48 − 48
Discriminant Nature of the roots
∆ = 0 ⇒ Roots are real and equal
( √ ) ( √ ) √ √
∆= Two equal real roots The roots are: , = ( )
, ( )
= ,
∆> √ √
Two distinct real roots = , ⇒ ,
√ √
∆< No real roots (iii) 2x2 - 6x + 3 = 0
a = 2, b = −6, c=3
Example 16 : Find the discriminant of the quadratic equation 2x2 – 4x + 3 = 0, and ∆ = b − 4ac
hence find the nature of its roots ∆ = (−6) − 4(2)(3) ⇒∆ = 36 − 24
a = 2, b = −4, c = 3 ⇒∆ = 12 ⇒ ∆ > 0 ⇒ Roots are real and distinct
P a g e 28 | 101 Available in ykoyyur.blogspot.com P a g e 29 | 101 Available in ykoyyur.blogspot.com
SSLC Mathematics Solutions in English YK SSLC Mathematics Solutions in English YK

The roots =
√∆
,
√∆ − 4 = (−40) − 4(1)(400) = 1600 − 1600 = 0
− 4 = 0 Roots are rea and equal
( ) √ ( ) √ √ √ ( ) ( )
= , = , Roots are: , = , = , = 20 , 20
( ) ( ) ( ) ( )
√ √ √ √ Length = 20 ; Breadth = 40 − = 40 − 20 = 20
= , = ,
2. Find the values of k for each of the following quadratic equations, so that they have two
equal roots (i) 2x2 + kx + 3 = 0 (ii) kx (k - 2) + 6 = 0 Summary:
(i) + + = a = 2, b = k, c=3
b − 4ac = 0
1. A quadratic equation in the variable x is of the form ax2 + bx + c = 0, where a, b, c are real
(k) − 4(2)(3) = 0 ⇒k − 24 = 0 ⇒k = 24
numbers and a ≠ 0
k = ±√24 = ±√4 × 6 = ±2√6
( − )+ = 2. A real number is said to be a root of the quadratic equation ax2 + bx + c = 0, if a2
(ii)
kx − 2kx + 6 = 0 ⇒a = k, b = −2k, c=6 + b + c = 0. The zeroes of the quadratic polynomial ax2 + bx + c and the roots of the quadratic
b − 4ac = 0 equation ax2 + bx + c = 0 are the same.
⇒(−2k) − 4(k)(6) = 0 ⇒4k − 24k = 0 3. If we can factorise ax2 + bx + c, a 0, into a product of two linear factors, then the roots of the
⇒4k(k − 6) = 0 ⇒4k = 0 , k − 6 = 0 quadratic equation ax2 + bx + c = 0 can be found by equating each factor to zero.
⇒k = 0, k = 6
4. A quadratic equation can also be solved by the method of completing the square.
3. Is it possible to design a rectangular mango grove whose length is twice its breadth,
and the area is 800 m2? If so, find its length and breadth 5. Roots of the quadratic equation ax2 + bx + c = 0 are given by the formula
The breadth of the mango grove = ;The length = 2 ±√
= where b2 - 4ac ≥ 0
The area of the grove = Length x breadth
6. For the quadratic equation ax2 + bx + c = 0 ,
⇒ ( )(2 ) = 800 ⇒2 = 800 ⇒ = = 400 ⇒ = ±√400 = ±20
(i) If b 2 - 4ac > 0 then roots are real and distinct
∴ The breadth of the mango grove = = 20 m
(ii) If b 2 - 4ac = 0 roots are real and equal
∴ The breadth of the mango grove = 2 = 2 × 20 = 40 m
4. Is the following situation possible? If so, determine their present ages.The sum of the ages (iii) If b2 - 4ac < 0 no real roots

of two friends is 20 years. Four years ago, the product of their ages in years was 48.
Let the age of A friend = Years
The age of B friend = (20 − ) years
The age of friend A before 4 = ( − 4)
The age of B friend before 4 years = (20 − − 4) = 16 −
( − 4)(16 − ) = 48
16 − − 64 + 4 = 48
− + 20 − 64 − 48 = 0
− 20 + 112 = 0
= 1, = −20, = 112
− 4 = (−20) − 4(1)(112)
= 400 − 448 = −48
The equation has no real roots. Therefore this situation is not possible
5. Is it possible to design a rectangular park of perimeter 80 m and area 400 m2? If so, find
its length and breadth.
Let the length and breadth of the rectangle be and ; The perimeter = 2( + ) = 80
+ = = 40 ⇒ = 40 −
Area × = 400 ⇒ (40 − ) = 400
⇒40 − = 400 ⇒ − 40 + 400 = 0
= 1, = −40, = 400

P a g e 30 | 101 Available in ykoyyur.blogspot.com P a g e 31 | 101 Available in ykoyyur.blogspot.com


SSLC Mathematics Solutions in English YK SSLC Mathematics Solutions in English YK

Example 2 :If B and Q are acute angles such that sin B = sin Q, then prove that
ITRODUCTION TO ∠B = ∠Q.
Sin B = SinQ ⇒ =
TRIGONOMETRY ⇒ = =k (1)
Trigonometry is the study of relationships between the sides and angles of a triangle. BC = √AB − AC [By Pythagoras theorem]
11.2 Trigonometric Ratios: ⇒ k PQ − k PR ⇒k. PQ − PR [ from (1)]
To know the trigonometric ratio we have to consider right angle triangle. QR = PQ − PR
.
⇒ = =k (2)
From (1) and (2),
= = ⇒ ∆ABC ~∆PQR
∴ ∠B = ∠Q
Example 3 : Consider ∆ACB , right-angled at C, in which AB = 29 units, BC = 21 units
and ∠ABC = θ (see Fig. 11.10). Determine the values of
Cos2 + Sin2 (ii) Cos2 - Sin2
In right angle triangle ACB, ∠ACB = 90 0
Therefore AC = √AB − BC ⇒ AC = √29 − 21
⇒ AC = √841 − 441 = √400 = 20
There are six trigonometric ratios: (i) Cos2 + Sin2

Triangle 1 = + = = =1
Trigonometric ratios Triangle 2
Opposite
(ii) Cos2 - Sin2
BC AB
SinA = - = = =1
Hypotenuse AC AC
Adjecent AB BC Example 4 : In a right triangle ABC, right-angled at B, if tan A = 1, then verify that
CosA
Hypotenuse AC AB 2sin A cos A = 1
Opposite BC AB In right angle triangle ACB ,
Tan A
Adjecent AB BC tan A = 1 ⇒ = 1 ⇒ AB = BC
Hypotenuse AC AC AC2 = AB2 + BC2 [By Pythagoras theorem]
CosecA
Opposite BC AB ⇒ AC2 = 2AB2 (1)
Hypotenuse AC AC
SecA Now, 2sin A cos A = 2. . = 2. = 2. =1
Adjecent AB BC
E xa mple 5 : In ∆OPQ r ight - a ngled a t P, OP = 7 cm and OQ – PQ = 1 cm (see Fig.
Adjecent AB BC
CotA 11.12). Determine the values of sin Q and cos Q.
Opposite BC AB
Example 1: tan A = find the other In ∆OPQ,
trigonometric ratios of the angle A OQ2 = PQ2 + OP2 [By Pythagoras theorem]
In ∆ABC, ∠ ABC = 900 ∴ By Pythagoras theorem, ⇒(1 + PQ)2 = PQ2 + 7 2 ⇒1 + PQ2 + 2PQ = PQ2 + 49
AC2 = AB2 + BC2 ⇒ AC2 = 42 + 3 3 = 16 + 9 = 25 ⇒AC = 5 ⇒1 + 2PQ = 49
SinA = = ; Cos A = = ; TanA = = ⇒2PQ = 49 – 1 = 48 ⇒PQ = 24cm
⇒ OQ = 1 + PQ= 1 + 24 ⇒ OQ = 25
CosecA = = ;SecA = = ; CotA = = ∴ sin Q = and cos Q =

P a g e 32 | 101 Available in ykoyyur.blogspot.com P a g e 33 | 101 Available in ykoyyur.blogspot.com


SSLC Mathematics Solutions in English YK SSLC Mathematics Solutions in English YK

4. Given 15 cot A = 8 find sin A and sec A


Inverse of trigonometric values According to question, cot A = =
Hypotenuse In ΔABC , ∠B = 900
BC = 15k, AB = 8k [ Here, Let k = 1 ]
Opposite
Hypotenuse ∴ By Pythagoras theorem,
AC2 = AB2 + BC2
Adjecent
AC2 = 82 + 152 = 64 + 225 = 289 ⇒ AC = 17
Adjecent
⇒sin A = = ; sec A = =
Opposite
Opposite 5. Given sec θ = calculate all other trigonometric values.
Hypotenuse According to question, sec θ = =
Adjecent
In ΔPMO , ∠M = 900
Hypotenuse
OM = 12k, OP = 13k [ Here, Let k = 1 ]
Opposite
∴ By Pythagoras theorem, PM2 = OP2 + OM2
Adjecent
PM2 = 132 + 12 2 = 169 - 144 = 25⇒ AC = 5
sin θ = = ; cos θ = = ; tan θ = =
Exercise 11.1 cot θ = = ; cosec θ = =
[ for solving problems, the value of constant k is taken as 1] 6. If A and B are acute angles such that cos A = cos B, then show that ∠A = ∠B
1. In ∆ABC, right-angled at B, AB = 24 cm, BC = 7 cm. Determine
In ΔABC , CD ⊥ AB.
i) sin A, cos A (ii) sin C, cos C
In Δ ABC, ∠B = 90º ∴ by Pythagoras theorem, According to question, cos A = cos B
AC = AB + BC = (24)2 + 7 2
2 2 2 ⇒ = ⇒ =
= (576+49) cm2 = 625 cm2 ⇒ AC = 25 = = k DVgÀ°.
(i) sin A = = , cos A = =
⇒ AD = kBD (1)
(ii) sin C = = ; cos C = = ⇒ AC = kBC (2)
2. In Fig. 8.13, find tan P – cot R. In ΔCAD and ΔCBD , By Pythagoras theorem,
In ΔPQR , By Pythagoras theorem,
CD2 = AC2 - AD2 (3)
QR2 = PR2 - PQ2 = (13)2 - (12)2 = 169 - 144 CD2 = BC2 - BD2 (4)
⇒ QR2 = 25 ⇒ QR = 5 cm From (3) and (4) , AC2 - AD2 = BC2 - BD2
FUÀ, tan P = =
⇒ (kBC)2 - (k BD)2 = BC2 - BD2 ⇒ k2 (BC2 - BD2) = BC2 - BD2
cot R = = ⇒ k2 = 1 ⇒ k = 1
tan P – cot R = - =0 Substitute k = 1 in (2),
AC = BC ⇒ ∠A = ∠B [ Angle opposite to the equal sides are equal]
3. If sin A = calculate the value of cos A and tan A ( )( )
7. If cot θ = then find the value of i) ( ii) cot2 θ
In ΔABC, ∠B = 900 According to question, )( )

sin A = = ⇒ AC = 4k, BC = 3k [ Here, Let k = 1 ] In Δ ABC , ∠B = 90º and ∠C = θ According to question,


∴ By Pythagoras theorem, cot θ = = ⇒ AB = 8 ªÀÄvÀÄÛ BC = 7 [ If k = 1 ]
AB2 = AC2 - BC2 ⇒ AB2 = AC2 - BC2 In Δ ABC By Pythagoras theorem,
AB2 = 42 - 32 = 16 – 9 = 7 ⇒AB = √7
AC2 = AB2 + BC2 ⇒AC2 = 8 2 + 72

∴cos A = = ⇒ AC2 = 64 + 49 ⇒AC2 = 113 ⇒AC = √113
tan A = = sin θ = = and cos θ = =
√ √ √

P a g e 34 | 101 Available in ykoyyur.blogspot.com P a g e 35 | 101 Available in ykoyyur.blogspot.com


SSLC Mathematics Solutions in English YK SSLC Mathematics Solutions in English YK
( )( )
(i) = ii) sec A = for some values of A
( )( )
iii) cos A is the abbreviation used for the cosecant of angle A

= = = = = iv) cot A is the product of cot and A
√ v) sin θ = for some values of θ
cot2θ = =
(i) False
8. If 3 cot A = 4 Check whether = cos2 A - sin2 A or not? In ∆ABC, ∠B = 90 0,
In Δ ABC , ∠B = 90º Given, Let AB = 3, BC = 4 and AC = 5 ⇒tan A = > 1
cot A = = ⇒AB = 4 ªÀÄvÀÄÛ BC = 3, [ taken k = 1] (ii) True
In Δ ABC by Pythagoras theorem, In ∆ABC , ∠B = 90 0,
AC2 = AB2 + BC2 ⇒AC2 = 42 + 32 AC =13k and AB = 5k [ k Positive real number]
⇒AC2 = 16 + 9⇒AC2 = 25 ⇒AC = 5 ⇒ AC =12,BC=5 and AB = 5 [If k = 1]
tan A = = ; sin A = = ; cos A = = BC2 = AC2 - AB2 [By Pythagoras theorem]
– ⇒BC2 = 122 - 52 ⇒BC2 = 144 – 25 ⇒BC2 = 119⇒sec A =
LHS = = = = = =
(iii) False
2 2
cosecA is a abbreviation of CosecantA and CosA is the abbreviation of CosineA.
R.H.S. = cos A – sin A = - = – =
(iv) False
⇒R.H.S. = L.H.S. ∴ = cos2 A - sin2 A cot A is not a product of cot and A . it is just a symbol
9. In triangle ∆ABC, right-angled at B, if tan A = then find the value of (v) False

i) sin A cos C + cos A sin C ii) cos A cos C - sin A sin C sin θ = In any triangle hypotenuse is the larger side.
In Δ ABC , ∠B = 90º Given, tan A = = ∴ sin θ is always ≤ 1 ⇒sin θ = , it is not possible for any value of θ

Let AB = √3 and BC = 1
In Δ ABC By Pythagoras theorem, 11.3 Trigonometric Ratios of Some Specific Angles:
AC2 = AB2 + BC2 ⇒AC2 = (√3)2 + (1)2 ⇒AC2 = 3 + 1 Trigonometric ratio of 450
⇒AC2 = 4 ⇒AC = 2 In ∆ABC, ∠B = 900,
√ √ ∠A = 450 ⇒ ∠C = 450 [Sum of interior angles is 1800]
sin A = = ; cos A = = ; sin C = = ;cos C = =

(i) sin A cos C + cos A sin C = x +



x

= + = =1 ⇒ Let AB = BC = 1 , By Pythagoras theorem,
∴ AC2 = AB2 + BC2 ⇒ AC2 = 1 2 + 12 = 1+1 = 2
√ √ √ √
(ii) cos A cos C – sin A sin C = x - x = - =0 ⇒ AC = √2
10. In .∆PQR, right-angled at Q, PR + QR = 25 cm and PQ = 5 cm. Determine the values of 1
Sin450 Cosec450 √2
sin P, cos P and tan P. √2
Given PR + QR = 25 , PQ = 5 1
Let PR = x. ∴ QR = 25 - x Cos450 Sec450 √2
√2
By Pythagoras theorem, PR2 = PQ2 + QR2
Tan450 1 Cot450 1
x2 = (5)2 + (25 - x)2 ⇒ x2 = 25 + 625 + x2 - 50x Trigonometric Ratios of 300 and 600
⇒ 50x = 650 ⇒ x = 13 In equilateral triangle, the angles are equal.
∴ PR = 13 cm ⇒ QR = (25 - 13) cm = 12 cm ⇒∠A = ∠B = ∠C = 600
sin P = = ; cos P = = ; tan P = = Draw AD⟘BC ⇒BD = CD
[sIn an equilateral triangle the perpendicular
11. State whether the following are true or false. Justify your answer.
i) The value of tan A is always less than 1. from the vertex bisects the base]

P a g e 36 | 101 Available in ykoyyur.blogspot.com P a g e 37 | 101 Available in ykoyyur.blogspot.com


SSLC Mathematics Solutions in English YK SSLC Mathematics Solutions in English YK

⇒∠BAD =∠CAD = 30 0 E xa mple 6 : In ∆ABC , r ight- a ngled a t B, AB = 5


Let AB = BC = CA = 2, ⇒BD = CD = 1 cm and ∠ACB = 30° (see Fig. 11.19). Determine the lengths
of the sides BC and AC.
In ∆ABD , By Pythagoras theorem,
Tan300 = ⇒ = ⇒ 5√3cm
AD2 = AB2 – BD2⇒ AD2 = 2 2 – 12 = 4 – 1 = 3 ⇒AD = √3 √

√3 2 1 Sin300 = ⇒ = ⇒ AC = 10cm
Sin600 Cosec600 Sin300 Cosec30 0 2
2 √3 2 Example 7 : In ∆PQR, right - angled at Q (see Fig. 11.20),
2 PQ = 3 cm and PR = 6 cm. Determine ∠QPR and ∠PRQ.
1 √3
Cos600 Sec600 2 Cos300 Sec30 0 SinR = =
2 2 √3
1 1 ⇒ ∠R = 300 ⇒ ∠PRQ = 300
Tan600 √3 Cot600 Tan300 Cot30 0 √3 ∴ ∠QPR = 600
√3 √3
Example 8: If sin (A - B) = and cos (A + B) = , 0 < A + B ≤ 90 , A > B find A and B
Trigonometric ratios of 00 and 900 If sin (A - B) = then sin 300 = ⇒ A – B = 300 (1)
If ∠A Closer to 00 then the length of BC closer to 0 If cos (A + B) = then cos 600 = ⇒ A + B = 600 (2)
and almost AB = AC (1) + (2) = 2A = 90 0 ⇒ A = 450
From (2) ⇒ 450 – B = 300 ⇒ B = 150
Sin00 0 Cosec00 ND
Exercise 11.2
Cos00 1 Sec00 1
1. Evaluate the following:
Tan00 0 Cot00 ND i) sin 600 cos 300 + sin 300 cos 600 ii) 2tan245 0 + cos230 0 - sin2600
Sin900 0 Cosec90 0 ND iii) iv) iv)
If ∠A closer to 900 Then the length of AB closer
i) sin 600 cos 30 0 + sin 300 cos 600
to 0 and almost AC = AC Cos900 1 Sec90 0 1 √ √
Let AB = AC = 1 and BC = 0 = + = + =1
0 0
Tan90 0 Cot90 ND ii) 2tan 450 + cos2300 - sin2600
2

√ √
= 2 (1) + - =2
Table 11.1
iii)
∠A 00 300 450 600 900
√ √ √ √ √ √ √ √ √
1 1 √3 = = √
= = = x =
0 1 √ ( √ ) √ √ √ √ √ √ √ √
Sin √ √
2 √2 2 √ √ (√ √ ) (√ √ ) √ √ √ √
1 = = = = =
√3 1
Cos 1 0
2 √2 2 iv)
1 √ √ √
Tan 0 1 √3 ND
= √
= √
= √
=

√3 √ √ √ √

2 √ √
osec ND 2 √2 1 √ √ √ √ √
√3 =

x

=
( ) √
2 √ √ √ √
Sec 1 √2 2 ND = = =
√3
1
Cot ND √3 1 0 iv)
√3

P a g e 38 | 101 Available in ykoyyur.blogspot.com P a g e 39 | 101 Available in ykoyyur.blogspot.com


SSLC Mathematics Solutions in English YK SSLC Mathematics Solutions in English YK

⇒ sin θ ≠ cos θ for all values of θ



=

= = = ∴ The statement is false
v) cot A is not defined for A = 00
2. Choose the correct option and justify your choice: The statement is true
i) 11.4 Trigonometric Ratios of Complementary Angles
Two angles are given and if their sum is equal to 900 then angles are called
A) sin 60 0 B) cos 600 C) tan 600 D) sin 30 0 complementary Angles
√ √ √ √
= = = = Trigonometric Trigonometric ratios of
√ ratios complementary angles
Ans: A) sin 600 SinA c Cos(90-A)
a
ii )
CosA b Sin(90-A)
A) tan 900 B) 1 C) sin 450 D) 0
a
= = 0 c
TanA Cot(90-A)
Ans: D) 0 b
iii) sin 2A = 2 sin A is true when A = CosecA a Sec(90-A)
A) 0 B) 30 C) 45 D) 60 c
sin 2x0 = 2 sin 0 ⇒ sin 0 = 2 sin0 ⇒ 0 = 0 a
SecA Cosec(90-A)
Ans: A) 0 b
iv) CotA b Tan(90-A)
A) cos 600 B) sin 600 C) tan 600 D) sin 30 0 c
√ √ √
= = = =√ Example 9 : Evaluate -


( )
Ans: C) tan 600 = = = 1
3. If tan (A + B) = √ and tan (A - B) = , 0< A + B ≤ 90 ; A > B find A and B Example 10 : If sin 3A = cos (A – 26°), where 3A is an acute angle, find the value

Given sin 3A = cos (A – 26)
tan (A + B) = √3 ⇒ A + B = 600 (1)
⇒Cos(90-3A) = cos(A-26 0) ⇒90-3A = A-260
tan (A - B) = ⇒ A – B = 300 (2) ⇒90 + 26 = A + 3A ⇒116 = 4A ⇒A = 290

(2) – (1) ⇒ 2B = 300 ⇒ B = 150 ⇒ (1) jAzÀ A = 60 – 15 = 450 Example 11 : Express cot 85° + cos 75° in terms of trigonometric ratios of angles
4. State whether the following are true or false. Justify your answer. between 00 and 450
i) sin (A + B) = sin A + sin B cot 850 = Cot(90-5 0) = tan50
Let A = 30 0 and B = 90 0 Cos750 = Cos(90 -15 0) = Sin15 0
√ √
sin (30 0 + 60 0) = sin 900 = 1 ⇒ sin 300 + sin 600 = + =
∴ sin (A + B) ≠ sin A + sin B Exercise 11.3
∴ The statement is false
1. Evaluate: i) ii) iii) cos480 – sin420 vi) cosec310 – sec59 0
ii) The value of sin θ increases as θ increases
sin 0 0 = 0 , sin 90 0 = 1 i)
∴ The statement is true ( )
iii)The value of cos θ increases as θ increases. = = =1
Cos 00 = 1 , Cos 900 = 0
Here, we observe that as θ increases the value of cos θ dicreases ii)
∴ The statement is false =
( )
= =1
iv)sinθ = cosθ for all values of θ
√ iii) cos48 – sin420
0
sin 300 = ; cos 300 = cos480 – sin(90-480) = cos480 – cos480 = 0
P a g e 40 | 101 Available in ykoyyur.blogspot.com P a g e 41 | 101 Available in ykoyyur.blogspot.com
SSLC Mathematics Solutions in English YK SSLC Mathematics Solutions in English YK

vi) cosec310 – sec59 0 ∴ tanA = ⇒ ⇒secA = ⇒


√ √
cosec310 – sec590 = cosec310 – sec( 90 - 310) = cosec310 – cosec31 0 = 0
2. Show that i) tan480 tan230 tan420 tan67 0 = 1 ii) cos380 cos52 0 – sin38=0 sin52 0 = 0 i) Example 13 : Prove that sec A (1 – sin A)(sec A + tan A) = 1.
tan480 tan230 tan42 0 tan670 = 1 LHS = sec A (1 - sin A)(sec A + tan A)
LHS = tan480 tan230 tan(90-480) tan(90-23 0)
= (1 - sin A) + = = = =1
= tan480 tan230 cot 480 cot 230 = tan48 0 x tan230 x x =1 − −
Example 14: Prove that =
ii) cos380 cos52 0 – sin380 sin520 = 0 + +
LHS = cos380 cos520 – sin380 sin520 cosec A − 1
= = = = =
= cos380 cos520 – sin(90 - 520) sin(90-38)0 = cos380 cos52 0 – cos520 cos38 0 cosec A + 1

= cos380 cos520 – cos520 cos38 0 = 0 RHS GzÁºÀgÀuÉ 15:Prove that = using the identity sec2θ = 1 + tan2 θ
3. If tan 2A = cot (A - 180) and 2A is an acute angle find the value of A
tan 2A = cot (A - 180) = =
⇒ cot(90-2A) = cot(A-180) ⇒ 900 -2A = A-18 0 ⇒3A = 1080 ⇒A = 360 ( ) ( )( ) ( )
4. If tan A = cot B, Prove that A + B = 900 = ( )
x = ( )( )
LHS = tan A = cot B ( ) ( ) )
= =(
⇒ cot(90-A) = cotB ⇒90 – A = B ⇒ A + B = 900 ( )( ) )( )
5. If sec 4A = cosec (A - 20 0) and 4A is an acute angle find the value of A =( =
)
sec 4A = cosec (A - 200)
⇒cosec(90 – 4A) = cosec(A – 200) ⇒90 – 4A = A – 200 ⇒5A = 110 ⇒A = 220 Exercise 11.4
( )
6. If A, B and C are the interior angles of ∆ABC then show that sin =cos 1. Express the trigonometric ratios sinA, secA ªÀÄvÀÄÛ tanA in terms of cotA
Let A , B and C are the interior angles of ∆ABC cosec2A - cot2A = 1
⇒A + B + C = 1800 ⇒ B + C = 180 – A ±
⇒ cosec2A = 1 + cot2A ⇒ = 1 + cot2A ⇒sin2A = ⇒sinA = √
⇒ = ⇒ = 90 −
( ) ( ) sin2A = ⇒ 1 - cos2A = ⇒cos2A = 1 – ⇒cos2A =
⇒sin = sin 90 − ⇒sin = cos
±√
7. Express Sin670 + cos75 0 in terms of the trigonometric ratios in between 00 and 45 0 ⇒ = ⇒ sec2A = ⇒ secA = ⇒ tan A =
Sin670 + cos75 0 2. Write all the trigonometric ratios ∠A in terms of sec A
= sin(90-230) + cos(90-150) =cos 230 + sin15 0 sec A = ⇒ cos A =
cos2A + sin2A = 1 ⇒ sin2A = 1 - cos2A ⇒ sin2A = 1 -
11.5 wæPÉÆãÀ«Äw ¤vÀå¸À«ÄÃPÀgÀtUÀ¼ÀÄ
±
⇒ sin2A = ⇒ sinA =
You may recall that an equation is called an identity when it is true for all values of the variables
±
involved. Similarly, an equation involving trigonometric ratios of an angle is called a sin A = ⇒cosec A = ⇒ cosec A =
trigonometric identity, if it is true for all values of the angle(s) involved.
Sin2A + cos2A = 1 sec A - tan A = 1 ⇒ tan A = sec A + 1
2 2 2 2

tan2 + 1 = sec2A ⇒ tanA = +


1 + cot2A = cosec2A tan A = ⇒ cot A = ⇒ cot A =
Note: = tanA
3. Evaluate:
= cotA i) ii) sin 25 0 cos 650 + cos 250 sin 650
i)
Example 12 : Express the ratios cos A, tan A and sec A in terms of sin A. ( )
= = = =1
cos2A + sin2A = 1 ⇒ cos2A = 1 - sin2A ( )

cos A = √1 − sin2A ii) sin 25 cos 650 + cos 250 sin 65 0


0

P a g e 42 | 101 Available in ykoyyur.blogspot.com P a g e 43 | 101 Available in ykoyyur.blogspot.com


SSLC Mathematics Solutions in English YK SSLC Mathematics Solutions in English YK

sin 25° cos 65° + cos 25° sin 65° iii) + = 1+ sec θ.cos θ
= sin(90°-25°) cos 65° + cos(90°-65°) sin 65°
[Hint : Write the expression in terms of sin and cos ]
= cos 65° cos 65° + sin 65° sin 65° = cos265° + sin265° = 1
4. Choose the correct option and justifyyour choice L.H.S. = +
i) 9 sec2A - 9 tan2A
A) 1 B) 9 C) 8 D) 0 = + = +
9 sec2A - 9 tan2A
2 2
= 9 (sec A - tan A)
= 9×1 = 9 [∵ sec2 A - tan2 A = 1] = ( )
+ ( )
Ans: B) 9
ii) (1 + tan θ + sec θ ) (1 + cot θ - cosec θ) = =
( )
-
( )
A) 0 B) 1 C) 2 D) -1
(1 + tan θ + sec θ) (1 + cot θ - cosec θ) = −
( )
= 1 + + 1 + +
= × =( ) .
( ) ( )( (
= = = = =2 =( =
. . . . ) . .
Ans C) 2
= = +1 = 1 + sec θ cosec θ = R.H.S.
iii) (secA + tanA) (1 - sinA) = . .
A) secA B) sinA C) cosecA D) cosA
iv) = = 2secA
(secA + tanA)(1 - sinA)
[Hint: simplify LHS and RHS separately]
(secA + tanA) (1 - sinA)
= + (1 - sinA) = (1 - sinA) = = = cos A L.H.S. = = =
Ans: D) cosA
= x = cos A + 1
iv) = ( )( )
A) 2
sec A B) -1 C) 2
cot A D) tan A2 R.H.S. = = = cos A + 1
L.H.S. = R.H.S.
v) Prove that = cosec A + cot A using the identity cosec2A = 1 + cot2A
= = x = = tan2A L.H.S. =

2 –
Ans: D) tan A –

5. Prove the following identities, where the angles involved are acute angles for which the = – = [ Divide both denominator and numerator by sin A]

expressions are defined.

i) (cosec θ - cot θ)2 = = –
(using cosec2A - cot2A = 1)
L.H.S. = (cosec θ - cot θ) 2
=
–( )
=
( )( – )
– –
= (cosec θ + cot θ - 2cosec θ cot θ) =
2 2
+ − = cot A + cosec A = R.H.S.
( )
= = = RHS vi) = sec A + tan A
( )( )

ii) + = 2secA ( )
= x =
L.H.S. = +
( )
( ) = = = + = secA + tanA=RHS
= =
( ) ( )

= = =
( )
= = 2 sec A = R.H.S. vii) = tan
( ) ( ) ( )
L.H.S. =
P a g e 44 | 101 Available in ykoyyur.blogspot.com P a g e 45 | 101 Available in ykoyyur.blogspot.com
SSLC Mathematics Solutions in English YK SSLC Mathematics Solutions in English YK
( ) [ ]
= ( )
= ( )
[ ] [ ]
= ( )
= ( )
= = tan θ = R.H.S. Hypotenuse
viii) (sin A + cosec A) + (cos A + sec A)2 = 7 + tan2 A + cot2 A
2
Adjacent side
L.H.S. = (sin A + cosec A)2 + (cos A + sec A)2
Adjacent side
= sin2A + cosec2A + 2 sin A cosec A + cos2A + sec2A + 2 cos A sec A
Opposite side
= (sin2A + cos2A) + 2 sin A + 2 cos A + 1 + tan2A + 1 + cot2A
2 2 2 2
= 1 + 2 + 2 + 2 + tan A + cot A = 7+tan A+cot A = R.H.S.
ix) (cosec A - sin A)(sec A - cos A) = 3. If one of the trigonometric ratios of an acute angle is known, the remaining trigonometric
ratios of the angle can be easily determined.
[Hint: simplify LHS and RHS separately] 4. The value of sin A or cos A never exceeds 1, whereas the value of sec A or cosec A is always
L.H.S. = (cosec A – sin A)(sec A – cos A) greater than or equal to 1.
= − sin A − cosA 5. sin (90° – A) = cos A, cos (90° – A) = sin A;
tan (90° – A) = cot A, cot (90° – A) = tan A; s
= = cos A sin A
ec (90° – A) = cosec A, cosec (90° – A) = sec A
R.H.S. = = 6. sec2 A - tan2 A = 1, 00 ≤ A < 900
.
cosec2 A = 1 + cot2 A, 0 0 ≤ A < 900
= = cos A. sin A
. sin2 A + cos2 A = 1,
L.H.S. = R.H.S.
x) = =
L.H.S. =
= = = = tan2A

= = ( ) = (−tanA) = tan2A

Summery:
1. In right angle triangle ABC, ∠B = 900

SinA

Adjacent side
CosA
Hypotenuse
Opposite side
Tan A
Adjacent

P a g e 46 | 101 Available in ykoyyur.blogspot.com P a g e 47 | 101 Available in ykoyyur.blogspot.com


SSLC Mathematics Solutions in English YK SSLC Mathematics Solutions in English YK

tan 600 = ⇒ √3 = ⇒ BC = 15√3 m


Some Applications of Trigonometry Example 2 : An electrician has to repair an electric fault on a pole of height 5 m. She
Trigonometry is one of the most ancient subjects studied by scholars all over the world. As we needs to reach a point 1.3m below the top of the pole to undertake the repair work (see Fig.
have said in Chapter 11, trigonometry was invented because its need arose in astronomy. Since 12.5). What should be the length of the ladder that she should use which, when inclined at an
then the astronomers have used it, for instance, to calculate distances from the Earth to the planets angle of 60° to the horizontal, would enable her to reach the required position? Also, how
and stars. Trigonometry is also used in geography and in navigation. The knowledge of far from the foot of the pole should she place the foot of the ladder? (You may take √
trigonometry is used to construct maps, determine the position of an island in relation to the = 1.73)
longitudes and latitudes. Surveyors have used trigonometr y for centuries. One such large Height of the PoleAD = 5m; The height in
surveying project of the nineteenth century was the ‘Great Trigonometric Survey’ of British which repair work to be done BD
India for which the two largest-ever theodolites were built. During the survey in 1852, the = 5 – 1.3 = 3.7m
highest mou nt a in in t he wor ld wa s discover ed. F r om a dist ance of over160 km, Height of the Ladder BC = ?.
the peak was observed from six different stations. In 1856, this peak was named after Sir George Distance between the foot of the pole and
Everest, who had commissioned and first used the giant theodolites (see the figure alongside). the foot of the ladder CD=?
The theodolites are now on display in the M us eum of t he S ur vey of Indi a in √
Dehradun. Sin 600 = ⇒
. . .
12.2 Height and distance: = ⇒ BC = = m≈ = 4.28m
√ √
. .
Thus, the line of sight is the line Tan60 = 0
⇒ √3 = = m ≈ 2.14m

drawn from the eye of an observer ∴ Height of the Ladder BC = 4.28m and
to the point in the object viewed by Distance between the foot of the pole and
the observer. The angle of the foot of the ladder CD = 2.14m
elevation of the point viewed is the Example 3 : An observer 1.5 m tall is 28.5
angle formed by the line of sight m away from a chimney. The angle of
with the horizontal when the point
elevation of the top of the chimney from
being viewed is above the
her eyes is 45°. What is the height of the
horizontal level, i.e., the case when
we raise our head to look at the chimney?
object Height of the observer CD = BE = 1.5m,
Thus, the angle of depression of a Distance from the chimney to the observer
DE = CB = 28.5m;
point on the object being viewed is
Height of the chimney AB = ?
the angle formed by the line of sight
with the horizontal when the point tan450 = ⇒ 1 = . ⇒ AE = 28.5m
is below the horizontal level, i.e., ∴ Height of the chimney AB = AE + BE = 28.5 + 1.5 = 30m
the case when we lower our head Example 4 : From a point P on the
to look at the point being viewed ground the angle of elevation of the top of
Example1 : A tower stands a 10 m tall building is 30°. A flag is hoisted
vertically on the ground. From a at the top of the building and the angle of
point on the ground, which is 15 elevation of the top of the flagstaff from
m away from the foot of the P is 45°. Find the length of the flagstaff
tower, the angle of elevation of the and the distance of the building from the
top of the tower is found to be point P. (you take √ = 1.732 )
60°. Find the height of the tower. Height of the building AB = 10m
Tan300 = ⇒ = ⇒ AP = 10√3 = 10x1.732 = 17.32m

Tan450 = ⇒ 1 = . ⇒ AD = 17.32m
Let Height of the tower = BC; AB = 15m ∴ Length of the flagstaff = AD – AB = 17.32 – 10 = 7.32m

P a g e 48 | 101 Available in ykoyyur.blogspot.com P a g e 49 | 101 Available in ykoyyur.blogspot.com


SSLC Mathematics Solutions in English YK SSLC Mathematics Solutions in English YK

Example 5 : The shadow of a tower standing on a level ground is found to be 40 m longer Width of the river = AD + BD = 3√3 + 3 = 3(√3 + 1)m
when the Sun’s altitude is 30° than when it is 60°. Find the height of the tower.
Let length of the shadow when Sun’s altitude 600 BC = x m
∴ length of the shadow when Sun’s altitude 300 BD = (40 + x)m
tan600 = ⇒√3 = ⇒ AB = √3 -------------------------- (1) Exercise 12.1
tan300 = ⇒ = ⇒ 40+x = √3AB
√ A circus artist is climbing a 20 m
⇒ 40 + x = √3 . √3 long rope, which is tightly stretched
⇒ 40 + x = 3x and tied from the top of a vertical
⇒ 2x = 40 ⇒ x = 20m pole to the ground. Find the height
∴ (1) ⇒ AB = √3 ⇒ AB = 20√3 m of the pole, if the angle made by the
Example 6 : The angles of depression of the top and the bottom of an 8 m tall building from rope with the ground level is 30° (see
the top of a multi-storeyed building are 30° and 45°, respectively. Find the height of the Fig. 9.11).
multi-storeyed building and the distance between the two buildings. Height of the pole BC
Height of the building = AB = 8m Sin300 = ⇒ = ⇒ BC = 10m
Height of the multi-storeyed building
∴ Height BC = 10m
PC = PD + CD = PD + AB = PD + 8m
------------ (1) 2. A tree breaks due to storm and the broken part bends so that the top of the tree touches
Distance between the buildings=AC = BD the ground making an angle 30° with it. The distance between the foot of the tree to the
PQǁBD, point where the top touches the ground is 8 m. Find the height of the tree.
∴ ∠BPQ = ∠PBD [ Alternate angles] Let BC is the broken part of the tree
∴tan30 0 = ⇒ = ∴ Total height of the tree = AB+BC

cos 30° =
⇒ BD = √3PD ------------------------(2)

PQǁAC, ⇒ = ⇒ BC =

∴ ∠APQ = ∠PAC [Alternate angles]
tan 30° =
∴tan45 0 = ⇒ 1 = [ From (1) and (2)]

(√ ) ⇒ = ⇒ AB = m
⇒√3PD = PD + 8 ⇒PD(√3 – 1) = 8 ⇒PD = = = 4(√3 + 1) √ √
√ – ∴ Height of the tree
∴ Height of the multi-storeyed building PC = PD + 8m =4(√3 + 1) + 8
= AB+ BC = + = m
= 4√3 + 12 = 4(3 + √3) √ √ √

∴ Distance between the buildings À = Distance between the buildings =4(3 + √3) 3. A contractor plans to install two slides for the children to play in a park. For the children
below the age of 5 years, she prefers to have a slide whose top is at a height of 1.5 m, and is
[Distance between the buildings = AC = BD ⇒ BD
inclined at an angle of 30° to the ground, whereas for elder children, she wants to have a
= 4√3 (√3 + 1) [ (2) jAzÀ ] ⇒ BD =4(3 + √3)m ] steep slide at a height of 3 m, and inclined at an angle of 60° to the ground. What should
Example: From a point on a bridge across a river, the angles of depr ession of the be the length of the slide in each case?
banks on opposite sides of the river are 30° and 45°, respectively. If the bridge is at a
height of 3 m from the banks, find the width of the river.
Width of the river = AD + BD
MNǁAB ⇒ ∠MPA = ∠A = 300 and
∠NPD = ∠B = 450 [ Alternate angles]
tan300 = ⇒ =

⇒ AD = 3√3 m ---------------------(1)
Tan450 = ⇒ 1 =
⇒BD = 3m --------------------(2)
From (1) and (2)

P a g e 50 | 101 Available in ykoyyur.blogspot.com P a g e 51 | 101 Available in ykoyyur.blogspot.com


SSLC Mathematics Solutions in English YK SSLC Mathematics Solutions in English YK

Let the length of the side making inclination 600 = AC and CD = OC - OD = (30 - 1.5) = 28.5 m, According to question
.
Length of the slide making inclination 30° = PR In right angle ΔADC, tan 30° = ⇒ = ⇒ AD = 28.5√3 m

According to question, . .
In right angle ΔCBD AiÀÄ°è, tan 60° = ⇒ √3 = ⇒ BD = = 9.5√3 m
In right angle triangle ΔABC, √

sin 30° = ⇒ =
.
⇒ PR = 3m ∴ MN = AB = x = (28.5√3 - 9.5√3 ) = 19√3 m
In right angle triangle ΔPQR, ∴The distance he walked towards the building = 19√3 m
√ 7. From a point on the ground, the angles of elevation of the bottom and the top of a
sin 60° = ⇒ = ⇒ AC = m = 2√3 m

transmission tower fixed at the top of a 20 m high building are 45° and 60° respectively.
∴ Length of the slides 3m and 2√3 m .
Find the height of the tower
4. The angle of elevation of the top of a tower from a point on the ground, which is 30 m
Height of the building = BC= 20 m
away from the foot of the tower, is 30°. Find the height of the tower
Let height of the tower = AB A point on the ground where the angle of
Distance from the foot of the tower to the point elevation measured is D
BC = 30m Height of the tansmission tower AB = AC - BC
In right angle triangle ΔABC , According to question,
tan300 = In right angle triangle ΔBCD,
⇒ = tan 45° = ⇒ 1 =

⇒ AB = = 10√3m ⇒ CD = 20 m
√ In right angle triangle ΔACD,
5. A kite is flying at a height of 60 m above
the ground. The string attached to the tan 60° = ⇒ √3 =
kite is temporarily tied to a point on the ⇒ AC = 20√3 m
ground. The inclination of the string Height of the tansmission tower AB = AC - BC = (20√3 - 20) m = 20(√3 - 1) m.
with the ground is 60°. Find the length 8. A statue, 1.6 m tall, stands on the top of a pedestal. From a point on the ground, the
of the string, assuming that there is no angle of elevation of the top of the statue is 60° and from the same point the angle of
slack in the string.
elevation of the top of the pedestal is 45°. Find the height of the pedestal.
Height of the kite BC = 60m
Let the height of the statue = AB
Length of the tread = AB,
The point where the angle of elevation measured is D
In right angle triangle ΔABC ,
√ Height of the pedestal BC = AC – AB
Sin600 = ⇒ =
By question, In right angle ΔBCD ,
⇒AB = = 40√3m tan 45° = ⇒ 1 =

6. A 1.5 m tall boy is standing at ⇒ BC = CD.
some distance from a 30 m tall In right angle ΔACD,
building. The angle of
tan 60° =
elevation from his eyes to the
top of the building increases ⇒ √3CD = 1.6 m + BC ⇒ √3BC = 1.6 m + BC
from 30° to 60° as he walks ⇒ √3BC - BC = 1.6 m ⇒ BC(√3 -1) = 1.6 m
towards the building. Find the ⇒ BC(√3 -1) =
.
⇒ BC = 0.8(√3 +1) m
distance he walked towards √
the building? ∴Height of the pedestal BC = 0.8 (√3+1) m.
Angle of elevation when the boy 9. The angle of elevation of the top of a building from the foot of the tower is 30° and the
is at M = 300 After walking x
angle of elevation of the top of the tower from the foot of the building is 60°. If the tower is
meter, the angle of elevation is
600 at N. 50 m high, find the height of the building.
∴ MN = AB = x.
Height of the building = OC = 30 m

P a g e 52 | 101 Available in ykoyyur.blogspot.com P a g e 53 | 101 Available in ykoyyur.blogspot.com


SSLC Mathematics Solutions in English YK SSLC Mathematics Solutions in English YK

Given: the height of the tower CD = 50 m According to question, In triangle ΔABD,


Let the height of the building = AB tan 30° = ⇒ =

Distance from the foot of the building to the ( )
⇒ AB = -------------------- (1)
tower = BC √
According to question In right angle triangle In right angle triangle ΔABC,
ΔBCD, tan 60° = ⇒ √3 =
tan 60° = ⇒ √3 = ⇒ BC = ⇒ AB = √3 BC ------------------------- (2)

In right angle triangle ΔABC, From equation (1) and (2)
( )
tan 30° = = √3 BC ⇒ 3 BC = 20 + BC ⇒ 2 BC = 20 ⇒ BC = 10 m

⇒ = ⇒ AB = ⇒ AB = m Substitute BC =10 in equation (2) AB = 10√3 m


√ √ The height of the tower = 10√3 m and width of the canal = 10 m.
Therefore height of the building = m = 16 m
12. From the top of a 7 m high building, the angle of elevation of the top of a cable tower is 60°
10. Two poles of equal heights are standing opposite each other on either side of the road, and the angle of depression of its foot is 45°. Determine the height of the tower
which is 80 m wide. From a point between them on the road, the angles of elevation of the Height of the building AB = 7 m ; Height of the tower = EC
top of the poles are 60° and 30°, respectively. Find the height of the poles and the
A is the point of elevation
distances of the point from the poles. EC = DE + CD
AB and CD are the two poles of equal height CD = AB = 7 m ªÀÄvÀÄÛ BC = AD
The point on the ground where the angle of elevation is measured is O.
By question, In right angle triangle ΔABC,
The distance between the poles = BD tan 45° = ⇒ 1= ⇒ BC = 7 m = AD
According to question, In right angle triangle ΔADE,
AB = CD, OB + OD = 80 m tan 60° = ⇒ √3 =
In right angle triangle ΔCDO,
⇒ DE = 7√3 m
tan 30° = ⇒ = Height of the tower = EC = DE + CD

= (7√3 + 7) m = 7√3+1) m.
⇒ CD = ------------------- (1)
√ 13. As observed from the top of a 75 m high lighthouse from the sea-level, the angles of
In right angle triangle ΔABO, depression of two ships are 30° and 45°. If one ship is exactly behind the other on the
tan 60° = ⇒ √3 = ⇒ AB = √3 (80-OD) same side of the lighthouse, find the distance between the two ships
Height of the lighthouse AB = 75 m.
AB = CD [Given]
Let the positons of the ships C and D
⇒ √3 (80-OD) = ⇒ 3(80-OD) = OD According to question,

⇒ 240 - 3 OD = OD ⇒ 4 OD = 240 ⇒ OD = 60 In right angle triangle ΔABC,
Substitute OD = 60 in (1) we get, tan 45° =
CD = ⇒ CD = 20√3 m ⇒ 1=

OB + OD = 80 m ⇒ OB = (80-60) m = 20 m ⇒ BC = 75 m
Therefore the height of the poles = 20√3 m and the distance from the point of elevation to In right angle triangle ΔABD
the poles = 60m and 20m tan 30° =
⇒ =
11.A TV tower stands vertically on a bank of a canal. From a point on the other bank directly √
opposite the tower, the angle of elevation of the top of the tower is 60°. From another ⇒ BD = 75√3m
point 20 m away from this point on the line joing this point to the foot of the tower, the Therefore the distance between the ships = CD = BD - BC
angle of elevation of the top of the tower is 300 (see Fig. 12.12). Find the height of the = (75√3 - 75)m = 75(√3-1)m.
tower and width of the canal.

Let Height of the TV tower = AB; CD = 20 m [Given]


P a g e 54 | 101 Available in ykoyyur.blogspot.com P a g e 55 | 101 Available in ykoyyur.blogspot.com
SSLC Mathematics Solutions in English YK SSLC Mathematics Solutions in English YK

14. A 1.2 m tall girl spots a balloon moving with the wind in a horizontal line at a height of The height of the tower AB
88.2 m from the ground. The angle of elevation of the balloon from the eyes of the girl at C and D are the points from the tower at the distance 4 m and 9 m respectively
any instant is 60°. After some time, the angle of elevation r educes to 30° (see
Fig.12.13). Find the distance travelled by According to question,
the balloon during the interval. In right triangle ΔADB,
Let the initial position of the balloon and the later position be A and B respectively tan x = ⇒ tan x =
The height of the ballon = 88.2 m - 1.2 m = 87 m ⇒ AB = 4 tan x ----------------- (1)
The distance travelled by the bolloon In right triangle ΔABD,
= DE = CE - CD tan (90°-x) = ⇒ cot x =
According to question in ΔBEC, ⇒ AB = 9 cot x ---------------- (2)
tan 30° = ⇒ = ⇒ CE = 87√3 m Multiplying equation (1) and (2) ,

In right triangle ΔADC, AB2 = 9 cot x × 4 tan x ⇒ AB2 = 36 ⇒ AB = ± 6
AB = -6 is not possible.
tan 60° = ⇒ √3=
Therefore height of the tower is 6m
⇒ CD = m = 29√3 m

∴ The distance travelled by the bolloon DE = CE - CD = (87√3 - 29√3 ) m = 58√3 m. Summary:
15.A straight highway leads to the foot of a tower. A man standing at the top of the tower 1. (i) The line of sight is the line drawn from the eye of an observer to the point in the object
observes a car at an angle of depression of 30°, which is approaching the foot of the viewed by the observer.
tower with a uniform speed. Six seconds later, the angle of depression of the car is found to (ii) The angle of elevation of an object viewed, is the angle formed by the line of sight with
be 60°. Find the time taken by the car to reach the foot of the tower from this point. the horizontal when it is above the horizontal level, i.e., the case when we raise our head to
Let the height of the tower = AB look at the object.
D is the initial position of the car and C is the later position (iii) The angle of depression of an object viewed, is the angle formed by the line of sight with
BC is the distance from the car to the tower the horizontal when it is below the horizontal level, i.e., the case when we lower our head to
According to question, In right triangle ΔABC, look at the object.
tan 60° = ⇒ √3 = ⇒ BC = m The height or length of an object or the distance between two distant objects can be

In right triangle ΔADB, determined with the help of t
tan 30° =
⇒ =

⇒ AB√3 = + CD

⇒ CD = AB√3 -

⇒ CD = AB √3 −

⇒ CD = AB

⇒ CD = m

Here, distance BC is half the distance CD
Therefore time taken to move BC is half the time taken to move CD
Given that the time taken by the car to move distance CD = 6 sec.
∴ The time taken to move the distance BC = 6/2 = 3 sec.
16. The angles of elevation of the top of a tower from two points at a distance of 4 m and
9 m from the base of the tower and in the same straight line with it are complementary.
Prove that the height of the tower is 6 m

P a g e 56 | 101 Available in ykoyyur.blogspot.com P a g e 57 | 101 Available in ykoyyur.blogspot.com


SSLC Mathematics Solutions in English YK SSLC Mathematics Solutions in English YK

Assumed Mean Method:

Statistics = – a [ Here, a = 47.5 ]


Class
Interval
( )
Mid-point
( )
= – 47.5

13.2 Mean of Grouped data 10-25 2 17.5 -30 -60


25-40 3 32.5 -15 -45

Average: = ∑
[ i = 1 to n ] 40-55 7 47.5 0 0
Example 1 : The marks obtained by 30 students of Class X of a certain school in a 55-70 6 62.5 15 90
Mathematics paper consisting of 100 marks are presented in table below. Find the mean 70-85 6 77.5 30 182
of the marks obtained by the students 85-100 6 92.5 45 270
∑ = 30 ∑ = 435
10 1 10 1 10 ∑
Average ̅ =a + ∑
= 47.5 + = 47.5 + 14.5 = 62
20 1 20 1 20
36 3 108 Step Deviation Method:
36 3
40 4 160 = – a [ Here, a = 47.5 ] and h = 15
40 4 50 3 150 ∑
Average ̅ = ∑
Class
( )
Mid-point
=
– .
50 3 56 2 112 Interval ( )
56 2 = 60 4 240 = 59.53 10-25 2 17.5 -2 -4
70 4 280 25-40 3 32.5 -1 -3
60 4
72 1 72 40-55 7 47.5 0 0
70 4
80 1 80 55-70 6 62.5 1 6
72 1
88 2 176 70-85 6 77.5 2 12
80 1
92 3 276 85-100 6 92.5 3 18
88 2
92 3 95 1 96 ∑ = 30 ∑ = 29
95 1 ∑ = 30 ∑ = 1779 Average ̅ =a +

x h = 47.5 + x 15 = 47.5 + = 47.5 + 14.5 = 62

Note: If all di have common multiple then step deviation method is the best method
Direct Method to find average: We get the same average in all three methods.
No.of Class Mid-point Assumed Mean and step deviation methods are the simplified form of Direct Method.
C.I. ( )
students Interval ( ) Example 2 : The table below gives the percentage distribution of female teachers in the
10-25 2 10-25 2 17.5 35.0
primary schools of rural areas of various states and union territories (U.T.) of India. Find
25-40 3 32.5 97.5
25-40 3 the mean percentage of female teachers by all the three methods discussed
40-55 7 47.5 332.5
a = 50, h = 10 =
40-55 7 55-70 6 62.5 375.0 C.I. =
Percenta − 50
70-85 6 77.5 465.0
55-70 6 ge of
85-100 6 92.5 555.0 Female 15-25 6 20 -30 -3 120 -180 -18
70-85 6
85-100 6 ∑ = 30 ∑ = 1860 teachers 25-35 11 30 -20 -2 330 -220 -22
15-25 6 35-45 7 40 -10 -1 280 -70 -7
25-35 11 45-55 4 50 0 0 200 0 0
∑ 35-45 7
Average ̅ = ∑
= = 62 55-65 4 60 10 1 240 40 4
45-55 4 65-75 2 70 20 2 140 40 4
55-65 4 75-85 1 80 30 3 80 30 3
65-75 2 1390 -360 -36
75-85 1 ∑ = 35

P a g e 58 | 101 Available in ykoyyur.blogspot.com P a g e 59 | 101 Available in ykoyyur.blogspot.com


SSLC Mathematics Solutions in English YK SSLC Mathematics Solutions in English YK

From the above table ∑ = 35, ∑ = 1390, ∑ = -360, ∑ = -36 2. Consider the following distribution of daily wages of 50 workers of a factory. Find the
∑ mean daily wages of the workers of the factory by using an appropriate method
Average in direct Method ̅ = ∑
= = 39.71
a = 75.5, h = 3

Average in Assumed Mean Method ̅ =a + ∑
= 50 - = 50 – 10.29 = 39.71 Daily No.of =
C.I. =
∑ wages workers −
Average in step deviation Method ̅ = a + ∑
x h=50 - x 10 = 50 - 10.29= 39.71 (Rs) 100-120 12 110 -40 -2 1320 -480 -24
Example 3 : The distribution below shows the number of wickets taken by bowlers in one- 100-120 12 120-140 14 130 -20 -1 1820 -280 -14
day cricket matches. Find the mean number of wickets by choosing a suitable method. 120-140 14 140-160 8 150 0 0 1200 0 0
What does the mean signify? 140-160 8 160-180 6 170 20 1 1020 120 6
= 160-180 6
C.I. = 180-200 10 190 40 2 1900 400 20
No.of No.of − 200 180-200 10 50 7260 -240 -12
Wickets Bowlers 20 -60 7 40 -160 -8 -56
20 -60 7 60 -100 5 80 -120 -6 -30 From the above table ∑ = 50, ∑ = 7260, ∑ = -240, ∑ = -12
60 -100 5 100 -150 16 125 -75 -3.75 -60 ∑
100 -150 16 Average from Direct Method ̅ = ∑
= = 145.2
150 -250 12 200 0 0 0
150 -250 12 ∑
250 -350 2 300 100 5 10 Average from assumed Mean Method ̅ =a + ∑
= 150 + = 150 – 4.8 = 145.2
250 -350 2 350 -450 3 400 200 10 30 ∑
350 -450 3 Average from step deviation method ̅ =a + x h =150+ x 20 =150 - 4.8 = 145.2
̅ = a 45 -106 ∑

∑ [Can use any method. But Assumed mean method is more suitable here]
+ ∑
x h = 200 - x 20 = 200 - 47.11 = 152.89
3. The following distribution shows the daily pocket allowance of children of a locality. The
Exercise – 13.1 mean pocket allowance is ` 18. Find the missing frequency f
1. A survey was conducted by a group of students as a part of their environment awareness Daily Pocket 11-13 13-15 15-17 17-19 19-21 21-23 23-25
programme, in which they collected the following data regarding the number of plants in 20 allowences(Rs)
houses in a locality. Find the mean number of plants per house No.of Children 7 6 9 13 f 5 4
Which method did you use for finding the mean, and why?
a = 18 , h = 2
a=7,h=2 =
C.I. = −
No.of No.of − C.I. = =
Plants Houses 0-2 1 1 -6 -3 1 -6 -3 11-13 7 12 -6 -3 84 -42 -21
0-2 1 2-4 2 3 -4 -2 6 -8 -4 13-15 6 14 -4 -2 84 -24 -12
2-4 2 4-6 1 5 -2 -1 5 -2 -1 15-17 9 16 -2 -1 144 -18 -9
4-6 1 6-8 5 7 0 0 35 0 0 17-19 13 18 0 0 234 0 0
6-8 5 8-10 6 9 2 1 54 12 6 19-21 f 20 2 1 20f 2f 1f
8-10 6 10-12 2 11 4 2 22 8 4 21-23 5 22 4 2 110 20 10
10-12 2 12-14 3 13 6 3 39 18 9 23-25 4 24 6 3 96 24 12
12-14 3 ∑ 0 162 22 11
=20 ∑ =44+ f 752+20f -40+2f -20+ f

From the above table ∑ =44+ f, ∑ = 752+ 20f, ∑ = -40 + 2f, ∑ =-20 + f
From the above table ∑ = 35, ∑ = 162, ∑ = 20, ∑ = 11

∑ Average from Direct Method ̅ =
Average from Direct Method ̅ = ∑
= = 8.1 ∑

∑ 18 = ⇒ 18(44 + f)= 752 + 20f


Average from assumed Mean Method ̅ =a + ∑
= 7+ = 7 + 1.1 = 8.1

⇒792 + 18f = 752 + 20f ⇒40 = 2f ⇒ f = 20
Average from step deviation method ̅ = a + ∑
xh = 7 + x 2 = 7 + 1.1 = 8.1 ∑
Average from assumed Mean Method ̅ =a + ∑
[You can use any method. Because of simple tabulation we can use direct method here]
18 = 18 + ⇒ 0 = (−40 + 2f) ⇒ 2f = 40 ⇒ f = 20

P a g e 60 | 101 Available in ykoyyur.blogspot.com P a g e 61 | 101 Available in ykoyyur.blogspot.com


SSLC Mathematics Solutions in English YK SSLC Mathematics Solutions in English YK

Average from step deviation method ̅ = a + xh = 57 + = 57 + 0.1875 = 57.1875 ≈ 57.19


⇒18 = 18 + x 20 ⇒ −20 + f = 0 ⇒ f = 20 Average from step deviation method ̅ = a + ∑
xh
[Wecan use any method here] = 57 + x 3 = 57 + 0.1875 = 57.1875 ≈ 57.19
4. Thirty women were examined in a hospital by a doctor and the number of heartbeats per Here, Assumed mean method is more suitable
minute were recorded and summarised as follows. Find the mean heartbeats per minute 6. The table below shows the daily expenditure on food of 25 households in a locality
for these women, choosing a suitable method Daily expenditure(Rs) 100-150 150-200 200-250 250-300 300-350
No.of Heart No.of households 4 5 12 2 2
65-68 68-71 71-74 74-77 77-80 80-83 83-86
beats/Minute
2 4 3 8 4 2 Find the mean daily expenditure on food by a suitable method.
No.of women 7
a = 225 , h = 50
a = 75.5 , h = 3
.
. C.I. = − =
C.I. = − . =
100-150 4 125 -100 -2 -400 -8
65-68 2 66.5 -9 -3 -18 -6
150-200 5 175 -50 -1 -250 -5
68-71 4 69.5 -6 -2 -24 -8
200-250 12 225 0 0 0 0
71-74 3 72.5 -3 -1 -9 -3
250-300 2 275 50 1 100 2
74-77 8 75.5 0 0 0 0
300-350 2 325 100 2 200 4
77-80 7 78.5 3 1 21 7
80-83 4 81.5 6 2 24 8 ∑ =25 -350 -7
83-86 2 84.5 9 3 18 6 ∑
Average from assumed Mean Method ̅ =a + ∑

=30 12 4
= 225 + = 225 - 14 = 211
From the above table ∑ = 30, ∑ = 12, ∑ = 4

∑ Average from step deviation method ̅ = a + xh
Average from assumed Mean Method ̅ = a + ∑
= 75.5 + = 75.5 + 0.4 = 75.9 ∑

∑ = 225 + x 50 = 225 - 14 = 211


Average from step deviation method ̅ = a + ∑
x h = 75.5 + x 3 = 75.5 + 0.4 = 75.9
For this problem step deviation method is more suitable
[Direct method is not suitable here] 7. To find out the concentration of SO2 in the air (in parts per million, i.e., ppm), the data
5. In a retail market, fruit vendors were selling mangoes kept in packing boxes. These boxes was collected for 30 localities in a certain city and is presented below
contained varying number of mangoes. The following was the distribution of mangoes Find the mean concentration of SO2 in the air
according to the number of boxes.
50-52 53-55 56-58 59-61 62-64 C.I.
No.of Mangoes Concentration Freequency 0.00 – 0.04 4 0.02 0.08
No.of boxes 15 110 135 115 25
of SO2 0.04 – 0.08 9 0.06 0.54
Find the mean number of mangoes kept in a packing box. Which method of finding the mean did 0.00 – 0.04 4
you choose? 0.08 – 0.12 9 0.10 0.90
0.04 – 0.08 9
0.12 – 0.16 2 0.14 0.28
a = 57 , h = 3 0.08 – 0.12 9
0.16 – 0.20 4 0.18 0.72
. 0.12 – 0.16 2
C.I. = − = 0.20 – 0.24 2 0.22 0.44
50-52 15 51 -6 -2 -90 -30 0.16 – 0.20 4
∑ =30 2.96
53-55 110 54 -3 -1 -330 -110 0.20 – 0.24 2
56-58 135 57 0 0 0 0
59-61 115 60 3 1 345 115
∑ .
62-64 25 63 6 2 150 50 Average from Direct Method ̅ = ∑
= = 0.099ppm
∑ =400 75 25 The mean concentration of SO2 in the air = 0.099ppm

Average from assumed Mean Method ̅ =a + ∑
8. A class teacher has the following absentee record of 40 students of a class for the whole
term. Find the mean number of days a student was absent

P a g e 62 | 101 Available in ykoyyur.blogspot.com P a g e 63 | 101 Available in ykoyyur.blogspot.com


SSLC Mathematics Solutions in English YK SSLC Mathematics Solutions in English YK

Example 5 : A survey conducted on 20 households in a locality by a group of students


No.of days No.of students C.I. resulted in the following frequency table for the number of family members in a household
0-6 11 3 33
0-6 11
6-10 10 8 80 Family size 1-3 3-5 5-7 7-9 9 - 11
6-10 10
10-14 7 12 84 No.of families 7 8 2 2 1
10-14 7 14-20 4 17 68
14-20 4 20-28 4 24 96 Find the mode of this data
20-28 4 28-38 3 33 99 Here the maximum class frequency is 8, and the class corresponding to this frequency is 3 – 5.
28-38 3 38-40 1 39 39 So, the modal class is 3 – 5
38-40 1 ∑ =40 499 modal class = 3 – 5, lower limit (l ) of modal class = 3, class size (h) = 2
frequency ( f1 ) of the modal class = 8,
From the above table ∑ = 40, ∑ = 499, frequency ( f0 ) of class preceding the modal class f0 = 7
∑ frequency ( f2 ) of class succeeding the modal class f2 = 2
Average from Direct Method ̅ = ∑
= = 12.475
Now substitute the values in the formula:
9. The following table gives the literacy rate (in percentage) of 35 cities. Find the mean
literacy rate. Mode = l + xh = 3+ x2
( )
C.I. = −
Literacy No.of = 3+ x 2 = 3 + = 3 .286
rate( %) 45-55 3 50 150 -20 -60
cities
55-65 10 60 600 -10 -100 ∴ Therefore, the mode of the data above is 3.286.
45-55 3
55-65 10 65-75 11 70 770 0 0 Example 6 : The marks distribution of 30 students in a mathematics examination are
65-75 11 75-85 8 80 640 10 80 given in Table 13.3 of Example 1. Find the mode of this data. Also compare and interpret
75-85 8 85-95 3 90 270 20 60 the mode and the mean.
85-95 3 ∑ =35 2430 0 -20 Class Intervals 10-25 25-40 40-55 55-70 70-85 85-100

From the above table ∑ = 35, ∑ = 2430, No.of students 2 3 7 6 6 6



Average from Direct Method ̅ = ∑
= = 69.43
Refer the table 13.3 of example . Maximum students are in the class interval 40-45, it is the

Average from Assumed mean method ̅ =a + ∑
= 70 + = 60.43 modal class,
∴ l = 40 , h = 15, f1 = 7 , f0 = 3 , f2 = 6
13.3 Mode of Grouped Data
A mode is that value among the observations which occurs most often, that is, the value of Mode = l + xh
the observation having the maximum frequency
Example: 4 The wickets taken by a bowler in 10 cricket matches are as follows: Mode = 40 + ( )
x 15 = 40 + x 15
2 6 4 5 0 2 1 3 2 3
= 40 + x 15 = 40 + 12
Find the mode of the data
No.of wickets 0 1 2 3 4 5 6 ∴ The mode of the given data is 52
No.of matches 1 1 3 2 1 1 1
Clearly, 2 is the number of wickets taken by the bowler in the maximum number (i.e., 3) of Exercise 13.2
matches. So, the mode of this data is 2
1. The following table shows the ages of the patients admitted in a hospital during a year:
Mode = l + xh 5 - 15 15-25 25-35 35-45 45-55 55-65
Age(in years)
No.of patients 6 11 21 23 14 5
L = lower limit of the modal class
Find the mode and the mean of the data given above. Compare and interpret the two
h = size of the class interval (assuming all class sizes to be equal),
measures of central tendency.
= frequency of the modal class,
= frequency of the class preceding the modal class, Maximum number of patients =23
= frequency of the class succeeding the modal class Therefore 35-45 is the modal class interval
∴ l = 35, h = 10, f1 = 23, f0 = 21, f2 = 14
P a g e 64 | 101 Available in ykoyyur.blogspot.com P a g e 65 | 101 Available in ykoyyur.blogspot.com
SSLC Mathematics Solutions in English YK SSLC Mathematics Solutions in English YK

Mode = l + xh C.I. = − =
Mode = 35 + x 10= 35 + x 10 = 35 + x 10 = 35 + 1.81 1000 - 1500 24 1250 -1500 -3 -72
( )
∴ The mode of the above data is 36.81 1500 - 2000 40 1750 -1000 -2 -80
(a = 30 , h = 10) 2000 - 2500 33 2250 -500 -1 -33
2500 - 3000 28 2750 0 0 0
C.I. = − = 3000 - 3500 30 3250 500 1 30
5-15 6 10 -20 -2 -12 3500 - 4000 22 3750 1000 2 44
15-25 11 20 -10 -1 -11 4000 - 4500 16 4250 1500 3 48
25-35 21 30 0 0 0 4500 - 5000 7 4750 2000 4 28
35-45 23 40 10 1 23 ∑ =200 -35
45-55 14 50 20 2 28 ∑
By step deviation method ̅ = a + ∑
xh
55-65 5 60 30 3 15
∑ =80 43 = 2750 + x 500 = 2750 - 87.5 = 2662.5
4. The following distribution gives the state-wise teacher - student ratio in higher

By step deviation method ̅ = a + ∑
x h = 30 + x 10 = 30 + 5.375 = 35.375 secondary schools of India. Fin d the mode and mean of this data. Interpret the two
So, we conclude that maximum number of patients admitted in the hospital are of the age 36.81 measures No.of
Maximum freeqency = 10, of the class interval 30 - 35 No.of
years(Approx) whereas the average age of the patient admitted in the hospital is 35.375years students per
Therefore 30 – 35 is the modal class interval state/U.Ts
2. The following data gives the information on the observed lifetimes (in hours) of 225 teacher
electrical components ∴ l = 30, h = 5, f1 = 10, f0 = 9, f2 = 3 15 - 20 3
Life time(in hours) 0 - 20 20-40 40-60 60-80 80-100 100-120 Mode = l + xh 20 - 25 8
10 35 52 61 38 29 25 - 30 9
Freequency Mode = 30 + x5
( ) 30 - 35 10
Determine the modal lifetimes of the components
Maximum frequency =61 = 30 + x5 35 - 40 3
40 - 45 0
It is in the class interval 60 – 80. So, 60 - 80 is the modal class interval. = 30 + x 5 = 30 + 0.625 = 30.625
45 - 50 0
∴ l = 60, h = 20, f1 = 61, f0 = 52, f2 = 38 ∴ The mode of the above data is 30.625 50 - 55 2
Mode = l + xh
.
C.I. = − . =
Mode = 60 + x 20 = 60 + x 20
( ) 15 - 20 3 17.5 -15 -3 -9
= 60 + x 20 = 60 + 5.625 = 65.625 20 - 25 8 22.5 -10 -2 -16
∴ The mode of the above given data = 65.625 25 - 30 9 27.5 -5 -1 -9
3. The following data gives the distribution of total monthly household expenditure of 200 30 - 35 10 32.5 0 0 0
families of a village. Find the modal monthly expenditure of the families. Also, find the 35 - 40 3 37.5 5 1 3
mean monthly expenditure. 40 - 45 0 42.5 10 2 0
Expenditure No.of
Maximum frequency = 40 45 - 50 0 47.5 15 3 0
(in Rs) families
Therefore the modal class interval is (1500 – 2000) 1000 - 1500 24 50 - 55 2 52.5 20 4 8
∴ l = 1500, h = 500, f1 = 40, f0 = 24, f2 = 33 1500 - 2000 40 ∑ =35 -23
2000 - 2500 33 ∑
Mode = l + xh By step deviation Method ̅ = a + ∑
xh = 32.5 + x 5 = 32.5 – 3.29 = 29.21
2500 - 3000 28
Mode = 1500 + x 500= 1500 + x 500 3000 - 3500 30 The students – teacher ratio is 30.625 and average ratio is 29.21
( )
3500 - 4000 22 5. The given distribution shows the number of runs scored by some top batsmen of the
= 1500 + x 500 = 1500 + 347.83 = 1847.83 world in one-day international cricket matches
4000 - 4500 16
∴ The mode of the given data = 1847.83 4500 - 5000 7 Find the mode of the data.

P a g e 66 | 101 Available in ykoyyur.blogspot.com P a g e 67 | 101 Available in ykoyyur.blogspot.com


SSLC Mathematics Solutions in English YK SSLC Mathematics Solutions in English YK

Maximum freequency = 18. It is in the class interval 4000 - 5000 Heights (in cm) No.of Girls C.I. f cf
Therefore 4000 -5000 is the modal class interval No.of Batsman Less than 140 4 Less than 140 4 4
Runs scored
∴ l = 4000, h = 1000, f1 = 18, f0 = 4, f2 = 9 Less than 145 11 140 - 145 7 11
3000 - 4000 4 145 - 150 18 29
Mode = l + xh Less than 150À 29
4000 - 5000 18
150 - 155 11 40
Mode = 4000+ ( ) x 1000 = 5000 - 6000 9 Less than 155 40
155 - 160 6 46
6000 - 7000 7 Less than 160 46
4000 + x 1000 160 - 165 5 51
7000 - 8000 6 51
8000 - 9000 3 Less than 165
= 4000 + x 1000 = 4000 + 608.7 = 4608.7
9000 - 10000 1 Now, n = 51, ∴ = 25.5 It is in the class interval 145 - 150
∴ The mode of the above data is 4608.7 10000 - 11000 1
∴ l (lower limit) = 145, cf = 11. f = 18, h = 5
6. A student noted the number of cars passing through a spot on a road for 100
periods each of 3 minutes and summarised it in the table given below. Find the mode of Median = l + xh
the data . .
MedianÀ = 145 + x 5 = 145 + = 149.03
No.of cars 0 - 10 10-20 20-30 30-40 40-50 50-60 60-70 70-80 Therefore median of the given data is 149.03
7 14 13 12 20 11 15 8 Example 8 : The median of the following data is 525. Find the values of x and y, if the total
Freequency
frequency is 100.
Maximum freequency = 20. It is in the class interval 40 - 50 Class interval Freequency C.I. f cf
Therefore 40 – 50 is the modal class interval 0 -100 2 0 -100 2 2
∴ l = 40, h = 10, f1 = 20, f0 = 12, f2 = 11 100-200 5 100-200 5 7
Mode = l + xh 200-300 x 200-300 x 7+x
300-400 12 300-400 12 19+x
Mode = 40+ ( )
x 10= 40 + x 10 400-500 17 400-500 17 36+x
500-600 20 500-600 20 56+x
= 40 + x 10 = 40 + 4.71 = 44.71
600-700 y 600-700 y 56+x+y
∴ Mode of the given data 44.71 700-800 9 700-800 9 65+x+y
13.4 Median of Grouped Data 800-900 7 800-900 7 72+x+y
the median is a measure of central tendency which gives the value of the middle-most 900-1000 4 900-1000 4 76+x+y
observation in the data. Recall that for finding the median of ungrouped data, we first arrange
the data values of the observations in ascending order, then, if n is odd, then the meadian is Here, n = 100 ∴ 76 + x + y = 100
th observation and if n is an even, then the dedian is the average of and + 1 th So, x + y = 24 ........................ (1)
Median is 525, which is lies in the class interval 500 – 600
observation. ∴ l = 500, f = 20, cf = 36 + x, h = 100
After finding the median class, we use the following formula for calculating the median.
Median = l + xh
Median of Grouped Data
525 = 500 + x 100
Median = l + xh
525 = 500 + [14 − x] x 5
l = lower limit of median class, 25 = 70 – 5x ⇒ 5x = 70 – 25 ⇒ 5x = 45 ∴ x = 9
n = number of observations From equation (1) 9 + y = 24 ⇒ y = 15
cf = cumulative frequency of class preceding the median class,. Remarks: There is a empirical relationship between the three measures of central
f = frequency of median class tendency
h = class size (assuming class size to be equal).
3 Median = Mode + 2 average
Example 7 : A survey regarding the heights (in cm) of 51 girls of Class X of a school was
conducted and the following data was obtained
Find the median height.

P a g e 68 | 101 Available in ykoyyur.blogspot.com P a g e 69 | 101 Available in ykoyyur.blogspot.com


SSLC Mathematics Solutions in English YK SSLC Mathematics Solutions in English YK

8.5x20 =(30 − 5 − x)10 ⇒ 170 =250 − 10x ⇒ 10x = 80 ⇒ x = 8


Substitute x = 8 in equation (1),
Exercise 13.3 ⇒8 + y = 15 ⇒y = 7
Therefore x = 8 and y = 7
1. The following frequency distribution gives the monthly consumption of electricity of 68 3. A life insurance agent found the following data for distribution of ages of 100 policy
consumers of a locality. Find the median, mean and mode of the data and compare them. holders. Calculate the median age, if policies are given only to persons having age 18
Now, n = 68, ∴ = 34 It is in the class interval 125 ªÀiÁ¹PÀ §¼ÀPÉ UÁæºÀPÀgÀ ¸ÀAavÀ years onwards but less than 60 year.
- 145 . (AiÀÄƤmïUÀ¼À°)è ¸ÀASÉå DªÀÈwÛ
Age(in years) Cumulative freequency C.I. f cf
∴ l = 125, cf = 22, f = 20, h = 20 65 - 85 4 4
85 - 105 5 9 15-20 2 2
Below 20 2
Median = l + xh 20-25 4 6
105 - 125 13 22 Below 25 6
Below 30 24 25-30 18 24
median = 125 + x 20 125 - 145 20 42
145 - 165 14 56 Below 35 45 30-35 21 45
= 125 + x 20 = 125 + 12 = 137units 165 - 185 8 64 Below 40 78 35-40 33 78
Therefore median is 137units 185 - 205 Below 45 89 40-45 11 89
4 68
Average: Below 50 92 45-50 3 92
C.I. = − Below 55 98 50-55 6 98
=
Below 60 100 55-60 2 100
65 - 85 4 75 -60 -3 -12
85 - 105 5 95 -40 -2 -10 Totoa frequency = 100
105 - 125 13 115 -20 -1 -13 Now, n = 100, ∴ = 50 This is in the class interval 35 - 40
125 - 145 20 135 0 0 0
145 - 165 14 20 1 14 So, l = 35, cf = 45, f = 33, h = 5
155
165 - 185 8 175 40 2 16
Median = l + xh
185 - 205 4 195 60 3 12
∑ =68 7 = 35 + x 5 = 35 + x 5 = 35 + = 35 + 0.76
∑ Median = 35.76
By step deviation method ̅ = a + ∑
x h = 135 + x 20 = 135 + 2.1 = 137.05
4. The lengths of 40 leaves of a plant are measured correct to the nearest millimetre, and
Mode: maximum freequency = 20, which lies in the class interval 125 – 145. the data obtained is represented in the following table . Find the median length of the
Therefore 125-145 is the modal class interval leaves.
∴ l = 125, h = 20, f1 = 20, f0 = 13, f2 = 14 (Hint : The data needs to be converted to continuous classes for finding the median, sin ce th
Mode = l + xh e for mula assumes con tin uous classes. Th e classes th en ch an ge to 117.5 - 126.5,
126.5 - 135.5, ...., 171.5 - 180.5.]
= 125+ x 20 = 125 + x 20 = 125 + x 20 = 125 + 10.77 = 135.77
( ) Length(in mm) No.of Leaves C.I. f cf
∴ Therefore mode of the given data is 135.77 118 - 126 3 117.5 – 126.5 3 3
So, we conclude that three measures are approximately same. 127 - 135 5 126.5 – 135.5 5 8
2. If the median of the distribution given below is 28.5, find the values of x and y 136 - 144 9 135.5 – 144.5 9 17
Total freequency = 45 + x + y ⇒ 60 = 45 + x + y Class cf 145 - 153 12 144.5 – 153.5 12 29
Freequency 153.5 – 162.5 5 34
⇒x + y = 15 --------------------------(1) interval 154 - 162 5
Now, n = 60, 0 - 10 5 5 163 - 171 4 162.5 – 171.5 4 38
∴ = 30 this is in the class interval 20 - 30 10 - 20 x 5+x 172 - 180 2 171.5 – 180.5 2 40
∴ l = 20 .cf = 5 + x, f = 20, h = 10 20 - 30 20 25+x
30 - 40 15 40+x Now, n = 40, ∴ = 20 This is in the class interval 144.5 – 153.5
Median = l + xh 40 - 50 y 40+x+y So, l = 144.5, cf = 17, f = 12, h = 9
( ) 50 - 60 5 45+x+y
28.5 = 20 + x 10 Total 60 Median = l + xh

P a g e 70 | 101 Available in ykoyyur.blogspot.com P a g e 71 | 101 Available in ykoyyur.blogspot.com


SSLC Mathematics Solutions in English YK SSLC Mathematics Solutions in English YK

= 144.5 + x 9 = 144.5 + x 9=144.5 + = 144.5 + 2.25 =146.75mm = 8.5 + x 3 = 8.5 - 0.18 = 8.32
5. The following table gives the distribution of the life time of 400 neon lamps . Find the To find the mode:
median life time of a lamp. Maximum freequency = 40, Which is in the class interval 7 - 10
Life time in No.of Lamps Therefore the modal class interval is 7 - 10
C.I. f cf
hours ∴ l = 7, h = 3, f1 = 30, f0 = 30, f2 = 16
1500-2000 14 14
1500-2000 14 2000-2500 56 70 Mode = l + xh
2000-2500 56 2500-3000 60 130 = 7+ x3= 7+ x3= 7+ x 3 = 7 + 0.88 = 7.88
2500-3000 60 3000-3500 86 216 ( )
3000-3500 86 3500-4000 74 290 ∴ The mode of the given data is 7.88
3500-4000 74 4000-4500 62 352 7. The distribution below gives the weights of 30 students of a class. Find the median
4000-4500 62 4500-5000 48 400 weight of the students
4500-5000 48 Weight in Kgs 40-45 45-50 50-55 55-60 60-65 65-70 70-75
Total freequencies = 400 No.of students 2 3 8 6 6 3 2
Now, n = 400, ∴ = 200 this is in the class interval 3000 – 3500
Total freequencies = 30 C.I. f cf
Now, l = 3000, cf = 130, f = 86, h = 500
Now, n = 30, ∴ = 15 40-45 2 2
Median= l + x h = 3000 + x 500= 3000 + x 500 which is in the class interval 55 – 60 45-50 3 5
So, l = 55, cf = 13, f = 6 , h = 5 50-55 8 13
= 3000 + 406.98 = 3406.98 55-60 6 19
6. 100 surnames were randomly picked up from a local telephone directory and the Meadian = l + xh 60-65 6 25
frequency distribution of the number of letters in the English alphabets in the surnames 65-70 3 28
was obtained as follows = 55 + x5 = 7 + x5 70-75 2 30
No.of Letters 1-4 4 - 7 7 - 10 10 - 13 13 - 16 16 - 19 Median= 55 + 1.67 = 56.67kg
No.of surnames 6 30 40 16 4 4 13.5 Graphical Representation of Cumulative Frequency Distribution
Determine the median number of letters in the surnames. Find the mean number of Example 9 : The annual profits earned by 30 shops of a shopping complex in a locality
letters in the surnames? Also, find the modal size of the surnames give rise to the following distribution. draw its ogive. Hence obtain the median profit.
Total freequencies = 100 C.I. f cf
1-4 6 6 Profit (in lakhs) No.of shopes(f) We first draw the coordinate axes, with
Now, n = 100, ∴ = 50 this is in the class interval 7 – 10 More than or equal to 5 30
4-7 30 36 lower limits of the profit along the
So, l = 7, cf = 36, f = 40, h = 3 7-10 40 76 More than or equal to 10 28 horizontal axis, and the cumulative
10-13 16 92 More than or equal to 15 16 frequency along the vertical axes. Then,
Median = l + xh More than or equal to 20 14
13-16 4 96 we plot the points (5, 30), (10, 28), (15, 16),
16-19 4 100 More than or equal to 25 10 (20, 14), (25, 10), (30, 7) and (35, 3). We
7 + x3 = 7 + x 3 = 7 + 1.05 = 8.05 More than or equal to 30 7 join these points with a smooth curve to get
To find the average: More than or equal to 35 3
the ‘more than’ ogive, as shown in Fig.
[a = 8.5, h = 3]
Now, let us obta in the cla ss es, their frequencies and the cumulative frequency from the
C.I. = − = table above
1-4 6 2.5 -6 -2 -12 C.I. 5-10 10-15 15-20 20-25 25-30 30-35 35-40
4-7 30 5.5 -3 -1 -30 f 2 12 2 4 3 4 3
7-10 40 8.5 0 0 0 cf 2 14 16 20 23 27 30
10-13 16 11.5 3 1 16
13-16 4 14.5 6 2 8
16-19 4 17.5 9 3 12
∑ =100 -6

By step deviation method ̅ = a + ∑
xh
P a g e 72 | 101 Available in ykoyyur.blogspot.com P a g e 73 | 101 Available in ykoyyur.blogspot.com
SSLC Mathematics Solutions in English YK SSLC Mathematics Solutions in English YK

Weight (in kgs) 38 40 42 44 46 48 50 52


No of students 0 3 5 9 14 28 32 35

Weight No of
(in kgs) students
Less than 38 0
Less than 40 3
Less than 42 5
Less than 44 9

Less than 46 14

Less than 48 28

Exercise 13.4 Less than 50 32


1. The following table gives the distribution of the life time of 400 neon lamps : Less than 52 35
Daily income(Rs) 100-120 120-140 140-160 160-180 180-200
No.of workers 12 14 8 6 10 3. The following table gives production yield per hectare of wheat of 100 farms of a village.
Production
50-55 55-60 60-65 65-70 70-75 75-80
Convert the distribution above to a less than type cumulative frequency distribution, Yield(kg/ha)
and draw its ogive. No of farms 2 8 12 24 38 16

Daily Change the distribution to a more than type distribution, and draw its ogive.
No.of
income cf
workers
(Rs) Production
f cf
Yield(kg/ha)
100-120 12 12
50 2 100
120-140 14 26
55 8 98
140-160 8 34

60 12 90
160-180 6 40

180-200 65 24 78
10 50

2. During the medical check-up of 35 students of a class, their weights were recorded as 70 38 54
follows:
Draw a less than type ogive for the given data. Hence obtain the median weight from the
graph and verify the result by using the formula 75 16 16

P a g e 74 | 101 Available in ykoyyur.blogspot.com P a g e 75 | 101 Available in ykoyyur.blogspot.com


SSLC Mathematics Solutions in English YK SSLC Mathematics Solutions in English YK

Summary:
1. The mean for grouped data can be found by :

Probabilty
Direct Method ̅ = ∑

14.2 Probability — A Theoretical Approach
Assumed mean method: ̅ =a + ∑

Suppose a coin is tossed at random When we speak of a coin, we assume it to be
Step deviation method: ̅=a+ ∑
xh the coin can only land in one of two ‘fair ’, that is, it is symmetrical so that there is no
with the assumption that the frequency of a class is centred at its mid-point, called its class possible ways — either head up or tail up. reason for it to come down more often on one
mark suppose we throw a die once. For us, a side than the other. We call this property of the
die will always mean a fair die. They are coin as being ‘unbiased’. By the phrase ‘random
2. The mode for grouped data can be found by using the formula: toss’, we mean that the coin is allowed to fall
1, 2, 3, 4, 5, 6. Each number has the
same possibility of showing up. freely without any bias or interference
Mode = l + xh
The experimental or empirical probability P(E) of an event E as
Where symbols have the meanings
3. The cumulative frequency of a class is the frequency obtained by adding the frequencies of all
the classes preceding the given class P(E) =
4. The median for grouped data is formed by using the formula:.
The theoretical probability (also called classical probability) of an event E, written as P(E), is
meadian = l + xh defined as
Where symbols have the meanings
P(E) =
5. Representing a cumulative frequency distribution graphically as a cumulative frequency curve, .
or an ogive of the less than type and of the more than type.
Example 1 : Find the probability of getting a head when a
6. The median of grouped data can be obtained graphically as the x-coordinate of the point of coin is tossed once. Also find the probability of getting a tail.
intersection of the two ogives for this data. Random experiment: Tossing a coin once
S - { Tossing a coin once};
S - {H, T} [Here, H - Head T - Tail] – n(S) = 2
A - { Getting Head }- n(A) = 1
( )
P(A) = =
( )
B - { Getting Tail }- n(B) = 1
( )
P(B) = ( )
=
Example 2 : A bag contains a red ball, a blue ball and a yellow ball, all the balls being of
the same size. Kritika takes out a ball from the bag without looking into it. What is the
probability that she takes out the
(i) Yellow ball (ii) Red ball (iii) Blue ball
S – {Total balls in a bag } ⇒ n(S) = 3
A – { Krthika picking up yellow ball} – n(A) = 1
( )
P(A) = ( ) =
B – { Krthika picking up red ball} – n(B) = 1
( )
P(B) = =
( )
C – { Krthika picking up blue ball} – n(C) = 1
( )
P(C) = =
( )

P a g e 76 | 101 Available in ykoyyur.blogspot.com P a g e 77 | 101 Available in ykoyyur.blogspot.com


SSLC Mathematics Solutions in English YK SSLC Mathematics Solutions in English YK

Observe that the sum of the probabilities of all the Example 6 : Savita and Hamida are friends. What is the probability that both will have
elementary events of an experiment is 1 (i) different birthdays? (ii) the same birthday? (ignoring a leap year)
Example 3 : Suppose we throw a die once. (i) What is (i) Favarable days that Savitha and Hamida have different birthdays 365-1 = 364
the probability of getting a number greater than 4 ?
Probabilty of having different birthdays P(A) =
(ii) What is the probability of getting a number less
than or equal to 4 ? Probabilty of having same birthdayP( ̅) = [P( ̅)= 1 – P(A)]
S – {Throwing a dice once } –{ 1,2,3,4,5,6}⇒ n(S) = 6 Example 7 : There are 40 students in Class X of a school of whom 25 are girls and 15 are
A – { Getting number more than 4 } – {5,6} - n(A) = 2 boys. The class teacher has to select one student as a class representative. She writes the
( )
P(A) = ( ) = = name of each student on a separate card, the cards being identical. Then she puts cards
in a bag and stirs them thoroughly. She then draws one card from the bag. What is the
B – { Getting a number equal or less than 4} – {1,2,3,4} - n(B) = 4
( ) probability that the name written on the card is the name of (i) a girl? (ii) a boy?
P(B) = ( ) = = Total number of students: n(S) = 40
P(A) = 1 – P( ) : where A is an event and is complement of an event A Number of Girls – n(A) = 25
That is, the probability of an event which is impossible to occur is 0. Such an event is called Number of boys – n(B) = 15
an impossible event ( )
The probability of drawn card with the name of a Girl P(A) = = =
Example: We know that there are only six possible outcomes in a single throw of a die. These ( )
( )
outcomes are 1, 2, 3, 4, 5 and 6. Since no face of the die is marked 8, so there is no outcome The probability of drawn card with the name of a BoyP(B) = ( )
= =
favourable to 8, i.e., the number of such outcomes is zero. In other words, OR P(B) = 1 - P(A) = 1 - =
getting 8 in a single throw of a die, is impossible
Example 8 : A box contains 3 blue, 2 white, and 4 red marbles. If a marble is drawn at
So, the probability of an event which is sure (or certain) to occur is 1. Such an event is called
random from the box, what is the probability that it will be (i) white (ii) blue (iii) red
a sure event or a certain event.
The number of marbles in a box = n(S) = 9
Example:Since every face of a die is marked with a number less than 7, it is sure that we will
The probability of getting white marble P(W) =
always get a number less than 7 when it is thrown once. So, the number of favourable outcomes
is the same as the number of all possible outcomes, which is 6. The probability of getting white blue P(B) =
0 ≤ P(E) ≤ 1
The probability of getting white red P(B) =
Example 4 : One card is drawn from a Now, let us take an example related to playing
well-shuffled deck of 52 cards. Calculate cards. Have you seen a deck of playing cards? Example 9 : Harpreet tosses two different coins simultaneously (say, one is of ` 1 and
the probability that the card will It consists of 52 cards which are divided into 4 other of ` 2). What is the probability that she gets at least one head?
(i) Be an aace (ii) Not be an ace suits of 13 cards each— spades, hearts, The two different coins are tossed, the outcomes are S = { HH, HT, TH, TT }⇒ n(S) = 4
(i) S – {Picking a card from a deck of 52} diamonds and clubs. Clubs and spades are of The favorable outcomes to get atleast one head – {HT, TH, TT}
n(S) = 52 black colour, while hearts and diamonds are
of red colour. The cards in each suit are ace, Therefore the probability of getting atleast one head -
E – { The picked card is an ace} [Example 10 and 11 are not solved because they are optional]
king, queen, jack, 10, 9, 8, 7, 6, 5, 4, 3 and 2.
P(E) = 4 [There are 4 aces in a deck of 52] Kings, queens and jacks are called face cards Example 12 : A carton consists of 100 shirts of which 88 are good, 8 have minor defects
( )
P(A) = = = and 4 have major defects. Jimmy, a trader, will only accept the shirts which are good,
( )
(ii) F – {The card picked is not an ace} but Sujatha, another trader, will only reject the shirts which have major defects. One
n(F) = 48 shirt is drawn at random from the carton. What is the probability that
( )
P(F) = = = (i) it is acceptable to Jimmy? (ii) it is acceptable to Sujatha?
( )

or P(F) = P(E) = 1 – p(E) = 1 - = Total number of shirts = n(S) = 100


The number of good shirts = 88
Example 5 : Two players, Sangeeta and Reshma, play a tennis match. It is known that the
(i) The number of outcomes favourable (i.e., acceptable) to Jimmy = 88
probability of Sangeeta winning the match is 0.62. What is the probability of Reshma winning Therefore, P (shirt is acceptable to Jimmy) = = 0.88
the match? (ii) The number of outcomes favourable to Sujatha = 88 + 8 = 96
The probability that Savith wins the match = P(A) = 0.62
So, P (shirt is acceptable to Sujatha) = = 0.96
The probability that Reshma wins the match P(A) = 1 – P(A) = 1 – 0.62 = 0.38

P a g e 78 | 101 Available in ykoyyur.blogspot.com P a g e 79 | 101 Available in ykoyyur.blogspot.com


SSLC Mathematics Solutions in English YK SSLC Mathematics Solutions in English YK

Example 13 : Two dice, one blue and one grey, are thrown at the same time. Write down 4. Which of the following cannot be the probability of an event?
all the possible outcomes. What is the probability that the sum of the two numbers appearing A) B) -1.5 C) 15% D) 0.7 3
on the top of the dice is (i) 8 (ii) 13 (iii) less than or equal to 12 The probability of an event is always greater than or equal to 0 and less than or equal to 1.
The total number of outcomes when two dice are thrown at the same time Thus, (B) -1.5 cannot be the probability of an event.
(1,1), (1,2), (1,3), (1,4),(1,5), (1,6), (2,1), (2,2), (2,3), (2,4), (2,5), (2,6), (3,1), (3,2), (3,3), (3,4),
5. If P(E) = 0.05, what is the probability of ‘not E’?
(3,5), (3,6), (4,1), (4,2), (4,3), (4,4), (4,5), (4,6), (5,1), (5,2), (5,3), (5,4), (5,6), (6,1), (6,2), The probability of ‘not E’=1 - P(E) = 1 – 0.05 = 0.95
(6,3), (6,4), (6,5), (6,6)
6. A bag contains lemon flavoured candies only. Malini takes out one candy without looking
n(S) = 6x6 = 36 into the bag. What is the probability that she takes out
(i) A – The sum of two numbers be 8 (i) an orange flavoured candy? (ii) a lemon flavoured candy?
A – { (2,6), (3,5), (4,4), (5,3), (6,2)} - n(A) = 5 Answer
∴ The probability of getting the sum of two numbers be 8 = (i) Since the bag contains only lemon flavoured.
(ii) B - The sum of two numbers be 13 - n(B) = 0 Therefore, No. of orange flavoured candies = 0
∴ The probability of getting the sum of two numbers be 13 = =0 Probability of taking out orange flavoured candies = = 0
(ii) The bag only have lemon flavoured candies.
(iii) C - B - The sum of two numbers be equal or less than 12
Probability of taking out lemon flavoured candies = = 1
∴ The probability of getting the sum of two numbers be equal or less than = =1
7. It is given that in a group of 3 students, the probability of 2 students not having the same
birthday is 0.992. What is the probability that the 2 students have the same birthday?
Exercise 14.1 Answer
Let E be the event of having the same birthday. P(E) = 0.992
1. Complete the following statements ⇒ P(E) + P(not E) = 1 ⇒ P(not E) = 1 – P(E) ⇒ 1 - 0.992 = 0.008
(i) Probability of an event E + Probability of the event ‘not E’ = _______________ The probability that the 2 students have the same birthday is 0.008
(ii) The probability of an event that cannot happen is Such an event is called 8. A bag contains 3 red balls and 5 black balls. A ball is drawn at random from the bag.
_______ . What is the probability that the ball drawn is (i) red? (ii) not red?
(iii)The probability of an event that is certain to happen is Such an event is called
. Total number of balls in a bag = n(S) = 3 + 5 = 8
(iv) The sum of the probabilities of all the elementary events of an experiment is (i) Number of red balls = n(A) = 3
. ( )
Probabilty of drawing red balls P(A) = =
(v) The probability of an event is greater than or equal to and less than or equal ( )

to ______ (ii) Probabilty of drawing ‘not red ball’ P(A) = 1 - P(A) = 1 - =


Ansewers: 9. A box contains 5 red marbles, 8 white marbles and 4 green marbles. One marble is taken
(i) 1 (ii) 0 , impossible event (iii) 1, Sure (iv) 1 (v) 0,1 out of the box at random. What is the probability that the marble taken out will be (i)
red ? (ii) white ? (iii) not green?
2. Which of the following experiments have equally likely outcomes? Explain. Total number of marbles in abox = n(S) = 5 + 8 + 4 = 17
(i) A driver attempts to start a car. The car starts or does not start (i) Number of red marbles = n(A) = 5
(ii) A player attempts to shoot a basketball. She/he shoots or misses the shot.
( )
(iii) A trial is made to answer a true-false question. The answer is right or wrong Probability of taking out red marbles P(A) = ( )
=
(iv) A baby is born. It is a boy or a girl. (ii) Number of white marbles = n(B) = 8
Answer ( )
(i) It does not have equally likely outcomes as it depends on various reasons like mechanical Probabilty of taking out white marbles P(B) = =
( )
problems, fuels etc. (iii) Number of green marbles = n(C) = 4
(ii) It does not have equally likely outcomes as it depends on the player how he/she shoots. ( )
(iii) It has equally likely outcomes. Probabilty of taking out green marbles P(C) = =
( )
(iv)It has equally likely outcomes. ( )
∴ Probabilty of ‘not green’ marbles P(C ) = 1-
1
= 1- =
3. Why is tossing a coin considered to be a fair way of deciding which team should get the ( )
ball at the beginning of a football game? 10. A piggy bank contains hundred 50p coins, fifty Rs1 coins, twenty Rs2 coins and ten Rs5
Yes, tossing of a coin is a fair way of deciding which team should get the ball at the beginning coins. If it is equally likely that one of the coins will fall out when the bank is turned upside
of a football game because it has only two outcomes either head or tail. A coin is always down, what is the probability that the coin (i) will be a 50 p coin ?(ii) will not be a Rs 5
unbiased coin?
P a g e 80 | 101 Available in ykoyyur.blogspot.com P a g e 81 | 101 Available in ykoyyur.blogspot.com
SSLC Mathematics Solutions in English YK SSLC Mathematics Solutions in English YK

Total number of coins in a piggy bank = 100 + 50 + 20 + 10 = 180 14. One card is drawn from a well-shuffled deck of 52 cards. Find the probability of getting
Total number of 50 p coins = (A) = 100 (i) a king of red colour (ii) a face card (iii) a red face card (iv) the jack of hearts (v) a spade
Number of Rs 5 coins = (B) = 10 (vi) the queen of diamonds
( ) Possible numbers of events = 52
(i) Probabilty of getting Rs 5 coins (A) = = =
( ) (i) Numbers of king of red colour = 2
( )
(ii)Probabilty of it will not be a Rs 5 coin 1 - (B) =1 - =1- = Probability of getting a king of red colour = = =
( )
11. Gopi buys a fish from a shop for his aquarium. The (ii) Numbers of face cards = 12
shopkeeper takes out one fish at random from a tank Probability of getting a face card = =
containing 5 male fish and 8 female fish (see Fig. 15.4). What
(iii) Numbers of red face cards = 6
is the probability that the fish taken out is a male fish?
Total number of fish in the tank = n(S) = 5+8 = 13 Probability of getting a king of red colour = =
Number of male fish in the tank = n(A) = 5 (iv) Numbers of jack of hearts =1
The probability of taking out the male fish Probability of getting a king of red colour =
( ) (v) Numbers of king of spade = 13
= P(A) = =
( )
12. A game of chance consists of spinning an arrow which comes to rest pointing at one of the Probability of getting a king of red colour = =
numbers 1, 2, 3, 4, 5, 6, 7, 8 (see Fig. 15.5), and these are equally likely outcomes. What is (vi) Numbers of queen of diamonds = 1
the probability that it will point at (i) 8 (ii) an odd number (iii) A number greater Probability of getting a king of red colour =
than 2 (iv) A number less than 9 15. Five cards the ten, jack, queen, king and ace of diamonds, are well-shuffled with their
Possible number of events = 8 face downwards. One card is then picked up at random.
(i) Possible chances that an arrow pointing number 8 = 1 (i) What is the probability that the card is the queen?
Probabilty of pointing 8 = (ii) If the queen is drawn and put aside, what is the probability that the second card
picked up is (a) an ace? (b) a queen?
(ii) Chances of pointing an odd number (1, 3, 5 ªÀÄvÀÄÛ 7) = 4 Total numbers of cards = 5
Probabilty of pointing an odd number = = (i) Numbers of queen = 1
(iii) Chances of pointing a number greater than 2 Probability of picking a queen =
(i,e.3, 4, 5, 6, 7 and 8) = 6 (ii) When queen is drawn and put aside then total numbers of cards left is 4
Probablity of pointing a number greater than 2 = = (a) Numbers of ace = 1
Probability of picking an ace =
(iv) Chances of pointing less than 9 (i.e, 1,2,3,4,5,6,7,8) = 8
Probability of pointing a number less than 9 = = 1 (a) Numbers of queen = 0
Probability of picking a queen = = 0
13. A die is thrown once. Find the probability of getting
(i) a prime number; (ii) a number lying between 2 and 6; (iii) an odd number. 16. 12 defective pens are accidentally mixed with 132 good ones. It is not possible to just look
Possible numbers of events on throwing a dice = 6 at a pen and tell whether or not it is defective. One pen is taken out at random from this
Numbers on dice = 1,2,3,4,5 and 6 lot. Determine the probability that the pen taken out is a good one.
(i) Prime numbers = 2, 3 and 5 Numbers of defective pens = 12
Favourable number of events = 3 Numbers of good pens = 132
Probability that it will be a prime number = = Total numbers of pen = 132 + 12 = 144 pens
Favourable number of events = 132
(ii) Numbers lying between 2 and 6 = 3, 4 and 5
Probability of getting a good pen = =
Favourable number of events = 3
Probability that a number between 2 and 6 = =
(iii) Odd numbers = 1, 3 and 5
Favourable number of events = 3
Probability that it will be an odd number = =

P a g e 82 | 101 Available in ykoyyur.blogspot.com P a g e 83 | 101 Available in ykoyyur.blogspot.com


SSLC Mathematics Solutions in English YK SSLC Mathematics Solutions in English YK

17. (i) A lot of 20 bulbs contain 4 defective ones. One bulb is drawn at random from the lot. Total numbers of pens = 144
What is the probability that this bulb is defective? Numbers of defective pens = 20
(ii) Suppose the bulb drawn in (i) is not defective and is not replaced. Now one bulb is Numbers of non defective pens = 144 - 20 = 124
drawn at random from the rest. What is the probability that this bulb is not defective? (i) Numbers of favourable events = 124 ; Probability that she will buy it = =
(i) Total numbers of bulbs = 20
Numbers of defective bulbs = 4 (ii) Numbers of favourable events = 20; Probability that she will not buy it = =
Probability of getting a defective bulb = = 22. Refer to Example 13. (i) Complete the following table
(ii) One non defective bulb is drawn in (i) then the total numbers of bulb left is 19 Event
2 3 4 5 6 7 8 9 10 11 12
Total numbers of events = 19 Sum on two dice
Favourable numbers of events = 19 - 4 = 15 1 5 1
Probability
Probability that the bulb is not defective = 36 36 36
(ii)A student argues that ‘there are 11 possible outcomes 2, 3, 4, 5, 6, 7, 8, 9, 10, 11 and 12.
18. A box contains 90 discs which are numbered from 1 to 90. If one disc is drawn at random
from the box, find the probability that it bears (i) a two-digit number (ii) a perfect square Therefore, each of them has a probability . Do you agree with this argument? Justify
number (iii) a number divisible by 5 your answer.
Total numbers of discs = 50 Events that can happen on throwing two dices are (1,1), (1,2), (1,3), (1,4),(1,5), (1,6), (2,1),
(i) Total numbers of favourable events = 81 (2,2), (2,3), (2,4), (2,5), (2,6), (3,1), (3,2), (3,3), (3,4), (3,5), (3,6), (4,1), (4,2), (4,3), (4,4), (4,5),
Probability that it bears a two-digit number = = (4,6), (5,1), (5,2), (5,3), (5,4), (5,6), (6,1), (6,2), (6,3), (6,4), (6,5), (6,6)
⇒ n(S) = 6x6 = 36
(ii) Perfect square numbers = 1, 4, 9, 16, 25, 36, 49, 64 and 81
Favourable numbers of events = 9; Probability of getting a perfect square number = = (i) To get sum as 2, possible outcomes = (1,1)
(iii) Numbers which are divisible by 5 = 5, 10, 15, 20, 25, 30, 35, 40, 45, 50, 55, 60, 65, 70, To get sum as 3, possible outcomes = (1,2) and (2,1)
75, 80, 85 and 90 To get sum as 4, possible outcomes = (1,3); (3,1); and (2,2)
Favourable numbers of events = 18; Probability of getting a number divisible by 5 = = To get sum as 5, possible outcomes = (1,4); (4,1); (2,3); and (3,2)
To get sum as 6, possible outcomes = (1,5); (5,1); (2,4); (4,2); and (3,3)
19. A child has a die whose six faces show the letters as given below:
To get sum as 7, possible outcomes = (1,6); (6,1); (5,2); (2,5); (4,3); and (3,4)
To get sum as 8, possible outcomes = (2,6); (6,2); (3,5); (5,3); and (4,4)
To get sum as 9, possible outcomes = (3,6); (6,3); (4,5); and (5,4)
The die is thrown once. What is the probability of getting (i) A? (ii) D? To get sum as 10, possible outcomes = (4,6); (6,4) and (5,5)
Total numbers of events = 6 To get sum as 11, possible outcomes = (5,6) and (6,5)
(i) Total numbers of faces having A on it = 2; Probability of getting A = = To get sum as 12, possible outcomes = (6,6)
(ii) Total numbers of faces having D on it = 1 ; Probability of getting A =
Event
20. Suppose you drop a die at random on the 2 3 4 5 6 7 8 9 10 11 12
rectangular region shown in Fig. 15.6. What Sum on two dice
is the probability that it will land inside the 1 2 3 4 5 6 5 4 3 2 1
Probability
circle with diameter 1m?[Not for 36 36 36 36 36 36 36 36 36 36 36
examination] (ii) No, i don't agree with the argument. It is already justified in (i).
Area of the rectangle = (3 × 2) m2 = 6m2 23. A game consists of tossing a one rupee coin 3 times and noting its outcome each time.
Area of the circle = πr2 = π 2 = m2 Hanif wins if all the tosses give the same result i.e., three heads or three tails, and loses
otherwise. Calculate the probability that Hanif will lose the game.
Probability that die will land inside the circle = = Events that can happen in tossing 3 coins
21. A lot consists of 144 ball pens of which 20 are defective and the others are good. Nuri will = HHH, HHT, HTH, THH, TTH, HTT, THT, TTT
buy a pen if it is good, but will not buy if it is defective. The shopkeeper draws one pen at Total number of events = 8
random and gives it to her. What is the probability that Hinif will lose the game if he gets HHT, HTH, THH, TTH, HTT, THT
(i) She will buy it? (ii) She will not buy it? Favourable number of elementary events = 6
Probability of losing the game = = =
P a g e 84 | 101 Available in ykoyyur.blogspot.com P a g e 85 | 101 Available in ykoyyur.blogspot.com
SSLC Mathematics Solutions in English YK SSLC Mathematics Solutions in English YK

24. A die is thrown twice. What is the probability that


(i) 5 will not come up either time? (ii) 5 will come up at least once? Surface Area and Volumes
[Hint : Throwing a die twice and throwing two dice simultaneously are treated as the
same experiment] You must have seen a truck with a container fitted
(i) Total number of possibilities = 6x6 =36 on its back (see Fig. 15.2), carrying oil or water
Possible outcomes: (1,1), (1,2), (1,3), (1,4),(1,6), (2,1), (2,2), (2,3), (2,4), (2,6), (3,1), (3,2), from one place to another. Is it in the shape of any
(3,3), (3,4), (3,6), (4,1), (4,2), (4,3), (4,4),(4,6), (6,1), (6,2), (6,3), (6,4), (6,6) of the four basic solids mentioned above? You may
guess that it is made of a cylinder with two
The possibility of 5 will not come either time = 25
hemispheres as its ends.
Required probability = = A test tube, is also a combination of a cylinder and a
(ii) Number of events when 5 comes at least once = 11 hemisphere. Similarly, while travelling, you may have
Probability = [ another way 1 - = ] seen some big and beautiful buildings or monuments
made up of a combination of solids mentioned above.
25. Which of the following arguments are correct and which are not correct? Give reasons
for your answer. 15.2 Surface Area of a Combination of Solids
(i) If two coins are tossed simultaneously there are three possible outcomes—two heads,
To find the surface area or the volume of a container or test tube we have to break it up two
two tails or one of each. Therefore, for each of these outcomes, the probability is
or more known solids. For example,
(ii) If a die is thrown, there are two possible outcomes—an odd number or an even Area of the container
number. Therefore, the probability of getting an odd number is = Area of the hemisphere + Area of the cylinder + Area of the hemisphere
(i) The statement is incorrect Example 1 : Rasheed got a playing top (lattu) as his birthday
Possible events = (H,H); (H,T); (T,H) and (T,T) present, which surprisingly had no colour on it. He wanted to
colour it with his crayons. The top is shaped like a cone
Probability of getting two heads = surmounted by a hemisphere (see Fig 13.6). The entire top is
Probability of getting one of the each = = 5 cm in height and the diameter of the top is 3.5 cm. Find the
(ii) Correct. The two outcomes considered are equally likely. area he has to colour. ( = )
TSA of the toy = CSA of hemisphere + CSA of cone
. .
Summary: CSA of hemisphere = 2 = 2x x x cm
Height of the cone = Height of the top – Radius of the hemisphere
1. The difference between experimental probability and theoretical probability.
2. The theoretical (classical) probability of an event E, written as P(E), is defined as = 5 – 1.75 = 3.25cm
Slant height of cone (l ) = √ + ℎ
P(E) =
= (1.75) + (3.25) ≈ 3.7cm
where we assume that the outcomes of the experiment are equally likely. .
∴ CSA of cone = rl = x x 3.7 cm
3. The probability of a sure event (or certain event) is 1.
. . . .
4. The probability of an impossible event is 0 ∴ TSA of the toy = 2x x x + x x 3.7 = x (3.5 + 3.7)
5. The probability of an event E is a number P(E) such that = 11 x 0.5(3.5 + 3.7) = 5.5 x 7.2 = 39.6cm2
0 ≤ P(E) ≤ 1
Example 2 : The decorative block shown in Fig. 13.7 is
6. An event having only one outcome is called an elementary event. The sum of the made of two solids — a cube and a hemisphere. The base
probabilities of all the elementary events of an experiment is of the block is a cube with edge 5 cm, and the hemisphere
7. For any event E, P(E) + P( ) = 1 where E stands for ‘not E’. E and E are called
complementary events. fixed on the top has a diameter of 4.2 cm. Find the total
surface area of the block ( = )
TSA of cube = 6a2 = = 6 × (5 × 5) = 150 cm2
The surface area of the block
= TSA of cube – base area of hemisphere + CSA of hemisphere
= (150 - +2 )cm2 = (150 + )cm2
= (150 + x 2.1 x 2.1 )cm2 = (150 + 13.86 )cm2 = 163.86cm2

P a g e 86 | 101 Available in ykoyyur.blogspot.com P a g e 87 | 101 Available in ykoyyur.blogspot.com


SSLC Mathematics Solutions in English YK SSLC Mathematics Solutions in English YK

Example 3 : A wooden toy rocket is in the shape of Breadth b = 4cm and height h = 4cm
a cone mounted on a cylinder, as shown in Fig. 15.8. The surface area of the cuboid = 2(lb+bh+hl)
The height of the entire rocket is 26 cm, while the
= 2(8x4+4x4+4x8)
height of the conical part is 6 cm. The base of the
= 2(32+16+32)= 2(80) = 160 cm2
conical portion has a diameter of 5 cm, while the
2. A vessel is in the form of a hollow hemisphere mounted by a hollow cylinder. The
base diameter of the cylindrical portion is 3 cm. If the
diameter of the hemisphere is 14 cm and the total height of the vessel is 13 cm. Find the
conical portion is to be painted orange and the
inner surface area of the vessel.
cylindrical portion yellow, find the area of the rocket
The inner surface area of the vessel =
painted with each of these colours. (Take = 3.14)
Let the the radius of the cone = r, the slant height = l the Inner surface area of the cylinder+ Inner surface area of the
height of the cone = h , radius of the cylinder = r1, and hemisphere = 2 ℎ+2
height of the cylinder = h1
= ; r= = 7cm; height of the cylinder h = 13 – 7 = 6cm
Then r = 2.5 cm, h = 6 cm, r1 = 1.5 cm, h1 = 26 - 6 = 20 cm ªÀÄvÀÄÛ l = √ + ℎ
= 2x x 7x 6 + 2x x 7x 7 = 2x 22x 6 + 2x22 x 7
= l = √2.5 + 6 = 6.5cm
Here, the conical portion has its circular base resting on the base of the cylinder, but the base = 264 + 308 = 572cm2
of the cone is larger than the base of the cylinder. So, a part of the base of the cone (a ring) is to 3. A toy is in the form of a cone of radius 3.5 cm mounted on a hemisphere of same radius.
be painted. The total height of the toy is 15.5 cm. Find the total surface area of the toy.
The area to be painted orange TSA of the Toy = CSA of cone + CSA of hemisphere = + 2
= CSA of the cone + base area of the cone– base area of the cylinder
= πrl + πr2 - π( )2 = ; r = 3.5 ; h = 15.5 – 3.5 = 12cm
= π [(2.5 × 6.5) + (2.5)2 - (1.5)2] cm2 l = √ℎ + = √12 + 3.5 = √144 + 12.25
= π [20.25] cm2 = 3.14 × 20.25 cm2 = 63.585 cm2 l =√156.25 = 12.5cm
Now, the area to be painted yellow = CSA of the cylinder+ area of one base of the cylinder TSA of the Toy = x 3.5 x 12.5 + 2x x3.5 2
= 2πr1h1 + π(r1 )2 = 22 x 0.5 x 12.5 + 2x 22x0.5x3.5
= πr1 (2h1 + r1) = (3.14×1.5) ( 2×20 +1.5) cm2 = 4.71× 41.5 cm2 = 195.465 cm2 = 11 x 12.5 + 11x7= 11 x 19.5
Example 4 : Mayank made a bird-bath for his garden in the
=214.5cm2
shape of a cylinder with a hemispherical depression at one
end (see Fig. 15.9). The height of the cylinder is 1.45 m and its 4. A cubical block of side 7 cm is surmounted by a hemisphere. What is the greatest
radius is 30 cm. Find the total surface area of the bird bath diameter the hemisphere can have? Find the surface area of the solid
The greatest diameter of the hemisphere = Side of the square = 7cm
(Take = ).
Surface area of the solid
Let h be height of the cylinder, and r the common radius of the
= Surface area of cube + CSA of hemisphere - The area of the circular base of the hemisphere
cylinder and hemisphere. Then,
= 6a2 + 2 -
The total surface area of the bird-bath = CSA of cylinder + CSA of hemisphere
= 6x72 + 2 x x - x
= 2πrh + 2πr2 = 2πr (h + r) = 2 × × 30 (145 + 30) cm2 = 33000 cm2 = 3.3m2
= 6x49 + 11x7 - 11 x
= 294 + 77 - 11 x
Exercise 15.1
= 371 – 38.5
(Unless stated otherwise, take π = ) = 332.5cm2
1. 2 cubes each of volume 64 cm3 are joined end to end. Find the surface area of the 5. A hemispherical depression is cut out from one face of a cubical wooden block such that
the diameter l of the hemisphere is equal to the edge of the cube. Determine the surface
resulting cuboid.
area of the remaining solid
The volume of the cube = 64cm3, Therefore length of the side = 4cm
The surface area of the solid = Surface area of cube + Surface area of hemisphere
∴ The length of the cuboid = 4+4 = 8 cm
– Area of the circular base of the hemisphere
= 6l2 + 2 -
P a g e 88 | 101 Available in ykoyyur.blogspot.com P a g e 89 | 101 Available in ykoyyur.blogspot.com
SSLC Mathematics Solutions in English YK SSLC Mathematics Solutions in English YK

9. A wooden article was made by scooping out a hemisphere from each end of a solid
= 6l2 + 2 -
cylinder, as shown in Fig. 15.11. If the height of the cylinder is 10 cm, and its base is of
= 6l2 + 2 - radius 3.5 cm, find the total surface area of the article.
TSA of the article = CSA of cylinder + 2x innere CSA of hemisphere
2 (24 + )
= 6l + = = 2πrh + 2x2πr
6. A medicine capsule is in the shape of a cylinder with two hemispheres stuck to each of its
= ; r = 3.5 ; h = 10m
ends (see Fig. 13.10). The length of the entre capsule is 14mm and the diameter of the
capsule is 5mm. Find its surface area. = 2πr(h + 2r)
Surface area of the capsule = = 2x x 3.5 (10 + 2x3.5) = 2x 22 x 0.5 (10 + 7)
2CSA of hemisphere CSA of cylinder = 2(2 )+2 ℎ = 22 (17)= 22 (17) = 374cm2
= ; r = 2.5 ; h = 9mm
= 2(2 ) + 2 ℎ = 2 (2 + ℎ) 15.3 Volume of a Combination of Solids
Example 5 : Shanta runs an industry in a shed which is in the shape of a cuboid
= 2x x (2x2.5 + 9) surmounted by a half cylinder (see Fig.15.12). If the base of the shed is of dimension 7
= (14) = 110 x 2 = 220mm2 m × 15 m, and the height of the cuboidal portion is 8 m, find the volume of air that the shed
can hold. Further, s up pose the ma chiner y in t he s hed occupies a total space of 300
7. A tent is in the shape of a cylinder surmounted by a conical top. If the height and
diameter of the cylindrical part are 2.1 m and 4 m respectively, and the slant height of the m3, and there are 20 workers, each of whom occu py ab ou t 0.08 m3 spa ce on a n
top is 2.8 m, find the area of the canvas used for making the tent. Also, find the cost of the average. Then, how much air is in the sshed ( = )
canvas of the tent at the rate of Rs 500 per m2 . (Note that the base of the tent will not be The volume of air inside the shed (when there are no people
covered with canvas.) or machinery) is given by the volume of air inside the cuboid
The area of the tent
and inside the half cylinder, taken together.
= CSA of the cylinder + CSA of cone
Now, the length, breadth and height of the cuboid are 15 m, 7
= 2 ℎ + m and 8 m, respectively. Also, the diameter of the half
= (2ℎ + ) cylinder is 7 m and its height is 15 m
= ; r=2 ; h = 2.1m ; l = 2.8m So, the required volume = volume of the cuboid + volume of the cylinder
= x 2(2x2.1 + 2.8) = 15 x 7 x 8 + x x x x 15 m3 = = 1128.75 m3
= x 7 (2x0.3 + 0.4) Next, the total space occupied by the machinery = 300 m3
And the total space occupied by the workers = 20 × 0.08 m3 = 1.6m3
= 44 (0.6 + 0.4) = 44m2
The total cost of the canvas at the rate of Rs 500/cm2 = 44x500 = Rs 22000 ∴ the volume of the air, when there are machinery and workers
= 1128.86 - (300.00 + 1.60) = 827.15 m3
8. From a solid cylinder whose height is 2.4 cm and diameter 1.4 cm, a conical cavity of the
Example 6:A juice seller was serving his customers using glasses
same height and same diameter is hollowed out. Find the total surface area of the
remaining solid to the nearest cm2 as shown in Fig. 15.13. The inner diameter of the cylindrical glass
Surface area of the solid = TSA of cylinder + Inner CSA of cone was 5 c m, b u t t he bot t om of t he gla s s ha d a hemispherical
raised portion which reduced the capacity of the glass. If the
– Area of one the circular face of the cylinder
height of a glass was 10cm, find the apparent capacity of the glass
= 2 ( + ℎ) + −
and its actual capacity. (Use = 3.14)
= ; r = 0.7 ; h = 2.4m Since the inner diameter of the glass = 5 cm and height = 10 cm,
the apparent capacity of the glass = πr2h
l = √ℎ + = √2.4 + 0.7 = √5.76 + 0.49 =√6.25= 2.5m
= 3.14 × 2.5 × 2.5 × 10 cm3 = 196.25 cm3
= 2x x0.7(0.7 + 2.4) + x 0.7x2.5 − x0.7x0.7 But the actual capacity of the glass is less by the
= 2x22x0.1(3.1) + 22x 0.1x2.5 − 22x0.1x0.7 volume of the hemisphere at the base of the glass.
= 4.4(3.1) + 2.2x2.5 − 2.2x0.7 = 13.64 + 5.5 − 1.54 i.e., it is less by = = × 3.14 × 2.5 × 2.5 × 2.5 = 32.71 cm3
So, the actual capacity of the glass = apparent capacity of glass – volume of the hemisphere
= 13.64 + 5.5 − 1.54 = 17.6m2 ≈ 18m2
= (196.25 - 32.71) cm3

P a g e 90 | 101 Available in ykoyyur.blogspot.com P a g e 91 | 101 Available in ykoyyur.blogspot.com


SSLC Mathematics Solutions in English YK SSLC Mathematics Solutions in English YK

E xample 7: A s olid t oy is in t he form of a hemisphere surmounted by a right circular Volume of the Jamun = 2 x Volume of two hemisphere + Volume of the cylinder
cone. The height of the cone is 2 cm and the diameter of the base is 4 cm. Determine the = 2x πr + πr h
volume of the toy. If a right circular cylinder circumscribes the toy, find the difference of
the volumes of the cylinder and the toy. ( take = 3.14). Given: π = ; r = 1.4cm; ℎ = 2.2cm;
Height of the cylinder h = radius of the hemisphere + height of the cone = 2+2= 4cm = 2x x x 1.4 x 1.4 x 1.4 + x 1.4 x1.4 x 2.2
= 3.14 ; radius of the hemisphere = radius of the cylinder = radius of the cone = 2cm
= 4x x 0.2 x 1.4x1.4 + 22 x 0.2 x1.4 x 2.2
The volume of the cylinder circumsubscried the toy = ℎ .
= 3.14 x 2 x 2 x 4 = 3.14 x 16 = 50.24cm3 = + 13.552 = 11.5 + 13.552 = 25.05 cm3
The volume of the toy = + ℎ Therefore the amount of sugar contained = 25.05 x = 7.515cm3
= [2 + ℎ] = x 3.14x 2 [4 + 2]= x 3.14x 4[6] ∴ The total amount of sugar contained in 45 jamun = 7.515x 45
= 338.175cm3 ≈ 338cm3
= x 3.14x 4[6] = 25.12cm3
4. A pen stand made of wood is in the shape of a cuboid with four conical depressions to
Hence, the required difference of the two volumes hold pens. The dimensions of the cuboid are 15 cm by 10 cm by 3.5 cm. The radius of each
= 50.24 - 25.12 cm3 = 25.12cm3 of the depressions is 0.5 cm and the depth is 1.4 cm. Find the volume of wood in the entire
stand (see Fig. 15.16)
Exercise 15.2 The radius of the conical deoressions r = 0.5cm, the depth ℎ = 1.4 cm
[Unless stated otherwise, take = ] Length of the cuboid shape l = 15cm,breadth b = 10cm height hÀ = 3.5cm

1. A solid is in the shape of a cone standing on a hemisphere with both their radii being 4 (Volume of the conical depressions) = 4 ℎ
equal to 1 cm and the height of the cone is equal to its radius. Find the volume of the solid in
=4 x x 0.5 x0.5 x1.4 = 4 x22x 0.5 x0.5 x 0.2
terms of
Volume of the solid = Volume of the cone + volume of the hemisphere =4 x22x 0.5 x0.5 x 0.2 = 1.47cm3
Given: π = ; h = 1cm; r = 1cm Volume of the wood in the pen stand
Volume of the solid = ℎ + = Volume of the cuboid shape - 4 (Volume of the conical depressions)
= 15x10x3.5 – 1.47 = 525 – 1.47 = 523.53cm3
= x1x1 + x 1 x1 x1
5. A vessel is in the form of an inverted cone. Its height is 8 cm and the radius of its top,
= + = = cm3 which is open, is 5 cm. It is filled with water up to the brim. When lead shots, each of which
2. Rachel, an engineering student, was asked to make a model shaped like a cylinder with is a sphere of radius 0.5 cm are dropped into the vessel, one-fourth of the water flows out.
two cones attached at its two ends by using a thin aluminium sheet. The diameter of the Find the number of lead shots dropped in the vessel.
model is 3 cm and its length is 12 cm. If each cone has a height of 2 cm, find the volume of The Volume of the lead shots =
air contained in the model that Rachel made. (Assume the outer and inner dimensions of
= x x x x = cm3
the model to be nearly same]
Volume of the air contained in the model = 2xVolume of the cone +Volume of the cylinder The volume of the water in the vessel = ℎ
V = 2x ℎ + ℎ = x x5 x8 = cm3
Given: π = ; r = 1.5cm; The volume of the water flows out
ℎ = 2cm; ℎ = 8cm = x = cm3
V = 2x x x (1.5) x 2 + x(1.5) 8 ∴ Number of lead shots =
= x 2.25x2 + x 2.25x8
∴ Number of lead shots = = 100 shots
= x 4.5 + x 18 = + = + = = 66cm3
3. A gulab jamun, contains sugar syrup up to about 30% of its volume. Find approximately 6. A solid iron pole consists of a cylinder of height 220 cm and base diameter 24 cm, which is
how much syrup would be found in 45 gulab jamuns, each shaped like a cylinder with two surmounted by another cylinder of height 60 cm and radius 8 cm. Find the mass of the pole,
hemispherical ends with length 5 cm and diameter 2.8 cm (see Fig. 15.15).
given that 1 cm3 of iron has approximately 8g mass. (Use = 3.14)

P a g e 92 | 101 Available in ykoyyur.blogspot.com P a g e 93 | 101 Available in ykoyyur.blogspot.com


SSLC Mathematics Solutions in English YK SSLC Mathematics Solutions in English YK

= 8cm; = = 12cm; ℎ = 60cm; ℎ = 220cm = 25.12 + 321.39 = 346.51cm3


Volume of the pole So, there is little difference in her measurment
= Volume of the first cylinder + Volume of the second cylinder 13.4 Conversion of Solid from One Shape to Another
= ℎ + ℎ We can convert one shape to another. When we convert the shape, the volume of the new
shape will be the same as the earliar shape.
= 3.14 x 12 x 12 x 220 + 3.14 x 8 x 8 x 60 Example 8: A cone of height 24 cm and radius of base 6 cm is made up of modelling clay.
= 99475.2 + 12057.6 = 111532.8cm3 A child reshapes it in the form of a sphere. Find the radius of the sphere.
The mass of the iron / 1 cm3 = 8g Volume of the cone = × × 6 × 6 × 24 cm3
∴ Mass of the iron pole = 111532.8 x 8 = 892262.4 g = 892.26kg If the radius of the sphere is `r ' then the volume of the sphere is
7. A solid consisting of a right circular cone of height 120 cm and radius Therefore, Volume of the cone = Volume of the sphere
60 cm standing on a hemisphere of radius 60 cm is placed upright ⇒ = × × 6 × 6 × 24 cm3
in a right circular cylinder full of water such that it touches the = 3 × 3 × 24 = 33 × 23 ⇒ r = 3 x 2 = 6
bottom. Find the volume of water left in the cylinder, if the radius of Radius of the sphere = 6 cm.
the cylinder is 60 cm and its height is 180 cm. Example 9: Selvi’s house has an overhead tank in the shape of a cylinder. This is filled
Radius of the cylinder r = 60cm; height h = 180cm by pumping water from a sump (an underground tank) which is in the shape of a
Height of the cone ℎ = 120 cuboid. The sump has dimensions 1.57 m × 1.44 m × 95cm. The overhead tank has
Volume of the cylinder = ℎ its radius 60 cm and height 95 cm. Find the height of the water left in the sump after
the overhead tank has been completely filled with water from the sump which had
= x 60 x 60 x 180 = 2036571.43cm3
been full. Compare the capacity of the tank with that of the sump (use = 3.14)
Volume of the cone = ℎ Volume of water in the overhead tank = Volume of the water removed from the sump.
= x 20 x 60 x 120 = 452571. 43cm3 Now, the volume of water in the overhead tank (cylinder) = r2 h
= 3.14 × 0.6 × 0.6 × 0.95 m3
Volume of the hemisphere = The volume of water in the sump when full = l × b × h = 1.57 × 1.44 ×0.95 m3
= x x 60 x60 x60 = 4525571.43cm3 The volume of water left in the sump after filling the tank
= [(1.57 × 1.44 × 0.95) - (3.14 × 0.6 × 0.6 × 0.95)] m3 = (1.57 × 0.6 × 0.6 × 0.95 × 2) m3
∴ The volume of the water left in the cylinder
So, the height of the water left in the sump h =
= 2036571.43 – (452571.43 + 452571.43) = 2036571.43 – 905142.86
. × . × . × . ×
= 1131428.57cm3 = 1.131m3 = . .
m = 0.475 m = 47.5 cm
Alternate method: Also , =
. . . .
=
The volume of the water left in the cylinder . . .
Therefore, the capacity of the tank is half the capacity of the sump
= ℎ − ℎ + = ℎ − ℎ + Example 10 : A copper rod of diameter 1 cm and length 8 cm is drawn into a wire of length
= x 60x60 180 − x120 + x60 = x 60x60[180 − (40 + 40)] 18 m of uniform thickness. Find the thickness of the wire.

= x 60x60x 100 = = 1131428.57cm3 = 1.131m3 The volume of the rod = × x 8 cm3 = 2 cm3
The length of the new wire of the same volume = 18 m = 1800 cm
8. A spherical glass vessel has a cylindrical neck 8 cm long, 2 cm in
diameter; the diameter of the spherical part is 8.5 cm. By If r is the radius (in cm) of cross-section of the wire, its volume = r2 × 1800 cm3
measuring the amount of water it holds, a child finds its volume Therefore, r2 × 1800 = 2 ⇒ r2 = ⇒r=
to be 345 cm3. Check whether she is correct, taking the above So, the diameter of the cross section, i.e., the thickness of the wire is cm
as the inside measurements, and = 3.14.
i.e., Approximately 0.67 mm
Height of the cylinder h = 8cm; Radius = = 1cm
. Example 11 : A hemispherical tank full of water is emptied by a pipe at the rate of 3
Radius of the sphere = cm litres per second. How much time will it take to empty half the tank, if it is 3m in
Volume of the Vessel diameter? (Take = )
= Volume of the cylinder + Volume of the sphere = ℎ +
Radius of the hemispherical tank = m
.
= 3.14x1 x 8 + x3.14x = 25.12 + x 8.5x8.5x8.5
P a g e 94 | 101 Available in ykoyyur.blogspot.com P a g e 95 | 101 Available in ykoyyur.blogspot.com
SSLC Mathematics Solutions in English YK SSLC Mathematics Solutions in English YK

Volume of the tank = x x m3 = m3 5. A container shaped like a right circular cylinder having diameter 12 cm and height 15 cm is
full of ice cream. The ice cream is to be filled into cones of height 12 cm and diameter 6 cm,
So, the volume of the water to be emtied = x m3 = x 1000 = ltr
having a hemispherical shape on the top. Find the number of such cones which can be filled
since, ltr of water emptied in 1 second with ice cream
The required time to empty ltrs of water = x seconds = 16.5minute Volume of ice cream to be filled in cone =
Volume of Hemisphere + Volume of cone
Exercise 15.3
= + ℎ
(Take π = unless stated otherwise)
1. A metallic sphere of radius 4.2 cm is melted and recast into the shape of a cylinder of = x x3 + x x 3 x 3 x 12
radius 6 cm. Find the height of the cylinder. = 2x x9+ x 3 x 12
Radius of the sphere = 4.2cm, Radius of the cylinder = 6cm
= + = cm3
πr = ℎ The ice-cream filled in the cylinder = Volume of the cylinder = x 6 x 6 x 15 = cm3
= x 4.2 = 6 h ⇒ 4 x 1.4 x 4.2 x 4.2= 36h ⇒ 4 x 1.4 x 4.2 x 4.2= 36h
⇒ 98.784 = 36h ⇒h = 2.744cm Therefore, number of cones = = = 10
2. Metallic spheres of radii 6 cm, 8 cm and 10 cm, respectively, are melted to form a single solid sphere. 6. How many silver coins, 1.75 cm in diameter and of thickness 2 mm, must be melted to form a
Find the radius of the resulting sphere. cuboid of dimensions 5.5 cm × 10 cm × 3.5 cm?
= 6cm, = 8cm, = 10cm
No.of silver coins = =
Let radius of the resulting sphere = r
Volume of silver coin lbh = 5.5 x 10 x 3.5 = 192.5cm3
= + + ⇒ = ( + + ) . .
Volume of cuboid ℎ = 0.2 = 0.48125
⇒ = (6 + 8 + 10 ) ⇒ = (216 + 512 + 1000) .
⇒ = 1728 ⇒ r = 12cm No.of silver coins = .
= 400
3. A 20 m deep well with diameter 7 m is dug and the earth from digging is evenly spread out to 7. A cylindrical bucket, 32 cm high and with radius of base 18 cm, is filled with sand. This
form a platform 22 m by 14 m. Find the height of the platform. bucket is emptied on the ground and a conical heap of sand is formed. If the height of the
Depth of the well h = 20m; radius of the well r = m conical heap is 24 cm, find the radius and slant height of the heap.
Length of the platform l = 22m; Breadth b = 14m; HeightÀ H = ? Height of the cylindrical bucket h = 32cm; radius of the base = 18cm
Volume of the well = Volume of the platform Height of heap of the sand H = 24cm
ℎ= ℎ Volume of cylindrical bucket = ℎ = x 18 x18 x 32 =
x x x 20 = 22 m × 14h Volume of heap of sand πr H = x x r x 24 =
⇒ 11 x 7 x 10 = 22 m × 14h ⇒ 10 = 4h ⇒ 14h = 22 x 7 x 5 ⇒ h = = 2.5m Volume of heap of sand = Volume of cylindrical bucket
4. A well of diameter 3 m is dug 14 m deep. The earth taken out of it has been spread evenly all
= ⇒ = = 1296
around it in the shape of a circular ring of width 4 m to form an embankment. Find the
height of the embankment . = √24 + 36 = √576 + 1296 = √1872 = √144x13 = 12√13 cm
Volume of the well = ℎ = x 1.5 x 1.5 x 14 8. Water in a canal, 6 m wide and 1.5 m deep, is flowing with a speed of 10 km/h. How much
= 22 x 2.25 x 2 = 44 x 2.25 =99cm3 area will it irrigate in 30 minutes, if 8 cm of standing water is needed?
Volume of the embankment = [ x - x ]h Speed of the water = 10km/h ⇒ 10 x 1000 m/h
Length of the water flows in 1 hour
= ℎ [30.25 - 2.25]= [28]h = 88h
l = 10 x 1000 m
Volume of the embankment = Volume of the well
Area of the rectangular canal = lb = 6 x 1.5 = 9m2
88h = 99 ⇒ h = = 1.125 ⇒ ℎ = 99 The volume of water flows in 1 hour
⇒ ℎ= = 0.26m = Area of canal x Length of water flows in 1 hour
= 9 x 10 x1000 m3

P a g e 96 | 101 Available in ykoyyur.blogspot.com P a g e 97 | 101 Available in ykoyyur.blogspot.com


SSLC Mathematics Solutions in English YK SSLC Mathematics Solutions in English YK

The volume of water flows in 30 minutes = = 45000m3 Example 13 : Hanumappa and his wife Gangamma are busy making jaggery out of
sugarcane juice. They have processed the sugarcane juice to make the molasses, which is
Hence, the area required for covering 8 cm = m of standing water poured into moulds in the shape of a frustum of a cone having the diameters of its two
= x 100 = 562500m3 = 56.25hec [ 1hec = 10000m3] circular faces as 30 cm and 35 cm and the vertical height of the mould is 14 cm (see Fig.
15.22). If each cm3 of molasses has mass about 1.2 g, find the mass of the molasses that
9. A farmer connects a pipe of internal diameter 20 cm from a canal into a cylindrical tank in
can be poured into each mould(take = )
her field, which is 10 m in diameter and 2 m deep. If water flows through the pipe at the
rate of 3 km/h, in how much time will the tank be filled? Since the mould is in the shape of a frustum of a cone, the quantity (volume) of molasses that
Speed of the water = 10km/h ⇒ 3 x 1000 m/h can be poured into it = ℎ( + + ) where, h = 14cm, = , = 15
Length of the water flows in 1 hour l = 3000 m Volume = x x 14(17.5x17.5 + 15x15 + 17.5x15)
Now, area of the pipe which is in the form of a circle = = x 22 x 2(17.5x17.5 + 15x15 + 17.5x15)
= x = 100 cm2 = m2 = x 22 x 2(306.25 + 225 + 262.5)
Volume of cylindrical tank = ℎ = x 5 x 5 x 2 = 50 m2 = x 22 x 2(793.75) = 11,641.7cm3
∴ Required time = Mass of molasses = 1.2 g
∴ The mass of the molasses that can be poured into each mould = (11641.7 × 1.2)g
= = = 1.67 hourss or 1.67x 60 = 100 min
= 13970.04 g = 13.97 kg ≈ 14 kg
Example 14 : An open metal bucket is in the shape of a frustum of a cone, mounted on a
15.5 Frustum of a Cone hollow cylindrical base made of the same me- tallic sheet (see Fig. 15.23). The diameters of
the two circular ends of the bucket are 45 cm and 25 cm, the total vertical height of the
Given a cone, when we slice (or cut) through it with a plane parallel to its base (see Fig. 15.20)
bucket is 40 cm and that of the cylindrical base is 6 cm. Find the area of the metallic
and remove the cone that is formed on one side of that plane, the part that is now left over on
the other side of the plane is called a frustum of the cone. sheet used to make the bucket, where we do not take into account the handle of the bucket.
Example 12 : The radii of the ends of a frustum of a cone 45 cm high are 28 cm and Also, find the volume of water the bucket can hold.(Take = )
7 cm (see Fig. 15.21). Find its volume, the curved s u r f a c e a r ea a n d t he t ot a l s Height of the bucket = 40 cm,(it include height of the base)
u f a c e a r ea (take = ) Height of the frustum of cone = h = (40 - 6) cm = 34 cm
Volume of frustum of cone = + + Slant height of frustum of cone = ℎ + ( − )
CSA of frustum of cone = ( + ) [ = + ( − ) Where, h = 34cm, = 22.5 , = 12.5cm
= 34 + (22.5 − 12.5) ⇒ = 34 + (10) = 35.44
TSA of frustum of cone = ( + ) + +
The area of metallic sheet used
h = 48cm, = 28cm, = 7cm = CSA of frustum of cone + Area of circular base + CSA of cylinder
= ℎ + ( − ) = [ × 35.44 (22.5 + 12.5) + × (12.5)2 + 2 × 12.5 × 6] cm2
⇒ = 48 + (28 − 7) ⇒ = (3x15) + (3x7) = (1240.4 + 156.25 + 150) cm2 = 4860.9 cm2
⇒ = 3√225 + 49 ⇒ = 3√225 + 49 = 49.65cm Now, the volume of water that the bucket can hold (also, known as the capacity
i) CSA of frustum of cone = ( + )
of the bucket)
= (28 + 7)49.65 = 22 x 5 x 49.65 = 5461.5cm2 = ℎ( + + )
ii) TSA of frustum of cone = ( + ) + + = x x 34 (22.5 + 12.5 + 22.5x12.5) = (506.25 + 156.25 + 281.25)
= (28 + 7)49.65 + x 28 x 28+ x7x7
= (943.75) = 33615.48cm3 = 33.62 °ÃlgïUÀ¼ÄÀ (¸Àj¸ÀĪÀiÁgÁV)
= 5461.5 + 22 x 4 x 28+ 22 x 7 = 5461.5 + 2464+ 154= 8079.5cm2
iii) Volume of frustum of cone = + +
= x x 45 (28x28 + 7x7 + 28x7) = x 15(784 + 49 + 196)
= x 15(784 + 49 + 196) = 48510cm3
Exercise 15.4
P a g e 98 | 101 Available in ykoyyur.blogspot.com P a g e 99 | 101 Available in ykoyyur.blogspot.com
SSLC Mathematics Solutions in English YK SSLC Mathematics Solutions in English YK

(use π = unless stated otherwise) = 16 + (8 − 20) = = √256 + 144 = √400 = 20cm


1. Now, the volume of water that the bucket can hold (also, known as the capacity of the TSA of frustum of cone
bucket) = CSA of frustum of cone + Area of the circular bottom = ( + ) +
Volume of the bucket that hold water
= 3.14(8 + 20)20 + 3.14 x 82 = 3.14(28)20 + 3.14 x 64
= ℎ( + + ) = 3.14(28)20 + 3.14 x 64 = 1758.4 + 200.96 = 1,959.36cm2
π = ; h = 14cm; = = 2cm; = 1cm Cost of metal for 100cm2 = Rs 8
, .
= x x 14(4 + 1 + 2) Therefore total cost of metals used = x 8 = Rs 156.75

= x x 14(7) = x 22x 14 = 2 5. A metallic right circular cone 20 cm high and whose vertical angle is 60° is cut into two
parts at the middle of its height by a plane parallel to its base. If the frustum so obtained
2. The slant height of a frustum of a cone is 4 cm and the perimeters (circumference) of its be drawn into a wire of diameter cm find the length of the wire.
circular ends are 18 cm and 6 cm. Find the curved surface area of the frustum
Circumference of the circular base = 18cm Cot 300 = ⇒ √3 = ⇒ BO = cm =

⇒ 2 = 18 ⇒ = cm Cot 300 = ⇒ √3 = ⇒ CD = cm =

Circumference of the circular top = 6cm Volume of the frustum of cone = ℎ( + + )


⇒ 2 = 6 ⇒ = cm = x x 10
10
+
20
+
10
x
20
√3 √3 √3 √3
CSA of frustum of cone = ( + ) 100 400 200 700
= x 10 + + = x 10 =
= ( + ) 3 3 3 3

3
Volume of the wire = Volume of the frustum of cone
= + 4= 4 = 48cm
ℎ = ⇒ ℎ =
3. A fez, the cap used by the Turks, is shaped like the frustum of a cone (see Fig. 15.24). If its
radius on the open side is 10 cm, radius at the upper base is 4 cm and its slant height is 15 ⇒ h= ⇒h= ⇒ h = 796444.44cm = 7964.44m
cm, find the area of material used for making it.
TSA of fez = CSA of fez + Area of circular top
CSA of frustum of cone = ( + ) +
Summary:
1. To determine the surface area of an object formed by combining any two of the basic solids,
= 10cm; = 4cm ; l = 15cm
namely, cuboid, cone, cylinder, sphere and hemisphere.
= (10 + 4)15 + x 42
2. To find the volume of objects formed by combining any two of a cuboid, cone,
= (14)15 + x 16 = + cylinder, sphere and hemisphere.
3. Given a right circular cone, which is sliced through by a plane parallel to its base,
= = 710 cm3
when the smaller conical portion is removed, the resulting solid is called a
4. A container, opened from the top and made up of a metal sheet, is in the form of a Frustum of a Right Circular Cone.
frustum of a cone of height 16 cm with radii of its lower and upper ends as 8 cm and 20 4. The formulae involving the frustum of a cone are
cm, respectively. Find the cost of the milk which can completely fill the container, at the Volume of frustum of cone = ℎ( + + )
rate of Rs 20 per litre. Also find the cost of metal sheet used to make the container, if it
costs Rs 8 per 100 cm2. (Take = 3.14) CSA of frustum of cone = ( + ) [ = ℎ + ( − )
TSA of frustum of cone = ( + ) + +
Volume of frustum of cone = ℎ( + + )
= x 3.14 x 16(8 x 8 + 20 x 20 + 8x20)
= x 3.14 x 16(64 + 400 + 160)
= x 3.14 x 16(624) = 10449.9cm2 ⇒ 10.45 ltr
Total amount required at the rate of Rs 20/ltr = 10.45 x 20 = Rs 209
= ℎ + ( − )

P a g e 100 | 101 Available in ykoyyur.blogspot.com P a g e 101 | 101 Available in ykoyyur.blogspot.com


Index
Real Numbers 1
Polynomials 3
Pair of Linear Equations 6
in Two Variables
My Concept Sheet Quadratic Equations 8
Arithmetic Progression 10
th Triangles 11
CLASS-10 Co-ordinate Geometry 13
Introduction to Trigonometry 14
Maths Circles
Areas Related to Circles
16
18
Surface Areas and Volumes 19
Statistics 20
Probability 22
1 2
th
Class 10 Maths Concept Sheet HCF (Highest common factor):
HCF of two positive integers can be find using the Euclid’s Division Lemma algorithm
Real Numbers We know that for any two integers a, b. we can write
following expression
a=bq + r , 0 ≤ r < b
Type of Numbers If r=0 ,then
HCF( a, b) =b
Natural Numbers: Whole number:
If r≠0 , then
N = {1,2,3,4,5……….} W= {0,1,2,3,4,5……..}
HCF ( a, b) = HCF ( b,r)
It is the counting numbers It is the counting numbers + zero
Again expressing the integer b,r in Euclid’s Division Lemma, we get
Integers: Positive integers: b=pr + r1
Z={…-7,-6,-5,-4,-3,-2,-1,0,1,2,3,4,5,6…} Z+= {1,2,3,4,5……..} HCF ( b,r)=HCF ( r,r1)
Similarly successive Euclid ‘s division can be written until we get the remainder zero, the
Negative integers: divisor at that point is called the HCF of the a and b.
Z-={…-7,-6,-5,-4,-3,-2,-1}
HCF ( a,b) = 1 Then a and b are co primes.
Rational Number: A number is called rational if it can be expressed in the form p/q where Fundamental Theorem of Arithmetic: Composite number = Product of primes
p and q are integers ( q =/ 0). HCF and LCM by prime factorization method:
Example : ½ , 4/3 ,5/7 ,1 etc. HCF = Product of the smallest power of each
Irrational Number: A number is called irrational if it cannot be expressed in the form p/q where common factor in the numbers
p and q are integers ( q =/ 0). LCM = Product of the greatest power of each prime
Example : √3,√2, √5, ฀ etc factor involved in the number
Real Numbers: All rational and all irrational number makes the collection of real number. Important Formula: HCF (a,b) X LCM (a,b) =a X b
It is denoted by the letter R.
Prime number has only two distinct factors : “1”and number itself. Eg-2,17 etc
Co Prime Number Two numbers that have only 1 as the common factor. eg - 2 and 3
Composite Number - A number that have more than 2 factors. Eg- 4, 15
3 4
th
Class 10 Maths Concept Sheet (iii) If α, β are roots of a quadratic polynomial p(x), then p(x) = x2 – (α + β) x + αβ
p(x) = x2 – (sum of roots) x + product of roots
Polynomials
Basic Concepts
• Zeroes of a polynomial. k is said to be zero of a polynomial p(x) if p(k) = 0
• Graph of polynomial.
1. Find the zeroes of the quadratic polynomial and verify the relationship between the zeroes and
(ii) Graph of a linear polynomial ax + b is a straight line. coefficient of polynomial p(x) = x2 + 7x + 12.
(ii) Graph of a quadratic polynomial p(x) = ax2 + bx + c is a parabola open upwards like , if a > 0.
Sol. p(x) = x2 + 7x + 12
(iii) Graph of a quadratic polynomial p(x) = ax2 + bx + c is a parabola open downwards like , if a < 0.
p(x) = (x + 3)(x + 4)
(iv) In general a polynomial p(x) of degree ‘n’ crosses the x-axis at atmost ‘n’ points.
p(x) = 0 if x + 3 = 0 or x + 4 = 0
x = – 3 or x = – 4
– 3 and – 4 are zeros of the p(x).
Now,

2. Find the zeroes of 4x2 – 7 and verify the relationship between the zeroes and its coefficients.
• Relationship between the zeroes and the coefficients of a Polynomial.
Sol. Let p(x) = 4x2 – 7
Here coefficient of x2 = 4,
(i) If α, β are zeroes / roots of p(x) = ax2 + bx + c, then Coefficient of x = 0 and constant term = –7.
4x²

(ii) If α, β and γ are zeroes / roots of p(x) = ax3 + bx2 + cx + d


5 6
th
3. Find a quadratic polynomial whose zeroes are
Class 10 Maths Concept Sheet
Sol. Let α, β are zeroes of quadratic polynomial p(x).
x² Pair of Linear Equations in Two Variables
Basic Concepts with Examples
• Linear Equation in Two Variables
An equation which can be put in the form
4. Find a quadratic polynomial, the sum of whose zeroes is 0 and one zero is 5. ax + by + c = 0
Sol. Let zeroes are α and β. where a, b and c are real numbers {a, b ≠ 0) is called a linear equation in two variables ‘x’ and ‘y’
α + β = Sum of zeroes
α + β = 0 5 + β = 0 β = –5 • General Form of a Pair of Linear Equations in Two Variables
Now product of zeroes = αβ = 5 × (–5) = –25 General form of a linear pair of equations in two variables is:
Let polynomial p(x) = ax2 + bx + c a1x + b1y + c1 = 0 and
a2x + b2 y + c2 = 0
where a1, b1, c1, a2, b2, c2 are real numbers such that
a12 + b12 =/ 0 and a22 + b22 =/ 0

• Solution of a Pair of Linear Equations in Two Variables


The solution of a linear equation in two variables ‘x’ and ‘y’ is a pair of values (one for ‘x’ and other for ‘y’)
which makes the two sides of the equation equal.
There are two methods to solve a pair of linear equations:
(i) algebraic method
(ii) graphical method.

• Algebraic Method
We have already studied (i) Substitution method and (ii) Elimination method. Here, we will study
cross-multiplication method also.
If a1 x + b1 y + c1 = 0
a2 x + b2 y + c2 = 0
7 8
th
form a pair of linear equations, then the following three situations can arise:
Class 10 Maths Concept Sheet
(i) If then the system is consistent.
Quadratic Equations
(ii) If then the system is inconsistent.
A Polynomial of the form p(x) = ax2 + bx + c, where a =/ 0 and a, b, c are real numbers and x is a real variable
(ii) If then the system is dependent and consistent. is called a quadratic polynomial.
An equation p(x) = 0, where p(x) is a quadratic polynomial is called a quadratic equation
• Graphical Method of Solution of a Pair of Linear Equations i.e. ax2 + bx + c = 0, a =/ 0
(i) If the graphs of two equations of a system intersect at a point, the system is said to have a unique Zeros of Quadratic Equations
solution, i.e., the system is consistent.
Those values of x for which ax2 + bx + c = 0 is satisfied are called zeros of quadratic equation.
(ii) If the graphs of two equations of a system are two parallel lines, the system is said to have no solution,
i.e., the system is inconsistent. Quadratic equation is classified into two categories
(iii) When the graphs of two equations of a system are two coincident lines, the system is said to have • Pure quadratic equation of type
infinitely many solutions, i.e., the system is consistent and dependent. ax2 + c = 0
by putting b = 0 in ax2 + bx + c = 0
• Affected quadratic equation of type ax2 + bx + c = 0, b =/ 0.
Roots of Quadratic Equations
If α, β are the zeros of the polynomial ax2 + bx + c. Then α, β are called roots of corresponding equation
ax2 + bx + c = 0
p(α) = p(β) = 0
i.e. aα2 + bα + c = 0
and aβ2 + bβ + c = 0
Pure quadratic ax2 + c = 0 can be solved by any one of the following methods:
• By Taking square root
• By factorisation
Affected quadratic equation can be solved by any one of the following method:
• By splitting middle term
• By method of completing the square
D = b2 – 4ac, is called the discriminant which decides the nature of roots.
• If D > 0, Roots are real and unequal.
• If D = 0, Roots are real and equal.
• If D < 0, No Real roots are possible.
9 10
th Concept Sheet
The quadratic formula or Sridharacharya’s formula to find the roots of ax2 + bx + c = 0 is
Class 10 Maths
Arithmetic Progression
A group of numbers connected by a definite law is known as sequence.

Arithmetic Progression (A.P.)


A Sequence in which each term is obtained from the preceeding term by adding a constant quantity to it.
A sequence is called a series if its terms are connected by the sign of addition or subtraction.
nth term of an Arithmetic Progression.
an = a + (n - 1) d = l
where ‘a’ is first term and ‘d’ is common difference ‘l’ is last term.

Selection of terms of an A.P.


• When odd number of terms are required. Take middle term as ‘a’ and common difference as ‘d’.
• When even number of terms are required take a - d, a + d as two middle terms and ‘2d’ as
common difference.
The condition for three terms to be in an Arithmetic Progression is that common difference between
them must be same.
t3 ‐ t2 = t2 – t1
Sum of n terms of an A.P.
l is the last term
a is the first term
d is the common difference
nth term from the end is : l - (n - 1)d.
where l is last term, d is common difference.
The Standard form of an Arithmetic Progression is
a + (a + d) + (a + 2d) + .... (l - d) + l
a is first term, l is last term, d is common difference

nth term of an Arithmetic Progression is the difference of the sum to first n terms and
the sum to first (n - 1) terms
an = Sn - Sn - 1
11 12
th
Class 10 Maths Concept Sheet
Triangles BASIC PROPORTIONALITY THEOREM
Basic Concepts with Examples
• SIMILAR TRIANGLES. Two triangles are said to be similar if
(i) their corresponding angles are equal and
(ii) their corresponding sides are proportional.
• All congurent triangles are similar but the similar triangles need not be congruent.
• Two polygons of the same numbers of sides are similar, if
(i) their corresponding angles are equal and
(ii) their corresponding sides are in the same ratio.
• BASIC PROPORTIONALITY THEOREM. In a triangle, a line drawn parallel to one side,
to intersect the other sides in distinct points, divides the two sides in the same ratio.
• CONVERSE OF BASIC PROPORTIONALITY THEOREM. If a line divides any two sides of
a triangle in the same ratio, the line must be parallel to the third side.
13 14
th Concept Sheet th Concept Sheet
Class 10 Maths Class 10 Maths
Co-ordinate Geometry Introduction to Trigonometry
Trigonometric ratios
•The certain ratios involving the sides of a right angled triangle are called Trigonometric ratios.
Suppose:
b is the base
h is the hypotenuse
p is perpendicular
then,
sin A = Perpendicular = p
hypotenuse h
cos A = Base = b
hypotenuse h
tan A = Perpendicular = p
Base b
Reciprocals of the ratios are:
Cosec A= 1/sin A= h/p
Sec A= 1/cos A= h/b
Cot A= 1/tan A= b/p
• Sin x is a single symbol and sin cannot be detached from ‘x’.
This remark is true for other ratios as well
Trigonometric /Ratios of some specific angles
The specific angles are 0°, 30°,45°, 60°, 90°. These are given in the following table
15 16
th Concept Sheet
The value of sin A increases from 0 to 1, as A increases from 0° to 90°
Class 10 Maths
The value of cosA decreases from 1 to 0, as A increases from 0° to 90°
The value of tan A increases from 0 to infinity, as A increases 0° to 90° Circles
√2 = 1.414 and √3 = 1.732
Trignometric identities
Facts that Matter
•Tangent to a Circle
• sin2 A + Cos2 A = 1
A tangent to a circle is a line that touches the circle at only one point.
• 1+tan2 A = sec2 A
• 1+Cot2 A = cosec2 A Theorem 1
The tangent at any point of a circle is perpendicular to the radius, through the point of contact.
Proof: We have the centre O of the given circle and XY is the tangent to the circle at a point P
Let us take a point Q on Xy other than P, Join OQ.

Obviously, Q lies outside the circle.


i.e., OQ > OP
Since, all the points on XY, except P lies outside the circle.
i.e., OP is smaller than all the distance of the point O from XY.
i.e., OP is the smallest distance of O from XY.
i.e., OP XY
17 18
th Concept Sheet
Class 10 Maths
Theorem 2 Areas Related to Circles
Lengths of two tangents drawn from an external
point to a circle are equal.

Given: AP and AQ are two tangents drawn from a point A to a circle C (0, r).
To prove: AP = AQ.
Construction: Join OP, OQ and OA.
Proof: In MOQ and APO
LOQP = LOPA [Tangent at any point of a circle is perp. to radius through the point of contact]
AO = AO [Common]
OQ = OP [Radius]
So by R.H.S. criterion of congruency AOQ ::: MOP .
AQ = AP. [By CPCT] Hence Proved
19 20
th th
Class 10 Maths Concept Sheet Class 10 Maths Concept Sheet
Surface Areas and Volumes Statistics
• Lets remember that the mean, mode and median are measures if central tendency ie numerical
representatives of the given data.
• Mean of the grouped data
1. Using direct method:
2. Using assumed mean method:
3. Using step deviation method:

EMPIRICAL FORMULA
MODE = 3 MEDIAN - 2 MEAN
21 22
th
•The mode of the grouped data is
Mode: l + ( f1–f2/ 2f1–f0–f2)x h
Class 10 Maths Concept Sheet
Where, l= lower limit of the class
h= size of the class
f1= frequency of the modal class
Probability
f0= frequency of the preceding class
• The Cumulative Frequency of a class-interval is the sum of frequencies of that class and the classes which
precede (come before) it.
• Median if the grouped data

Median = l + ((N/2–cf)/f) X h
Where, l = lower limit of median class
N = number of observations
cf = cumulative frequency of the class preceding the median class
f = frequency of median class
h = class size
23

DECK OF CARDS (52)


RED BLACK
26 26
HEART DIAMOND CLUB SPADE
FACE CARDS

KING KING KING KING


QUEEN QUEEN QUEEN QUEEN
JACK JACK JACK JACK
10 10 10 10
9 9 9 9
8 8 8 8
7 7 7 7
6 6 6 6
5 5 5 5
4 4 4 4
3 3 3 3
2 2 2 2
ACE ACE ACE ACE
S.S.L.C. OLD QUESTION
PAPERS WITH SOLUTIONS

“ MATHEMATICS GIVES US HOPE THAT EVERY


PROBLEM HAS A SOLUTION ”
MATHEMATICS

BY :
SRI. VEERENDRA H.M.
G.P.U.C. HS MEGARAVALLI, THIRTHAHALLI TQ
SMT. SHASHIPRABHA K.S.
G.H.S. ARASALU, HOSANAGARA TQ SHIMOGA.

Sri.Veerendra.H.M. Asst.Teacher. G.P.U.C. Megaravalli. Thirthahalli Tq Ph: 8762624683 Page-1 Sri.Veerendra.H.M. Asst.Teacher. G.P.U.C. Megaravalli. Thirthahalli Tq Ph: 8762624683 Page-2
6) If the n-th term of an arithmetic progression is 5𝑛+3, its 3rd term is
UNIT-1. ARITHMETIC PROGRESSIONS: 𝐴) 11 𝐵) 18 𝐶) 12 𝐷) 13 June : 2019
7) If the n-th term of an arithmetic progression 𝑎n = 4n2-1, then its 8th term is
Arithmetic progression : An arithmetic progression is a list of numbers in which
A) 32 B) 31 C) 256 D) 255 Model-2 : 2019-20
each term is obtained by adding a fixed number to the
preceding term except the first term. 8) If the n-th term of an arithmetic progression 𝑎n = 3n+6 , then its 8th term is
This fixed number is called the common difference(d) of the A.P. A) 25 B) 24 𝐶) 30 𝐷) 17 Prep ∶ 2020
It can be positive, negative or zero. 9) If the nth term of an arithmetic progression an = 3n – 2, then its 9th term is
Example : 1) 1, 2, 3, 4, .................... A) -25 B) 5 𝐶) -5 𝐷) 25 Sept ∶ 2020
2) 100, 80, 60, ................. 10) If the n term of an arithmetic progression 𝑎n = 4𝑛 + 5, then its 3rd term is
th

3) -3, -2, -1, .................. 𝐴) 5 𝐵) 9 𝐶) 13 𝐷) 17 Model-1 ∶ 2020-21


The terms of the arithmetic progression are denoted by a1, a2, a3, a4, ............ an. 11) The n-th term of an arithmetic progression 𝑎n = 4𝑛 + 5, then its 5th term is
Common difference(d) = a2 – a1 = a3 – a2 = ................... = an – an-1 = an+1 – an 𝐴) 20 𝐵) 14 𝐶) 25 𝐷) 24 July ∶ 2021
General form of an A.P. : a, a+d, a+2d, ................., a + (n-1)d 12) In an arithmetic progression, if 𝑎n =2𝑛 +1, then the common difference of
The first term of an A.P.is ‘a’ and common difference is ‘d’ then, the given progression is June : 2020
nth term of an A.P. : an = a + (n-1)d 𝐴) 0 𝐵) 1 𝐶) 2 𝐷) 3
nth term from the last term = l - (n-1)d [ l – last term ] 13) The nth term of an Arithmetic progression is given by an = 7 - 4n then the
𝐧
Sum of ‘n’ terms of an A.P.: Sn = {2a + (n-1)d } common difference is Model-1 ∶ 2020-21
𝐧
𝟐 A. 4 B. -4 C. 3 D. -3
Sum of ‘n’ terms : Sn = {a + an } [When first term is ‘a’ and last term is ‘an’] 14) 10th term of an arithmetic progression 5, 9, 13,......... is Model-1 : 2019-20
𝟐
𝐧(𝐧 𝟏)
Sum of ‘n’ natural numbers Σn or Sn = A) 36 B) 31 𝐶) 41 𝐷) 21
𝟐
Sum of ‘n’ even numbers : Sn = n(n+1) 15) The common difference of an A.P. 3, 1, −1, −3, .....
Sum of ‘n’ odd numbers: Sn = n2 A) −2 B) 2 𝐶) −5 𝐷) 5
th
Sum of ‘n’ and (n-1) terms is given, then an = Sn - Sn-1 16) The 20 term of an Arithmetic progression 1, 5, 9, 13 ........ is
𝐚 𝐜 A. 77 B. 75 C. 76 D. 74 Model-2 ∶ 2021
Arithmetic mean : a, b and c are in A.P. then, b = ( b-Arithmetic mean)
𝟐 17) The 11th term of the arithmetic progression -3, -1, 1, 3, ........... is
MULTIPLE CHOICE QUESTIONS : 𝐴) 23 𝐵) -23 𝐶) -17 𝐷) 17 July ∶ 2021
1) Which of the following is an Arithmetic Progression ? July ∶ 2021 18) The 10th term of an Arithmetic Progression 0, 4, 8, 12, . ...... is,
A) 1, -1, -2, ............... 𝐵) 1, 5, 9, ................ A) 36 B) 40 C) 44 D) 32.
𝐶) 2, -2, 2, -2, .......... 𝐷) 1, 2, 4, 8, ............ 19) Sn = 50, Sn-1 = 42, Then the value of an is Model-1 ∶ 2020-21
2) The common difference of the Arithmetic progression 8, 5, 2, −1, ..... is
A) 50 B) 42 𝐶) 20 𝐷) 8
A) −3 B) -2 𝐶) 3 𝐷) 8 March-2022
20) The sum of first 15 terms of an arithmetic progression is 465 and the sum
3) The common difference of the arithmetic progression 100, 93, 86, .......... is
of first 14 terms of the same arithmetic progression is 406. Then its 15th
A) 4 B) 8 C) 7 D) -7 Model-QP ∶ 2022 term is Model-2 ∶ 2021
4) In an arithmetic progression 5, 3, 1, – 1, .... the common difference is A. 95 B. 59 C. 69 D. 58
A) (–2) B) 2 C) (–3) D) 5. JUNE∶ 2022 21)The sum of first 10 terms of an arithmetic progression is 155 and the sum of
5) If the n-th term of an arithmetic progression 𝑎n = 24 − 3𝑛, then its 2nd term is the first 9 terms of the same progression is 126 then the 10th term of the
𝐴) 18 𝐵) 15 𝐶) 0 𝐷) 2 March ∶ 2019 progression is July ∶ 2021
Sri.Veerendra.H.M. Asst.Teacher. G.P.U.C. Megaravalli. Thirthahalli Tq Ph: 8762624683 Page-3 Sri.Veerendra.H.M. Asst.Teacher. G.P.U.C. Megaravalli. Thirthahalli Tq Ph: 8762624683 Page-4
𝐴) 27 𝐵) 126 𝐶) 29 𝐷) 25 ONE MARK QUESTIONS :
22) The first term and the last term of an arithmetic progression are ‘a’ 1) What is an Arithmetic progression ? June∶ 2022
and ‘l’ respectively, then the sum of its first ‘n’ terms is Model-2 ∶ 2021 Ans. : An arithmetic progression is a list of numbers in which each term is
( ) ( ( ) ) ( ) ( )
A. Sn = B. Sn = C. Sn = D. S n = obtained by adding a fixed number to the preceding term, except the first term.
23) The formula to find the sum of first ‘n’ terms of positive odd numbers is 2) In an Arithmetic Progression if ‘a’ is the first term and ‘d’ is the common
3 2 difference, then write its nth term. March-2022
A) Sn = n ( n+ 1) B) Sn = n ( n- 1) C) Sn = n D) Sn = n PREP∶ 2022
Ans : an = a + (n – 1)d
24) In an A.P. S1 = 5, S2 = 12 Then the common difference is Model-1 ∶ 2020-21
3) The first and last terms of an Arithmetic Progression are ‘a’ and ‘an’, then
A) −2 B) 1 𝐶) 2 𝐷) 3
write the formula to find the sum of ‘n’ terms. June∶ 2019
25) If 2, x, 14 are in Arithmetic Progression, then the value of ‘x’ 𝐧
Ans : Sn = {𝐚 + an }
𝟐
𝐴) 28 𝐵) 16 𝐶) 7 𝐷) 8 Model-2 ∶ 2020-21
4) The first term and common difference of an Arithmetic Progression are
26) If 8, x, 20 are in arithmetic progression, the value of ʻxʼ is Model-2 ∶ 2021
6 and 5. Find the 3rd term of the progression. Model-1: 2020
A. 10 B. (−10) C. 14 D. 8 Ans : a = 6, d = 5, n = 3, a3 = ?
27) If 4, x, 10 are in Arithmetic Progression, then the value of ‘x’
a3 = a + 2d
𝐴) 14 𝐵) -6 𝐶) -7 𝐷) 7 July ∶ 2021
a3 = 6 + 2x5
28) The terms of an arithmetic progression are 18, a, b, -3, so a+b =
∴ a3 = 16
A) 19 B) 15 𝐶) 11 𝐷) 7
5) If the n-th term of an arithmetic progression is an = 3n + 2, Find its 12th term.
29) If x, 8, 11, y are the consecutive terms of an Arithmetic progression. The
Ans : 𝑎n = 3𝑛 + 2 Prep ∶ 2020
values of ‘x’ and ‘y’ are respectively equal to Model-2 ∶ 2021
A. 6 and 13 B. 4 and 15 C. 3 and 16 D. 5 and 14 𝑎12 = 3x12 + 2
30) If 4, a, b, 28 are in Arithmetic progression then the value of ‘b’ is 𝑎12 = 36 + 2
∴ 𝒂12 = 38
A) 20 B) 19 𝐶) 23 𝐷) 32 Model-I ∶ 2021
6) If the n-th term of an arithmetic progression is an = 3n - 2, Find its 2nd term.
31) 7, a, b, 22 are the consecutive terms of an Arithmetic Progression , Ans : 𝑎n = 3𝑛 - 2 Model-2 ∶ 2021
then the values of ‘a’ and ‘b’ are respectively equal to, 𝑎2 = 3x2 - 2
A) 11 and 16 B) 12 and 17 C) 13 and 17 D) 12 and 16 𝑎2 = 6 - 2
32) Two arithmetic progressions has the same common difference. If the first ∴ 𝒂2 = 4
term of the first progression is 3 and that of the other is 8, then the 7) In an Arithmetic Progression, the 8th term is 17 and 19th term is 39.
difference between their 3rd term is Model-1 ∶ 2021
Find the common difference. Prep ∶ 2020
A. 2 B. 3 C. 4 D. 5
Ans : a8 = 17, a19 = 39, d = ?
33) The sum of first ‘n’ terms of an arithemetic progression is given by the 𝐚𝐪 𝐚𝐩
d=
formula Sn = 3n2 + n, then its 3rd term is Model-1 ∶ 2021 𝐪 𝐩
A. 14 B. 16 C. 22 D. 42 d=

Answers : 1.B, 2.A, 3.D, 4.A, 5.A, 6.B, 7.D, 8.C, 9.D, 10.D, 11.C, 12.C, 13.B, 14.C, 15.A, d= ∴d=2
16.A, 17.D, 18.A, 19.D, 20.B, 21.C, 22.C, 23.D, 24.C, 25.D, 26.C, 27.D, 28.B, 8) In an Arithmetic progression the sum of first four terms is 20 and the sum
29.D, 30.A, 31.B, 32.B, 33.B.
of first three terms is 12 then find the fourth term of the arithmetic
progression. Model-QP ∶ 2022
Sri.Veerendra.H.M. Asst.Teacher. G.P.U.C. Megaravalli. Thirthahalli Tq Ph: 8762624683 Page-5 Sri.Veerendra.H.M. Asst.Teacher. G.P.U.C. Megaravalli. Thirthahalli Tq Ph: 8762624683 Page-6
Ans : S4 = 20, S3 = 12, a4 = ? From given, a7 = a5 + 12
an = Sn – Sn-1 a + 6d = a + 4d + 12
a4 = S4 – S3 = 20 – 12 ⇒ a4 = 8 2d = 12
9) The 17th term of an Arithmetic Progression exceeds its 10th term by 7. d=6
Substitute d = 6 in Eqn(1), we get
Write the common difference of this progression. PREP∶ 2022
Ans : a17 = a10 + 7 a + 2x6 = 16
a + 16d = a + 9d +7 a = 16 – 12
16d -9d = 7 a =4
7d = 7 ∴ Arithmetic Progression is : a , a+d, a+2d, ..... = 4, 10, 16, 22, ..........
d=1 5) Verify whether 130 is a term of the arithmetic progression 3, 7, 11 .........
TWO MARKS QUESTIONS : Ans : Here, a = 3, d = 7-3 = 4, 𝑎n = 130, n = ? MODEL-QP-2022
th
1) Find the 10 term of arithmetic progression 2, 7, 12 .... using the formula. 𝑎n = a +(n-1)d
Ans : Given A.P.is: 2, 7, 12, ....... Model-2 ∶ 2021 130 = 3+(n-1)x4
Here, a = 2, d = 7-2 = 5, n = 10, 𝑎10 = ? 130 = 3+4n – 4
𝑎n = a +(n-1)d 130 = 4n - 1
𝑎10 = 2+(10-1)x5 4n = 131
𝑎10 = 2+9x5 n=
∴ 𝑎10 = 47
Here, ‘n’ is not an integer.
2) Find the 25th term of the A.P. 2, 6, 10, 14, .........… Model-1∶ 2021
∴ 130 is not a term of the arithmetic progression 3, 7, 11 .........
Ans : Here, a = 2, d = 6-2 = 4, n = 25, 𝑎25 = ?
6) Find the sum of first 20 terms of Arithmetic series 2 + 7 + 12 + ... using
𝑎n = a +(n-1)d
suitable formula. March ∶2019
𝑎25 = 2+(25-1)x4
Ans. : Here, a = 2, d = 7 – 2 = 5, n = 20, S20= ?
𝑎25 = 2+24x4 𝐧
∴ 𝑎25 = 98 Sn = {𝟐𝐚 + (𝐧 − 𝟏)𝐝 }
𝟐
3) How many two-digit numbers are divisible by 3 ? Model ∶ 2019 S20 = { 2x2 + (20-1)5}
Ans : Given A.P.is : 12, 15 , 18, ........ , 99 S20 = 10{4 + 19x5}
S20 = 10{ 4 + 95}= 10 x 99
𝑎 = 12, d = 3 , an = 99 , n = ?
∴ S20 = 990
𝑎n = a +(n-1)d
99 = 12 +(n-1)3 7) Find the sum of first 15 terms of 3 + 6 + 9 .................. using the formula
99 = 12 +3n-3 Ans. : Here, a = 3, d = 6-3 = 3 , n = 15 , S15 = ? MODEL-QP-2022
99-9 = 3n 𝐧
Sn = {𝟐𝐚 + (𝐧 − 𝟏)𝐝 }
3n = 90 𝟐
n = 30 S15 = {2x3 + (15 − 1)x3 }
∴ there are 30 two-digit numbers divisible by 3. S15 = {6 + 14x3 }
4) Determine the AP whose third term is 16 and the 7th term exceeds the S15 = {6 + 42 }
5th term by 12. = x48
Ans : From given, a3 = 16 ∴ S15 = 360
a + 2d = 16 --------- (1)
Sri.Veerendra.H.M. Asst.Teacher. G.P.U.C. Megaravalli. Thirthahalli Tq Ph: 8762624683 Page-7 Sri.Veerendra.H.M. Asst.Teacher. G.P.U.C. Megaravalli. Thirthahalli Tq Ph: 8762624683 Page-8
8) Find the 20th term from the last term of the A.P. 10, 7, 4, ..........., -62. EXERCISE :
Ans : If we write the given A.P. in the reverse order : -62, .......,4,7,10 1) Find the 15th term of the arithmetic progression 3, 8, 13 .................. using
Here, a = (-62), d = 7-4 = 3, n = 11, 𝑎11 = ? the formula. PREP∶ 2022
Ans : ∴ 𝑎15 = 73
𝑎n = a +(n-1)d
2) Find the 30th term of the arithmetic progression 5, 8, 11 .............. using
𝑎11 = -62+(11-1)x3
the formula. March-2022
𝑎11 = -62+10x3
Ans : ∴ 𝑎30 = 92
∴ 𝑎11 = -62+30 = (-32)
3) Find the 15th term of the arithmetic progression 6, 10, 14 ............... using
9) Find the sum of first 20 positive integers using formula. March-2022
( ) the formula. MODEL-QP-2022
Ans : Sn = Ans : ∴ 𝑎15 = 62
n = 20 4) Find the sum of first 20 terms of Arithmetic series 3 + 7 + 11 + ... using
( )
S20 = suitable formula. Ans. : ∴ S20 = 820 Prep ∶ 2020
= 5) Find the sum of 5+8+11+⋯ to 10 terms using formula. June-2020
= 10x21 Ans. : ∴ S10 = 185
S20 = 210 6) Find the sum of first 20 terms of Arithmetic series 5 + 10 + 15 + ... using
10) Find the sum of the first 30 positive integers divisible by 6. Model-1 ∶ 2021
suitable formula. Ans. ∴ S20 = 1050 Sept ∶ 2020
Ans : Given A.P.is : 6, 12, 18, ............. 7) Find the sum of first 20 terms of Arithmetic series 3 + 8 + 13 + ... using
Here, a = 6, d = 12-6 = 6 , n = 30 , S30 = ? suitable formula. Ans : ∴ S20 = 1010 Model-1 ∶ 2021
𝐧
Sn = {2a + (n-1)d} 8) Find the sum of 2+5+8+................. to 20 terms using the formula.
𝟐
S30 = {2x6 + (30 − 1)x6 } Ans. : ∴ S20 = 610
9) Find the sum of first 20 terms of the Arithmetic series 2 + 7 + 12 + .........
S30 = {12 + 29x6 }
using the formula. Ans. : ∴ S20 = 990 PREP∶ 2022
S30 = 15{12 + 174 }= 15x186
∴ S30 = 2790 10) Find the sum of first 20 terms of the Arithmetic progression 10, 15, 20, .........
11) Find how many terms of the arithmetic progression 3, 6, 9, .... must be using the formula. March-2022
added to get the sum 165. JUNE∶ 2022 Ans. : ∴ S20 = 1150
Ans. : Here a = 3, d = 6-3 = 3, Sn = 165, n = ? 11) Find the 12th term of the Arithmetic progression 2, 5, 8, ..... using formula.
𝐧
Sn = {2a + (n-1)d} Ans. : ∴ 𝑎12 = 35 March-2022
𝟐
165 = {2x3 + (n − 1)x3 } 12) Find the sum of arithmetic progression 7, 11, 15, ...... to 16 terms using
165 = {6 + 3n − 3 } formula. June∶ 2022
Ans. : ∴ S16 = 592
165 = {3n + 3 }
165 = 3{n + 1 }
= n{n + 1 }
n{n + 1 } = 110
n{n + 1 } = 10x11
n = 10 ∴ The sum of first 10 terms of the A.P. is 165
Sri.Veerendra.H.M. Asst.Teacher. G.P.U.C. Megaravalli. Thirthahalli Tq Ph: 8762624683 Page-9 Sri.Veerendra.H.M. Asst.Teacher. G.P.U.C. Megaravalli. Thirthahalli Tq Ph: 8762624683 Page-10
THREE MARKS QUESTIONS : Eqn(3) + Eqn(4)
1) The seventh term of an Arithmetic progression is four times its second 12a -6d = 0
term and twelfth term is 2 more than three times of its fourth term. 2a+6d = 28
Find the progression. March-2019 14a = 28
Ans : 𝑎n = a +(n-1)d a=2
Substitute a = 2, in Eqn (1)
From data, 𝑎7 = 4(𝑎2)
4x2 - 2d = 0
a +6d = 4x(a+d) -2d = -8
a +6d = 4a+4d d=4
3a - 2d = 0 ----------- (1) ∴ Total length of the line segment = a + a+d + a+2d + a+3d = 4a+ 6d
From data, 𝑎12 = 3(𝑎4)+2
= 4x2 + 6x4 = 8+24
a +11d = 3(a+3d)+2, ∴ Total length of the line segment = 32 cm
a +11d = 3a+9d+2
a- 3a + 11d -9d = 2 3) Sum of first ‘n’ terms of an Arithmetic Progression is 210 and sum of
2a -2d = -2 --------- (2)
Eqn (1) - Eqn (2) (n-1) terms is 171. First term of the A.P. is 3, then find A.P. Model-2 : 2020
3a - 2d = 0 Ans : Sn = 210, Sn-1 = 171, a = 3
𝐧
(-)2a –(+)2d = -(+)2 Sn = {2a + (n-1)d}
𝟐
a = 2 210 = {2x3 + (n − 1)xd}
Substitute a = 2 in Eqn (1)
420 = n{6 + (n − 1)xd} ------------- (1)
3x2 – 2d = 0
Sn - Sn-1 = an
-2d = -6
d=3 210-171 = an
𝑎n = a +(n-1)d
General form of A.P. : a , a+d, a+2d, .....
39 = 3 + (n -1)d
∴ Required A.P.is : 2, 5, 8, ..........
(n -1)d = 36 --------- (2)
Substitute Eqn(2) in (1),
2)A line segment is divided into four parts forming an Arithmetic progression. 420 = n{6 + 36}
The sum of the lengths of 3rd and 4th parts is three times the sum of the 42n = 420
lengths of first two parts. If the length of 4th part is 14 cm, find the total n = 10
length of the line segment. March-2019 Substitute n = 10 in Eqn (2)
Ans. : Four parts of line segment are : a , a+d, a+2d, a+3d. (10-1)d = 36
From data, a3 + 𝑎4 = 3(𝑎1 + 𝑎2) 9d = 36
a+2d+ a+3d = 3(a+ a+d) d=4
2a+5d = 6a + 3d ∴ General form Arithmetic Progression is : a , a+d, a+2d, .....
4a -2d = 0 ------ (1) ∴ Arithmetic Progression : 3, 7, 11, ..........
From data, 𝑎4 = 14 ⇒ a+3d = 14 --------(2)
Eqn (1) x 3 and Eqn (2) x 2 , then 4) The 12th term of an A.P. is -13 and sum of first four terms is 24. Find sum
(4a -2d = 0) x3 ⇒ 12a - 6d = 0 ------ (3) of first 20 terms. Prep ∶ 2020
(a+3d = 14) x2 ⇒ 2a +6d = 28 ------ (4) Ans : 𝑎n = a +(n-1)d
Sri.Veerendra.H.M. Asst.Teacher. G.P.U.C. Megaravalli. Thirthahalli Tq Ph: 8762624683 Page-11 Sri.Veerendra.H.M. Asst.Teacher. G.P.U.C. Megaravalli. Thirthahalli Tq Ph: 8762624683 Page-12
From given, 𝑎12 = -13 Sum of first 20 terms :
a +11d = -13 ----------- (1) Sn = {2a + (n − 1)d }
Sum of first four terms = 24.
S20 = {2x2 + (20 − 1)x − 2}
a+ a+d+a+2d+a+3d = 24
4a +6d = 24 --------- (2) S20 = 10x{4 + 19x − 2 }
Multiply Eqn (1) by 4, S20 = 10 x {4 − 38 }= 10 x -34
∴ S20 = (-340)
4a +44d = -52 ----------- (3)
Eqn(3) - Eqn(2)
6) An arithmetic progression consists of 37 terms. The sum of the middle
4a + 44d = -52
(-)4a +(-)6d = (-)24
term and its adjacent terms is 225 and the sum of its last 3 terms is 429,
38d = -76 then find the first three terms of the progression.
d = -2 Ans : Arithmetic Progression is : a , a+d, a+2d, ........., a+36d
Substitute d = -2 in Eqn(1) Middle term = 𝑎19 = a +18d
a + 11x-2 = -13 Its adjacent term are = 𝑎18 = a +17d, and 𝑎20 = a +19d
a = -13 + 22
Sum of the middle term and its adjacent terms = 225
a=9
Sum of first 20 terms : 𝑎18+ 𝑎19 +𝑎20 = 225
∴ a +18d+ a +17d+ a +19d = 225
Sn = {2a + (n − 1)d } 3a + 54d = 225 ------ (1)
S20 = {2x9 + (20 − 1)x − 2} Sum of its last 3 terms = 429
S20 = 10x{18 + 19x − 2 } a35 + a36 + a37 = 429
S20 = 10 x {18 − 38 }= 10 x -20 a +34d+ a +35d+ a +36d = 429
∴ S20 = (-200) 3a + 105d = 429 -------- (2)
5) The 14th term of an Arithmetic Progression is twice the 8th term. The 6th Subtract Eqn(1) by Eqn(2)
term is -8, then find sum of first 20 terms. Prep ∶ 2020 3a + 105d = 429
Ans : 𝑎n = a +(n-1)d (-)3a +(-)54d = (-)225
51d = 204
From given, 𝑎14 = 2x 𝑎8
d=4
a + 13d = 2(a+7d) Substitute d = 4 in Eqn(1)
a + d = 0 ----------- (1) 3a + 54x4 = 225
From given, 𝑎6 = (-8) 3a = 225-216
a +5d = -8 --------- (2) a=3
Eqn(1) - Eqn(2) ∴ Arithmetic Progression is : a , a+d, a+2d = 3, 7, 11, .......
a + d = 0
(-) +(-)5d = -(+)8
a 7) The ratio of 11th and 8th term of an A.P. is 2:3. Then find the ratio of 5th
-4d = 8
and 21st term and ratio of sum of first 5 terms and sum of first 21 terms.
d = (-2)
Ans : 𝑎n = a +(n-1)d
Substitute d = (-2) in Eqn(1)
a +(-2) = 0 From given, 𝑎11 : 𝑎8 = 2 : 3
a=2 =
Sri.Veerendra.H.M. Asst.Teacher. G.P.U.C. Megaravalli. Thirthahalli Tq Ph: 8762624683 Page-13 Sri.Veerendra.H.M. Asst.Teacher. G.P.U.C. Megaravalli. Thirthahalli Tq Ph: 8762624683 Page-14
3(a+10d) = 2(a+7d) 248 = 12 + (n -1)x4
3a + 30d = 2a+14d 248 = 12 + 4n - 4
a = -16d 248 = 4n + 8
𝑎5 : 𝑎21 = = = = 4n = 248 -8
𝑎5 : 𝑎21 = 3 : 1 n=
Sn = {2a + (n − 1)d } n = 60
S5 : S21 = {2a + 4d } : {2a + 20d } Sn = {a + a }
= x2{a + 2d } : x 2{a + 10d } Sn = {12 + 248}
= 5(-16d+2d) : 21(-16d + 10d) Sn = 30{260}
= 5(-14d) : 21(-6d)
Sn = 7800
= -70d : -126d
∴ S5 : S21 = 5 : 9
10) The sum of first 9 terms of an Arithmetic progression is 144 and its 9th
8) The sum of Rs.700 is to be used to give seven cash prizes to students of a term is 28 then find the first term and common difference of the Arithmetic
school for their overall academic performance. If each prize is Rs.20 less than progression. March-2022
its preceding prize, Find the value of each of the prizes. MODEL-QP-2022 Ans : n = 9, a9 = 28, S9 = 144, a = ?, d = ?
Ans : Sn = [ a + l ]
Let the first prize be ‘a’
S9 = [ a + 28 ]
The amount of second prize = a – 20
The amount of third prize = a – 40 144 = [ a + 28 ]
This is an AP with common difference (-20) and first term ‘a’
= a + 28
d = (-20), S7 = 700
32 = a + 28
Sn = {2a + (n − 1)d }
a = 32 – 28
700 = {2a + (7 − 1)(−20)} a=4
1400 = 7{2a -120} 𝑎n = a +(n-1)d
2a – 120 = 𝑎9 = 4 +(9-1)d
28 = 4 +8d
2a – 120 = 200 8d = 28 – 4
2a = 200 + 120 d=
a = 160
d=3
So, the values of prizes Rs 160, 140, 120, 100, 80, 60 and 40.
11) Find the arithmetic progression whose third term is 16 and its 7th term
exceeds the 5th term by 12. JUNE∶ 2022
9) Find the sum of all the multiples of 4 between 10 and 250. PREP-QP-2022
Ans. : 𝑎n = a +(n-1)d
Ans : Let the given Arithmetic progression is 12, 16, 20, ................., 248.
Here a = 12, d = 16-12= 4, an = 248, n = ? and Sn = ? According to given, a3 = 16
an = a + (n-1)d a + 2d = 16 -------------- (1)
Sri.Veerendra.H.M. Asst.Teacher. G.P.U.C. Megaravalli. Thirthahalli Tq Ph: 8762624683 Page-15 Sri.Veerendra.H.M. Asst.Teacher. G.P.U.C. Megaravalli. Thirthahalli Tq Ph: 8762624683 Page-16
According to given, a7 = a5 +12 2) Sum of the first three terms of an A.P. is 33. Product of first and 3rd term
a + 6d = a + 4d +12 exceeds the 2nd term by 29. Find the A.P. Model-1 : 2020
6d - 4d =12 Ans : Let first term of an A.P. is = (a –d),
2d =12 Second term = a, and third term = (a+d)
d= Sum of the first three terms = 33
d=6 a –d+ a+ a+d = 33
Substituting d = 6 in equation (1) a + 2d = 16 3a = 33
a + 2 (6) = 16 a = 11
a + 12 = 16 From given, Product of first and 3rd term exceeds the 2nd term by 29,
a = 16 – 12 (a-d)(a+d) = a + 29
a=4 a2 – d2 = a + 29
∴ Arithmetic progression is a, a + d, a + 2d, .............. = 4, 10, 16, ..................... 112 - d2 = 11 + 29
2
121 - d = 40
FOUR MARKS QUESTIONS: d2 = 121-40
1) The sum of the fourth and eighth terms of an arithmetic progression is 24 d2 = 81
and the sum of the sixth and tenth terms is 44. Find the first three terms of d=9
the Arithmetic progression. June : 2019 General form of A.P.is : a - d, a , a + d,.....
Ans : 𝑎n = a +(n-1)d ∴ Required A.P.is : 11-9, 11, 11+9, .......... = 2, 11, 20,.........
From given, a4 + 𝑎8 = 24 3) Sum of first 8 terms of an Arithmetic Progression is 136 and Sum of first
a +3d + a + 7d = 24 15 terms is 465. Find the sum of first 25 terms. Model-2 : 2020
2a + 10d = 24 -----------(1) Ans : From given, S8 = 136
From given, 𝑎6 + 𝑎10 = 24
Sn = {2a + (n − 1)d }
a +5d + a+9d = 44
2a + 14d = 44 ---------(2) S8 = {2a + (8 − 1)d}
Eqn(2) - Eqn(1) 136 = 4{2a + 7d}
2a + 7d = 34 --------- (1)
2a + 14d = 44
S15 = 465
(-)2a +(-)10d =(-)24
465 = {2a + (15 − 1)d}
4d = 20
465 = {2a + 14d}
d= d=5
2a + 14d = 62 --------- (2)
Substitute d = 5 in Eqn(1) 2a + 10d = 24, Eqn(2) - Eqn(1)
2a + 10x5 = 24 2a + 14d = 62
(-)2a +(-)7d =(-)34
2a + 50 = 24 7d = 28
2a = 24 – 50 a = -13 d=4
General form of A.P.is a , a+d, a+2d,..... Substitute d = 4 in Eqn(1),
∴ Required A.P.is : -13, -8, -3, .......... 2a + 7(4) = 34 ⇒ 2a = 34 – 28 ⇒ a = 6
Sri.Veerendra.H.M. Asst.Teacher. G.P.U.C. Megaravalli. Thirthahalli Tq Ph: 8762624683 Page-17 Sri.Veerendra.H.M. Asst.Teacher. G.P.U.C. Megaravalli. Thirthahalli Tq Ph: 8762624683 Page-18
Sum of first twenty five terms : -a + 4d = 5
Sn = {2a + (n − 1)d } put a = 11, -11 + 4d = 5
4d = 5 + 11
S25 = {2x6 + (25 − 1)4}
d=
S25 = { 12 + 96 } d=4
S25 = x 108 ∴ The required A.P. is a-2d, a-d, a, a+d , a+2d = 3, 7, 11, 15, 19.
S25 = 25x54 6) In an Arithmetic Progression 6th term is one more than twice the 3rd
∴ S25 = 1350
4) The first term of two Arithmetic progressions are same and the ratio term. The sum of the 4th and 5th terms is five times the second term.
Find the tenth term of the Arithmetic Progression. June : 2020
between their common difference is 1 : 2. The 7th term of first progression Ans. : From given, a6 = 2a3 + 1
and 21st term of second progression are 23 and 125 respectively. Find the
a + 5d = 2( a +2d) +1
two Arithmetic progressions. PreP ∶ 2020
a + 5d = 2a +4d +1
Ans : Let ‘a’ and ‘d’ are first term and common difference of first A.P.
a - 2a + 5d -4d = 1
Let ‘A’ and ‘D’ are first term and common difference of second A.P. -a + d = 1 ------- (1)
From given, a = A; D = 2d; a7 = 23 ⇒ a + 6d = 23 ------- (1) From given, a4 + a5 = 5a2
A21 = 125 ⇒ A + 20D = 125 ⇒ a + 40d = 125 ------ (2)
Eqn(2) - Eqn(1) a + 3d + a +4d = 5(a + d)
a + 40d = 125 2a + 7d = 5a + 5d
2a – 5a + 7d – 5d = 0
(-)a +(-)6d =(-) 23
34d = 102 -3a + 2d = 0
3a – 2d = 0 ------- (2)
d= Multiply Eqn(1) by 3
d=3 (-a + d = 1) x 3
Substitute d = 3 in Eqn(1), -3a + 3d = 3 ------- (3)
a + 6(3) = 23 ⇒ a = 23 – 18 ⇒ a = 5 Add Eqn(2) and Eqn (3)
and D = 2d = 2x 3 = 6 3a -2d = 0
∴ The first A.P. is : a, a+d, a+2d = 5, 8, 11 . . . -3a +3d = 3
The second A.P. is : A, A+D, A+2D = 5, 11, 17. . . d=3
Substitute d = 3 in Eqn (1)
5) There are 5 terms in an Arithmetic Progression. The sum of these terms -a + d = 1
is 55, and the fourth term is five more than the sum of the first two terms. -a + 3 = 1
Find the terms of the Arithmetic progression. June : 2020 -a = 1-3
Ans. : Five terms of A.P. are : a-2d, a-d, a, a+d and a+2d. -a = -2
From given, sum of 5 terms = 55 a=2
th
a-2d +a-d+a+a+d+a+2d = 55 10 term of the Arithmatic Progression:
5a = 55 𝑎n = a +(n-1)d
a = 11 a10 = a + 9d
From given, a4 = a1 + a2 +5 a10 = 2 + 9x 3
a + d = a-2d + a - d + 5 a10 = 2 + 27
a + d – a +2d –a + d = 5 ∴ a10 = 29
Sri.Veerendra.H.M. Asst.Teacher. G.P.U.C. Megaravalli. Thirthahalli Tq Ph: 8762624683 Page-19 Sri.Veerendra.H.M. Asst.Teacher. G.P.U.C. Megaravalli. Thirthahalli Tq Ph: 8762624683 Page-20
7) The sum of the first 7 terms of an arithmetic progression is 140 and the
=
sum of the next 7 terms of the same progression is 385 then find the
15a - 135d2 = 7a2 -7d2
2
arithmetic progression. Model-1 ∶ 2021
15a2-7a2 - 135d2 + 7d2 = 0
Ans : The Arithmetic progression is : a , a+d, a+2d, .........
8a2 - 128d2 = 0 8x82 -128d2 = 0 128d2 = 512 d2 = 4 d=2
Sum of first seven terms : Arithmetic progression : a-3d, a - d, a+d, a+3d
Sn = {2a + (n − 1)d } ∴ Arithmetic progression = 8-3x2, 8-2, 8+2, 8+3x2 = 2, 6, 10, 14
S7 = {2a + (7 − 1)d }
9) In an Arithmetic Progression of 50 terms, the sum of first 10 terms is 210.
140 = {2a + 6d }
And the sum of last 15 terms is 2565. Find the A.P. Model ∶ 2019
140 = 7a + 21d
Ans : Sn = {2a + (n − 1)d }
a + 3d = 20 ---------- (1)
From given, Sum of 14 terms = Sum of first 7 terms + Sum of last 7 terms. From given, Sum of first 10 terms = 210
Sum of 14 terms = 140+385 = 525 S10 = {2a + (10 − 1)d } = 210 {2a + 9d } = 42 ------------(1)
S14 = {2a + (14 − 1)d } = 525 Sum of last 15 terms = 2565 S50 – S35 = 2565
7{2a + 13d } = 525
2a + 13d = 75 ------- (2) {2a + 49d } - {2a + 34d } = 2565
Multiply Eqn(1) by 2 50(2a + 49d) – 35(2a+34d) = 2565x2
2a + 6d = 40 ---------- (3) 100a + 2450d – 70a - 1190d = 5130
Subtract Eqn(3) by Eqn(2) 30a +1260d = 5130 ( ÷ 15)
2a + 13d = 75 2a + 84d = 342 -------------- (2)
(-) 2a +(-) 6d =(-)40 Eqn(2) - Eqn(1),
7d = 35
2a + 84d = 342
d=5 2a +(-)9d = (-)42
(-)
Substitute d = 5 in Eqn (1), 75d = 300
a + 3x5 = 20 d=4
a = 20 - 15 Substitute d = 4 in Eqn (1),
a=5 2a + 9x4 = 42
∴ The required A.P. is : a , a+d, a+2d = 5, 10, 15, .......... 2a = 42-36 = 8
8) Sum of four consecutive terms of an Arithmetic progression is 32 and a=4
∴ Arithmetic progression : a , a+d, a+2d = 4, 8, 12, ..........
the ratio of product of first and last term to product of middle terms is
7 : 15. Find the four terms. Model ∶ 2019 10)In anArithmetic Progression pth, qth and rth terms are a,b and c respectively.
Ans: Let four terms of Arithmetic progression are a-3d, a - d, a+d, a+3d.
Prove that a(q-r) + b(r-p) + c(p-q) = 0 Model-1 : 2020
From given, Sum of four terms = 32 Ans: ap = a, aq = b, ar = c
a-3d+a-d+a+d+a+3d = 32 𝑎n = a +(n-1)d
4a = 32 a=8 th
P term = 𝑎p = a +(p-1)d = a
The ratio of product of first and last term to product of middle terms = 7 : 15
qth term = 𝑎q = a +(q-1)d = b
(a-3d) x (a+3d) : (a-d)(a+d) = 7 : 15
rth term = 𝑎r = a +(r-1)d = c
Sri.Veerendra.H.M. Asst.Teacher. G.P.U.C. Megaravalli. Thirthahalli Tq Ph: 8762624683 Page-21 Sri.Veerendra.H.M. Asst.Teacher. G.P.U.C. Megaravalli. Thirthahalli Tq Ph: 8762624683 Page-22
a(q-r) + b(r-p) + c(p-q) =
={ a +(p-1)d}(q-r) + { a +(q-1)d}(r-p) + { a +(r-1)d}(p-q)
= { a + pd – d}(q-r) + { a +qd -d}(r-p) + { a +rd-d}(p-q) =
= {aq+pqd-qd-ar-prd+dr+ar+qrd-dr-ap-pqd+pd+ap+prd-pd-aq-qrd+qd} =
( )
=0 ( )
( )
∴ a(q-r) + b(r-p) + c(p-q) = 0 =
( )
OR
∴ the ratio of mth and nth terms = (2m - 1) : (2n - 1)
ap = a, aq = b, ar = c,
EXERCISE :
d= = ------- (1) th th
1) The sum of 5 term and 9 term of an Arithmetic Progression is 40
d= = ------- (2) and sum of 8th term and 14th term is 64. Find the sum of first twenty terms.
Ans: d = 3, a = 2, ∴ S20 = 610 Model-2 : 2020
From Eqn (1) and (2)
2) An arithmetic progression consists of 37 terms. The sum of the first 3
= terms of it is 12 and the sum of its last 3 terms is 318, then find the first
(b-a)(r-q) = (c-b)(q-p) and last terms of the progression. Model-1 ∶ 2021
br – bq – ar + aq = cq – cp – bq + bp Ans. : d = 3 , a = 1, a37 = 109
aq – ar + br – bp + cp –cq = 0 3)The third term of an arithmetic progression is 8 and its ninth term exceeds
∴ a(q-r) + b(r-p) + c(p-q) = 0 three times the third term by 2 find the sum of the first 19 terms.
11) The ratio of sum of ‘m’ and ‘n’ terms of an Arithmetic Progression is Ans : d = 3, a = 2, ∴ S19 = 551 Prep ∶ 2019 & Model-2 : 2021
m2 : n2. Show that the ratio of mth and nth terms is (2m - 1) : (2n - 1). 4) The sum of first eight terms of an A.P. is 100 and sum of first 19 terms
Ans : From given, Sm : Sn = m2 : n2 Prep ∶ 2020
is 551. Find the Arithmetic progression. Board Prep ∶ 2019
Sn = {2a + (n − 1)d } Ans. : d = 3, a = 2 , Arithmetic progression is : 2, 5, 8, .......
Sm = {2a + (m − 1)d } 5) The 14th term of an Arithmetic Progression is twice the 8th term. The 6th term
=
{ ( ) } is -8. Find the sum of first 20 terms.
{ ( ) } Ans : d = -2 , 𝑎 = 2 , ∴ S20 = -340
{ ( ) }
=
{ ( ) }
= 6) In an arithmetic progressive the sum of the three terms is 24, and their
{ ( ) }
= product is 480, write three terms of the arithmetic progression?
{ ( ) } Ans : a = 8, d = 2, Arithmetic progression : 6, 8, 10, ......... Model-2 ∶ 2021
2an +mnd-nd = 2am +mnd-md
7) In an arithmetic progression first term is 5, last term is 45 and sum is 400. Then
2an -nd = 2am -md
find the number of terms and common difference.
2an - 2am = nd –md 𝟖
2a(n-m) = d(n-m) Ans : n = 16, d = 𝟑
d = 2a -------(1) 8) In an A.P. the 7 times of 7th term is equal to 11 times of 11 term. Then find
𝑎n = a +(n-1)d its 18th term.
Ans : a18 = 0
𝑎m = a +(m-1)d
( )
9) In an A.P. an = 10 -3n.Then find S40 .
= ( ) Ans : S40 = -2060
=
( )
[ ∵ d = 2a ] 10) For what value of ‘n’ the two A.P.s 63, 65, 67, .................. and 3, 10, 17, .....
( ) nth terms are equal ? Ans : n = 13
Sri.Veerendra.H.M. Asst.Teacher. G.P.U.C. Megaravalli. Thirthahalli Tq Ph: 8762624683 Page-23 Sri.Veerendra.H.M. Asst.Teacher. G.P.U.C. Megaravalli. Thirthahalli Tq Ph: 8762624683 Page-24
FIVE MARKS QUESTION : UNIT-2. : TRIANGLES:
Similar Figures : Two polygons of the same number of sides are similar, if
1) The common difference of two different arithmetic progressions are equal.
* their corresponding angles are equal and.
The first term of the first progression is 3 more than the first term of second
progression. If the 7th term of first progression is 28 and 8th term of second * their corresponding sides are in the same ratio (proportion).
progression is 29, then find the both different arithmetic progressions. Two triangles are similar, if
5−SEPT ∶ 2020 * their corresponding angles are equal and.
Ans. : First Arithmetic Progression : a, a+d, a+2d, ....... * their corresponding sides are in the same ratio.
Second Arithmetic Progression : b, b+d, b+2d, ....... Basic proportionality theorem : (Thales theorem)
From data, a = b + 3 --------- (1) “ If a line is drawn parallel to one side of a triangle to intersect other two
a7 = 28 sides in distinct points, the other two sides divided in the same ratio”
a + 6d = 28 ---------- (2) In ∆ABC, DE ║BC then, By Thales theorem, A
b8 = 29
b + 7d = 29 ---------------- (3) =
Substitute Eqn (1) in (2) By corollary of Thales theorem, D E
a + 6d = 28 = = B C
b + 3+ 6d = 28 Converse of Thales theorem :
b + 6d = 28 - 3 “If a line divides any two sides of a triangle in the same ratio, then the line is
b + 6d = 25 ---------------- (4) parallel to the third side”
Substract (4) from (3) Criteria for similarity of Triangles :
b + 7d = 29
(-)b +(-)6d =(-)25
Angle-Angle-Angle(A-A-A) Criterion of similarity of two triangles :
d=4 “If in two triangles, corresponding angles are equal, then their corresponding
Substitute d = 4 in (2) sides are in the same ratio(or proportion) and hence the two triangles are similar”
a + 6d = 28 Side-Side-Side(S-S-S) Criterion :
a + 6x4 = 28 “If in two triangles, sides of one triangle are proportional (i.e. in the same ratio)
a + 24 = 28
to the sides of the other triangle, then their corresponding angles equal and
a = 28 – 24
a =4 hence the two triangles are similar”
Substitute d = 4 in (3) Side-Angle-Side(S-A-S) Criterion :
b + 7d = 29 “If in two triangles, one angle of a triangle is equal to one angle of the other
b + 7x4 = 29 triangle and the sides including these angles are proportional, then the two
b + 28 = 29 triangles are similar”
b = 29 – 28
Areas of similar triangles :
b =1
∴ First Arithmetic Progression is, : a, a+d, a+2d, ....... “The ratio of the areas of two similar triangles is equal to the square of the
= 4, 4+4, 4+2(4) = 4, 8, 12, ....... ratio of their corresponding sides”
Second Arithmetic Progression: b, b+d, b+2d, ....... Pythagoras theorem :
= 1, 1+4, 1+2(4) = 1, 5, 9, ........ “In a right triangle, the square on the hypotenuse is equal to sum of the squares
of the other two sides”
Sri.Veerendra.H.M. Asst.Teacher. G.P.U.C. Megaravalli. Thirthahalli Tq Ph: 8762624683 Page-25 Sri.Veerendra.H.M. Asst.Teacher. G.P.U.C. Megaravalli. Thirthahalli Tq Ph: 8762624683 Page-26
A 7) In the ∆ABC, if DE ǁ AC, then the correct relation is Sept-2020
In ∆ABC, ∠B = 900 then A
A) = = B) = =
AC2 = AB2 + BC2
D
Converse of Pythagoras theorem : C B C) = = D) = =
“In a triangle, if square of one side is equal to the sum of the squares of the B E C
other two sides, then the angle opposite to the first side is a right angle” 8) In the given figure ∆ABC ∼ ∆DEF and ∠ABC=∠DEF = 600 then, the length of
AC : Model-1:2021
MULTIPLE CHOICE QUESTIONS : A. 2.4 cm
1) The true statement in the following Model-2:2021 B. 2.6 cm
A. two similar triangles are always congruent. C. 3.9 cm
B. a square and a rectangle are always similar. D. 3.2 cm
C. two equiangular triangles are always similar.
D. a square and a rhombus are always similar.
2) Which of the following pair of triangles are always similar Model QP : 2022 9) In the ∆ABC, if DE || BC then the relation which is true is Model-2:2021

A) Two isosceles triangles B) Two scalene triangles A. = B. = C. = D. =


C) Two equilateral triangles D) Two right angle triangles
𝐏𝐒
10) Sides of two similar triangles are in the ratio 1:4. Areas of these triangles
3) In the given figure ST ǁ QR then is equal to: P Model-2:2021
are in the ratio Model-1:2021
𝐒𝐐
A) B) C) D) A) 1 : 2 B) 1 : 16 C) 1 : 4 D) 16 : 1
11) Observe the given two triangles then identify the length of DF in the
S T
following : D
Q R
A
4) In ∆ABC, if XY ǁ BC then July ∶ 2021 A
7.6 cm ?
(A) = 3.8cm 3√2 cm
(B) = X Y 600 600
B 6cm C E 12cm F
(C) =
(D) = B C (A) 6√2 cm (B) 3√2 cm (C) 4.2 cm (D) 8.4 cm July ∶ 2021
5) In ∆𝐴𝐵𝐶, 𝐷𝐸 ∥ 𝐵𝐶, 𝐷E = 5cm, 𝐵C = 8cm, and A 12) ΔADE ~ ΔABC, if AD = 1 cm and AB = 2cm then A
𝐴D = 3.5cm, then 𝐴B = ---- MODEL-2019 the area of ΔABC : area of ΔADE is PREP QP : 2022
A ) 5.6cm B) 4.8cm c) 5.2cm D) 6.4cm D E A) 4 : 1 B) 1 : 4 C) 1 : 2 D) 2 : 1 D E

B C B C
6) In the figure, if Δ POQ ~ Δ SOR June -2020 R 13) In the ∆ABC, DE || BC. If AB : AD = 5 : 3 then area of ∆ABC : area of ∆ADE is
and PQ : RS = 1 : 2, then OP : OS is P A. 3 : 5 B. 6 : 10 C. 9 : 25 D. 25 : 9 Model-1:2021
A ) 1 : 2 B) 2 : 1 C) 3 : 1 D) 1 : 3 O 14) ∆ABC ∼ ∆PQR and their areas are in the ratio 25 : 9. If BC = 5cm,
Q S
the length of QR is Model-2:2021
A. 8 cm B. 3 cm C. 3.5 cm D. 9 cm
Sri.Veerendra.H.M. Asst.Teacher. G.P.U.C. Megaravalli. Thirthahalli Tq Ph: 8762624683 Page-27 Sri.Veerendra.H.M. Asst.Teacher. G.P.U.C. Megaravalli. Thirthahalli Tq Ph: 8762624683 Page-28
15) ΔABC ~ ΔPQR. Area of ΔABC = 64cm2 and the area of ΔPQR = 100 cm2 ONE MARK QUESTIONS :
If AB = 8 cm then the length of PQ is July ∶ 2021
1) State “Basic proportionality theorem”.
A) 12 cm B) 15 cm C) 10 cm D) 8 cm
June 2020 & 𝑲𝑺𝑬𝑬𝑩-1 & Model-1:2021. Prep-2022, March-2022 , JUNE-2022
16) In the given figure ∠ABC= 900 , BD ⊥ AC. If Thales theorem : “If a line is drawn parallel to one side of a triangle to intersect
AB = 6cm, BC=8cm, CA=10cm then the length of
the other two sides in distinct points , the other two sides
AD is :
A. 6.3 cm B. 3.6 cm are divided in the same ratio”
C. 3 cm D. 4 cm 2) State “Pythagoras’s” theorem Model QP : 2022
17) The sides of some triangles are given below. Ans : Pythagoras’s theorem: “In a right triangle, the square of the hypotenuse is
Identify which does not form a Right Triangle Model-2:2021 equal to the sum of the squares of the other two sides”
A. 5cm, 12cm, 13cm B. 8cm, 15cm, 17cm 3) In figure DE ∥ BC , 𝐴E = 3cm., 𝐶E = 6cm. A
C. 3cm, 8cm, 6cm D. 7cm, 24cm, 25cm
18) In the given figure ∠B = 900 , AB = BC = 2cm, and AD = 4cm. then find AB. Prep ∶ 2020

then the length of AC is Model-2:2021 Ans: = D E


A. 2√2cm B. 4√3 cm C. 2 cm D. 4 cm
= B C
19) The measure of three angles of a triangle are in the ratio 1:2:3, then the x
AB = =
triangle is Model-2:2021
A. equilateral B. right angled C. isosceles D. obtuse angled AB = 12cm
20) In the ΔABC, ∟B = 900 and BD ⊥ AC. If AB = 6 cm, BC = 8 cm then the length 4) In given fig. AB ∥ DE , 𝐴𝐷 = 6cm, 𝐶𝐷 = 4cm. and 𝐵𝐶 = 15cm then, find 𝐵𝐸.
of CD is A July ∶ 2021
Ans : Let 𝐵𝐸 = x
(A) 10 cm
= ( ∵ Thale’s corollary)
(B) 6.4 cm D
(C) 4.8 cm 6cm 6 cm 10 cm =
(D) 3.6 cm
15 - BE =
B 8 cm C
BE = 15-6
BE = 9 cm .
𝐀𝐁 𝐁𝐃
21) In the ∆ABC, it is given that 𝐀𝐂 = 𝐂𝐃.
If ∠B=700 , ∠C=500 then ∠BAD is : Model-1:2021 TWO MARKS QUESTIONS :
A. 300 B. 400 C. 450 D. 500 1) In the given figure if PQ || RS, prove that Δ POQ ~ Δ SOR. March-2019

Answers : 1.C, 2.C, 3.A, 4.B, 5.A, 6.A, 7.B, 8.B, 9.C, 10.B, 11.A, 12.A, 13.D, 14.B, 15.C, Ans : In Δ POQ and Δ SOR, P
16.B, 17.C, 18.A, 19.B, 20.B, 21.A
∟P = ∟S ( Alternate angles ) R
∟Q = ∟R ( Alternate angles )
∟POQ = ∟ROS ( V.O.A. ) O
Δ POQ ~ Δ SOR. ( A.A. criterion )
Q S

Sri.Veerendra.H.M. Asst.Teacher. G.P.U.C. Megaravalli. Thirthahalli Tq Ph: 8762624683 Page-29 Sri.Veerendra.H.M. Asst.Teacher. G.P.U.C. Megaravalli. Thirthahalli Tq Ph: 8762624683 Page-30
2) In Δ ABC, DE || BC. If AD = 5 cm, BD = 7 cm and AC = 18 cm, find the = ( ∵ QH+HP = QP = 9)
length of AE. March ∶ 2019
12x BH = 18x10
Ans : In ∆ ABC, DE || BC
BH =
=
∴ BH = 15 cm
= ∴ AB = AH + BH = 10+15
∴ AB = 25cm
AE =
5) In triangle 𝐴𝐵𝐶, PQ ∥ BC and BD = DC. Prove that PE = EQ. Prep ∶ 2020
AE = 7.5 cm Proof : In ∆ABD,
𝐀𝐘 𝟏
3) In figure XY ∥ BC , 𝐴X = P- 3., BX = 2P-2 and = , then find P. According to Thale’s corollary,
𝐂𝐘 𝟒
Solution : MODEL-2019 AP
= = -------- (1)
According to Thale’s theorem, BP
In ∆ADC, According to Thale’s corollary,
= AQ
= = -------- (2)
QC
=
From Eqn(1) and Eqn(2)
4(P-3) = 2P-2
=
4P – 12 = 2P - 2
PE x CD = EQ x BD [ ∵ BD = DC ]
4P – 2P = 12 - 2
∴ PE = EQ
2P = 10
6) 𝐸 and 𝐹 are points on the sides PQ and PR respectively of a ∆𝑃𝑄𝑅. 𝑃𝐸 = 1cm
∴P=5
4) In figure PC ∥ QK and BC ∥ HK. 𝐴Q = 6cm, QH = 4cm, HP = 5cm, and 𝑄𝐸 = 2cm., 𝑃𝐹 = 3cm., and 𝑅𝐹 = 6cm, then show that 𝐸𝐹 ∥ PQ.
Proof : In ∆𝑃𝑄𝑅, P
K𝐶 = 18cm, then find AK and AB. MODEL-2019
PE 1
= E F
Solution : In ∆𝐴P𝐶, PC ∥ QK A EQ 2
PF 3
Acording to Thale’s theorem, FR
= 6
=
Q R
= Q
= =
= ( ∵ QH+HP = QP = 9) H K ∴ 𝐸𝐹 ∥ PQ (∵ Converse of Thale’s theorem)
9x AK = 18x6 P 𝐁𝐅 𝐁𝐄
7) In the adjoining figure DEǁAC and DFǁAE, then prove that = .
𝐅𝐄 𝐄𝐂
AK =
PREP-2019 A
∴ AK = 12 cm B C Proof : In ∆ABC, DE ∥ AC ( ∵ Given )
In ∆𝐴B𝐶, BC ∥ HK
According to Thale’s theorem, D
According to Thale’s theorem,
= --------(1)
=
In ∆ABE, DF ǁ AE ( ∵ Given ) B F E C
Sri.Veerendra.H.M. Asst.Teacher. G.P.U.C. Megaravalli. Thirthahalli Tq Ph: 8762624683 Page-31 Sri.Veerendra.H.M. Asst.Teacher. G.P.U.C. Megaravalli. Thirthahalli Tq Ph: 8762624683 Page-32
According to Thale’s theorem, Construction : Draw AL ┴ BC and DM ┴ BC
= --------(2) Proof : Area of triangle = x base x height
From Eqn(1) and Eqn(2) ∆
𝐁𝐅 𝐁𝐄 =
= (∵ Axiom-1 ) ∆
𝐅𝐄 𝐄𝐂

= -------- (1)
8) In the figure 𝐿𝑀 ∥ 𝐴𝐵, AL = 𝑥−3, 𝐴𝐶 = 2𝑥, 𝐵𝑀 = 𝑥 −2 C ∆
In ∆ALO and ∆DMO
and 𝐵𝐶 = 2𝑥 +3, find ‘x’.
∠A𝑂L = ∠D𝑂M (∵ Vertically opposite angles)
Solution : According to Thale’s corollary,
∠ALO = ∠DMO = 900 (∵ Heights )
= L M ∴ ∆ALO ~ ∆DMO (∵ A-A similarity criteria )

= ∴ = ------- (2)

= A B From Eqn(1) and (2)


𝐀𝐫𝐞𝐚 𝐨𝐟 ∆𝐀𝐁𝐂 𝐀𝐎
=
= 𝐀𝐫𝐞𝐚 𝐨𝐟 ∆𝐃𝐁𝐂 𝐃𝐎
11) D is a point on the side BC of a triangle ABC such that ∠ADC = ∠BAC.
2x (x+5) = (x+3)(2x+3)
2x2 +10x = 2x2 +3x + 6x+ 9 Show that CA2 = CB.CD Prep ∶2020

10x = 9x+ 9 Solution : In ∆ABC and ∆ADC,


10x - 9x =9 ∠BAC = ∠ADC (∵ Given )
x=9 ∠C = ∠C (∵ common angle)

∴ ∆ABC ≈ ∆ADC (∵ A-A similarity criteria)


9) The diagonal BD of parallelogram ABCD intersects AE at F as shown in
CA
the figure. If E is any point on BC, then prove that 𝐷𝐹 ×𝐸𝐹 = 𝐹𝐵 ×𝐹𝐴. ∴ =
CD
Ans : In ∆𝐴DF and ∆𝐸BF June : 2019 & Prep ∶ 2020 ⇒ CA2 = CB. CD
∠ADF = ∠BEF ( ∵ AD║BE, alternate angles )
12) Diagonals AC and BD of a trapezium 𝐴𝐵𝐶D with 𝐴𝐵 ∥ 𝐶D intersect each

∠AFD = ∠BFE ( ∵ vertically opposite angles) other at the point O. Using a similarity criterion for two triangles,
∆𝐴DF ~ ∆𝐸BF ( ∵ A-A similarity criteria) show that AO.OD = BO.OC Prep ∶ 2020

DF
Solution : In ∆AOB and ∆COD,
=
FB ∠AOB = ∠COD (∵ Vertically opposite angles)
𝐷𝐹 ×𝐸𝐹 = 𝐹𝐵 ×𝐹𝐴
∠ABO = ∠CDO (∵Alternate angles)
10) In the given figure ∆𝐴𝐵𝐶 and ∆𝐷BC are two triangles on the same base ∴ AOB ≈∆COD (∵A-A similarity criteria)
𝐀𝐫𝐞𝐚 𝐨𝐟 ∆𝐀𝐁𝐂 𝐀𝐎 AO
BC. If AD intersect BC at ‘O’, show that = MODEL-2020 ∴ =
𝐀𝐫𝐞𝐚 𝐨𝐟 ∆𝐃𝐁𝐂 𝐃𝐎 OC
⇒ AO.OD = BO.OC
Sri.Veerendra.H.M. Asst.Teacher. G.P.U.C. Megaravalli. Thirthahalli Tq Ph: 8762624683 Page-33 Sri.Veerendra.H.M. Asst.Teacher. G.P.U.C. Megaravalli. Thirthahalli Tq Ph: 8762624683 Page-34
13) In the given triangle, ∠𝐵𝐴𝐶 = 90° and 𝐴𝐷 ⊥ 𝐵𝐶. Prove that 𝐴𝐶2 = 𝐵𝐶.𝐷𝐶. Solution :

=

Solution : In ∆BAC and ∆ADC,
0 =
∠BAC = ∠ADC = 90 (∵ Given )
( . )
∠C = ∠C (∵ common angle) 𝐵𝐶2 =

∴ ∆BAC ≈ ∆ADC (∵ A-A similarity criteria) ( . ) . .


𝐵𝐶 = = =
∴ = 𝐵𝐶 = 11.2 cm
2
⇒ 𝐴𝐶 = 𝐵𝐶.𝐷𝐶
17) In ΔABC, AD ⊥ BC and AD2 = BD × CD. Prove that AB2 + AC2 = (BD +CD )2
14) In the given figure ∆ABC and ∆AMP are two right angled triangles with Ans. : In ∆𝐴𝐵D, By Pythagoras thereom March-2019
right angles B and M. Then prove that i) ∆ABC ≈ ∆AMP and 𝐴𝐵2 = BD2 +AD2 --------- (1)
𝐂𝐀 𝐁𝐂
= . PREP-2019 In ∆𝐴DC, By Pythagoras thereom
𝐏𝐀 𝐌𝐏
𝐴C2 = DC2 +AD2 --------- (2)
Solution : (1) In ∆ABC and ∆AMP, Eqn (1) + (2)
∠A = ∠A (∵ common angles) AB2 + AC2 = BD2 +AD2 + DC2 +AD2
∠ABC = ∠AMP = 90
0 AB2 + AC2 = BD2 +2AD2 + DC2

∴ ∆ABC ≈∆AMP (∵ A-A similarity criteria) AB2 + AC2 = BD2 +2. 𝐵𝐷.𝐶𝐷 + DC2 ( ∵ 𝐴𝐷2 = 𝐵𝐷 ×CD )
𝐂𝐀 𝐁𝐂 AB2 + AC2 = ( BD + DC )2
∴ = (∵ A-A similarity criteria)
𝐏𝐀 𝐌𝐏
18) A vertical pole of height 6m casts a shadow 4m long on the ground, and at
15) ∆𝐴𝐵𝐶~∆𝐷𝐸𝐹 and their areas are 64 cm2 and 100 cm2 respectively. If
the same time a tower on the same ground casts a shadow 28 m long. Find
𝐸𝐹 = 12cm then find the measure of BC. June : 2019
the height of the tower. June : 2019
Ans : Area of (∆𝐴𝐵𝐶) = 𝐵𝐶2 D
Ans : Height of the vertical pole = AB = 6m
Area of (∆𝐷𝐸𝐹) 𝐸𝐹2 A
Length of its shadow = BC = 4m D
=
Length of tower’s shadow = EF = 28m A
B C E F
Height of the tower = DE = ?
𝐵𝐶2 =
∆𝐴𝐵𝐶≈∆𝐷𝐸𝐹 B C E F

𝐵𝐶 = = = ∴ = ( ∵ A-A criteria)
𝐵𝐶 = 9.6 cm = DE = DE = 42m
∴ Height of the tower = DE = 42 m
16) Let ∆𝐴𝐵𝐶~∆𝐷𝐸𝐹 and their areas be, 64 𝑐𝑚2 and 121 𝑐𝑚2 respectively. If
𝐸𝐹 = 15.4𝑐𝑚 find 𝐵𝐶.
Sri.Veerendra.H.M. Asst.Teacher. G.P.U.C. Megaravalli. Thirthahalli Tq Ph: 8762624683 Page-35 Sri.Veerendra.H.M. Asst.Teacher. G.P.U.C. Megaravalli. Thirthahalli Tq Ph: 8762624683 Page-36
THREE MARKS QUESTION : FOUR MARKS QUESTIONS :
1) In the ∆ABD, C is a point on BD such that BC : CD = 1 : 2, and ∆ABC is an 1) Prove that “The ratio of the areas of two similar triangles is equal to the ratio
equilateral triangle. Then prove that AD2 = 7 AC2. SEPT ∶ 2020 of their corresponding sides” April-2019 &Macrh-2022
Ans : 2) Prove that “In a right triangle, the square of the hypotenuse is equal to the sum
Data : In ∆ ABD, BC : CD = 1 : 2 of the squares of the other two sides” ( Pythagoras Theorem )
In ∆ ABC, AB = BC = AC June-2019 & Prep-2019 & PREP-2020
To prove : AD = 7AC2
2
3) Prove that “If a line is drawn parallel to one side of a triangle to intersect the
Construction : Draw AE ┴ BC. other two sides in distinct points , the other two sides are divided
Proof : In ∆ ABC, in the same ratio” ( Thales theorem) Model-2019


4) State and prove converse of the Pythagoras theorem. Model-2020
BE = EC = and AE =
5) Prove that “ If in two triangles, corresponding angles are equal, then their
In ∆ ADE ∟AED = 900 , By Pythagoras theorem
corresponding sides are in the same ratio (or proportion) and hence the two
AD2 = AE2 + ED2
√ triangles are similar Sept∶ 2020
AD2 = ( ) + ( 2a + )2
FIVE MARKS QUESTIONS :
2 2
AD = +( ) 1) State and prove Pythagoras theorem. June : 2020 & PREP-2022

AD =2
+ 2) State and prove Thales theorem. Model-2 ∶2021 & Model- QP ∶ 2022

AD2 = 3) Prove that “The ratio of the areas of two similar triangles is equal to the ratio

AD2 = 7a
2 of their corresponding sides” Model-1 ∶2021, March-2022
4) Prove that “If in two triangles, corresponding angles are equal, then their
AD2 = 7 AC2 [ ∵ AC = a ]
corresponding sides are in the same ratio ( or proportion ) and hence the two
triangles are similar”. 5-June-2022

Sri.Veerendra.H.M. Asst.Teacher. G.P.U.C. Megaravalli. Thirthahalli Tq Ph: 8762624683 Page-37 Sri.Veerendra.H.M. Asst.Teacher. G.P.U.C. Megaravalli. Thirthahalli Tq Ph: 8762624683 Page-38
U-3. : PAIR OF LINEAR EQUATIONS WITH TWO VARIABLES: MULTIPLE CHOICE QUESTIONS :
1) If the lines drawn to the linear equations of the type 𝑎1𝑥 +𝑏1𝑦 +𝑐1 = 0 and
Linear equation with two variables :
𝑎2𝑥 +𝑏2𝑦 +𝑐2 = 0 are coincident on each other, then the correct
Equation of the form ax + by + c = 0, where a, b and c are real numbers, relation among the following is June-2019
(a ≠ 0, b ≠ 0 ) is called a linear equation in two variables x and y. 𝐴) = = 𝐵) ≠ ≠
Pair of linear equations with two variables:
Equations of the form a1x + b1y + c1 = 0 and a2x + b2y + c2 = 0, where 𝐶) = ≠ 𝐷) ≠ =
a1 , b1 , c1 , a2 , b2 and c2 are real numbers, (a1 ≠ 0, b1 ≠ 0 , a2 ≠ 0, b2 ≠ 0 ) 2) The pair of lines a1 x+b1 y+c1 = 0 and a2 x+b2 y+c2 = 0 are intersecting
are called a pair of linear equations with two variables x and y. lines then the ratio of their coefficients is : Model-2 ∶ 2021
a1x + b1y + c1 = 0 and a2x + b2y + c2 = 0 are a pair of linear equations, then 𝐴) ≠ 𝐵) = ≠
𝐶) = = 𝐷) =
Type of Number of Type of 3) If the pair of linear equations in two variables a1x + b1y + c1 = 0 and
Comparision Figure
line solutions equation
a2x + b2y + c2 = 0 are parallel lines then the correct relation of their
Intersecting Unique or
≠ Consistent coefficients is : July ∶ 2021
lines one solution
Infinitely 𝐴) = ≠ 𝐵) = =
= = Coincident Consistent &
many ≠ =
lines dependent 𝐶) 𝐷)
solutions
4) In the pair of linear equations 𝑎1𝑥 +𝑏1𝑦 +𝑐1 = 0 and 𝑎2𝑥 +𝑏2𝑦 +𝑐2 = 0
= ≠ Parallel
No solution Inconsistent 𝐢𝐟
𝐚𝟏

𝐛𝟏
then the June-2020
lines 𝐚𝟐 𝐛𝟐
A) equations have no solution (B) equations have unique solution
C) equations have three solutions (D) equations have infinitely many solutions.
Methods of solving a pair of linear equations :
5) A pair of linear equations a1x+b1y+c1 = 0 a2x+b2y+c2 = 0 have unique
1) Substitution method : Solution. Then correct relation among the following is : Model & PREP-QP-2022
2) Elimination method : 𝐴) = 𝐵) = = 𝐶) ≠ 𝐷) = ≠
3) Cross multiplication method : 6) The lines represented by 2x + 3y – 9 = 0 and 4x + 6y – 18 = 0 are April-2019
4) Graphical method: (A) Intersecting lines (B) Perpendicular lines to each other
(C) Parallel lines (D) Co-incident lines
Cross- multiplication method :
7) The lines represented by 𝑥 +2𝑦 – 4 = 0 and 2𝑥 +4𝑦 − 12 = 0 are
x y 1 𝐴) Intersecting lines 𝐵) Perpendicular lines to each other Sept ∶ 𝟐𝟎20
b1 c1 a1 b1 C) Parallel lines 𝐷) Coincident lines
8) The pair of equations 2x-5y+4=0 and 2x+y-8=0 has Model-1 ∶ 2021
b2 c2 a2 b2
A. Exactly two solutions B. Infinity many solutions
C. A unique solution D. No solution
x= , y= 9) x – 2y = 0 and 3x + 4y – 20 = 0 are July ∶ 2021
A)Intersecting lines B)Coincident lines C)Parallel lines D)Perpendicular lines

Sri.Veerendra.H.M. Asst.Teacher. G.P.U.C. Megaravalli. Thirthahalli Tq Ph: 8762624683 Page-39 Sri.Veerendra.H.M. Asst.Teacher. G.P.U.C. Megaravalli. Thirthahalli Tq Ph: 8762624683 Page-40
10) The graphical representation of the pair of lines x + 2y – 4 = 0 and A. x = 3, y = 5 B. x = 4, y = 4 C. x = 5, y = 3 D. x = −5, y = -3
2x + 4y – 12 = 0 March-2022 21) The pair of equations of lines as shown in the graph are July ∶ 2021
A) Intersecting lines B) Parallel lines (A) x + y = 1 and 2x – y = 1
C) Co-incident lines D) Perpendicular lines (B) 2x + y = 2 and x+y=2
11) Lines represented by the pair of linear equations x-y = 8 and 3x–3y = 16 are (C) 2x - y = 2 and 4x – y = 4
A) intersecting lines B) parallel lines June-2022 (D) y - x = 0 and x–y=1
C) perpendicular lines D) coincident lines.
12) If the pair of Linear equations x + 2y = 3 and 2x + 4y = k are coincide
Answers : 1.A, 2.A, 3.A, 4.B
4.B, 5.C, 6.D, 7.C, 8.C, 9.A, 10.B, 11.B,, 12.
12.B, 13.C, 14.B, 15.C,
then the value of ‘k’ is : Model-1 ∶ 2021
16.A, 17.C, 18.B, 19.A, 20.C,
20.C 21.C.
𝐴) 3 𝐵) 6 𝐶) -3 𝐷) -6
13) If the pair of Linear equations 3𝑥 +2𝑘𝑦 = 2 and 2𝑥 +5𝑦 +1 = 0 are ONE MARK QUESTIONS :
parallel then the value of ′𝑘′ is Model-1 ∶ 2021 1) Write the general form for a pair of linear equations in
in two variables
𝐴) 𝐵) 𝐶) 𝐷) ‘x’ and ‘y’ ? Model ∶ 2019
Ans : 𝑎1𝑥 +𝑏1𝑦 +𝑐1 = 0 and 𝑎2𝑥 +𝑏2𝑦 +𝑐2 = 0
14) The pair of coincident lines in the following are Model-2 ∶ 2021
2) If a pair of linear equations in two variables are inconsistent then write how
A. x − 2y = 0 B. 2x + 3y = 9
many solutions do they have. Model ∶ 2022 & April : 2022, June ∶ 2022
3x + 4y = 20 4x + 6y = 18
Ans : No solution
C. x + 2y = 4 D. x + y = 8
3) Write number of solutions that the pair of linear equations a1x+b1y+c1 = 0
2x + 4y = 12 x−y=4
and a2x+b2y+c2 = 0 have (c1 ≠ c2 ) PREP QP-2022
15) If the pair of lines 2x + 3y + 7 = 0 and ax + by +14 = 0 are coincident
Ans : If c1 ≠ c2 then the given pair of equations are parallel.
lines then the values of ‘a’ and ‘b’ are respectively equal to July ∶ 2021
Hence they have no solution.
(A) 2 and 3 (B) 3 and 2 (C) 4 and 6 (D) 1 and 2
16) Solutions for the equations 𝑥 +𝑦 = 9 and 𝑥 −𝑦 = 1 are MODEL-2019
4) Inn the pair of linear equations 𝑎1𝑥 +𝑏1𝑦 +𝑐1 = 0 and 𝑎2𝑥 +𝑏2𝑦 +𝑐2 = 0
𝐚𝟏 𝐛𝟏
𝐴) 𝑥 = 5, 𝑦 = 4 𝐵) 𝑥 = 4, 𝑦 = 5 𝐶) 𝑥 = 6, 𝑦 = 3 𝐷) 𝑥 = 3, 𝑦 = 6 𝐢𝐟 𝐚𝟐
≠ 𝐛𝟐
then how many solutions they have ? Sept : 2020
17) The values of ‘x’ and ‘y’ when a point lies on the linear equation Ans : Equations have unique solution (one)
2x - 3y = 12 Model-1 ∶ 2021 5) The given graph represents a pair of linear equations
A. x = 0, y = -3 B. x = 2, y = 3 C. x = 3, y = -2 D. x = -2, y = +3 in two variables. Write how many solutions these pair of
18) Identify the wrong statement with respect to a pair of linear equations equations have. April : 2019
Ans. : one or unique solutions.
A. If lines are parallel there is no solution Model-1 ∶ 2021
B. If the lines are perpendular to each other, there is no solution
6) If a pair of linear equations represented by lines has no solutions
C. Many solutions if the lines coincide each other
D. A unique solution if they intersect ( inconsistent ) then write what kinds of lines are these. June-2019

19)The values of ‘x’and ‘y’which satisfy the linear equation 2x + 3y = 16 are Ans : Parallel lines.
7) If the pair of Linear equations 4x + py + 8 = 0 & 4x + 4y + 2 = 0
A. x = 5, y = 2 B. x = 2, y = 5 C. x = −5, y = −2 D. x = −5, y = 2
20) By solving a pair of linear equations x + y = 8 and 2y − x = 1, the values represent parallel lines then find the value of ‘p’. PREP ∶ 2020
Ans : If two lines are parallel then,
of ‘x’ and ‘y’ are Model-2 ∶ 2021
Sri.Veerendra.H.M. Asst.Teacher. G.P.U.C. Megaravalli. Thirthahalli Tq Ph: 8762624683 Page-41 Sri.Veerendra.H.M. Asst.Teacher. G.P.U.C. Megaravalli. Thirthahalli Tq Ph: 8762624683 Page-42
= ≠ 2) 2𝑥 + 𝑦 = 6 and 2𝑥 − 𝑦 = 2 then find 𝑥 and 𝑦. Model ∶ 2020

⇒ = ⇒ 4p = 16 Ans : 2𝑥 + 𝑦 = 6 ------(1)
P=4 2𝑥 − 𝑦 = 2 ------(2)
8) How many solutions have the pair of linear equations 2x+3y-9 = 0 and Add Eqn(1) and (2) we get,
2𝑥 + 𝑦 = 6
4x + 6y - 18 = 0? Model-1 ∶ 2021
2𝑥 −𝑦 = 2
Ans : 𝑎1 = 2 , 𝑏1 = 3, 𝑐1 = -9 and 𝑎2 = 4, 𝑏2 = 6, 𝑐2 = -18
4𝑥 = 8
𝐚𝟏
= = , = = , = = 𝑥= 2
𝐚𝟐
𝐚𝟏
= = Substitute x = 2 in Eqn(1), 2𝑥 + 𝑦 = 6
𝐚𝟐
2x2 + y = 6
⇒ They are co-incident lines.
y=6–4
So they have infinitely many solutions. y=2
9) In equation x + y = 7, if x = 3, then find the value of y ? Model-2 ∶ 2021 3) Find the solution for the pair of linear equations :
Ans : 𝑥 + 𝑦 = 7 ⇒ 3 + 𝑦 = 7 3x + 2y = 11 & 5x – 2y = 13 PREP ∶ 2020
⇒ 𝑦 = 7-3 Ans : 3x + 2y = 11 ----- (1)
⇒𝑦=4 5x – 2y = 13 ----- (2)
10) 5 pencils and 7 pens together cost Rs. 50, whereas 7 pencils and 5 pens Add Eqn(1) and (2), then we get,
together cost Rs. 46. Form a pair of linear equations. 3𝑥 + 2𝑦 = 11
Ans : 5𝑥 + 7𝑦 = 50 & 7𝑥 + 5𝑦 = 46 5𝑥 − 2𝑦 = 13
8x = 24
TWO MARKS QUESTIONS :
1) Find the solution for the pair of linear equations : June-2019
x=
𝑥 + 𝑦 = 14 & 𝑥 − 𝑦 = 4 x=3
Ans : 𝑥 + 𝑦 = 14 ------(1) Substitute x = 3 in Eqn(1)
3 x 3 + 2y = 11
𝑥 −𝑦 = 4 ------(2)
2y = 11 - 9
Add Eqn(1) and (2), we get 2y = 2
𝑥 + 𝑦 = 14
y=1
𝑥 −𝑦 = 4 4) Solve : 2𝑥 + 𝑦 = 11 and 𝑥 + 𝑦 = 8 June : 2020
2𝑥 = 18 Ans : 2𝑥 + 𝑦 = 11 -----(1)
𝑥 = 9 𝑥 + 𝑦 = 8 ------(2)
Substitute x = 9 in Eqn(1), 𝑥 + 𝑦 = 14 Eqn(1) - Eqn (2) then,
9 + y = 14 2𝑥 + 𝑦 = 11
y = 14–9 (-)𝑥(-)+𝑦 =(-)8

y=5 𝑥 = 3
Sri.Veerendra.H.M. Asst.Teacher. G.P.U.C. Megaravalli. Thirthahalli Tq Ph: 8762624683 Page-43 Sri.Veerendra.H.M. Asst.Teacher. G.P.U.C. Megaravalli. Thirthahalli Tq Ph: 8762624683 Page-44
Substitute x = 3 in Eqn(2) , 𝑥 + 𝑦 = 8 = , = =
3+𝑦=8
𝑦=8-3 Here ≠ ⇒ They are intersecting lines.
y=5 8) The difference of two numbers is 26 and one number is thrice of other.
5) Solve the following pair of linear equations by any suitable method : Find the two numbers. PREP-2019
x + y = 5 ------(1) March-2019 Ans:
2x – 3y = 5 ------(2) Let one number be ‘x’ and the other is ‘y’.
Ans. : Multiply Eqn(1) by 3, we get 3𝑥 + 3𝑦 = 15 ------(3)
From given, x - y = 26 ---------- (1)
Add Eqn(2) and Eqn(3)
From given, x = 3y
2x - 3y = 5
x – 3y = 0 ------------ (2)
3𝑥 + 3𝑦 = 15
Subtract Eqn(2) from (1),
5x = 20
x=4
x - y = 26
Substitute the value of x = 4 in equation (1), x + y = 5 (-) x–(+)3y
= 0
2y = 26
4+y=5
y = 13
y=1 Substitute y = 13 in Eqn(1),
6) Solve : 10𝑥 + 3𝑦 = 75, 6𝑥 - 5𝑦 = 11 Model ∶ 2020 x - 13 = 26
Ans : (10𝑥 + 3𝑦 = 75) ------ (1) x 5 x = 26 + 13
x = 39
(6𝑥 − 5𝑦 = 11) ------ (2) x 3
∴ One number x = 39
50x + 15y = 375 ------ (3) ∴ Other number y = 13
18𝑥 – 15𝑦 = 33 ------ (4)
Add Eqn(3) and (4) we get 9) Five years ago, age of Gouri thrice the age of Ganesh. After ten years,
50x + 15y = 375 age of Gouri twice the age of Ganesh. Find the age of Gouri and Ganesh.
18𝑥 – 15𝑦 = 33 Ans: MODEL ∶2019
68x = 408 Let the present age of Gouri is ‘x’and
x =6
Let the present age of Ganesh is ‘y’
Substitute x = 6 in Eqn(1),
10𝑥 + 3𝑦 = 75 From given, 5 years ago, x-5 = 3(y-5)
10x6 + 3y = 75 x - 5 = 3y - 15 x - 3y = -10 --------- (1)
60 + 3y = 75 After 10 years, x + 10 = 2(y+10)
3y = 75 - 60 x+ 10 = 2y + 20 x - 2y = 10 ------------- (2)
y=5 Subtract Eqn(2) from (1),
7) Show that the lines represented by linear pair of equations 2x + 3y = 1 and x - 3y = -10
5x + 6y = 2 are intersecting lines by comparing their co-efficients.
(-)x (+)- 2y =(-)10
Ans : 𝑎1 = 2 , 𝑏1 = 3, 𝑐1 = -1 and 𝑎2 = 5, 𝑏2 = 6, 𝑐2 = -2
- y = -20
Sri.Veerendra.H.M. Asst.Teacher. G.P.U.C. Megaravalli. Thirthahalli Tq Ph: 8762624683 Page-45 Sri.Veerendra.H.M. Asst.Teacher. G.P.U.C. Megaravalli. Thirthahalli Tq Ph: 8762624683 Page-46
y = 20 6) Solve the pair of linear equations by elimination method. PREP-2022

Substitute y = 20 in Eqn (1), 2x + y = 14 & x – y = 4 Ans : x = 6, y = 2


x – 3x20 = -10 7) Solve: 3𝑥 + 2𝑦 – 11 = 0, 5𝑥 - 2𝑦 -13 = 0 PREP ∶ 2020
x – 60 = -10 Ans. : 𝑥 = 3, y = 1
x = -10+ 60 8) Solve the pair of linear equations by elimination method.
x = 50 2x + y = 8 and x – y = 1 Ans : 𝑥 = 3 , y = 2 March-2022
∴ Gouri’s age = x = 50 , Ganesh’s age = y = 20 9) “The difference between two positive numbers is 26 and one number is 3
times the other.” Represent this statement in the form of the pair of linear
10) Find the value of k, if the pair of linear equations 2x – 3y = 8 and equations. PREP-2022

2(k – 4 )x – ky = k + 3 are inconsistent. June : 2019 10) Solve the given equations by elimination method : 2x + 3y = 7 2x + y = 5
2x + 3y = 7 & 2x + y = 5 Ans. : x = 2, y = 1
Ans : 2𝑥 - 3𝑦 = 8 and 2(k – 4) x - ky = k + 3
THREE MARKS QUESTIONS :
a1 = 2, b1 = -3, c1 = -8 , a2 = 2(k – 4) , b2 = - k , c2 = -(k + 3)
1) After 10 years, X will be twice as old as Y. Ten years ago, the age of X was
If the pair of linear equations inconsistent then, =
six times that of Y. Then find their present age? MODEL ∶ 2020
= Ans : From given,
𝟐(𝐤 – 𝟒)
-2k = -3 x 2(k-4) x+10 = 2(y+10)
-2k = -3( 2k – 8) x+10 = 2y+20
-2k = -6k +24 x - 2y = 10 ---------- (1)
6k -2k = 24 From given,
4k = 24 x - 10 = 6(y-10)
k=6 x-10 = 6y - 60
EXERCISE : x - 6y = -50 ---------- (2)
Subtract Eqn(2) from Eqn(1),
1) Solve : 3x + y = 15 , 2x ˗ y = 5 Ans : x = 4, y = 3 Model-1 ∶ 2021 x - 2y = 10
2) Solve by elimination method Model-2 ∶ 2021 (-) x–(+)6y = -(+)50
x + y = 8 & 2x ˗ y = 7 Ans: x = 5 , y = 3 4y = 60
3) Solve the pair of linear equations by elimination method. Model QP ∶ 2022 y = 15
Substitute y = 15 in Eqn(1),
2x + y = 3 & 4x – y = 9 Ans : 𝑥 = 2 , y = (-1)
x – 2x15 = 10
4) From suitable method find ‘x’ and ‘y’: Model ∶ 2019 x = 10 + 30
2𝑥 +3𝑦 +5 = 0 , 3𝑥 −2𝑦 −12 = 0 Ans. : x=2 , y = -3 x = 40
∴ Present age of x = 40 and present age of y = 15
5) Using suitable method, find the solution for the pair of linear
𝟖
equations : 2𝑥 +3𝑦 = 11, 2𝑥 −4𝑦 = -24 Sept : 2020 2) A fraction becomes if 2 is added to both the numerator and
𝟏𝟏
Ans : 𝑥 = -2, y = 5 denominator. If 3 is subtracted from both numerator and denominator
𝟐
it becomes . Find the fraction. Model ∶ 2020
𝟓
Sri.Veerendra.H.M. Asst.Teacher. G.P.U.C. Megaravalli. Thirthahalli Tq Ph: 8762624683 Page-47 Sri.Veerendra.H.M. Asst.Teacher. G.P.U.C. Megaravalli. Thirthahalli Tq Ph: 8762624683 Page-48
Ans : Let the fraction be . FOUR MARKS QUESTIONS :
From given, = 1) Find the solution of the following pairs of linear equation by the
graphical method : April : 2019
11(x+3) = 8(y+3) 2x + y = 6
11x + 33 = 8y + 24 2x – y = 2
11x - 8y = -9 --------- (1) Ans : 2x + y = 6 ⇒ y = 6 – 2x
x 0 1 2 3
From given, = y 6 4 2 0
5(x-3) = 2(y - 3) (x,y) (0,6) (1,4) (2,2) (3,0)

5x - 15 = 2y - 6 2x – y = 2 ⇒ -y = 2 – 2x ⇒ y = 2x - 2
5x - 2y = 9 ---------- (2) x 0 1 2 3
Multiply Eqn(2) by 4 y -2 0 2 4
20x - 8y = 36 -------- (3) (x,y) (0,-2) (1,0) (2,2) (3,4)
Subtract Eqn(1) from (2)
20x - 8y = 36
(-) 11x –(+)8y =(+)-9
9x = 45
x=5
Substitute x = 5 in Eqn (2),
5x5 - 2y = 9
25 – 2y = 9
-2y = 9-25
-2y = -16
y=8
𝐱 𝟓
∴ The fraction is =
𝐲 𝟖

Sri.Veerendra.H.M. Asst.Teacher. G.P.U.C. Megaravalli. Thirthahalli Tq Ph: 8762624683 Page-49 Sri.Veerendra.H.M. Asst.Teacher. G.P.U.C. Megaravalli. Thirthahalli Tq Ph: 8762624683 Page-50
2) Find the solution of the following pairs of linear equation by the
graphical method : 2𝑥 +𝑦 = 8 , 𝑥 −𝑦 = 1 June : 2019
UNIT- 4. CIRCLE :
Circle : The collction of all the points in a plane which are at a fixed distance
3) Find the solution of the following pairs of linear equation by the
from a fixed point in the plane is called a circle. The fixed point is the
graphical method : 𝑥 + 𝑦 = 7 , 3𝑥 - 𝑦 = 1 JUNE-2020
cenre and the fixed length is called radius of the circle.
4) Find the solution of the following pairs of linear equation by the
Chord: The line segment joining any two points on the circumference of a circle.
graphical method : 2𝑥 +𝑦 = 8 , 𝑥 + 𝑦 = 5 SEPT ∶ 2020
5) Find the solution of the following pairs of linear equation by the Arc : A piece of a circle between two points called an arc.
graphical method : 2x + y = 5 & x + y = 4 PREP ∶ 2020 Segment : The region between a chord and either of its arc is called a segment.
6) Find the solution of the following pairs of linear equation by the Sector : The region between an arc and two radii is called a segment.
graphical method : 2x + 3y = 12 & x – y = 1 PREP-2019
Secant of a circle : A line which intersects a circle
7) Find the solution of the following pairs of linear equation by the
in two distinct points is called secant M N
graphical method : 2x + y = 6 & 2x – y = 2 PREP-2020
of a circle. Here MN is a secant.
8) Find the solution of the following pairs of linear equation by the Tangent to a circle : A tangent to a circle is straight line P A Q
graphical method : Model-1 ∶2021
which intersects (touches) the circle at only one point.
2x + y = 10 , x + y = 6
9) Find the solution of the following pairs of linear equation by the Here PQ is a tangent touches the circle at ‘A’
* Tangent touches the circle at only one point.
graphical method : Model-2 ∶ 2021
x + y = 5, 2x ˗ y = 4 * We can draw infinitely many tangents to a circle.
10) Solve graphically : MODEL-QP-2022 * The common point of a tangent to a circle and the circle is called point of contact.
x+y=5 & x–y=1 Theorem 4.1 : “The tangent at any point of a circle is perpendicular to the radius
11) Find the solution of the given pair of linear equations by graphical
through the point of contact”
method : x + y = 5 and 2x – y = 4 PREP-2022
12) Find the solution of the given pair of linear equations by graphical Theorem 4.2 : “The lengths of tangents drawn from an external point to a circle
method : x + 2y = 6 & x+y=5 March-2022 are equal”
13) Find the solution of the pair of linear equations by graphical method :
2x – y = 7 & x – y = 2 June-2022
MULTIPLE CHOICE QUESTIONS :
1) A straight line which passes through two points on a circle is April-2019

(A) a chord (B) a secant (C) a tangent (D) the radius


2) A straight line passing through a point on a circle is April-2020

A) a tangent 𝐵) secant 𝐶) radius 𝐷) a transversal. A C


3) In given figure BC is Model-1:2021 O

A) radius 𝐵) chord 𝐶) diameter 𝐷) a transversal


D B E
4) Four statements are given below with respect to the tangents. The wrong
statement is: Model-1:2021
Sri.Veerendra.H.M. Asst.Teacher. G.P.U.C. Megaravalli. Thirthahalli Tq Ph: 8762624683 Page-51 Sri.Veerendra.H.M. Asst.Teacher. G.P.U.C. Megaravalli. Thirthahalli Tq Ph: 8762624683 Page-52
A. There are exactly two tangents that can be drawn to a circle from a point lying 12) In the following figure, PA, PC and CD are
outside the circle. tangents drawn to a circle of centre O. If AP = 3 cm,
B. There is only one tangent passing through a point lying on a circle. CD = 5 cm, then the length of PC is June : 2019
C. Only two tangents can be drawn from a point lying inside a circle. 𝐴) 3 cm 𝐵) 5 cm. 𝐶) 8 cm. 𝐷) 2 cm.
D. The lengths of the tangents drawn from an external point to a circle are equal.
13) In the figure, TP and TQ are tangents drawn to a
5) The lengths of the tangents drawn to a circle from a point outside it
circle of centre O. measure of ∠𝑃𝑇𝑄 Model-2020
A. are unequal B. are equal Model-2:2021
𝐴) 90° 𝐵) 110° 𝐶) 70° 𝐷) 40°
C. are equal to radius of the circle D. are equal to diameter of the circle
14) AB and CD are two parallel tangents to the circle of
6) The wrong statement in the following is JULY-2021
radius 6 cm. Then the distance between the tangents
(A) a tangent to a circle touches the circle exactly at one point.
(B) when a straight line is drawn to a circle it always passes through a is Model-2020

point on the Circle. 𝐴) 10𝑐𝑚 𝐵) 12𝑐𝑚 𝐶) 13𝑐𝑚 𝐷) 15𝑐𝑚


(C) the point common to the circle and its tangent is called the point of 15) In the given figure PA and PB are the tangents
contact.
(D) the tangent drawn at any point to a circle is to a circle with centre ‘O’. If PA= 5cm and
perpendicular to the radius drawn at the point of contact. ∠APB=600 then the length of the chord AB is :
A. 5√2 cm B. 5√3 cm C. 5cm D. 5.2cm
7) In a circle with centre ‘O’ the secant is 16) A tangent can be drawn from an external point ‘A’ to a circle with centre
A. PQ B. XY C. QR D. AB Model-2:2021 ‘O’. AB = 24cm and OA=25cm then the radius is Model-2:2021
A. 12cm
B. 7cm
8) In the figure the ‘Chord’ is , C. 15cm
A) AB B) CD C) MN D) OK D. 16cm

9) In a circle, the angle between the tangent and the radius at the point of 17) In the given circle with centre ‘O’ ACB, AE and BD
contact is March-2022 are the tangents. If AB = 12cm, AE=3cm the length of
A) 30
0
B) 60
0
C) 90
0
D) 180
0 BD is Model-2:2021
A. 6 cm B. 3 cm C. 8 cm D. 9 cm

10) AP and BP are the tangents to a circle of centre ‘O’. 18) In the given figure PA, PBC and CD are the tangents to
0
If ∟APB = 30 then the measure of ∟OAP is , a circle with centre O. If PC = 8 cm and AP = 5 cm,
A) 300 B) 1200 C) 600 D) 900 the length of the tangent CD is JULY-2021
(A) 5 cm (B) 3 cm (C) 8 cm (D) 13 cm

11) The angle between the two radii of a circle is 1300. Then the angle between 19) A line intersecting a circle in two points is : Model-1:2021
the tangents drawn at the ends of the radii is Model-1:2021 A. secant B. tangent C. radius D. perpendicular
A. 650 B. 400 C. 700 D. 500

Sri.Veerendra.H.M. Asst.Teacher. G.P.U.C. Megaravalli. Thirthahalli Tq Ph: 8762624683 Page-53 Sri.Veerendra.H.M. Asst.Teacher. G.P.U.C. Megaravalli. Thirthahalli Tq Ph: 8762624683 Page-54
20) In the adjoining figure, tangents are drawn 500 + 900 + ∟BAC = 1800
0 0
from an external point ‘P’ to two internally ∟BAC = 180 - 140
0
touching circles. If AP = 8 cm, then the measure ∟BAC = 40
of CP is 4) In the adjoining figure, AP = 3cm and PC = 8cm
A) 7.6 cm B) 7.8 cm C) 8cm D) 8.2 cm then, find the length of tangent CD PREP-2019 D C
Ans : AP = 3 cm , PC = 8 cm , CD = ?
21) In the figure, secant is June-2022 M X S Y
A) AB P Q BP + BC = PC
B) PQ But BP = AP (∵ The tangents drawn from an external point A B
C) XY A O B
D) MN ∴ AP + BC = PC to the circle are same)
N 3 + BC = 8
22) In the given figure AT is a tangent drawn at the BC = 8 – 3 P
point A to the circle with centre O such that BC = 5 cm
0
OT = 4 cm. If ∟OTA = 30 ,then AT is JULY-2021 O But CD = BC (∵The tangents drawn from an external point to the circle are same)
( A) 4 cm (B) 2 cm (C) 2√3 cm (D) 4√3 cm 300
CD = 5cm
A T
Answers : 1.B, 2.A, 3.B, 4.C, 5.B, 6.B, 7.B, 8.C, 9.C, 10.D, 11.D, TWO MARKS QUESTIONS :
12.C, 13.C, 14.B, 15.C, 16.B, 17.D, 18.B, 19.A, 20.C, 21.D, 22.C
1) In the given circle 𝐴𝑃, 𝐴𝑋 and 𝐴𝑌 are the
tangents. Then prove that 𝐴𝑌 = A𝑋.
ONE MARK QUESTIONS :
Ans : In circle centre with ‘D’, 𝐴𝑌 = 𝐴𝑃 ----- (1)
1) How many tangent/s can be drawn to circle in a given point. Prep : 2020
( ∵ The tangents drawn from an external point to the circle are same)
Ans : One
In circle centre with ‘C’, AX = 𝐴𝑃 ------(2)
2) In the figure AB and AC are the two B From (1) and (2)
tangents drawn from the point A to the circle 𝐴𝑌 = A𝑋
with centre O. If∟BOC = 1300 then find∟BAC O 130 0
A 2) Prove that “ The lengths of tangents drawn from an external point to a
Ans : April : 2020 circle are equal” ( Theorem-6) 3-April ∶ 2019 , June ∶2020 , SEPT∶ 20, PREP ∶ 19, PREP ∶ 20
C March-2022 , June-2022
0
Angle between tangents = 180 – Angle between the radii 3) In the given figure PQ and Rs are two parallel tangents to a circle wityh
0 0
∟BAC = 180 -130 centre O and another tangents AB with point of contact C intersecting PQ
∴ ∟BAC = 500 at A and Rs at B. Prove that ∠𝐴𝑂𝐵 = 90°. 3-April : 2019
3) In the given circle with centre ‘O’ AC is diameter. To prove : ∠𝐴𝑂𝐵 = 90°.
∟ACB = 500 then find ∟BAC?
Proof : Join O and C.
Ans : AC is a diameter. ∟AOC = 1800 ∴ ∟ABC = 900
In ∆OXA and ∆OCA
∴ ∟ACB + ∟ABC + ∟BAC = 1800 AX = AC (∵ tangents)
[ ∵ sum the three angles of a triangle is 180° ]
OX = OC (∵ radii )
Sri.Veerendra.H.M. Asst.Teacher. G.P.U.C. Megaravalli. Thirthahalli Tq Ph: 8762624683 Page-55 Sri.Veerendra.H.M. Asst.Teacher. G.P.U.C. Megaravalli. Thirthahalli Tq Ph: 8762624683 Page-56
OA = OA (∵ common side ) 7) To the circle with centre ‘O’, TP and TQ are the two tangents drawn from
∴ ∆OXA ∆OCA (∵ S.S.S. Congruence rule) an external point ‘T’. Prove that ∠PTQ = 2∠OPQ PREP-2019

∟XOA = ∟COA .................. (1) P


Similarly ∆OCB ∆OYB (∵ S.S.S. Congruence rule) Data : ‘O’ is the centre. TP and TQ
∟COB = ∟YOB .................. (2) are the two tangents drawn T
∟XOA +∟COA +∟COB + ∟YOB = 1800 ------(3) (∵ XY is a straight line.) O
to the circle at P and Q.
Substitute (1) and (2) in Eqn(3) To prove : ∠PTQ = 2∠OPQ
0 Q
2∟AOC + 2∟COB = 180
∟AOC + ∟COB = 90 0 Proof : Let ∠PTQ = θ.
0
∟AOB = 90 But TP = TQ (∵The tangents drawn from an external point to the circle are same.)
4) Two concentric circles of radii 5cm and 3cm are drawn. Find the length of ∴ TPQ is an isosceles triangle.
the chord of the larger circle which touches the smaller circle. June-2019 ∴ ∠TPQ = ∠TQP = [ 180 - θ ]
Ans : By Pythagoras theorem, ∠TPQ = ∠TQP = 90 - θ ---------- (1)
2 2 2
OA = AB + OB O 0
But ∠OPT = 90 (∵The tangents at any point of a circle is perpendicular to the
52 = AB2 + 32 A B radius through the point of contact)
25 = AB2 + 9
∠OPQ = ∠OPT - ∠TPQ
AB2 = 25-9
AB2 = 16 ∠OPQ = 900 – (90 - θ)
AB = 4cm ∠OPQ = θ = x ∠PTQ
∴ Length of the chord = AC = 8cm ∴ ∠PTQ = 2 x ∠OPQ
5) Prove that “The tangents at any point of a circle is perpendicular to the
radius through the point of contact”(Theorem-5) Model ∶ 2020
6) In the given circle with centre ‘O’ AB is diameter and
AP is a tangent. ∟OCB = 250 then find ∟APB? Model∶2019
Ans : ∠𝑂𝐶𝐵 = 25°
OB = OC (∵ Radii )
∠𝑂BC = ∠𝑂𝐶𝐵 = 25° (∵ ∆OBC is a isosceles triangle )
∠PAB = 90° (∵The tangents at any point of a circle is perpendicular to the
radius through the point of contact)
In ∆PAB, ∠APB + ∠A𝐵P + ∠PAB = 180°
∠APB + 25° + 90° = 180°
∠APB = 180° -1150
∠APB = 65°
Sri.Veerendra.H.M. Asst.Teacher. G.P.U.C. Megaravalli. Thirthahalli Tq Ph: 8762624683 Page-57 Sri.Veerendra.H.M. Asst.Teacher. G.P.U.C. Megaravalli. Thirthahalli Tq Ph: 8762624683 Page-58
UNIT-5. AREA RELATED TO CIRCLES: 2) In the figure find the length of an arc AB of a circle
1. Circumference of a circle = C = 2 π r
2
centre ‘O’ if ∟AOB = 900 MODEL-QP-2022 O
2. Area of a circle = A = π r ϴ
Ans : Length of an arc = x2 𝜋𝑟 A 900 B
3. Area of a Semi circle = A =
= x2x x7
4. Area of a quadrant = A =
= x2x22
5. Area of a square = A = a2
= 11cm
6. Area of a rectangle = A = lxb
3) If the diameter of a circle is 7cm then
7. Area of a triangle = A = xbxh
√ write the length of its semicircular arc.
8. Area of an equilateral triangle = A = a2 ϴ
Ans : Length of an arc = x2 𝜋𝑟 PREP-2022
9. Length of an arc of a circle of radius ‘𝑟’ and angle ′𝜃′ = x 2𝜋𝑟
= x2x x3.5
10. Area of the sector of a circle of radius ‘𝑟’ and angle ′𝜃′= x 𝜋𝑟2
= x22
MULTIPLE CHOICE QUESTIONS : length of its semicircular arc = 11cm
1) If the area of a circle is 49π sq.units then its perimeter is April : 2019
(A) 7 π units (B) 9 π units (C) 14 π units (D) 49 π units TWO MARKS QUESTIONS :
2) Area of two congruent circles with radius ‘𝑟’ is MODEL-2019
1) Find the length of an arc of a sector of a circle with radius 21cm and
2 2 2
𝐴) 2𝜋𝑟 𝐵) 2 C) 𝜋𝑟 𝐷) 𝜋𝑟 angle subtended at the centre is 60°. April : 2019
3) Length of an arc of a sector of a circle of radius ‘r’ and angle ‘θ is Ans : Radius = r = 21cm, Angle = 𝜃 = 60°
𝛉 ϴ
A) x 𝜋𝑟2 B) x 2𝜋𝑟2 C) x 2𝜋𝑟 D) x 2𝜋𝑟 June-2020 Length of an arc = x2 𝜋𝑟
𝟑𝟔𝟎
4) The area of the sector of a circle with radius ‘r’ and angle ‘p’ (in degree) = x2x x 21
𝐩
A) x2πr2 B) x2πr C) x πr
2
D) x 2πr PREP-2019 = x x 21
𝟕𝟐𝟎
𝟏
5) The area of the sector of a circle with radius ‘r’ is PREP-2020 ∴ Length of an arc = 22 cm.
𝟒

A)
πr2
sq.units B)
πr2
sq.units C)
𝛑𝐫𝟐
sq.units D)
πr2
sq.units 2) Find the area of the sector of a circle with radius 4cm and angle 450
𝟒
(use 𝜋= 3.14). April : 2019
ONE MARK QUESTION :
Ans : Radius = r = 4cm, Angle = 𝜃 = 45°
1) Write the formula to find area of a sector of a circle, if angle at the
ϴ
centre is ‘θ’ degrees. June : 2019 Area of the sector = x 𝜋𝑟2
Ans : A = x 𝜋𝑟2 = x x 42

= x x16

Sri.Veerendra.H.M. Asst.Teacher. G.P.U.C. Megaravalli. Thirthahalli Tq Ph: 8762624683 Page-59 Sri.Veerendra.H.M. Asst.Teacher. G.P.U.C. Megaravalli. Thirthahalli Tq Ph: 8762624683 Page-60
∴ Area of the sector = 6.28 cm2 Area of the shaded region = Area of the square – 4 x Area of the circle
= Side x Side – 4 x 𝜋𝑟2
3) In the figure, ABCD is a square of side 14 cm. A, B, C and D are the centres
of four congruent circles such that each circle touches externally two of the = 14 x 14 – 4 x x 3.5 x 3.5
remaining three circles. Find the area of the shaded region. ∴ Area of the shaded region = 196 – 154 = 42cm2
Ans : Area of a square = ( side )2 = ( 14 )2 6) In the given figure 𝐴𝐵 = 36 𝑐𝑚. M is the midpoint of 𝐴𝐵. AM, BM and
Area of the square = 196 cm2
AB are the diameters of 3 semi circles. Find the area of the shaded region
Area of a quadrant = πr2
Ans : MODEL-2019
4× Area of quadrant = 4 × πr2
Radius of the circle with diameter AB = R = 18cm
= 4 × x x 72
Radius of the circle with diameter AM = r = 9cm
= 22 × 7
4 × Area of quadrant = 154 cm2 Area of the = Area of the semicircle - 2x Area of the
Area of the shaded region = Area of square – 4 x area of quadrant Shaded region with diameter AB semicircle with diameter AM & BM
Area of shaded region = 196 – 154 π
Area of shaded region = 42 cm2 = – 2x
= πx - π x 9x9
4) The areas of two circles are 92 cm2 and 62cm2 resp. Find radius of the circle
having the area of sum of the areas of these circles. June : 2019 = 162π - 81π = 81π = 81 x 3.14
Ans : Sum of the areas of two circles = 92+62 = 154cm2 ∴ Area of the shaded region = 254.34 cm2
7) In the given figure, two congruent circles are
Area of a circle = A = 𝜋𝑟2
touching externally and touches the sides of
𝜋𝑟2 = 154
ABCD. AB = 28 cm. and BC = 14 cm then, find
𝑟2 = 154 x
the area of the shaded region. PREP-2020
𝑟2 = 49 Ans :
∴ Area of a circle = r = 7 cm Area of the shaded region = Area of rectangle ABCD – 2x Area of circle.
= Length x Breadth – 2𝜋𝑟2
5) ABCD is a square of side 14 cm. Four
= 28 x 14 – 2x x7
congruent circles are drawn in the square as
= 392 – 308
shown in the figure. Calculate the area of
the shaded region.[ Circles touch each other ∴ Area of the shaded region = 84 cm2
externally and also sides of the square ] 8) A right angled triangle is inscribed in a circle
June : 2019 of radius 5cm with centre ‘O’. The sides which
Ans : Radius = = 3.5 cm. make right angles are 8 cm and 6cm. Find the
Length of the side of the square = 14 cm area of the shaded region in the figure.
Ans : PREP-2020
Sri.Veerendra.H.M. Asst.Teacher. G.P.U.C. Megaravalli. Thirthahalli Tq Ph: 8762624683 Page-61 Sri.Veerendra.H.M. Asst.Teacher. G.P.U.C. Megaravalli. Thirthahalli Tq Ph: 8762624683 Page-62
Area of the shaded region = Area of the circle – Area of the triangle Ans :
= 𝜋𝑟2 - x AC x BC Area of the hand fan = Area of the sector

= 3.14(5)2 - x 8 x 6 Area of the sector = x 𝜋𝑟2


= 3.14 x 25 - 24 Area of the hand fan = x x 21x 21
= 78.5 – 24 = 22 x 21
∴ Area of the shaded region = 54.5 cm2
= 462 cm2
THREE MARKS QUESTIONS :
Total length of the required wire = Length of the arc + 2 x radius
1) ABCD is a rectangle of length 20 cm and breadth 10 cm. OAPB is a sector
= x 2𝜋𝑟 + 2𝑟
of a circle of radius 10 √2 cm. Calculate the area of the shaded region.
[ Take π = 3·14 ] 3-JUNE-2020 = x2 x x 21+ 2 x 21
Ans : In ∆AOB, = 44 + 42
AB = 20 cm = 86 cm
OA = 10√2 cm  Area of the required cloth = 462 cm2
OB = 10√2 cm  Length of required wire = 86 cm
2 2
AB = 400 cm
2 3) AB and CD are the arcs of two concentric circles with centre O of radius
OA2  OB2= 100 ( 2 ) + 100 ( 2 ) = 400 cm
AB2 = OA2  OB2 21 cm and 7 cm resp. If ∟AOB = 300 as shown in the figure, find the area of
 AOB is a right triangle, ∟AOB = 90° the shaded region. SEPT-2020

Area of the shaded region = Area of the rectangle – Area of the segment APB Ans : Area of the sector = x 𝜋𝑟2

= l x b - [ Area of sector OAPB - Area of AOB ] Area of the sector OAB = x x 21x 21
θ 2
= 20 x 10 - [ x 𝜋𝑟 - x OA x OB ] =
= 20 x 10 - [ x 3.142 x 10√2 x 10√2 - x 10√2 x 10√2 ] = cm2
= 200 -[ 157 – 100 ]
Area of the sector OCD = x 𝜋𝑟2
= 200 – 57
Area of the shaded region = 143 cm2 = x x 7x 7
=
2) A hand fan is made up of cloth fixed in between the
= cm2
metallic wires. It is in the shape of a sector of a circle
of radius 21 cm and of angle 120° as shown in the Area of shaded region = area of sector OAB - area of sector OCD
figure. Calculate the area of the cloth used and also find = −
the total length of the metallic wire required to make
such a fan. JUNE-2020 = −

Sri.Veerendra.H.M. Asst.Teacher. G.P.U.C. Megaravalli. Thirthahalli Tq Ph: 8762624683 Page-63 Sri.Veerendra.H.M. Asst.Teacher. G.P.U.C. Megaravalli. Thirthahalli Tq Ph: 8762624683 Page-64
= = cm2 Area of the sector OAC = x x 21x 21
∴ Area of shaded region = 102.6 cm2 = x 22 x 3x 21
4) In the figure, ABCD is a square, and two semicircles touch each other Area of the sector OAC = 231 cm2
externally at P. The length of each semicircular arc is equal to 11 cm. Find
the area of the shaded region. SEPT-2020
Ans : Length of the semicircular arc = 𝜋𝑟 6) In the figure AXB and CYD are the arcs of two concentric circles with
centre O. The length of the arc AXB is 11 cm. If OC = 7 cm and ∟AOB = 300,
11 = x𝑟 𝟐𝟐
then find the area of the shaded region. [ Take π = ] 4-March-2022
𝟕
∴ 𝑟 = = 3.5 cm
11 cm
Area of two semicircle = 𝜋𝑟𝑟2 X
= x 3.5 x 3.5 A B
= 11 x 3.5
= 38.5 cm2
C Y D
The diameter of circle is equal to side of the square ABCD 7cm 300
 Side AB = 2x radius
= 2 x 3.5 O
AB = 7 cm Ans : Length of an arc = 11 cm
Area of square ABCD = Side x Side
= 7 x 7 = 49 cm2 Length of an arc = x 2π r
Area of shaded region = area of ABCD - area of two semi-circles
circles 11 = x 2x xr
= 499 – 38.5
11 = x 2x xr
Area of shaded region = 10.5 cm2
11x12x7 = 2x22x r
0
5) In a circle of radius 21cm an arc AC subtends an angle 60 at the centre as r=
shown, then find PREP-2022 r = 21 cm
i) the length of the arc AC
Area of sector OAXB = A1 = x π r2
ii) area of the sector OAC
Ans : Radius of the
he circle = r = 21cm O = x x 212
0
Angle = θ = 60 600 3
= x x 21x21
ϴ
i) Length of an arc = x 2 𝜋𝑟 A C
= cm2
Length of arc AC = x2x x 21
Area of sector OCYD = A2 = x π r2
= x2x22x3 = 22cm
= x x 72
ii) Area of the sector = x 𝜋𝑟2
= x x 7x 7
Sri.Veerendra.H.M. Asst.Teacher. G.P.U.C. Megaravalli. Thirthahalli Tq Ph: 8762624683 Page-65 Sri.Veerendra.H.M. Asst.Teacher. G.P.U.C. Megaravalli. Thirthahalli Tq Ph: 8762624683 Page-66
A2 = cm2 Radius of the sector = r = 4cm, θ = 600
Area of shaded region = A1 - A2 Area of shaded region = Area of equilateral triangle– 3x Area of the sector

= - = x a2 – 3x x π r2
= =

x 82 – 3x x π x 42
= cm2 =

x 8x82 – 3x x π x 4x4
2
= cm 6

= 102.66 cm 2 = 16√3 - 8x π
7) ABCD is a square of side 14 cm. A circle is drawn = 16x1.71 – 8x 3.14
inside it which just touches the mid-points of sides of = 27.36 – 25.12
the square, as shown in the figure. If P, Q, R and S ∴ Area of shaded region = 2.24cm2
are the mid RS and SP are the arcs of the circle
region. 4-June-2022
Ans. : 9) Find the area of the shaded design where ABCD is a square of side 10cm and
Sides of a square = a = 14 cm semicircles are drawn with each side of the square as diameter( Use π = 3.14)
Radius of circle = radius of quadrant = r = = 7 cm
Area of shaded region = A B
[Area of square – Area of circle] + [Area of square – 4 x Area of quadrant
= [ a2 – π r2 ] + [ a2 – 4 x π r2 ]
= [ a2 – π r2 ] + [ a2 – π r2 ]
= 2 [ a2 – π r2 ]
D C
= 2 [ 142 – x7x7 ] = 2 [ 196 – 154]
Ans. : Area of the shaded design = 57 cm2
= 2 [ 42]
Area of shaded region = 84 cm2
10) In the figure ABCD is a square of side 14cm with
8) In the given figure, ∆PQR is an equilateral with sides 8 cm. D, E, F are Centre A,B,C and D four circles are drawn such that
each circle touch externally two of the remaining three
midpoints of the triangle, and joined with circular arcs. Find the area of the
circles as shown in the figure. Find the area of the shaded
shaded region. (π = 3.14, √3 = 1.71) P region. MODEL-QP-2022
2
Ans : Area of shaded region = 42 cm

D E 11) ABCD is a square of side 14cm. P, Q, R and S are


the mid points of AB, BC, CD and AD respectively.
Also PS, PQ, QR and SR are the arcs of the circles.
Q F R Find the area of the shaded region. PREP-2022
Ans. : Each Side of the triangle = a = 8cm Ans : Area of shaded region = 42 cm2
Sri.Veerendra.H.M. Asst.Teacher. G.P.U.C. Megaravalli. Thirthahalli Tq Ph: 8762624683 Page-67 Sri.Veerendra.H.M. Asst.Teacher. G.P.U.C. Megaravalli. Thirthahalli Tq Ph: 8762624683 Page-68
UNIT-6. CONSTRUCTIONS : 11) Draw a pair of tangents to a circle of radius 5cm which are inclined to each
DIVISION OF A LINE SEGMENT : other at an angle of 600 and write the measure of its length. 2-PREP-2019
1) Draw a line segment of length 8cm and divide it in the ratio 3 : 2 12) Draw a circle of radius 4 cm and construct a pair of tangents such that
geometrically. 2-MODEL-2021 the angle between radii is 80°.
2) Divide the line segment AB = 10cm in the ratio 2 : 3 geometrically. MOD-2022
3) Draw a line segment of length 8cm and divide it in the ratio 2 : 3 CONSTRUCTION OF SIMILAR TRIANGLES :
geometrically. 2-PREP-2022 1) Construct a triangle with sides 5 cm, 6 cm and 7 cm and then construct
4) Draw a line segment of length 10cm and divide it in the ratio 2 : 3 𝟓
another triangle whose sides are of the corresponding sides of the first
𝟕
geometrically. March-2022 triangle. 3-APRIL-2019
5) Draw a line segment of length 8·4 cm and divide it in the ratio 1 : 3 by 2) Construct a triangle with sides 5 cm, 6 cm and 7 cm and then construct
geometric construction. 𝟑
another triangle whose sides are of the corresponding sides of the given
𝟓
CONSTRUCTION OF A TANGENT TO THE CIRCLE : triangle. 3-June-2019
1) Construct a tangent to a circle of radius 4cm at any point P on its 3) Construct a triangle with sides 4 cm, 5 cm and 6 cm and then construct
𝟐
circumference. 3-MODEL-2021 another triangle whose sides are of the corresponding sides of the given
𝟑
2) Draw a circle of radius 3 cm. From a point 5cm away from circle, triangle. 𝟑−−MODEL1-2020

construct the pair of tangents to the circle. 2-APRIL-2019 4) Construct an isosceles triangle ABC whose base BC = 8 cm and height 4 cm.
𝟓
3) Construct two tangents to a circle of radius 3 cm from a point 8 away Then construct a triangle whose sides are of the corresponding sides of the
𝟑
from its centre. 𝟐− MODEL2-2020 & 3-March-2022
triangle ABC. 𝟑− MODEL2-2020
4) Construct a pair of tangents to a circle of radius 4cm from a point 9cm 5) Construct a triangle ABC with sides BC = 3 cm, AB = 6 cm and AC = 4·5 cm.
away from its centre. 3-MODEL-QP-2022 𝟒
Then construct a triangle whose sides are of the corresponding sides of
5) Draw a circle of radius 4 cm. From a point 8cm away from its centre, 𝟑
the triangle ABC. 3-June-2020
construct the pair of tangents to the circle. 2-JUNE-2019
6) Construct a triangle with sides 6 cm, 7 cm and 8cm and then construct
6) Draw a circle of radius 4 cm and construct a pair of tangents such that the
𝟑
angle between them is 60°. 2-April-2020 & 2−SEPT-2020 another triangle whose sides are of the corresponding sides of the given
𝟒
7) Draw a pair of tangents to a circle of radius 3·5 cm which are inclined to triangle. 3-Sept-2020
each other at an angle of 60°. 2-June : 2019 7) Construct a triangle with sides 4 cm, 6 cm and 7 cm and then construct
8) Draw a pair of tangents to a circle of radius 5 cm which are inclined to 𝟑
another triangle whose sides are of the corresponding sides of the given
𝟒
each other at an angle of 60°. 2- MODEL1-2020 triangle. 3-PREP-2020
9) Draw a pair of tangents to a circle of radius 3 cm which are inclined to 8) Construct a triangle ABC with sides BC = 6 cm, AB = 5 cm and ∠ABC = 600
each other at an angle of 60°. 2-June-2020 & 3- JUNE-2022 𝟑
Then construct a triangle whose sides are of the corresponding sides of
𝟒
10) Draw a pair of tangents to a circle of radius 4 cm which are inclined to
the triangle ABC. 3-PREP-2019
each other at an angle of 70°. 2-PREP-2020

Sri.Veerendra.H.M. Asst.Teacher. G.P.U.C. Megaravalli. Thirthahalli Tq Ph: 8762624683 Page-69 Sri.Veerendra.H.M. Asst.Teacher. G.P.U.C. Megaravalli. Thirthahalli Tq Ph: 8762624683 Page-70
9) Construct a triangle with sides 4cm, 5cm, and 6cm and then another
𝟓 UNIT-7. CO-ORDINATE GEOMETRY:
triangle whose sides are of the corresponding sides of the first triangle
𝟑
4-MODEL-2021 Coordinate geometry has been developed as an algebraic tool for studying
10) Construct a triangle ABC of its sides BC=4cm, AB=6cm and AC=4.5cm geometry of figures. It helps us to study geometry using algebra, and understand
algebra with the help of geometry.
𝟐
then construct a triangle similar to it, whose sides are of the * The distance of a point from the y-axis is called its x-coordinate or abscissa.
𝟑
corresponding sides of the triangle ABC. 4-MODEL-2021
* The distance of a point from the x-axis is called its y-coordinate or ordinate.
* Coordinates of origin are (0,0)
11) Construct a triangle ABC with sides AB = 5cm, BC = 7cm and AC = 6cm, Distance formula: The distance between the points P(x1, y1) and Q(x2, y2) is
𝟑
then construct an another triangle whose sides are of the corresponding PQ = d = (𝑥 − 𝑥 ) + (𝑦 − 𝑦 )
𝟒
sides of the triangle ABC. 4-PREP-2022 The distance of a point P(x, y) from the origin(0, 0) is : d = 𝑥 + 𝑦
12) Construct a triangle with sides 4·5 cm, 6 cm and 8 cm. Then construct Section formula : The coordinates of the point P(x, y) which divides the line
𝟑
another triangle whose sides are of the corresponding sides of the first segment joining the points A(x1, y1) and B(x2, y2), internally, in the ratio
𝟒
triangle. 4-March-2022 m1 : m2 are
13) Construct a triangle BC = 6 cm, AB = 5 cm and AC = 4·5 cm. Then P(x, y) = ( , )
𝟒
construct a triangle whose sides are corresponding sides of the triangle . Mid-point formula : If A(x1, y1) and B(x2, y2) are two points, P is the midpoint,
𝟑
4-June-2022
Coordinates of midpoint of line joining A & B is : P(x, y) = ( , )
Area of a triangle : If A(x1, y1), B(x2, y2) and C(x3, y3) are three vertices of a
triangle, then Area of the triangle = A = { 𝑥1(y2-y3) + x2(y3-y1) + x3(y1-y2) }
Note : If three points are collinear, then the area of the triangle formed by them
must be zero(0).

MULTIPLE CHOICE QUESTIONS :


1) The distance of the co-ordinate p(4, 3) from the x- axis is : JULY 2021
A. 2 units B. 3 units C. 4 units D. 5 units
2) The distance of the co-ordinates (3, 4) from the x-axis is PREP-2022

𝐴) 3 units 𝐵) 4 units 𝐶) 5 units 𝐷) 7 units


3) In the graph given the length AB is Model-1:2021

A. 1 unit B. 5 units C. 3 units D. 4 units


4) The distance between origin and coordinates (𝑥, y)
𝐴) 𝑥2 +𝑦2 𝐵) 𝑥 −𝑦 𝐶) 𝑥2 –𝑦2 𝐷) 𝐱𝟐 + 𝐲 𝟐
𝑱𝒖𝒏𝒆 ∶ 𝟐𝟎𝟏𝟗, PREP-2019, APRIL-2022, MODEL-2022
5) Distance of the point P ( a, b ) from the origin is June-2022
𝐴) √𝑎 + 𝑏 units. 𝐵) √𝑎 − 𝑏 units. 𝐶) √𝑎 + 𝑏 units. 𝐷) √𝑎 − 𝑏 units.
Sri.Veerendra.H.M. Asst.Teacher. G.P.U.C. Megaravalli. Thirthahalli Tq Ph: 8762624683 Page-71 Sri.Veerendra.H.M. Asst.Teacher. G.P.U.C. Megaravalli. Thirthahalli Tq Ph: 8762624683 Page-72
6) In the given graph the length of ʻBPʼ is Model-2:2021 A) ½ [ x1 ( y2 + y3 ) ─ x2 ( y3 + y1 ) ─ x3 ( y1 + y2 )]
A. 2 units B. 5 units C. 3 units D. 4 units B) ½ [ x1 ( y2 + y3 ) + x2 ( y3 + y1 ) + x3 ( y1 + y2 )]
7) The co-ordinates of origin are MODEL-2019
C) ½ [ x1 ( y2 - y3 ) ─ x2 ( y3 - y1 ) ─ x3 ( y1 - y2 )]
D) ½ [ x1 ( y2 - y3 ) + x2 ( y3 - y1 ) + x3 ( y1 - y2 )]
𝐴) (1, 1) B) (2, 2) 𝐶) (0, 0) 𝐷) (3, 3)
18) In the adjoining fig. the distance between the
8) The distance between origin and the point (8, -6)
point ‘P’ and the origin ‘O’ is…………. Units.
A) 10 𝐵) 100 𝐶) √28 𝐷) √14
A) 2√3 B) 4 C) 2√2 D) 2.5
9) The distance between the points (𝑥1, 𝑦1) and (𝑥2, 𝑦2) MODEL-2020, JULY 2021
Answers : 1.B, 2.B, 3.C, 4.D, 5.A, 6.A, 7.C, 8.A, 9.B, 10.C, 11.A, 12.C, 13.B, 14.C, 15.A,
𝐴) (𝑥 + 𝑥 ) + (𝑦 + 𝑦 )2 𝐵) (𝑥 − 𝑥 ) + (𝑦 − 𝑦 )2
16.B, 17.D, 18.C.
𝐶) (𝑥 − 𝑥 ) − (𝑦 − 𝑦 )2 𝐷) (𝑥 + 𝑥 ) − (𝑦 + 𝑦 )2
ONE MARK QUESTIONS :
10) The distance between the points A(0, 5) and B(-5, 0) is
1) Write the co-ordinates of the midpoint of the line segment joining the points
A. 5 units B. 2√5 units C. 5√2 units D. √10 units
A(x1 , y1 ) and B (x2 , y2 ). PREP-2022, Model-1 : 2020, June ∶ 2020
11) The formula to find the mid-point of the line segment joining the points
Ans : Co-ordinates of midpoint of AB is : P(x, y) = [ , ]
A(x1, y1) and B(x2, y2 ) is JULY 2021
A) ( , ) B) ( , ) 2) In the given figure “P” is a midpoint of BC, write the formula to find the
Co-ordinate of P ? Model-2 A(x1 ,y1)
C) ( , ) D) ( , )
12) If origin is the mid point of the line joining of the points A(4,-6) and B(a,b) Ans: P(x,y) = [ , ]
the values of ‘a’ and ‘b’ are equal to Model-1:2021
A. a = 4 and b = 6 B. a = -4 and b = -6 B(x2 ,y2) P C(x3 ,y3)
C . a = -4 and b = 6 D. a = 6 and b = 4 3) A point ‘P’ divides the line joining of points A (x1 ,y1 ) and B (x2 ,y2 ) in the
13) If P (x, y) divides the line join A (x1 ,y1 ) B (x2 ,y2 ) in the ratio m1 : m2 then ratio m1 : m2 internally then write the co-ordinates of P.
x and y are equal to Model-2:2021 Ans : P(x, y) = ( , ) Model-2022, June-2022
A. x = ,y= B. x = ,y= 4) Write the distance of the point ( 4, 3 ) from x-axis. March-2022
C. x = ,y= D. x = ,y= Ans. : 3
TWO MARKS QUESTIONS :
14) If the points A(1,2), O(0,0) and C(a,b) are collinear then. Model-1:2021

A. a = b B. b = 2a C. a = 2b D. a+b = 0 1) Find the distance between the point ( 3, 4 ) and origin. April-2019

15) The mid point of the line segment joining the points A (0, 5) and B (4, 1) is Ans : Distance between origin and a point = 𝑥 +𝑦
A) ( 2, 3) B) (4, 3) C) (2, 6) D) (4, 6) Model-1:2021 = √3 + 4
16) The distance between the points A (2, 3) and B (5, 7) is _______ units. = √9 + 16
A) 2√5 B) 5 C) 5√2 D) √14 = √25
17) The area of a triangle ABC, formed by the co-ordinates A (x1 , y1), ∴ Distance between origin and a point = 5 units
B (x2 , y2) and C (x3 , y3) is,
Sri.Veerendra.H.M. Asst.Teacher. G.P.U.C. Megaravalli. Thirthahalli Tq Ph: 8762624683 Page-73 Sri.Veerendra.H.M. Asst.Teacher. G.P.U.C. Megaravalli. Thirthahalli Tq Ph: 8762624683 Page-74
2) Find the co-ordinates of the midpoint of (2, 3) and (4, 3) June-2019 d = (4) + (−11)
Ans : d = √16 + 121
x1 y1 x2 y2
2 3 4 3 d = √𝟏𝟑𝟕 units
6) Verify whether the points (5, -2), (6, 4) and (7, -2) are vertices of an
Mid point formula = P(x, y) = ( , ) equilateral triangle ? 2-PREP-2019
=( , ) Ans : A(5,-2)

∴ Co-ordinates of the midpoint = (3 , 3) Distance formula = d = (𝑥 − 𝑥 ) + (𝑦 − 𝑦 )


3) Find the distance between the points ( 2, 3 ) and ( 4, 1 ). June-2019 AB = (6 − 5) + (4 − (−2))
Ans : = √1 + 6 = √1 + 36 = √37
x1 y1 x2 y2
2 3 4 1 BC = (7 − 6) + (−2 − 4) B(6,4) C(7,-2)

Distance formula = d = (𝑥 − 𝑥 ) + (𝑦 − 𝑦 ) = 1 + (−6) = √1 + 36 = √37


CA = (5 − 7) + (−2 − (−2)) = (−2) + 0 = √4 = 2
d = (4 − 2) + (1 − 3)
Here AB = BC ≠ CA
d = (2) + (−2)
∴ ∆ ABC is not an equilateral triangle.
d = √4 + 4
∴ d = √𝟖 = 2√𝟐 units 7) Find the coordinates of the point which divides the line joining the points
4) Find the distance between the points ( -5, 7 ) and ( -1, 3 ). June-2020
( 1, 6 ) and ( 4, 3 ) in the ratio 1 : 2 June-2020
Ans : x1 y1 x2 y2 Ans :
-5 7 -1 3 x1 y1 x2 y2 m1 m2
1 6 4 3 1 2
Distance formula = d = (𝑥 − 𝑥 ) + (𝑦 − 𝑦 )
d = (−1 − (−5) + (3 − 7) Section formula = P(x, y) = ,
d = (−1 + 5) + (−4) Co-ordinates of the point= P(x, y) = ,
d = √16 + 16
P(x, y) = ,
d = √32
∴ AB = 4√2 units P(x, y) = ,
∴ Co-ordinates of the point = P(x, y) = ( 2, 5)
5) Find the distance between the points A(2, 3) and B(6, −8) Model-1:2020

Ans : 8) Find the co-ordinates of point which divides the line segment joining the
X1 Y1 X2 Y2
2 3 6 -8 points A ( 4, – 3 ) and B ( 8, 5 ) in the ratio 3 : 1 internally.
Ans. : Let P ( x, y ) be the required point
Distance formula = d = (𝑥 − 𝑥 ) + (𝑦 − 𝑦 )
x1 y1 x2 y2 m1 m2
d = (6 − 2) + (−8 − 3) 4 -3 8 5 3 1
Sri.Veerendra.H.M. Asst.Teacher. G.P.U.C. Megaravalli. Thirthahalli Tq Ph: 8762624683 Page-75 Sri.Veerendra.H.M. Asst.Teacher. G.P.U.C. Megaravalli. Thirthahalli Tq Ph: 8762624683 Page-76
Section formula = P(x, y) = , Ans. : Using section formula,
( )
P(x, y) = ( , )
Co-ordinates of the point = P(x, y) = ,
(-4, 6) = ( , )
P(x, y) = ,
Equating 'x' coordinates , we get
P(x, y) = , -4 =
∴ Co-ordinates of the point P(x, y) = ( 7, 3) -4m1 -4m2 = 3m1 -6m2
6m2-4m2 = 3m1 +4m1
9) Find the co-ordinates of the points of trisection ( i.e. points divides in three
2m2 = 7m1
equal parts) of the line segment joining the points A(1, -2) and B(-3, -4)
=
Ans :
m1 : m2 = 2 : 7
A(1,-2) P Q B(-3, -4)
P and Q are trisection of AB. 11) If A(2, 3), B(-1, 0) and C(2, -4), are vertices of a triangle ABC.
i.e. AP = PQ = QB Find its area? PREP-2019
So, P divides AB internally in the ratio 1 : 2. Ans :
x1 y1 x2 y2 x3 y3
Section formula = P(x, y) = , 2 3 -1 0 2 -4
( ) ( ) ( )
Co-ordinates of point ‘P’ = P(x, y) = , Area of a triangle = A = { 𝑥 1(y2-y3) + x2(y3-y1) + x3(y1-y2) }
A = { 2(0-(-4)) + (-1)(-4-3) + (2)(3-0) }
P(x, y) = ,
A = { 2(4) + (-1)(-7) + (2)(3) }
P(x, y) = ,
A = { 8 + 7+ 6 }
−𝟏 −𝟖
Co-ordinates of point ‘P’ = P(x, y) = ,
𝟑 𝟑 A = {21}
Similarly Q divides AB internally in the ratio 2 : 1
∴ Area of a triangle = A = 10.5 sq.units
Section formula = P(x, y) = ,
( ) ( ) ( ) 12) Find the area of a triangle whose vertices are ( 1, – 1 ), ( – 4, 6 ) and ( – 3, – 5 ).
Co-ordinates of point ‘Q’ = Q(x, y) = ,
Ans : June-2019
Q(x, y) = , X1 Y1 X2 Y2 X3 Y3
1 -1 -4 6 -3 -5
Q(x, y) = , Area of a triangle = A = { 𝑥 1(y2-y3) + x2(y3-y1) + x3(y1-y2) }
−𝟓 −𝟏𝟎
Co-ordinates of point ‘Q’ = Q(x, y) = , 𝟑 A = { 1(6-(-5)) + (-4)(-5-(-1) + (-3)(-1-6) }
𝟑

10) Find in what ratio the point P ( – 4, 6 ) divides the line segment joining the A = { 1(6+5) + (-4)(-4) + (-3)(-7) }
points A ( – 6, 10 ) and B ( 3, – 8 ). June-2022
A = { 1(11) + (-4)(-4) + (-3)(-7) }

Sri.Veerendra.H.M. Asst.Teacher. G.P.U.C. Megaravalli. Thirthahalli Tq Ph: 8762624683 Page-77 Sri.Veerendra.H.M. Asst.Teacher. G.P.U.C. Megaravalli. Thirthahalli Tq Ph: 8762624683 Page-78
A = { 11 + 16 + 21}
THREE MARKS QUESTIONS :
A = {48}
1) The vertices of a Δ ABC are A( – 3, 2 ), B( – 1, – 4 ) and C( 5, 2 ). If M and N
∴ Area of a triangle = A = 24 Sq.units
are the mid-points of AB and AC respectively, show that 2 MN = BC.
13) The points A ( 1, 1 ), B ( 3, 2 ) and C ( 5, 3 ) cannot be the vertices of the April-2019
Ans : A(-3,2)
triangle ABC. Justify. June-2020
Ans : x1 y1 x2 y2 x3 y3 Co-ordinates of mid point = P(x, y) = ( , )
1 1 3 2 5 3 ( )
Co-ordinates of Mid point of AB = ( , ) M N
Area of a triangle = A = { 𝑥 1(y2-y3) + x2(y3-y1) + x3(y1-y2) } =( , )
A = { 1(2-3) + 3(3-1) + 5(1-2) } Co-ordinates of point M = (-2, -1) B(-1,-4) C(5, 2)
A = { 1(-1) + 3(2) + 5(-1) } Co-ordinates of Mid point of AC = ( , )
A = { −1 + 6 - 5} = ( , )
A = {0} Co-ordinates of point N = (1, 2 )
Area of a triangle = A = 0 Distance of MN = d = (𝑥 − 𝑥 ) + (𝑦 − 𝑦 )
∴ Area of a triangle ∆ 𝐴𝐵𝐶 is zero. Hence the points A(1, 1), B(3, 2) and d = (1 − (−2)) + (2 − (−1))
C(5, 3) cannot be the vertices of the triangle ABC. d = (3) + (3)
d = √9 + 9
14) Find the value of ‘k’ if the points A(2, 3), B(4, 𝑘) and 𝐶(6, −3) are
Distance of MN = d = √18
collinear. Model-1:2020
Distance of BC = d = (𝑥 − 𝑥 ) + (𝑦 − 𝑦 )
Ans : x y x y x y
1 1 2 2 3 3 d = (5 − (−1)) + (2 − (−4))
2 3 4 k 6 -3
d = (6) + (6)
Since the given points are collinear, then area of the triangle formed by them must
d = √36 + 36
be 0.
d = √72
Area of a triangle = A = { 𝑥1(y2-y3) + x2(y3-y1) + x3(y1-y2) } = 0 d = √18𝑥4
i.e. { 2(k-(-3)) + 4(-3-3) + 6(3-k) } = 0 Distance of BC = d = 2√18
{ 2(k+3) + 4(-6) + 6(3-k) } = 0 Distance of BC = 2 x Distance of MN
{ 2k+6 - 24 + 18-6k} = 0 BC = 2 x MN
{ 0-4k} = 0
2) The vertices of a Δ ABC are A ( – 5, – 1 ), B ( 3, – 5 ), C ( 5, 2 ). Show that the
k =0
area of the Δ ABC is four times the area of the triangle formed by joining
the mid-points of the sides of the triangle ABC. April-2019

Sri.Veerendra.H.M. Asst.Teacher. G.P.U.C. Megaravalli. Thirthahalli Tq Ph: 8762624683 Page-79 Sri.Veerendra.H.M. Asst.Teacher. G.P.U.C. Megaravalli. Thirthahalli Tq Ph: 8762624683 Page-80
Ans : Midpoint Formula = P(x, y) = ( , ) A(−5, −1) = { 3𝑝 − 3𝑝 + 3 − 3𝑝 + 6𝑝 − 2𝑝 + 3𝑝 − 1 − 4𝑝 − 4 }
Coordinates of D, the mid point of AB = ( , ) = {6𝑝 − 3𝑝 + 3p − 3p + 3p − 2p − 4p + 3 − 1 − 4 }
Coordinates of point D = (-1 , -3) D F F
𝟏
Area of ∆ABC = 𝟐 { 𝟑𝒑𝟐 − 𝟑𝒑 − 𝟐}
Coordinates of point E = (4 , -1.5) When P = 2
Coordinates of point F = (0, 0.5) B(3, −5) E C(5, 2)
Area of ∆ABC = { 3𝑝 − 3𝑝 − 2}
Area of a triangle = A = { 𝑥 1(y2-y3) + x2(y3-y1) + x3(y1-y2) }
= { 3.22 -3.2-2}
Area of ∆ABC = { −5(-5-2) + 3(2-(-1)) + 5(-1-(-5)) }
= { 12 - 6 - 2}
= { −5(-7) + 3(3) + 5(4) }
= { 4}
= { 35 + 9 + 20 } =2
= { 64 } ∆≠0
Area of a ∆ABC = 32 Sq.units ∴ The points are non-collinear when P =2.
𝟏
Area of a triangle = A = { 𝑥 1(y2-y3) + x2(y3-y1) + x3(y1-y2) } When P =
𝟐

Area of ∆DEF = { −1(0.5-(-1.5)) + 0(-1.5-(-3)) + 4(-3-(0.5)) } Area of ∆ABC = { 3𝑝 − 3𝑝 − 2}

= { −1(2) + 0 + 4(-3.5) } = { 3.( )2 -3.( ) - 2}


= {−2 − 14 } = { + - 2}
= { −16} = { }
= -8 =
Area of ∆DEF = 8 Sq.units ∆ ≠0
∴ Area of ∆ABC = 4 x Area of ∆DEF 𝟏
∴ 𝐓𝐡𝐞 𝐩𝐨𝐢𝐧𝐭𝐬 𝐚𝐫𝐞 𝐧𝐨𝐧 − 𝐜𝐨𝐥𝐥𝐢𝐧𝐞𝐚𝐫 𝐰𝐡𝐞𝐧 𝑝 =
𝟐

3) Find the area of the triangle whose vertices are (𝑝+1, 1), (2𝑝+1, 3) and
4) In ∆𝐴𝐵𝐶 (3,1), (5,6) and (−3,2) are midpoints of 𝐴𝐵,𝐵𝐶 and 𝐶𝐴
(2𝑝+2, 3𝑝). And also verify whether the points are collinear when 𝑝 = 2 or
respectively. Then find coordinates of 𝐴, and 𝐶. 𝟑− Model-1:2020
𝟏
𝑝= A(𝑝+1, 1) 𝟑−MODEL-2020
Ans : A(x1, y1)
𝟐

D (3,1) F(−3,2)
B(2𝑝+1, 3) C(2𝑝+2, 3𝑝)

Ans : Area of a triangle = A = { 𝑥 1(y2-y3) + x2(y3-y1) + x3(y1-y2) }


Area of ∆ABC = { (𝑝 + 1)(3-3p) + (2p+1)(3p-1) + (2p+2)(1-3) } B(x2, y2) E(5,6) C(x3, y3)
Sri.Veerendra.H.M. Asst.Teacher. G.P.U.C. Megaravalli. Thirthahalli Tq Ph: 8762624683 Page-81 Sri.Veerendra.H.M. Asst.Teacher. G.P.U.C. Megaravalli. Thirthahalli Tq Ph: 8762624683 Page-82
Midpoint formula = P(x, y) = ( , ) 5) In the figure, the vertices of Δ ABC are A ( 0, 6 ), B ( 8, 0 ) and C ( 5, 8 ).

Midpoint of AB = (3, 1) = ( , ) If CD ⊥ AB, then find the length of altitude CD. 𝟑− SEPT-2020
Ans :
= 3 and =1
x1 y1 x2 y2 x3 y3
x1 + x2 = 6 -------------------(1)
0 6 8 0 5 8
y1 + y2 = 2 -------------------(2)

Midpoint of BC = (5, 6) = ( , )
Area of a triangle = A = { 𝑥1(y2-y3) + x2(y3-y1) + x3(y1-y2) }
= 5 and =6
x2 + x3 = 10 -------------------(3) Area of the triangle ABC = { 0(0-8) + 8(8-6) + 5(6-0) }
y2 + y3 = 12 -------------------(4) = { 0 + 16 + 30 }
Midpoint of CA = (-3, 2) = ( , ) = 𝑥 46
= -3 and =2 Area of triangle ABC = 23 Sq.units.
x1 + x3 = -6 Length of AB : d = (𝑥 − 𝑥 ) + (𝑦 − 𝑦 )
x3 = -6- x1 ------------------(5) d = (8 − 0) + (0 − 6)
y1 + y3 = 4
y3 = 4 - y1 -------------------(6) d = (8) + (−6)
Substitute the value of x3 and y3 in eqn(3) and eqn(4) resp. we get, d = √64 + 36
x2 -6 - x1 = 10 d = √100
- x1 + x2 = 16 ------------------ (7) AB = d = 10 cm
y2 + 4 - y1 = 12 Area of triangle ∆ABC = x base x height
- y1 + y2 = 12-4
23 = x AB x CD
- y1 + y2 = 8 ---------------(8)
Add Eqn(1) and Eqn(7) we get, Add Eqn(2) and Eqn(8) we get, 23 = x 10 x CD
x1 + x2 = 6 y1 + y2 = 2 23 = 5 x CD
- x1 + x2 = 16 - y1 + y2 = 8 ∴ Length of altitude 𝐶D = 4.6 cm
2x2 = 22 2y2 = 10 6) Show that the triangle whose vertices are A ( 8, – 4 ), B ( 9, 5 ) and C ( 0, 4 ) is
x2 = 11 y2 = 5 an isosceles triangle. 3-SEPT-2020
Substitute x2 = 11 in Eqn (1) Substitute y2 = 5 in Eqn (2) A(8, -4)
x1 + 11 = 6 y1 + 5 = 2
x1 = -5 y1 = -3
x3 = -6- (-5) y3 = 4 – (-3)
x3 = -6+5 y3 = 4 + 3 B(9, 5) C(0, 4)
x3 = -1 y3 = 7 Ans : Distance formula = d = (𝑥 − 𝑥 ) + (𝑦 − 𝑦 )
∴ Co-ordinates of ∆ 𝐴𝐵𝐶 = (-5, -3) , (11, 5) and (-1, 7) AB = d = (9 − 8) + (5 − (−4)) = √1 + 9 = √1 + 81 = √82

Sri.Veerendra.H.M. Asst.Teacher. G.P.U.C. Megaravalli. Thirthahalli Tq Ph: 8762624683 Page-83 Sri.Veerendra.H.M. Asst.Teacher. G.P.U.C. Megaravalli. Thirthahalli Tq Ph: 8762624683 Page-84
BC = d = (9 − 0) + (4 − 5) = 9 + (−1) = √81 + 1 = √82 Co-ordinates of D : ( , ) = (2, 2) [D is the midpoint of BC]
CA = d = (0 − 8) + (4 − (−4)) = (−8) + 8 = √64 + 64 = √128 G divides AD in the ratio 2 : 1.
Here, AB = BC = √82 ≠ CA G(x,y) = ,
∴∆ ABC is an isosceles triangle.
( ) ( ) ( ) ( )
G(x,y) = [ , ]
7) A( 0, 5 ), B( 6, 11 ), and C(10 , 7 ) are vertices of ∆ABC. D and E are mid
G(x,y) = [ , ]
points of AB and AC. Find the area of ∆ADE 3-PREP-2020
G(x,y) = [ , ]
Ans : Midpoint formula : P(x,y) = ( , )
G(x,y) = ( 3, 4 )
Coordinates of D : ( , ) = (3, 8) ∴ Co-ordinates of G : ( 3, 4 )
Coordinates of E : ( , ) = (5, 6)
9) Find the area of the triangle ABC whose co-ordinates are A (4, -6),
B(3, -2) and C(5, 2) and find the length of the median AD. 3-Model-1-2021
Area of a triangle A = { 𝑥1(y2-y3) + x2(y3-y1) + x3(y1-y2) } Ans : A(4, -6)
x1 y1 x2 y2 x3 y3
0 5 6 11 10 7
Area of ∆ADE = [0(11-3) + 3(6 − 5) + 5(5 − 8)]
= [0 + 3(1) + 5(−3)]
B(3, -2) D C(5, 2)
= [0 + 3 − 15] Mid point formula : P(x,y) = [ , ]
= [−12] Co-ordinates of mid point of BC : D(x, y) = [ , ]
= (−6)
D(x, y) = [ , ]
∴ Area of the triangle = 6 Sq.units
D(x, y) = [ 4 , 0]
8) Let A ( 5 , 8), B ( 0, -1 ) and C ( 4, 5 ) be the vertices of ∆ABC. The median
Length of median AD :
from A meets BC at D. The point ‘G’ on AD divides in the ratio
Distance formula =d = (𝑥 − 𝑥 ) + (𝑦 − 𝑦 )
AG : GD = 2 : 1. Then find the co-ordinates of ‘G’. 3-PREP-2020
d = (4 − 4) + (0 − (−6))
d = 0 + (6)
d = √36
d = 6 units.
Area of the triangle ABC :
x1 y1 x2 y2 x3 y3
4 -6 3 -2 5 2
Ans : Mid point formula : P(x,y) = [ , ]

Sri.Veerendra.H.M. Asst.Teacher. G.P.U.C. Megaravalli. Thirthahalli Tq Ph: 8762624683 Page-85 Sri.Veerendra.H.M. Asst.Teacher. G.P.U.C. Megaravalli. Thirthahalli Tq Ph: 8762624683 Page-86
Area of the triangle = A = { 𝑥 1(y2-y3) + x2(y3-y1) + x3(y1-y2) } 8) Find the distance between the points A ( 2, 6 ) and B ( 5, 10 ) by using
distance formula. Ans : d = 5 Units March-2022
= { 4(-2-2) + 3(2-(-6)) + 5(-6-(-2))}
9) Find the distance between the points P ( 2, 3 ) and Q ( 4, 1 ) using distance
= { 4(-4) + 3(2+6) + 5(-6+2) } formula. Ans. : d = 2√2 Units June-2022
10) Find the co-ordinates of the point P, which divides the line joining A(0, 0)
= { 16 + 24 - 20 }
and B(5, 10) in the ratio of 2:3
= { 20 } Ans : Co-ordinates of ‘P’ = P(x, y) = (2, 4)
∴ Area of the ∆ABC = 10 Sq.units 11) Find the co-ordinates of point which divides the line segment joining the
10) A circle whose centre is at P ( 2, 3 ) passes through the points A ( 4, 3 ) and points A ( 4, – 3 ) and B ( 8, 5 ) in the ratio 3 : 1 internally. PREP-2022
B ( x, 5 ). Then find the value of ‘x’. June-2022 Ans. : Co-ordinates of the point P(x, y) = ( 7, 3)
Ans. : PA = PB
12) Find the coordinates of the point on the line segment joining the points
Distance Formula = d = (𝑥 − 𝑥 ) + (𝑦 − 𝑦 ) A ( – 1, 7 ) and B ( 4, – 3 ) which divides AB internally in the ratio 2 : 3.
(4 − 2) + 0 = (𝑥 − 2) + (5 − 3) Ans : Coordinates of the point = P(x, y) = ( 1, 3)
22 = (x -2 )2 + 22 13) Find the coordinates of the points of “trisection” of the line joining the
(x -2 )2 = 0 points (6, –2) and (10, 8). Model-2022
x=2 22 4
Ans : Co-ordinates of point ‘P’ = P(x, y) = ,
3 3
EXERCISE: 14
Co-ordinates of point ‘Q’ = Q(x, y) = 6,
3
1) Find the coordinates of the mid-point of the line segment joining the 14) Find the area of a triangle ABC whose vertices are A(2, 2) B(3,4) and C (–1,3).
points (2,3) and (4,7) sept ∶ 2020 Ans : Model-2022
Ans : ∴ Co-ordinates of the midpoint = (3 , 5) ∴ Area of a triangle = A = 3.5 sq.units
2) Find the coordinates of the mid-point of the line segment joining the points 15)Find the area of triangle PQR with vertices P ( 0, 4 ), Q ( 3, 0 ) and R ( 3, 5 ).
P( 3, 4 ) and Q( 5, 6 ) by using ‘mid-point’ formula. March-2022 Ans. : Area of a triangle = A = 7.5 Sq. units March-2022
Ans : Coordinates of the mid-point = (4 , 5) 16) Find the value of ‘k’. If the co-ordinates of the points A(2, -2), B(-4, 2)
3) Find the distance between the points A( 8, 3 ) and B( 2, 11 ) using formula. and C(-7, k) are collinear. 3-Model-1-2021
Ans : d = 10 units PREP-2020
Ans : k = 4
4) Find the distance between the co-ordinate of the points A(2, 3) and
17) Find the value of ‘k’ if the points A(7, -2), B(5, 1) and 𝐶(3, k) are collinear.
B(10, -3). Ans : d = 10 units 2-Model-1
Ans : k = 4 PREP-2022
5) Find the distance between the points A(3, 6) and B(5, 7) using distance 18) Find the value of ‘k’ if the points P ( 2, 3 ), Q ( 4, k ) and R ( 6, – 3 ) are
formula. Ans : d = √5 units 2-Model-2 collinear. Ans. : k = 0. June-2022
6) Find the distance between the points (3, 1) and (6, 2) using distance formula. 19) If A(-5, 7), B(-4, -5), C(-1, -6) and D(4, 5) are the vertices of a quadrilateral,
Ans : d = √10 units Model-2022 find the area of the quadrilateral ABCD. Ans. : Area = 72sq.units
7) Find the distance between the points (2, 3) and (6, 6) using distance formula. 20) Show that the points (1, 7), (4, 2), (-1, -1) and (-4, 4) are the vertices of a
Ans : d = 5 Units PREP-2022 square.

Sri.Veerendra.H.M. Asst.Teacher. G.P.U.C. Megaravalli. Thirthahalli Tq Ph: 8762624683 Page-87 Sri.Veerendra.H.M. Asst.Teacher. G.P.U.C. Megaravalli. Thirthahalli Tq Ph: 8762624683 Page-88
UNIT-8. REAL NUMBERS : ONE MARK QUESTIONS :
Euclid’s Division Lemma : Given positive integers a and b, there exist unique 1) State Euclid’s division lemma. MODEL : 2018-19

integers q and r satisfying a = bq + r, 0 ≤ 𝑟 < 𝑏. Ans. : Given positive integers ‘a’ and ‘b’, there exist unique integers q and r

The fundamental theorem of Arithmetic : Every composite number can be satisfying a = bq + r, 0 ≤ r < b.
2) Express 140 as a product of its prime factors. MODEL : 2018-19
expressed as a product of primes, and this factorization is unique, apart
Ans. :
from the order in which the prime factors occur.
* For any two positive integers ‘a’ and ‘b’, H.C.F.(a, b) x L.C.M.(a, b) = a x b. 2 140 140 = 2x2x5x7
2 70 140 = 22x51x71
* Let x = be a rational number, such that the prime factorisation of q is of the 5 35
n m
form 2 5 , where n, m are non-negative integers. Then x has a decimal 7 7
expansion which terminates. 1
* Let x = be a rational number, such that the prime factorisation of q is not of 3) Without actually performing the long division, write the decimal expansion
𝟑𝟓
the form 2n5m, where n, m are non-negative integers. Then x has a decimal of
𝟓𝟎
PREP-2019
expansion which is non-terminating repeating(recurring). Ans. : = = = 0.70

MULTIPLE CHOICE QUESTIONS : 4) Compare 17 = 6×2+5 to Euclid’s Division Lemma 𝑎 = 𝑏𝑞 +𝑟, then write
the remainder ? March-2019
1) The HCF of 95 and 152 is PREP-2019
Ans. : 5
A) 57 B) 38 C) 19 D) 8
5) Write 96 as the product of its prime factors. June-2019
2) If a and b are any two positive integers then HCF ( a, b ) × LCM ( a, b ) is
Ans. :
equal to APRIL : 2019
2 96
𝐴) 𝑎 +𝑏 𝐵) 𝑎 −𝑏 𝐶) 𝑎 ×𝑏 𝐷)
2 48
3) The HCF of 72 and 120 is 24, then their LCM is JUNE : 2019 2 24
𝐴) 36 𝐵) 720 𝐶) 360 𝐷) 72 2 12
4) In the following numbers, irrational number is MODEL1 : 2019-20 2 6
3 3 96 = 2x2x2x2x2x3
A) √16 - √9 B) C) 0.3333.......... D) 2 + √𝟑
1 96 = 25 x 3
5) The product of prime factors of 120 is MODEL2 : 2019-20
6) In Euclid’s division lemma, if a = 3q + r, then write all the possible values
3 2 1 2 1 1 3 1 2 3 1 1
A) 2 x 3 x 5 B) 2 x 3 x 5 C) 2 x 3 x 5 D) 2 x 3 x 5
of r. MODEL-1: 2019-20
6) If 180 = 2x x 32 x 5 then the value of ‘x’ is PREP : 2020
Ans. : r = 0, 1 and 2
A) 1 B) 2 C) 3 D) 4
7) The LCM of 24 and 36 is 48 and hence find their HCF. MODEL-2: 2019-20
7) The HCF of (12,15) is 3. Then LCM of (12,15) is
Ans. : LCM of (24 , 36) x HCF of (24 , 36) = 24 x 36
𝐴) 60 𝐵) 45 𝐶) 36 𝐷) 90
48 x HCF = 24 x 36

Sri.Veerendra.H.M. Asst.Teacher. G.P.U.C. Megaravalli. Thirthahalli Tq Ph: 8762624683 Page-89 Sri.Veerendra.H.M. Asst.Teacher. G.P.U.C. Megaravalli. Thirthahalli Tq Ph: 8762624683 Page-90
HCF = = 18 Rearranging this equation
𝟐𝟑 √3 =
8) Express the denominator of in the form of 2n x 5m and state whether the
𝟐𝟎
Since p and q are integers we get ( ) is rational
given fraction is terminating or non-terminating repeating decimal.June-2020
So √3 is rational.
Ans. , Factorising the denominator, 20 = 2x2x5 = 22 x 51
But this contradicts the fact that √3 is irrational.
Here , the factors of 20 is of the form 2n x 5m . This contradiction has arisen because of our incorrect assumption that 5 - √3
∴ It is a terminating decimal expansion. is rational.
9) Find the H.C.F. of the smallest prime number and the smallest composite ∴ 5 - √3 is irrational.
number. SEPT ∶ 2020
3) Prove that √𝟐 + √𝟑 is an irrational number. MODEL-1 : 2019-20
Ans. : Smallest prime number = 2
Smallest composite number = 4 Ans. : Let us assume, contrary that √2 + √3 is rational number.
∴ H.C.F. of ( 2, 4 ) is 2 √2 + √3 = where p, q ∈ z , q ≠ 0
10) Convert 0.375 into fractional form. √2 = - √3

Ans. : 0.375 = = Squaring on both sides, we get


2 2
( - √3 ) = (√2 )
TWO MARKS QUESTIONS : + 3 – 2( )( √3 ) = 2
1) Prove that 3 + √5 is an irrational number. MODEL : 2018-19& March-2019
+ 3 –2 = 2( )( √3 )
Ans. : Let us assume 3 + √5 is a rational number
+ 1 = 2√3 ( )
3 + √5 = where p, q ∈ z , q ≠ 0
√5 = − 3 = 2√3 ( )
( )
Rearranging this equation = √3
√5 = ( )
= √3
Since p and q are integers we get ( ) is rational ( )
Since p and q are integers, is rational number.
So √5 is rational.
So √3 is a rational number.
But this contradicts the fact that √5 is irrational.
But this contradicts the fact that √3 is irrational.
This contradiction has arisen because of our incorrect assumption that 3 + √5
This contradiction has arisen because of our incorrect assumption that
is rational.
√2 + √3 is rational.
∴ 3 + √5 is irrational.
∴ √2 + √3 is irrational.
2) Prove that 5−√𝟑 is an irrational number PREP-2019

Ans. : Let us assume (5 - √3) is a rational number THREE MARKS QUESTIONS :


5 - √3 = where p, q ∈ z , q ≠ 0
1) Prove that √𝟓 is an irrational number. June-2020
√3 = 5 - Ans. : Let us assume, contrary that √5 is rational number.

Sri.Veerendra.H.M. Asst.Teacher. G.P.U.C. Megaravalli. Thirthahalli Tq Ph: 8762624683 Page-91 Sri.Veerendra.H.M. Asst.Teacher. G.P.U.C. Megaravalli. Thirthahalli Tq Ph: 8762624683 Page-92
We can find integers a and b (b ≠ 0 ) such that √5 = ii) LCM of 15 and 20 :
Suppose a and b have a common factor other than 1, then we can divide 2 15, 20
by the common factor and assume that a and b are co-prime. 2 15, 10
So, b√5 = a 3 15, 5
Squaring on both sides, we get 5 5, 5
5b2 = a2 1, 1
a2 is divisible by 5, then a is also divisible by 5. ∴ LCM of 15 and 20 = 2x2x3x5 = 60
So, we can write a = 5c for some integer c.
Substituting for a, we get 3) Find the HCF of 24 and 40 using Euclid’s division algorithm. Hence find
5b2 = (5c)2 the LCM of HCF of (24, 40) and 20. June-2020
5b2 = 25c2 Ans. : a = 40 , b = 24
i.e. b2 = 5c2 According to Euclid’s Division algorithm, a = bq + r 0≤𝒓 <𝑏
Means b2 is divisible by 5, then ‘b’ is also divisible by 5.
1
∴ ‘a’ and ‘b’ have atleast 5 as a common factor.
24 40
But this contradicts the fact that a and b are co-prime.
24
This contradiction has arisen because of our incorrect assumption that √5
16 40 = (24 x 1) + 16
is rational.
∴ √5 is irrational. 1
16 24
2) Find the HCF of 135 and 75 by the prime factorization method. Hence find
16
the LCM of HCF of (135, 75) and 20. PREP-2020 08 24 = (16 x 1) + 8
Ans. : i) HCF of ( 135, 75) : 2
3 135 8 16
16
3 45
00 16 = (8 x 2) + 0
3 15
5 5
∴ HCF of 24 and 40 = 8
1
LCM of 8 and 20 :
3 75
2 8, 20
5 25
2 4, 10
5 5
2 2, 5
1
5 1, 5
135 = 3 x 3 x 3 x 5 = 33 x 51
1, 1
75 = 3 x 5 x 5 = 31 x 52
∴ LCM of (8, 20) = 2 x 2 x 2 x 5 = 8 x 5 = 40
∴ HCF of ( 135, 75) = 3x5 = 15

Sri.Veerendra.H.M. Asst.Teacher. G.P.U.C. Megaravalli. Thirthahalli Tq Ph: 8762624683 Page-93 Sri.Veerendra.H.M. Asst.Teacher. G.P.U.C. Megaravalli. Thirthahalli Tq Ph: 8762624683 Page-94
4) Find L.C.M. of H.C.F. ( 306, 657) and 12. SEPT-2020

Ans. : i) H.C.F. of ( 306, 657)


UNIT-9. POLYNOMIALS:
2 306 Degree of the Polynomial : The highest power of x in p(x) is called the degree of
3 153 the polynomial.
3 51 * A polynomial of degree 1 is called a linear polynomial. Ex : 2x + 3
17 17 Standard form : p(x) = ax + b , a & b ∈ R a≠0
1 * A polynomial of degree 2 is called a quadratic polynomial. Ex : 3x2 -4x+5
3 657 Standard form : p(x) = ax2 + bx + c , a,b & c ∈ R a ≠ 0
3 219 * A polynomial of degree 3 is called a cubic polynomial. Ex: 4x3 +2x2- 4x +1
73 73 Standard form : p(x) = ax3 + bx2 + cx + d , a,b,c & d ∈ R a≠0
1
Zeroes of a polynomial : A real number ‘k’ is said to be a zero of a polynomial
306 = 3 x 3 x 2x 17 p(x), if p(k) = 0.
657 = 3 x 3 x 73 * The zeroes of a polynomial p(x) are precisely the x-coordinates of the points,
HCF of ( 306, 657) = 3 x 3 = 9 where the graph of y = p(x) intersects the x-axis.
* If 𝛼 and 𝛽 are the zeroes of the quadratic polynomial ax2 + bx + c, then
ii) L.C.M. of 9 and 12 :
Sum of the zeroes = 𝛼 + 𝛽 =
3 9, 12
Product of the zeroes = 𝛼 𝑥 𝛽 =
3 3, 4
4 1, 4 Division algorithm of polynomials : If p(x) and g(x) are any two polynomials
1, 1 with g(x) ≠ 0, then there are polynomials q(x) and r(x) such that
∴ LCM of 9 and 12 = 3x3x4 = 36 p(x) = g(x). q(x) + r(x) where r(x) = 0 or degree r(x) < degree g(x).

EXERCISE : MULTIPLE CHOICE QUESTIONS :


1) Prove that 5+√𝟑 is an irrational number. June-2019
1) The degree of the polynomial p(𝑥) = 𝑥2 −3𝑥 +4𝑥3 − 6 is Model : 2018-19
2) Prove that 7 + √𝟓 is an irrational number. MODEL-2 : 2019-20 𝐴) 2 𝐵) 1 𝐶) 3 𝐷) 6
3) Prove that √𝟑 is an irrational number. PREP-2020 & SEPT-2020 2) The quadratic polynomial having zeroes -2 and 4 is PREP-2019
2 2 2 2
A) x + 2x + 8 B) x – 2x – 8 C) x + 2x – 8 D) x – 2x + 8
3) In the given graph y = P(x), the number of
zeroes is June-2019
𝐴) 4 𝐵) 3 𝐶) 2 𝐷) 7
4) If one of the zeroes of a polynomial p(𝑥) = 𝑥2- 𝑥 +𝑘 is
2, then the value of k is Model-1: 2019-20
A) 2 B) -2 C) -6 D) 6

Sri.Veerendra.H.M. Asst.Teacher. G.P.U.C. Megaravalli. Thirthahalli Tq Ph: 8762624683 Page-95 Sri.Veerendra.H.M. Asst.Teacher. G.P.U.C. Megaravalli. Thirthahalli Tq Ph: 8762624683 Page-96
5) In the figure a polynomial y = p ( x ) is represented 7) Find the sum of the zeroes of the polynomial P(x) = 2x2 – 9x + 10. PREP ∶ 2020
through a graph, the number of zeroes of the b ( 9) 𝟗
Ans. : Sum of the zeroes : α + β = = =
polynomial is. Prep ∶ 2020 a 2 𝟐
A) 4 B) 2 C) 3 D) 1 8) If, P(𝑥) = 2𝑥3 +3𝑥2 -11𝑥 +6 then find the value of P(1). Sept : 2020
3 2
6) The degree of a linear polynomial is June-2020 Ans. : P(𝑥) = 2𝑥 +3𝑥 -11𝑥 +6
A) 0 B) 1 C) 2 D) 3 P(1) = 2(1)3 +3(1)2 -11(1) + 6
7) In the given graph, the number of zeroes of P(1) = 2 + 3 -11 + 6
P(1) = 0
the polynomial y = p ( x ) is SEPT ∶ 2020
9) Sum and product of the zeroes is -3 and 2. Find the quadratic polynomial.
A) 3 B) 5 C) 4 D) 2 Ans. : PREP-2019
8) If sum of the zeroes of the polynomial p(𝑥) = 𝑘𝑥2 +2𝑥 +3𝑘 is equal to Sum of the zeroes of the polynomial = -3
product of zeroes of the polynomial, the value of ′𝑘′ is Product of the zeroes of the polynomial = 2
𝟐
𝐴) 𝐵) 𝐶) 𝐷) Sum of the zeroes of the polynomial = = −( )
𝟑
Product of the zeroes of the polynomial = =
ONE MARK QUESTIONS :
On comparing
1) Find the number of zeroes of the polynomial p(𝑥) from the graph given ∴a=1 ,b=3 &c=2
Y Model : 2018-19
Required polynomial = p(𝑥) = 𝑎𝑥2 +𝑏𝑥 +𝑐
X1 o X p(𝑥) = 𝑥2 +3𝑥 +2
10) Find the zeroes of the polynomial, P( x ) = x2 − 3 . April-2019
Ans. : P( x ) = 0,
Y1 p(𝑥) = 𝑥2 −3
Ans. : 4 𝑥2 –(√3)2 = 0
( x + √3 ) ( x − √3 ) = 0 [ ∵ a2 – b2 = (a + b)(a – b) ]
2) The following graph represents the polynomial ( x + √3 ) = 0 and ( x - √3 ) = 0
y = P( x ). Write the number of zeroes that P( x ) x = -√3 and x = + √3
has. June-2020 TWO MARKS QUESTIONS :
Ans.: 3 1) Write the general form of the following : Model2: -2019-20
a) Linear polynomial
3) Write the degree of the polynomial P ( x ) = 2x2 –x3 + 5 April-2019
b) Cubic polynomial
Ans. : 3
4) Find the degree of the polynomial p(𝑥) = 𝑥3 +2𝑥2 −5𝑥 −6 June-2019 Ans. : General form of Linear polynomial is P(x) = ax + b, a≠ 0
Ans. : 3 General form of Cubic polynomial is P(x) = ax3 + bx2 + cx + d, a≠ 0
5) Write the number of zeroes of the polynomial p(𝑥) = 𝑥3 +2𝑥2 +𝑥 + 6 2) Find the zeroes of polynomial P(x) = 6x2 - 3 -7x Model: 2018-19
Ans. : 3 Model-1: 2019-20
Ans. : P(x) = 6x2 -7x -3 = 0 -9x
6) Write the degree of the polynomial p(𝑥) = 3𝑥3 −4𝑥2 +5𝑥4 −3𝑥 + 4. 6x2 - 9x +2x - 3 = 0 6x2
Ans. : 4 Model-2 : 2019 3x(2x – 3) +1(2x -3) = 0 +2x
(2x – 3)(3x + 1) = 0 -7x
Sri.Veerendra.H.M. Asst.Teacher. G.P.U.C. Megaravalli. Thirthahalli Tq Ph: 8762624683 Page-97 Sri.Veerendra.H.M. Asst.Teacher. G.P.U.C. Megaravalli. Thirthahalli Tq Ph: 8762624683 Page-98
(2x – 3) = 0 and (3x + 1) = 0 6) The sum and product of the zeroes of a quadratic polynomial
2x = 3 and 3x = -1 P(x) = ax2 + bx + c are – 3 and 2 respectively. Show that b + c = 5a.
3 1 APRIL-2019
x = and x =
2 3 Ans. : Let α and β are the zeroes of the quadratic polynomial P ( x )
𝟑 𝟏
∴ the zeroes are and α+β=–3
𝟐 𝟑
b
3) Find the zeroes of polynomial p(𝑥) = 4𝑥2 −4𝑥 −3 - =–3
a
Ans. : −6𝑥 – b = – 3a
p(𝑥) = 4𝑥 −4𝑥 -3
2 -12𝑥 2 b = 3a ........ (i)
p(𝑥) = 4𝑥 −6𝑥 +2𝑥 -3
2 +2𝑥 αβ = 2
c
p(𝑥) = 2𝑥 (2𝑥 -3) + 1(2𝑥 -3) −4𝑥 =2
a
p(𝑥) = (2𝑥 -3)(2𝑥 + 1) c = 2a ........ (ii)
(2𝑥 -3) = 0 and (2𝑥 + 1) = 0 (i) + (ii) gives
2𝑥 = 3 and 2𝑥 = -1 b + c = 3a + 2a
𝟑 𝟏 b + c = 5a.
𝑥 = and 𝑥 = 7) Sum and product of the zeroes of a quadratic polynomial P(x) = ax2 + bx -4
𝟐 𝟐
4) Find the zeroes of polynomial x2 + 7x + 10 and verify the relationship 𝟏
are and – 1 respectively. Then find the values of a and b. JUNE-2019
𝟒
between the zeroes and the coefficients. PREP-2019 1
Ans : P(x) = 0 Ans. : Sum of the zeroes of the polynomial =
4
x2 + 7x + 10 = 0 Product of the zeroes of the polynomial = -1
x2 + 5x + 2x + 10 = 0 b 1
x(x + 5) + 2(x + 5) = 0 Sum of the zeroes of the polynomial = = −( 4 )
a
(x + 5)(x + 2) = 0 c 4
Product of the zeroes of the polynomial = =
The zeroes are x = -5 and x = -2 a 4
7
Sum of zeroes = -5 + (-2) = -7 = =
b On comparing
1 a
c ∴ a = 4 , b = -1
Product of zeroes = (-5)x(-2) = 10 =
a
𝟏 8) Find the value of k, in which one of its zeroes is (–4) of the polynomial
5) Find the quadratic polynomial whose sum and product of zeroes are and
𝟒 p(𝑥) = 𝑥2 −𝑥 −(2𝑘 +2) JUNE-2019
-1 respectively. Model-2018-19
Ans. : Ans. :
1 p(𝑥) = 𝑥2 −𝑥 −(2𝑘 +2)
Sum of the zeroes of the polynomial =
4 One of its zeroes is – 4
Product of the zeroes of the polynomial = -1 ⇒ p(𝑥) = 0 for x = - 4
b 1 4
Sum of the zeroes of the polynomial = = −( 4 ) = ∴ (-4)2 –(-4) −(2𝑘 +2) = 0
a 4
On comparing 16 +4 −(2𝑘 +2) = 0
∴ a = 4 , b = -1 & c = - 4 (2𝑘 +2) = 20
Required polynomial = p(𝑥) = 𝑎𝑥2 +𝑏𝑥 +𝑐 2𝑘 = 20 -2
p(𝑥) = 4𝑥2 -𝑥 -4 2𝑘 = 18
𝑘=9
Sri.Veerendra.H.M. Asst.Teacher. G.P.U.C. Megaravalli. Thirthahalli Tq Ph: 8762624683 Page-99 Sri.Veerendra.H.M. Asst.Teacher. G.P.U.C. Megaravalli. Thirthahalli Tq Ph: 8762624683 Page-100
9) If 𝜶 and 𝜷 are the zeroes of the polynomial P(x) = 3x2 - 12x + 15, Product of the zeroes = 𝛼 𝑥 𝛽 =
find 𝜶𝟐 + 𝜷𝟐 Model2: -2019-20
b 12 𝛼𝑥𝛽=
Ans. : Sum of the zeroes of the polynomial = 𝛼 + 𝛽 = = −( )=4
a 3
From given, 𝛼 + 𝛽 = 𝑥 (𝛼 𝑥 𝛽)
c 15
Product of the zeroes of the polynomial = 𝛼 x 𝛽 = = =5
a 3 3= x
(𝛼 + 𝛽)2 = 𝛼 + 𝛽 + 2 𝛼 x 𝛽
42 = 𝛼 + 𝛽 + 2x5 3=
16 = 𝛼 + 𝛽 + 10 ∴ k = 12
𝛼 + 𝛽 = 16 – 10 12) Find the quotient and remainder when p(x) = x4 - 3x2 + 4x + 5 is divided
∴ 𝜶𝟐 + 𝜷𝟐 = 6 by g(x) = x2 – x + 1 PREP-2019
Ans. :
10) If one zero of the polynomial p(𝑥) = 𝑥2 −6𝑥 + k is twice the other then find p(𝑥) = 𝑥4 −3𝑥2 +4𝑥 +5
the value of k. JUNE-2020
g(𝑥) = 𝑥2 -𝑥 +1
Ans. : P(𝑥) = 𝑥2 −6𝑥 + k
q(x) = ? & r(x) = ?
Let 𝛼 and 𝛽 be zeroes of P(𝑥).
𝑥2 + 𝑥 - 3
From given, 𝛽 = 2 𝛼
𝑥2 -𝑥 +1) 𝑥4 − 3𝑥2 +4𝑥 +5
Sum of the zeroes = 𝛼 + 𝛽 =
(-)𝑥4-(+)𝑥3 +(-)𝑥2
( )
𝛼 + 2𝛼 = 𝑥3 - 4𝑥2 + 4𝑥 + 5
3𝛼=6 (-)𝑥3 -(+)𝑥2 +(-)𝑥

𝛼=2 -3 𝑥2 + 3 𝑥 + 5
-(+)3 𝑥2 +(-)3 𝑥 -(+)3
Product of the zeroes = 𝛼 𝑥 𝛽 =
+8
𝛼 𝑥 2𝛼 = ∴ Quotient = q(x) = 𝑥 + 𝑥 - 3 & Remainder = r(x) = 8
2

2𝛼 2 = k
13) Find the quotient and the remainder when P(x) = 3x3 + x2 + 2x +5 is
2 x (22) = k
divided by g ( x ) = x2 + 2x + 1. April-2019
∴ k=8 Ans. : p(𝑥) = 3𝑥3 +𝑥2 +2𝑥 +5
11) Find the value of k of the polynomial p(𝑥) = 2𝑥2 −6𝑥 + k , such that the g(𝑥) = 𝑥2 +2𝑥 +1
sum of zeros of it is equal to half of the product of their zeros. SEPT ∶ 2020 q(x) = ? & r(x) = ?
Ans. : P(𝑥) = 2𝑥2 −6𝑥 + k 3𝑥 -5
Let 𝛼 and 𝛽 be zeroes of P(𝑥).
𝑥2 +2𝑥 +1) 3𝑥3 +𝑥2 +2𝑥 +5
P(𝑥) = a𝑥2 + b𝑥 + c
(-)3𝑥3+(-)6𝑥2+(-)3𝑥
a = 2, b= -6 , c = k
-5𝑥2 -1𝑥 +5
Sum of the zeroes = 𝛼 + 𝛽 =
-(+)5𝑥2-(+)10𝑥 –(+)5
( )
𝛼+ 𝛽= 9 𝑥 +10
𝛼+ 𝛽=3 Quotient = q(x) = 3x – 5 Remainder = r(x) = 9x + 10
Sri.Veerendra.H.M. Asst.Teacher. G.P.U.C. Megaravalli. Thirthahalli Tq Ph: 8762624683 Page-101 Sri.Veerendra.H.M. Asst.Teacher. G.P.U.C. Megaravalli. Thirthahalli Tq Ph: 8762624683 Page-102
𝑥2 + 6𝑥 + 9
14) Find the quotient and remainder when p(𝑥) = 2𝑥2 +3𝑥 +1 is divided by
g(x) = x + 2. JUNE-2019
𝑥2 -2𝑥 +1) 𝑥4 + 4𝑥3 −2𝑥2 −12𝑥 +9
Ans. : (-)𝑥4-(+)2𝑥3+(-)𝑥2

p(𝑥) = 2𝑥2 +3𝑥 +1 6𝑥3 - 3𝑥2 - 12𝑥 +9


g(𝑥) = 𝑥 +2 (-)6𝑥3-(+)12𝑥2 +(-)6𝑥

q(x) = ? & r(x) = ? 9 𝑥2 - 18 𝑥 + 9


2𝑥 -1 (-)9 𝑥2 –(+)18 𝑥 +(-)9
0
𝑥 +2) 2𝑥2 +3𝑥 + 1
∴ Quotient = q(x) = 𝑥 + 6𝑥 + 9 & Remainder = r(x) = 0
2
(-)2𝑥2+(-)4𝑥
-1𝑥 + 1 THREE MARKS QUESTIONS :
-(+)1𝑥-(+)2
3 1) If 3 and -3 are two zeroes of the polynomial p(x) = x4 + x3 -11x2 - 9x + 18,
∴ Quotient = q(x) = 2x -1 & Remainder = 3 Then find the remaining two zeroes of the polynomial. Model-1: -2019-20
Ans. :
15) Find the polynomial of least degree that should be subtracted from 3 and -3 are two zeroes of the polynomial p(x) = x4 + x3 -11x2 - 9x + 18
p(𝑥) = x3-2𝑥2 +3𝑥 + 4 so that it is exactly divisible by g(𝑥) = 𝑥2 -3𝑥 + 1 (x +3) and (x -3) are the factors of x4 + x3 -11x2 - 9x + 18
Ans. : JUNE-2020
(x +3)(x -3) = x2 – 9. Divide x4 + x3 -11x2 - 9x + 18 by x2 – 9
P(𝑥) = x3-2𝑥2 +3𝑥 + 4. 𝑥2 + 𝑥 - 2
g(𝑥) = 𝑥2 -3𝑥 + 1 𝑥2 -9) 𝑥4 + x3 − 11𝑥2 -9𝑥 + 18
x+1 (-)𝑥4-(+)9𝑥2
𝑥 -3𝑥 + 1) x3-2𝑥2 +3𝑥 + 4
2
𝑥3 -2𝑥2 - 9𝑥 + 18
x3-3𝑥2 +𝑥 (-)𝑥3 -(+)9𝑥
(-) (+) (-)
-2 𝑥2 + 18
𝑥2 +2𝑥 + 4
-(+)2 𝑥2 +(-)18
(-)𝑥 –(+)3𝑥 +(-)1
2
0
2
Remainder 5x + 3 The zeroes of 𝑥 + 𝑥 – 2 are
∴ (5x + 3) is subtracted from x3-2𝑥2 +3𝑥 + 4. 𝑥2 + 2𝑥 - 𝑥 – 2 = 0
𝑥(𝑥 +2) -1(𝑥 +2) = 0
16) Find the quotient and remainder when p(𝑥) = 𝑥4 +4𝑥3 −2𝑥2 −12𝑥 +9 is (𝑥 +2) (𝑥 -1) = 0
divided by g(𝑥) = 𝑥2 −2𝑥 +1. (𝑥 +2) = 0 and (𝑥 -1) = 0
Ans. : 𝑥 = -2 and 𝑥 = 1
p(𝑥) = 𝑥4 +4𝑥3 −2𝑥2 −12𝑥 +9 ∴ The remaining zeroes are -2 and 1.
g(𝑥) = 𝑥2 -2𝑥 +1 2) If two zeroes of the polynomial p(x) = x3 + 2x2 - 9x - 18 are 3 and -3, find
q(x) = ? & r(x) = ? other zero of the polynomial P(x).

Sri.Veerendra.H.M. Asst.Teacher. G.P.U.C. Megaravalli. Thirthahalli Tq Ph: 8762624683 Page-103 Sri.Veerendra.H.M. Asst.Teacher. G.P.U.C. Megaravalli. Thirthahalli Tq Ph: 8762624683 Page-104
Ans.: UNIT-10. : QUADRATIC EQUATIONS.
3 2
3 and -3 are two zeroes of the polynomial p(x) = x + 2x - 9x - 18
Standard form of a quadratic equation : ax2 + bx + c = 0
(x +3) and (x -3) are the factors.(x +3)(x -3) = x2 – 9.
Divide p(x) = x3 + 2x2 - 9x - 18 by x2 – 9 Standard form of a pure quadratic equation: ax2 + c = 0
±√
x2 -9 x3 + 2x2 - 9x – 18 x+2 Roots of a quadratic equation (Shreedhar’s Formula) : x =
x3 - 9x Discriminant of a quadratic equation : ∆ = b2 – 4ac
0 + 2x2 - 0 -18
2 Sl.
(-)2x - (+)18 Discriminant Nature of roots
No.
0
01. b2 – 4ac > 0 Two distinct real roots
So (x + 2) is the third factor.
The another zero of p(x) = x3 + 2x2 - 9x - 18 is- 2 02. b2 – 4ac = 0 Two equal real roots

3) On dividing p(x) = 3x3 + x2 + 2x + 5 by polynomial g(x), the quotient and b2 – 4ac < 0 No real roots
03.
remainder obtained are ( 3x – 5 ) and ( 9x + 10 ) respectively g(x).
Ans. : p(x) = g(x).q(x) + r(x) PREP ∶ 2020 MULTIPLE CHOICE QUESTIONS :
( ) ( )
⇒ g(x) = 1) “The product of two consecutive positive integers is 30.” This can be
( )

⇒ g(x) =
𝟑𝐱 𝟑 𝐱𝟐 𝟐𝐱 𝟓 (𝟗𝐱 𝟏𝟎) expressed algebraically as April-2019
A) x ( x + 2 ) = 30 B) x ( x – 2 ) = 30 C) x ( x – 3 ) = 30 D) x ( x + 1 ) = 30
𝟑𝐱 𝟑 𝐱 𝟐 𝟕𝐱 𝟓
g(x) = 2) The sum of the squares of two consecutive odd numbers is 394. The
mathematical equation for the above statement is
x2 + 2x + 1
A. x2 +(x+1)2 =394 B. x2 +(x+2)2 =394
3𝑥 - 5) 3x3 + 𝑥2 -7x -5 2 2
C. (x+1) + (x+2) =394 D. x+(x+2)2 =394
(-)3𝑥
3
-(+)5𝑥2 3) Degree of a linear equation is JUNE ∶ 2020
𝐴) 0 𝐵) 1 𝐶) 2 𝐷) 3
6𝑥2 - 7𝑥 - 5
2- 4) The standard form of quadratic equation is : Model-2-2020
(-)6𝑥 (+)10x
2 2 2 2
A) ax – bx + c = 0 B) ax + bx + c = 0 C) ax – bx - c = 0 D) ax + bx - c = 0
3x – 5
(-)3x – (+)5
5) The standard form of 2x2 = x - 7 April-2022
2 2 2 2
0 A) 2x - x = -7 B) 2x + x -7 = 0 C) 2x - x + 7 = 0 D) 2x + x + 7 = 0
∴ g(x) = x2 + 2x + 1 6) When the quadratic equation 5x2 = 2(2x + 3) is expressed in the standard
form, the constant term obtained is Model-2-2020
(A) 5 (B) 6 (C) 4 (D) -6
7) The constant term in the quadratic equation 3x2 -3(2x-4)=0, after
reducing it to the standard form ax2 +bx+c=0 is Model-1-2020

Sri.Veerendra.H.M. Asst.Teacher. G.P.U.C. Megaravalli. Thirthahalli Tq Ph: 8762624683 Page-105 Sri.Veerendra.H.M. Asst.Teacher. G.P.U.C. Megaravalli. Thirthahalli Tq Ph: 8762624683 Page-106
A. 3 B. 4 C. -12 D. 12 20) The discriminant of the Quadratic equation px2 + qx + r = 0 is
8) The quadratic equation in the following, PREP-2022 2
A) q – 4pr
2
B) q + 4pr
2
C) p – 4pr
2
D) p + 4qr JULY-2021
A) x2 - 3x + 2 = 0 B) 2x + 3 = 0 C) x2 - 5x + 6 3
D) 2x + 7x + 1= 0 2
21) If the discriminant of quadratic equation is 𝑏 − 4𝑎𝑐 = 0 then the nature
9) The quadratic equation in the following,
of roots MODEL-2019
A) 3x + y – 8 = 0 B) x2 + 2x + 3 = x2 C) x2 + = 0 D) x + =0
𝐴) Real and distinct 𝐵) real and equal
10) The roots of the quadratic equation ax2 + bx + c = 0 are Model-II-2021 , JULY- 21
𝐶) no real roots D) Roots are unequal and irrational
A) x=
±√
B) x=
±√
22) The nature of the roots of the equation 2x2 – 4x – 3 = 0 is _____

√ √ A) real and distinct roots B) real and equal roots


C) x = D) x=
C) no real roots D) none of these
11) Roots of quadratic equation x2 + bx + c = 0 PREP ∶ 2020
23) The nature of the roots of the quadratic equation x2 -2x+1=0 are
±√ ±√
𝐴) x = 𝐵) x= A. real and equal B. real, rational and distinct
√ √ C. real, irrational and distinct D. complex
𝐶) x= 𝐷) x = 24) If the value of the discriminant of a quadratic equation is zero then the
12) The roots of the equation (x -3)(x + 2) = 0 are JULY-2021 nature of the roots are MODEL-2022
A) -3, 2 B) 3, -2 C) -3, -2 D) 3, 2 A) Real distinct and irrational B) Real and equal
13) If the sum of two consecutive integers is 27, then the integers are JULY-2021 C) Real distinct and rational D) Not real
2
A) 7 and 20 B) 13 and 14 C) 1 and 26 D) -13 and -14 25) If the roots of the quadratic equation x + 6x + k = 0 are equal, then the
14) One root of the quadratic equation (2x-3) (x+5)=0 is -5, then the other value of ‘k’ is: Model-1-2020
root is Model-1-2021 A) 9 B) -9 C) 8 D) 5
A. 5 B. C. D. 26) x ( x + 1 ) = 5 is a June-2022
𝟏
A) linear equation B) quadratic equation
15) x2 – 2x + 1= 0 then x + is equal to PREP-2019 C) cubic equation D) quadratic polynomial
𝐱
A) 1 B) 3 C) 2 D) 4
16) The roots of the quadratic equation 4x2 - 81 = 0 are Model-II-2021 Answers : 1.D, 2.B, 3.B, 4.B, 5.C, 6.D, 7.D, 8.A, 9.D, 10.A, 11.A, 12.B, 13.B, 14.C, 15.C,
16.D, 17.A, 18.A, 19.C, 20.A, 21.B, 22.A, 23.A, 24.B, 25.A, 26.B.
A. ± B. ± C. ± D. ±
ONE MARK QUESTIONS :
17) In finding the roots of the quadratic equation 6x2 − x − 2 = 0 by the
1) Write the standard form of quadratic equation.
method of factorisation, the middle term ‘−x’ can be written as
Model-1-2021, March-2022 & June-2022
A. 3x and −4x B. −3x and +4x C. −3x and −4x D. −5x and 4x
2 Ans. : Standard form of quadratic equation : a𝑥2 +b𝑥 + c = 0 , a ≠ 0
18) The roots of the quadratic equation x + 3x + 2 = 0 are Model-II-2020
A) -1 & -2 B) 1 & 2 C) -2 & -3 D) 2 & 3 2) Write the discriminant of a𝑥2 +b𝑥 + c = 0 MODEL-2019
2
19) If one root of the equation 2x2 + ax + 6 = 0 is 2, then the value of ‘a’ is Ans. : Discriminant (∆) = b - 4ac
3) Write the discriminant of 𝑝𝑥2 +𝑞𝑥 − 𝑟 = 0. MODEL-2020
(A) 7 (B) (C) -7 (D) JULY-2021 2
Ans. : Discriminant (∆) = b -4ac
Sri.Veerendra.H.M. Asst.Teacher. G.P.U.C. Megaravalli. Thirthahalli Tq Ph: 8762624683 Page-107 Sri.Veerendra.H.M. Asst.Teacher. G.P.U.C. Megaravalli. Thirthahalli Tq Ph: 8762624683 Page-108
∆ = q2 + 4pr x=
𝟓 √𝟏𝟕
and x =
𝟓 √𝟏𝟕
𝐱 𝟏 𝟏 𝟐 𝟐
4) Write = in the standard form of a quadratic equation. June ∶2020
𝟐 𝐱 3) Find the roots of the equation 2𝑥2 −5𝑥 +3 = 0 using formula.
Ans. : = Ans. : Here a = 2 , b = -5 & c = 3 April : 2019
x(x+1) = 2x1 ±√
x2 + x = 2 x=
∴ x2 + x – 2 = 0 x=
( )± ( )

5) Find the value of discriminant of the quadratic equation 2x2 –4x +3 = 0


±√
Ans. : Here a = 2, b = -4, c = 3 April-2019
x=
Discriminant(∆) = b2 - 4ac x=
±√
= ( -4 )2- 4x 2x 3
±
= 16 – 24 x=
∆ =–8
6) One root of the equation (x+4)(x+3) = 0 is -4 then, find the another root x= and x =

of the equation. SEPT ∶ 2020 x= and x =


Ans. : (x+4)(x+3) = 0 x=
𝟑
and x = 1
𝟐
One root is -4 then,
4) Find the roots of the equation 𝑥2 −3𝑥 −10 = 0 using formula.
Another root is : (x+3) = 0
Ans. : Here, a = 1 , b = -3 & c = -10 June : 2019
x = -3
±√
TWO MARKS QUESTIONS : x=
1) Write (𝑥 −2)2 + 1 = 2𝑥 + 3 in to standard form of quadratic equation. x=
( )± ( ) ( )

Ans. : April : 2019 ±√


2
x=
(𝑥 −2) +1 = 2𝑥 +3
2 ±√
𝑥 +4 - 4𝑥 + 1 = 2𝑥 +3 { ∵ (a-b)2 = a2+b2-2ab } x=
2
𝑥 +4 - 4𝑥 +1 -2𝑥 -3 = 0 ±
x=
Standard form : 𝑥2 -6𝑥 +2 = 0
2) Find the roots of x2 + 5x + 2 = 0 by using quadratic formula. March-2022 x= and x=
Ans : Here, a = 1 , b = 5 & c = 2
x= and x =
±√
x = x = 5 and x = -2
2
x =
± ( ) 5) Solve x − 2x + 3 = 0 by using the quadratic formula. Model-2-2020

±√ Ans. : Here, a = 1 , b = -2 & c = 3


x =
±√
±√ x=
x =
( )± ( )
x=
Sri.Veerendra.H.M. Asst.Teacher. G.P.U.C. Megaravalli. Thirthahalli Tq Ph: 8762624683 Page-109 Sri.Veerendra.H.M. Asst.Teacher. G.P.U.C. Megaravalli. Thirthahalli Tq Ph: 8762624683 Page-110
x=
±√ Area of the field = 147 Sq.metres.

±√
Area of rectangle = Length x Breadth
x= 147 = 3x.x
𝟐 √ 𝟖 𝟐 √ 𝟖
x= 𝟐
and x =
𝟐 x2 =
6) Find the roots of the quadratic equation 𝑥2 − 𝑥 − = 0
𝟐 𝟑
x2 = 49
𝟓 𝟓
x = ±7
Ans. : 𝑥2 − 𝑥 − = 0
Length is positive so, x = 7
Multiply by 5 then, ∴ Length of the rectangular field = 3x7 = 21 metre.
. 5.𝑥2 – 5.𝑥 − . 5 = 0 Breadth of the field = 7 metre.
2
2x -5x – 3 = 0 9) Find the discriminant of the equation 2𝑥2 −5𝑥 +3 = 0 and hence write
Here, a = 2 , b = -5 & c = -3
the nature of the roots. June : 2020
±√
x= Ans. : Here, a = 2 , b = -5 & c = 3
x=
( )± ( ) ( ) Discriminant (∆) = b2 - 4ac
±√ ∆ = (-5)2 - 4x2x3
x=
∆ = 25-24 = 1 > 0
±√
x= ∴ roots are real and disctinct.
x= and x = 10) Find the nature of the roots of the equation 4𝑥2 −4𝑥 +1 = 0 June : 2019

x= and x = Ans. : Here, a = 4 , b = -4 & c = 1


𝟏 Discriminant (∆) = b2-4ac
x = 3 and x = 𝟐 ∆ = (-4)2-4x4x1
7) Find the roots of the equation 𝑥2 +7𝑥 +12 = 0 by factorisation.
∆ = 16-16 = 0
Ans. : June : 2019
∴ Roots are real and equal.
𝑥2 +7𝑥 +12 = 0 +4𝑥
2 2 11) Find the value of discriminant of 2𝑥2 −5𝑥 -1 = 0, then write the nature
𝑥 +4𝑥 +3𝑥 +12 = 0 +12 𝑥
of roots. Sept : 2020
(𝑥 +4) +3(𝑥 +4) = 0 +3𝑥
Ans. : 2𝑥2 −5𝑥 -1 = 0
(𝑥 +3)(𝑥 +4) = 0 +7x
(𝑥 +3) = 0 and (𝑥 +4) = 0 Here, a = 2 , b = -5 & c = -1
𝑥 = -3 and 𝑥 = -4 Discriminant (∆) = b2 - 4ac
8) In a rectangular field, the length is thrice the breadth. The area of the ∆ = (-5)2-4x2x(-1)
field is 147 sq.metre, then find its length and breadth. April : 2019
∆ = 25+8 = 33
Ans. : Let the breadth of rectangular field is ‘x’.
∴∆>0
Length of the rectangular field = 3x ∴ roots are real and disctinct.
Sri.Veerendra.H.M. Asst.Teacher. G.P.U.C. Megaravalli. Thirthahalli Tq Ph: 8762624683 Page-111 Sri.Veerendra.H.M. Asst.Teacher. G.P.U.C. Megaravalli. Thirthahalli Tq Ph: 8762624683 Page-112
12) Find the value of discriminant of, 2x2 + x + 4 = 0 then, write the nature 3) Solve 2x2 - 7x + 3 = 0 using formula PREP-2019
𝟏
of roots. PREP : 2020 Ans. : x = 3 and x =
𝟐
Ans. : 2𝑥2 +𝑥 +4 = 0 4) Solve using formula : x2 ˗3x+1= 0. Model-1-2020
Here, a = 2 , b = 1 & c = 4 𝟑 √𝟓 𝟑 √𝟓
Ans. : x = and x =
Discriminant (∆) = b2-4ac 𝟐 𝟐
5) Solve by factorisation. x2 + 5x + 6 = 0 Model-2-2020
∆ = (1)2 - 4x2x(4) Ans. : x = -2; x = - 3
∆ = 1-32 = -31 < 0 6) Using the quadratic formula solve the equation 12x2 – 17x + 6 = 0.
∴ Roots are imaginary( no real roots) Ans. : x =
𝟑
and x =
𝟐

13) Show that the roots of x2 + ax−4=0 are real and disctinct.
𝟒 𝟑
2
7) Solve 3x – 2x – 3 = 0 by using quadratic formula. MODEL-2022
Ans. : Here, a = 1 , b = a & c = (-4) Model-1-2020
Ans. : x=
𝟏 √𝟏𝟎
and x =
𝟏 √𝟏𝟎
𝟑 𝟑
Discriminant (∆) = b2 - 4ac 8) Find the roots of the quadratic equation 2x2 + x – 4 = 0 using quadratic
∆ = (a)2- 4x1x(-4) formula. PREP-2022
𝟏 √𝟑𝟑 𝟏 √𝟑𝟑
∆ = a2 + 16 > 0 Ans. : x = and x =
𝟒 𝟒
∆ = Square number + 16 > 0 9) Find the roots of the equation x2 – 3x + 1 = 0 using quadratic formula.
𝟑 √𝟓 𝟑 √𝟓
∴ Roots are real and disctinct. Ans. : x = and x =
𝟐 𝟐
10) Find the value of discriminant of, 2x2 −6x+3=0 then, write the nature
14) Find, for what value of ‘k’ the equation kx2 + 6x + 1 = 0 has equal roots.
PREP-2022
of roots. Model-1-2020
Ans : If the roots of an equation are equal, then discriminant is 0. Ans. : ∆ = 12 > 0 ∴ Roots are real and disctinct.
Here a = k , b = 6 & c = 1 11) Find the discriminant of the equation 3x2 −5x + 2 = 0 and hence write
Discriminant (∆) = b2- 4ac = 0 the nature of roots. Model-2-2020
Ans. : ∆ = 1 ∴ Roots are real and disctinct.
Discriminant (∆) = 62 – 4xkx1 = 0
12) Find the value of the discriminant and hence write the nature of roots of
36 – 4k = 0
4k = 36 the equation x2 + 3x + 2 = 0 MODEL-2022
Ans : ∴ Roots are real and disctinct.
k=9
13) Find the value of the discriminant of the equation 3x2 - 5x + 2 = 0
and hence write the nature of roots. PREP-2022
Ans : ∴ Roots are real and disctinct.
EXERCISE : 14) Find the value of the discriminant and hence write the nature of roots of
1) Find the roots of the equation 3𝑥2 − 5𝑥 +2 = 0 using formula. MODEL-2020
the quadratic equation x2 + 4x + 4 = 0. March-2022
𝟐 Ans. : Nature : Two equal real roots
Ans. : x = 1 and x = 𝟑
2) Find the roots of the equation 4𝑥2 −2𝑥 +1 = 0 using formula. 15) Find the value of the discriminant of the equation 4x2 – 12x + 9 = 0 and
hence write the nature of the roots. JUNE-2022

Ans. : x =
𝟏 √ 𝟑
and x =
𝟏 √ 𝟑
MODEL-2020 Ans. : ∆ = 0 ∴ The roots are real and equal.
𝟒 𝟒

Sri.Veerendra.H.M. Asst.Teacher. G.P.U.C. Megaravalli. Thirthahalli Tq Ph: 8762624683 Page-113 Sri.Veerendra.H.M. Asst.Teacher. G.P.U.C. Megaravalli. Thirthahalli Tq Ph: 8762624683 Page-114
THREE MARKS QUESTIONS : According to given condition, + =
1) A two digit number is four times the sum of the digits. It is also equal to 3 ( ) ( )( )
=
times the product of digits. Find the number. 3-MODAL QP-2019 ( )
Ans. :
=
Let the two digit number be ‘xy’.
Then the number = 10x + y. =
Sum of the digits = (x + y) 40x2 - 40x -120 = 29x2 + 58x
According to given condition, 10x+y = 4(x+y) 40x2 -29x2 - 40x -58x – 120 = 0
10x+y = 4x+4y 11x2 -98x – 120 = 0
10x+y - 4x- 4y = 0 Here a =11 , b = -98 & c = -120
±√
6x - 3y = 0 ÷ 3 Formula : x =
2x - y = 0 ± ( )
2x = y ------------- (1) x=
According to given condition, 10x+y = 3.x.y ±√
x=
Substitute the value of y = 2x, then ±√ ,
10x+2x = 3x(2x) x=
±
12x = 6x2 x=
6x2 -12x = 0 x= and x =
6 (𝑥 -2) = 0 x= and x =
(𝑥 -2) = 0
𝑥=2 x = 10 and x = [x = is not possible]
Substitute 𝑥 = 2 in eqn (1), y = 2x2 = 4 𝐱 𝟑 𝟕
The given fraction is ( )=
𝐱 𝟏𝟎
∴ The two digit number = xy = 24
3) The sum of the areas of two squares is 640m2. If the difference between
2) The numerator of a fraction is 3 less than its denominator. If 2 is added to their perimeters is 64m, then find sides of the square. 3-MODAL QP-2020
both the numerator and the denominator, then the sum of the new fraction
𝟐𝟗 Ans. :
and original fraction is . Find the original fraction. 3-MODAL QP-2019
𝟐𝟎 Let the side of first square be ‘x’ m and the side of the second square be ‘y’m.
Ans. : Let the Denominator of the fraction be ‘x’.
Area of the first square = x2 and the second square = y2
The numerator is 3 less than its denominator. Perimeter of the first square = 4x and the second square = 4y
So the given fraction is ( ) According to question,
If 2 is added to both the numerator and the denominator, then the new Difference of perimeters of two squares = 64
fraction becomes = = 4x – 4y = 64m ÷ 4
x – y = 16
Sri.Veerendra.H.M. Asst.Teacher. G.P.U.C. Megaravalli. Thirthahalli Tq Ph: 8762624683 Page-115 Sri.Veerendra.H.M. Asst.Teacher. G.P.U.C. Megaravalli. Thirthahalli Tq Ph: 8762624683 Page-116
x = (y + 16) -------(1) Discriminant (∆) = (b - c)2 – 4(𝑎 - b)x (𝑐 - a)= 0
According to question, Sum of areas of two squares = 640 m2 b2 + c2 - 2bc – 4(ac - a2 – bc + ab) = 0
x2 + y2 = 640
b2 + c2 - 2bc – 4ac + 4a2 + 4bc - 4ab = 0
Substitute x = (y + 16) in above eqn, then we get
(y +16)2 + y2 = 640 4a2 + b2 + c2 - 4ab + 2bc – 4ac = 0
y2 + 32y + 256 + y2 = 640 ( -2a + b +c)2 = 0 { ∵ (a+b+c)2 = a2+b2+ c2+2ab+2bc+2ca}
2y2 + 32y -384 = 0 +24y -2a + b +c = 0
y2 + 16y -192 = 0 -192y2 b +c = 2a
y2 +24y - 8y -192 = 0 -8y
y(y +24) - 8(y + 24) = 0 +16y 6) The sum of the reciprocals of Rehman’s age ( in years ) 3 years ago and his
(y + 24)(y - 8) = 0 𝟏
age 5 years from now is . Find his present age. June-2022
y = -24 or y = 8 [ y = -24 is not possible] 𝟑
Ans. : Let the present age of Rehman be ' x ' years.
∴ the side of the second square is 8m and 3 years ago, his age was ( x – 3 ) years.
the side of first square ‘x’ = 8 +16 = 24m After 5 years from now, his age will be ( x + 5 ) years.
According to the condition,
4) The roots of the quadratic equation (a2 + b2 )x2 + 2(bc– ad)x + c2 + d2 = 0
+ =
are equal, show that ac + bd = 0 3-MODAL QP-2020
Ans. : If roots of an equation are equal, then the discriminant is 0. =

Discriminant (∆) = b2- 4ac = 0 =


Here a = (𝑎2 +𝑏2) , b = 2(b𝑐 −𝑎d) & c = 𝑐2 +𝑑2 3(2x + 2) = x2 + 2x - 15
x2 + 2x – 15 -6x – 6 = 0
Discriminant (∆) = (2(bc-ad))2 – 4x(𝑎2 +𝑏2)x (𝑐2 +𝑑2)= 0
x2 - 4x – 21 = 0
4(b2c2 + a2d2 -2abcd) – 4x(𝑎2 𝑐2+ a2𝑑2 +b2c2 + b2d2) = 0 ±√
x=
(4b2c2 + 4a2d2 -8abcd – 4𝑎2 𝑐2 - 4a2𝑑2 - 4b2c2 - 4b2d2) = 0
( )± ( ) ( )
-8abcd – 4𝑎2 𝑐2 - 4b2d2 = 0 ÷ -4 x=
2abcd + 𝑎2 𝑐2 + b2d2 = 0 ±√
x=
(ac + bd )2 = 0 { ∵ (a+b)2 = a2+b2+2ab } ±√
x=
ac + bd = 0
±
2
x=
5) The roots of the quadratic equation (a - b)x + (b – c)x + (c – a) = 0 are
x= or x =
equal. Then prove that 2a = b + c. 3-PREP-2019
x = 7 or x = -3 (Age cannot be negative)
Ans. : If roots of an equation are equal, then the discriminant is 0.
So x = 7
Discriminant (∆) = b2- 4ac = 0 ∴ Present age of Rehman is 7 years.
Here a = (𝑎 - b) , b = (b - c) & c = (𝑐 – a)

Sri.Veerendra.H.M. Asst.Teacher. G.P.U.C. Megaravalli. Thirthahalli Tq Ph: 8762624683 Page-117 Sri.Veerendra.H.M. Asst.Teacher. G.P.U.C. Megaravalli. Thirthahalli Tq Ph: 8762624683 Page-118
𝟏 𝟏 𝟏 𝟏
7) Solve x. = + + { a ≠ 0, b ≠ 0, x ≠ 0, x ≠ -(a+b)} PREP-2019 =
(𝐚 𝐛 𝐱) 𝐚 𝐛 𝐱

Ans. : = + + =
( )
x2 +2x = 48
- = + 2
( ) ∴ x +2x – 48 = 0
( )
= x2 +8x – 6x – 48 = 0
( )
x(x+8) – 6(x + 8) = 0
( )
= (x+8) (x- 6) = 0
x = -8 and x = 6
( )
= ∴ Speed of A is 6 km/h and Speed of B is 8 km/h
( ) ( ) 𝐝𝐢𝐬𝐭𝐚𝐧𝐜𝐞 𝟏𝟐
= ∴ Time taken by A = t1 = = = 2 hours.
( ) 𝐬𝐩𝐞𝐞𝐝 𝟔
𝟏𝟐
x(a+b+x) = -ab Time taken by B = t2 = = 1.5 hours.
𝟔 𝟐
ax + bx + x2 = -ab
x2 + ax+bx + ab = 0 9) The diagonal of a rectangular playground is 60 m more than the smaller
x(x+a) + b(x+a) = 0 side of the rectangle. If the longer side is 30 m more than the smaller side,
(x+a)(x+b) = 0 find the sides of the playground. SEPT ∶ 2020 & March-2022
x +a = 0 or x +b = 0 Ans. :
∴ x = (-a) or x = (-b) Let the smaller side BC = x.

8) To save fuel, to avoid air pollution and for good health two persons A and Diagonal is 60m more than smaller side.

B ride bicycle for a distance of 12 km to reach their office. As the cycling Diagonal AC = x + 60
speed of B is 2 km/h more than that of A, B takes 30 minutes less than that Longer side is 30m more than the smaller side,
of A to reach the office. Find the time taken by A and B reach the office . AB = x + 30
Ans. : June ∶ 2020 In ∆ ABC, ∟B = 900 By Pythagoras theorem,
Let the speed of A be ‘x’ km/h. 2
AC = AB + BC2 2

∴ The time taken by A to reach office = t1 = = hours ( x + 60)2 = ( x +30)2 + x2


x2 + 120x + 3600 = x2 + 60x + 900 + x2
The speed of ‘B’ is 2 km/h more than that of A. x2 + 120x + 3600 - 2x2 - 60x - 900 = 0
∴ Speed of B = (x+2) km/h - x2 + 60x +2700 = 0 -90x
2
∴ time taken by B, t2 = hours x2 - 60x - 2700 = 0 -2700x
x2 - 90x + 30x - 2700 = 0 +30x
According to given, t1 – t2 = 30 minutes
x(x - 90) +30(x - 90) = 0 -60x
- = [ 30 minutes = hour] (x - 90)(x + 30) = 0
( ) (x - 90) = 0 & (x + 30) = 0
=
( )
Sri.Veerendra.H.M. Asst.Teacher. G.P.U.C. Megaravalli. Thirthahalli Tq Ph: 8762624683 Page-119 Sri.Veerendra.H.M. Asst.Teacher. G.P.U.C. Megaravalli. Thirthahalli Tq Ph: 8762624683 Page-120
x = 90m. & x = -30m. 180 = 81x – 9x2
∴ BC = x = 90 m x2 -9x +20 = 0
±√
AB = x + 30 = 90+30 = 120 m x=
Diagonal AC = x + 60 = 90 + 60 = 150 m. ( )± ( )
x=
±√
10) The altitude of a triangle is 6 cm more than its base. If its area is 108 cm2, x=
find the base and height of the triangle. ±
SEPT ∶ 2020 x=
Ans. : Let base BC = x.
x = 5 and x = 4
∴ Altitude is 6 more than its base.
∴ The two natural numbers are 4 and 5
∴ AD = x + 6
Area of triangle = 108 cm2 12) The perimeter and area of a rectangular play ground are 80m and 384m2
A= xbxh respectively. Find the length and breadth of the play ground. Model-1-2021
Ans. : Let the length = x, Breadth = y
108 = . x . (x+6)
Perimeter of the playground = 2x + 2y = 80
216 = x2 + 6x +18x
x + y = 40
x2 + 6x – 216 = 0 -216x2
y = 40 – x -------(1)
x2 + 18x -12x – 216 = 0 -12x
Area of the playground = xy = 384
x(x +18) -12(x+18) = 0 +6x
x +18 = 0 & x-12 = 0 Substitute, y = 40 – x in above equation.
x = -18 & x = 12 x(40-x) = 384
∴ Base of triangle BC = x = 12 cm 40x-x2 = 384 -24x
2 2
x -40x +384 = 0 384x
Altitude AD = x + 6 = 12+6 = 18 cm
x2 -16x-24x +384 = 0 -16x
𝟗
11)The sum of two natural numbers is 9 and the sum of their reciprocals is 𝟐𝟎 x(x-16) -24(x-16) = 0 -40x
Find the numbers. Model-1-2021 (x-24)(x-16) = 0
Ans. : x = 24 or x = 16
Let the two natural numbers be ‘x’ and ‘y’. ∴ Length of the plyground x = 24m and breadth y = 40 – 24 = 16m

According to given, x + y = 9 13) A train travels 480 km at a uniform speed. If the speed had been
y=9–x
10km/h more, it would have taken 4 hours less for the same journey,
According to given, + = find the speed of the train ? Model-2-2021
Ans. : Let the speed of the train be ‘x’ km/h.
+ =
20(9-x) + 20x = 9x(9-x) Time =
180-20x +20x = 81x -9x2
Sri.Veerendra.H.M. Asst.Teacher. G.P.U.C. Megaravalli. Thirthahalli Tq Ph: 8762624683 Page-121 Sri.Veerendra.H.M. Asst.Teacher. G.P.U.C. Megaravalli. Thirthahalli Tq Ph: 8762624683 Page-122
According to given, - = 4 132 = x2 + (x +7)2
480(x+10) – 480(x) = 4(x + 10)(x) 169 = x2 + x2 + 14x + 49
480x + 4800 – 480x = 4x2 + 40x x2 + x2 + 14x + 49 -169 = 0
4x2 + 40x - 4800 = 0 2x2 + 14x -120 = 0
x2 + 10x - 12000 = 0 x2 + 7x -60 = 0
x2 + 40x – 30x - 12000 = 0 x2 -5x + 12x -60 = 0
x(x + 40) - 30(x + 40) = 0 x(x – 5) + 12(x – 5) = 0
(x + 40)(x - 30) = 0 (x – 5) (x + 12) = 0
x = -40 or x = 30 (x – 5) = 0 or (x + 12) = 0
∴ x = 30 x = 5 or x = -12 (Not considered )
∴ Speed of the train = 30 km/h ∴ length of one side of a right angled triangle = x = 5cm ,
Length of another side = (x +7) = 12 cm
14) Find two consecutive odd positive integers, sum of whose squares is 290
Ans. : Model-2-2021 FOUR MARKS QUESTIONS :
Let the two consecutive positive integers be x and (x+2) 1) The ages of two students A and B are 19 years and 15 years respectively.
According to given, x2 + (x + 2)2 = 290
Find how many years it will take so that the product of their ages becomes
x2 + x2 + 4x + 4 = 290
equal to 480. JUNE-2019
2x2 + 4x – 286 = 0
Ans. : Let the required years be ‘x’
x2 + 2x – 143 = 0
x2 + 13x – 11x – 143 = 0 According to given, (19+x)(15+x) = 480
x (x + 13) – 11(x + 13) = 0 285 +19x + 15x +x2 = 480
(x – 11)(x + 13) = 0 x2+34x +285 – 480 = 0 +39x
x – 11 = 0; x + 13 = 0 x2+34x -195 = 0 -195x2
2
x = 11 and x = -13( x = -13 is not possible) x +39x -5x-195 = 0 -5x
∴ The two consecutive integers are 11 and 13 x(x +39) -5(x +39) = 0 +34x
(x +39)(x -5) = 0
(x +39) = 0 and (x -5) = 0
15) In a right angled triangle, the length of the hypotenuse is 13 cm. Among
x = -39 and x = 5
the remaining two than the other side. Find the sides of the triangle.
Age is positive so, x = 5
Ans : March-2022 P
∴ Required years = 5 years
Let the length of one side of a right angled triangle be ‘x’.
Length of another side = (x + 7) 2) If the quadratic equation (𝑏 −𝑐)x2 +(𝑐 −𝑎)𝑥 +(𝑎 −𝑏) = 0 has equal roots,
Length of hypotenuase = 13cm (x + 7) 13m then show that 2𝑏 = 𝑎 +𝑐. JUNE-2019
In ∆ PQR, By Pythagoras theorem Ans. :
PR2 = PQ2 + QR2 Q x R
If the roots of the equation are equal then disciminant is zero.
Sri.Veerendra.H.M. Asst.Teacher. G.P.U.C. Megaravalli. Thirthahalli Tq Ph: 8762624683 Page-123 Sri.Veerendra.H.M. Asst.Teacher. G.P.U.C. Megaravalli. Thirthahalli Tq Ph: 8762624683 Page-124
Discriminant (∆) = b2- 4ac = 0 Ans. : Speed of boat in still water = 𝑥 km/h
Here a = (b-c) , b = (𝑐 −𝑎) & c = (𝑎 −𝑏) Speed of the stream = 5 km/h, Distance = 30km.
2
Discriminant (∆) = (𝑐 −𝑎) – 4x(b-c)x(a-b) Speed of the boat in upstream = (𝑥 − 5)km/h
∆ = c2 + a2 – 2ac -4(ab-b2-ac+bc) = 0 Speed of the boat in downstream = (𝑥 + 5)km/h
c2 + a2 – 2ac -4ab+4b2 + 4ac-4bc = 0 Time =
a2 +4b2 + c2 -4ab-4bc+ 2ac = 0
( a -2b + c)2 = 0 Time taken by the boat towards upstream = hours
( a -2b + c) = 0 Time taken by the boat towards downstream = hours
∴ 2b = a + c
According to given, + =4
3) Sanvi purchased some books for Rs. 120. If she purchased 3 more books ( ) ( )
( )(
=
)
for the same amount each book would have cost her Rs. 2 less. Find the
30(𝑥 + 5)2 + 30(𝑥 − 5)2 = 9(𝑥 + 5)(𝑥 − 5)
number of books purchased by Sanvi and the price of each book. PREP-2020
60x + 300 + 60x – 300 = 9x2 – 225
Ans. : Let the books purchased by Sanvi be ‘x’
120x = 9x2 - 225
According to given, - =2 9x2 – 120x - 225 = 0 ÷ 3
( ) 3x2 – 40x - 75 = 0 -45x
=2 3x2 – 45x + 5x – 75 = 0 -225x 2
( )
120𝑥 + 360 − 120𝑥 = 2x + 6x 2 3x(x – 15) + 5(x – 15) = 0 +5x
360 = 2x2 + 6x (3x+ 5)(x – 15) = 0; -40x
2x2 + 6x – 360 = 0 +15x (3x+ 5) = 0 is not possible
x2 + 3x – 180 = 0 -180x 2
∴ x – 15 = 0 ⇒ x = 15
x2 + 15x –12x – 180 = 0 -12x ∴ Speed of boat in still water = 15 km/h
x( x + 15) – 12(x + 15) = 0 +3x
( x + 15) (x– 12) = 0 ; 5) A train travels 360 km at a uniform speed. If the speed had been 5 km/h
( x + 15) = 0 and x -12 = 0 more it would have taken 1 hour less for the same journey. Find the speed
of the train. MODEL-2022, June-2022
x = -15 is not possible , (x– 12) = 0 ⇒ x = 12 ,
Ans. : Let the speed of the train be ‘x’ km/h.
∴ The number of books purchased by Sanvi = 12
Distance = 360km
𝟏𝟐𝟎
And price of each book = = Rs.10 Time =
𝟏𝟐

4) A motor boat goes down the stream 30km and again returns to the starting Time =
point in a total time of 4 hours and 30 minutes. If the speed of the stream is According to given, - = 1
5 km/hr, then the speed of the motor boat in still water. PREP-2020 360(x+5) – 360(x) = 1(x + 5)(x)
360x + 1800 – 360x = x2 + 5x
Sri.Veerendra.H.M. Asst.Teacher. G.P.U.C. Megaravalli. Thirthahalli Tq Ph: 8762624683 Page-125 Sri.Veerendra.H.M. Asst.Teacher. G.P.U.C. Megaravalli. Thirthahalli Tq Ph: 8762624683 Page-126
x2 + 5x - 1800 = 0 7) An express train takes 1 hour less than a passenger train to travel 132km
x2 + 45x – 40x - 1800 = 0 between two towns A and B. The average speed of the express train is
x(x + 45) - 40(x + 45) = 0 11 km/hr more than that of a passenger train, find the average speed of
(x + 45)(x - 40) = 0 these trains. PREP-2022
x = -45 or x = 40 Ans : Let the speed of the passenger train be ‘x’ km/h
∴ x = 40 and x = -45 is not possible Speed of the express train = (x + 11) km/h
∴ Speed of the train = 40 km/h Distance = 132 km
Time =
6) By selling an article for Rs.18.75 a person losses as much percent as it cost
him in Rupees. Find the cost price of the article. MODEL-2022 Time taken by passenger train to travel 132 km = t1 = hours
Ans. : Let the cost price of the article be ‘x’. Selling Price = Rs.18.75 Time taken by express train to travel 132 km = t2 = hours
By given, percentage loss = cost price = x From given, t1 –t2 = 1
Loss = Cost Price – Selling Price - =1
Loss = x - 18.75 ( )
=1
% loss = x100 ( )

( . ) 132x + 1452 -132x = x2 +11x


x= . 100 1452 = x2 +11x
x2 = (x -18.75)100 x2 +11x – 1452 = 0
x2 = 100x - 1875 x2 -33x + 44x – 1452 = 0
x2 -100x + 1875 = 0 x(x-33) +44(x -33) = 0
x=
±√ (x-33)(x+44) = 0
x- 33 = 0 and x+44 = 0
( )± ( )
x= x= 33 and x = (-44) is not possible
±√ x = 33
x= So, the speed of the passenger train = x = 33 km/h
x=
±√ Speed of the express train = (x + 11) = 44 km/h
𝟑𝟑 𝟒𝟒 𝟕𝟕
± Average speed = = = 38.5 km/h
x= 𝟐 𝟐

x= and x = 8) Find the roots of the following equation :


𝟏 𝟏 𝟏𝟏
x= and x = - = ( x ≠ -4 and x ≠ 7 ) PREP-2022
𝐱 𝟒 𝐱 𝟕 𝟑𝟎
x = 75 and x = 25
Ans : - =
∴ The Cost Price of the article = 75 or 25
( ) ( )
( )(
=
)

Sri.Veerendra.H.M. Asst.Teacher. G.P.U.C. Megaravalli. Thirthahalli Tq Ph: 8762624683 Page-127 Sri.Veerendra.H.M. Asst.Teacher. G.P.U.C. Megaravalli. Thirthahalli Tq Ph: 8762624683 Page-128
= UNIT-11. INTRODUCTION TO TRIGONOMETRY:
( )( )
The trigonometric ratios : A
=

Opposite side
( )( ) 1. Sinθ = =

( )(
= 2. Cosθ = = Hypotenuse
)

(x+4)(x-7) = -30 3. tanθ = = θ

x2 -7x +4x -28 = -30 4. Cosecθ = = B Adjacent side C


x2 -3x -28 +30 = 0 5. Secθ = =
2
x -3x + 2 = 0 6. Cotθ = =
Here, a = 1 , b = -3 & c = 2 Reciprocal ratios :
±√
x= 1. Sinθ = , 2. Cosθ = , 3. tanθ = ,
( )± ( )
x=
4. Cosecθ = , 5. Secθ = , 6. Cotθ =
±√
x=
Trigonometric Ratios of some Specific Angles :
±√
x= θ 00 300 450 600 900
± 1 1 √3
x= sinθ 0
2 √2 2
1
√3 1 1
x = and x = cosθ 1 2
0
2 √2
1
x = and x = tanθ 0 1 √3 N.D.
√3
2
x=2 and x = 1 cosecθ N.D. 2 √2 1
√3
2
secθ 1 √2 2 N.D.
√3
1
cotθ N.D. √3 1 0
√3
Trigonometric Ratios of Complimentary Angles :
1.Sinθ = Cos(90-θ) 2.Cosθ = Sin(90- θ) 3.tanθ = Cot(90- θ)
4.Cotθ = tan(90- θ) 5.Cosecθ = Sec(90- θ) 6. Secθ = Cosec(90- θ)
Trigonometric Identities :
1. Sin2θ + Cos2θ = 1 2. 1+ tan2θ = Sec2θ 3.1+ Cot2θ =Coseec2θ
Sin2θ = 1 - Cos2θ Cos2θ = 1- Sin2θ Sec2θ - tan2θ = 1

1. Sin θ.Cosec θ = 1 2. Cos θ.Sec θ = 1 3. tan θ.Cot θ = 1


4. tan θ = 5. cot θ =
Sri.Veerendra.H.M. Asst.Teacher. G.P.U.C. Megaravalli. Thirthahalli Tq Ph: 8762624683 Page-129 Sri.Veerendra.H.M. Asst.Teacher. G.P.U.C. Megaravalli. Thirthahalli Tq Ph: 8762624683 Page-130
MULTIPLE CHOICE QUESTIONS : A. B. C. D.
𝟓
1) If 𝑠𝑖𝑛𝜃 = , then the value of 𝑐𝑜s𝜃 MODEL-QP-2019
𝟏𝟑
14) In a circle with centre ‘O’ AC is a tangent at ‘A’. If OC = 4cm and
𝐴) 𝐵) C) 𝐷)
∠ACO=300 then the radius of the circle is Model-I:2021
2) The value of cos 48° – sin 42° is APRIL-2019
A. √3 cm B. 4√3 cm C. 2cm D. 3cm
𝐴) 0 𝐵) C) 𝐷) 1
𝐱
3) The value of 𝑠𝑖𝑛 30°+𝑐𝑜𝑠 60° JUNE-2019 15) If Sinθ = then Cosθ is Model-I:2021
𝐲
A. B.
𝐴) 𝐵) C) 𝐷) 1

4) If SinA =
𝟏
, the magnitude of ∟A is MODEL∶ 2020
C. D.
√𝟐
A) 900 B) 600 C) 300 D) 450 16) If SinA+Sin2A= 1 then the value of Cos2A+ Cos4A is Model-I:2021
5) In the figure, the value of sinC is A MODEL-QP-2020
A. B. 2 C. 3 D. 1

A)

B) C) D) 1 17) In the right angle ∆ABC, ∠B=900 . If tan C = √3 ,the value of the angle
𝟑
6) If sinθ = , then the value of cosecθ PREP-2020 1 2 ‘A’ is Model-II:2021
𝟓
A. 300 B. 60 0
C. 45 0
D. 15 0
A) B) C) D) B √3 C 𝟑 2
18) If Sin θ = 𝟓 the value of (1 − Cos θ) is Model-II:2021
7) If 13𝑠𝑖𝑛𝜃 = 12 , then the value of 𝑐𝑜sec 𝜃 JUNE : 2020
A. B. C. D.
𝐴) 𝐵) C) 𝐷)
8) The value of sec2260 – tan2260 is SEPT ∶ 2020
19) If Sin (∝ + β) = 1 and cos (∝ − β) = 1 where ∝ + β < 90, then the value of
∝ and β are respectively equal to Model-II:2021
A) B) 0 C) 2 D) 1 0 0 0 0 0 0
A. 60 and 30 B. 30 and 60 C. 90 and 0 D. 45 and 450
0

9) The value of sin 600 × cos 300 is : MODEL-I- 2021 20) In the figure ABC is a right angle in which B= 900, BC = 6cm and A = 300

A) B) C) D) then the length of AC is Model-II:2021

10) Sin(90-θ) is equal to : MODEL-II-2021 A. 6√3 cm B. 12cm C. 2√3 cm D. 12√3 cm


21) Which one of the following is a correct relation
A. Cos θ. B. tan θ. C. Sec θ. D. Cot θ.
11) The value of tan45 is : 0
MODEL-II-2021 A. tan2θ = Sec2θ + 1 B. Sinθ =
A. √3 B. 0 C. 1 D. A C. tanθ = D. Sin2θ + Cos2θ =1

12) If 2Sin 2θ = √3 the value of ‘θ’ is Model-I:2021 22) The value of ( Sin300 + cos600 – tan450 ) is JULY: 2021

A. 900 B. 600 C. 300 D. 450 A) 1 B) -1 C) 2 D) 0


2
23) (3 + sec θ) is equal to JULY: 2021
13) In the given figure ∠B=900 , AB= 6cm, 6cm 10cm 2 2 2 2
A) 4 + tan θ B) 4 + cot θ C) 2 + cot θ D) 3 + cot θ
BC= 8cm and AC= 10cm then the θ 24) The value of ( sinθ x cosecθ ) is JULY: 2021
value of Sin(90-θ) is Model-I:2021 B 8cm C √
(A) 2 (B) 1 (C) – (D)
Sri.Veerendra.H.M. Asst.Teacher. G.P.U.C. Megaravalli. Thirthahalli Tq Ph: 8762624683 Page-131 Sri.Veerendra.H.M. Asst.Teacher. G.P.U.C. Megaravalli. Thirthahalli Tq Ph: 8762624683 Page-132
25) In the figure, the value of sinθ is A JULY: 2021 ONE MARK QUESTIONS :

A) B) 2 1) In ∆ABC , ∟ABC = 900 and ∟ACB = 300 , then find AB : AC JUNE-2019

C) √3 D) 1 θ Ans. : In ∆ABC , ∠𝐴𝐵𝐶 = 90° and ∠𝐴𝐶𝐵 = 30° A



√3
𝑠𝑖𝑛𝜃 = =
26) In a right angle ∆ABC, ∟𝐁 = 900 . If tanC = 1. Then ∟A is,
A) 450 B) 300 C) 600 D) 750 𝑠𝑖𝑛300 = B C
𝟑
27) If sinθ = , the value of 2sinθ, =
𝟓
A) B) 1 C) D) AB : AC = 1 : 2
𝟑 𝟒
28) In the figure, ABC is a right angle triangle in which 2) If sin𝜃 = and cos𝜃 = , find the value of Sin2𝜃 + Cos2𝜃. MODEL I-2020
𝟓 𝟓
∟𝐁 = 900 , BC = 6cm and ∟𝐂 = 600 then the length Ans. : Sin2𝜃 + Cos2𝜃 =( )2 + ( )2
of AC is, = +
A) 12cm B) cm C) 6√2 cm D) 10cm 2 2
√ Sin 𝜃 + Cos 𝜃 = 1
29) Which of the following is a correct relation 3) Find the value of Sin300 + Cos600 MODEL I-2020
0 0
A) sin2θ + 1 = cos2θ B) sinθ = Ans. : Sin30 + Cos60 = + =1
0 0
C) cosθ = D) tanθ = 4) Find the value of Sin90 + tan45 MODEL II-2020

30) The value of cosec450 is MODEL-2022


Ans. : Sin90 + tan45 0 0
= 1+1=2
𝐒𝐢𝐧𝟐𝟖𝟎
A) 1 B) √2 C) D) 0 5) Find the value of PREP-2020
√ 𝐂𝐨𝐬𝟔𝟐𝟎
31) If A= 300, then the value of sin2A is PREP-2022 Ans. : =
( )

A) B) C) 1 D)
√ = =1
32) The value of Cos( 900 - 30 0 ) is March-2022
6) In a ∆ABC, AB⊥BC, ∠ABC = 300 , AB = 15 m, then find the length of BC.
A) -1 B) C) 0 D) 1 A PREP-2020
33) 1 + tan2θ is equal to June-2022 Ans. : tan θ =
2 2 2
A) cosec θ B) C) sec θ D) -sec θ
θ tan 300 =
34) Value of cot90° is June-2022
= ⇒ BC = 15 √𝟑m. B 300 C
A) B) 1 C) √3 D) 0 √

7) Find the value of tan45° + cot45°. JUNE-2020
Answers : 1.D, 2.A, 3.D, 4.D, 5.C, 6.A, 7.D, 8.D, 9.C, 10.A, 11.C, 12.C, 13.D, 14.C, 15.D,
16.D, 17.A, 18.C, 19.D, 20.B, 21.D, 22.D, 23.A, 24.B, 25.A, 26.A, 27.C, 28.A, Ans. : tan45° + cot45° = 1 + 1 = 2
29.D, 30.B, 31.B, 32.B, 33.C, 34.D. 8) If cos𝜃 =
𝟐𝟒
,then write the value of sec𝜃. SEPT ∶ 2020
𝟐𝟓
𝟐𝟓
Ans. : cos𝜃 = ∴ sec𝜃 =
𝛉 𝟐𝟒

Sri.Veerendra.H.M. Asst.Teacher. G.P.U.C. Megaravalli. Thirthahalli Tq Ph: 8762624683 Page-133 Sri.Veerendra.H.M. Asst.Teacher. G.P.U.C. Megaravalli. Thirthahalli Tq Ph: 8762624683 Page-134
9) If sin2A = 0 , then find the value of cosA. SEPT ∶ 2020 Ans. : A
Ans. : sin2A + cos2A = 1
12cm ?
cos2A = 1- sin2A 300
cosA = √1 − 𝑆𝑖𝑛 𝐴 B C
cosA = √1 − 0
𝑠𝑖𝑛𝜃 = =
∴ cosA = √𝟏 = 1
10) Find the value of tanθ - cot (900 - θ ) MODEL-I- 2021 𝑠𝑖𝑛300 =
Ans. : tanθ - cot (900 - θ ) = tanθ – tanθ [∵cot (900 - θ ) = tanθ ] =
tanθ - cot (900 - θ ) = 0 AC = 24cm
0
11) In the figure ∠B=90 , ∠A = ∠C and BC=10cm, then find the value of TWO MARKS QUESTIONS :
𝟏𝟐
tan 450. MODEL-I- 2021
1) If sin θ = , find the values of cos θ and tan θ. APRIL-2019
𝟏𝟑
Ans. : By Pythagoras thereom A
Ans. : ∠A = ∠C = 450, So AB = BC A AB2 = AC2 + BC2
tan θ = 132 = 122 + BC2
169 = 144 + BC2
tan450 = =1 BC2 = 169 – 144 13 12
2
B 10cm C BC = 25
BC = 5 θ
12) If, 15 cot A = 8 , then find the value of tanA ? MODEL-II- 2021
B 5 C
Ans. : 15 Cot A = 8 .
cosθ = = =
Cot A = . 𝟏𝟐
𝟏𝟓
tanθ = .
= =
𝟓
tan A = =
𝟖
13) Find the value of sin2θ + cos2θ + 1 MODEL-2022 2) If 𝑐𝑜𝑠𝑒𝑐𝜃 =
𝟏𝟑
, then find the value of cos𝜃. JUNE-2019
𝟏𝟐
Ans : W.K.T. sin2θ + cos2θ = 1
∴ sin2θ + cos2θ + 1 = 1 + 1 = 2 Ans : 𝑐𝑜𝑠𝑒𝑐𝜃 = A
𝟑
14) If sinA = write the value of cosec A. PREP-2022 co𝑠ec𝜃 = = =
𝟒
Ans : W.K.T. cosec A = By Pythagorus theorem, 12k 13k
2 2 2
AB + BC = AC
Cosec A =
144k2 + BC2 = 169k2 θ
15) Write the value of
𝐒𝐢𝐧𝟏𝟖𝟎
March-2022 BC2 = 169 k2- 144 k2 B 5k C
𝐂𝐨𝐬𝟕𝟐𝟎 BC2 = 25 k2
Ans : = = =1 BC = 5k
( )
𝟓
𝑐𝑜𝑠𝜃 = = = =
16) In the figure, ABC is a right angled triangle. If ∠ C = 30° and AB = 12 cm 𝟏𝟑
then find the length of AC. June-2022

Sri.Veerendra.H.M. Asst.Teacher. G.P.U.C. Megaravalli. Thirthahalli Tq Ph: 8762624683 Page-135 Sri.Veerendra.H.M. Asst.Teacher. G.P.U.C. Megaravalli. Thirthahalli Tq Ph: 8762624683 Page-136
3) In the figure given below find the value of sinθ and cos∝ ? 8) In the figure ABC is a right angled triangle. If AB = 24 cm, BC = 7 cm and
AC = 25 cm, then write the value of sin α and cos α. June-2022
Ans : P MODEL-I-2021
A
θ
Sin θ = = 12cm 13cm 24cm 25cm

cos α = = α B 7cm C
Q 5cm R Ans. : Sin ∝ = = =
4) In the given figure, A MODEL-II-2021
Cos ∝ = = =
find the value of sin∝ + cosθ ? 𝜶
9) Evaluate : 2tan2450+ cos2300 – sin2600 MODEL-2019
Ans : Sin α = = Ans. :
4cm 5cm √ √
tan450 = 1, Cos300 = , Sin600 =
cos θ = = θ √ √
2tan2450+ cos2300 – sin2600 = 2x 12 + ( )2 - ( )2
𝟔
sin α + cosθ = + = B 3cm C =2
𝟓
5) From the given figure find the value of MODEL-2022
10) If √3 tan θ =1 and θ is acute, find the value of sin 3θ + cos 2θ. April-2019
a) sinθ A Ans :
b) tan∝ α √3 tan θ = 1
.
Ans : sinθ = = = 1 √5 tan θ =
√ √
. tan θ = tan 30°
tan∝ = = = =2 θ
. θ = 30°
B 2 C sin 3θ = sin (3 × 30°) = sin 90° = 1
6) In the figure find the value of sinP and tanR PREP-2022 cos 2θ = cos(2 × 30°) = cos 60° =
R
. sin 3θ + cos 2θ = 1 + = 1
Ans : sin P = = = 𝟑
Sin 3θ + Cos 2θ =
𝟐
1 2 (𝟏 𝐜𝐨𝐬𝛉)
. √
11) Prove that = (𝑐𝑜𝑠𝑒𝑐𝜃 +𝑐𝑜𝑡𝜃)2 April-2019
(𝟏 𝐜𝐨𝐬𝛉)
tan R = = = = √3
. Ans :
Q √3 P ( )
7) In the figure find the value of sinθ and tanα. March-2022 L.H.S. =
( )
A ( ) ( )
= x
. ( ) ( )
Ans : sin θ = = =
( )
=
12 α 13
. = (1 + cos2θ + 2cosθ) ( ∵ 1-cos2θ = Sin2θ )
tan α = = = θ
. Sin2θ
B 5 C
Sri.Veerendra.H.M. Asst.Teacher. G.P.U.C. Megaravalli. Thirthahalli Tq Ph: 8762624683 Page-137 Sri.Veerendra.H.M. Asst.Teacher. G.P.U.C. Megaravalli. Thirthahalli Tq Ph: 8762624683 Page-138
= + + 𝐭𝐚𝐧𝐀 – 𝐬𝐢𝐧𝐀 𝐬𝐞𝐜𝐀 – 𝟏
15) Prove that = SEPT ∶ 2020
2 2 𝐭𝐚𝐧𝐀 𝐬𝐢𝐧𝐀 𝐬𝐞𝐜𝐀 𝟏
= cosec θ + cot θ + 2cosecθ. cotθ Ans :
= (𝑐𝑜𝑠𝑒𝑐𝜃 +𝑐𝑜𝑡𝜃)2

= R.H.S. L.H.S. =

12) Show that (tan𝐴×sin𝐴)+cos𝐴 = sec𝐴. June-2019


=
Ans :
L.H.S. = (tan𝐴×sin𝐴)+cos𝐴 ( )
=
=( ×sin𝐴)+cos𝐴 ( )

= + cosA =
= = R.H.S.
= 2
{ ∵ cos 𝜃 + sin2𝜃 = 1 } 16) Prove that CosecA (1- cosA) (cosecA + cotA) = 1 SEPT ∶ 2020
Ans : L.H.S. = cosec A (1- cos A) (cosec A + cot A)
= SecA = R.H.S.
= (1-cosA)( + )
13) If cos𝜃 = 0.6 Show that 5 Sin𝜃 – 3 tan𝜃 = 0 MODEL-2020

Ans : = ( )
L.H.S. = 5 Sin𝜃 – 3 tan𝜃 = 5 Sin𝜃 – 3x =
= = =1
.
= L.H.S. = R.H.S
.
= 𝐭𝐚𝐧𝟔𝟓𝟎 𝐬𝐢𝐧𝟐𝟓𝟎
. 17) Evaluate : +
𝐜𝐨𝐭𝟐𝟓𝟎 𝐜𝐨𝐬𝟔𝟓𝟎
=0 Ans :
= R.H.S.
+ = +
( ) ( )
4 2 2 4
14) P.T. (sec 𝜃 - sec 𝜃) = tan 𝜃 + tan 𝜃 MODEL-2020
= + { ∵ cot(90-𝜃) = tan𝜃 & cos(90-𝜃) = sin𝜃 }
Ans :
4 2 = 1+1=2
L.H.S. = (sec 𝜃 - sec 𝜃)
= sec2θ(sec2 θ – 1) 𝟏
= (1 + tan2θ)(tan2 θ) [ ∵ 1 + tan2θ = sec2θ ] 18) If cos(𝐴+𝐵) = 0 and sin(𝐴−𝐵) = then find A and B
𝟐
= tan2𝜃 + tan4𝜃 Ans : cos(𝐴+𝐵) = 0
= R.H.S. cos(𝐴+𝐵) = cos900 {∵ cos900 = 0}
Sri.Veerendra.H.M. Asst.Teacher. G.P.U.C. Megaravalli. Thirthahalli Tq Ph: 8762624683 Page-139 Sri.Veerendra.H.M. Asst.Teacher. G.P.U.C. Megaravalli. Thirthahalli Tq Ph: 8762624683 Page-140
2
(𝐴+𝐵) = 90 --------(1) AC2 = 25k
sin(𝐴−𝐵) = AC = 5k

sin(𝐴−𝐵) = sin30 𝑠𝑖𝑛𝜃 = = =

(𝐴−𝐵) = 30 ------(2) 𝑐𝑜𝑠𝜃 = = =


Add Eqn(1) and Eqn(2) then,
[ ]
𝐴+ 𝐵 = 90 Substitute A = 600 in Eqn (1) =
[ ]
𝐴−𝐵 = 30 60 + B = 90
2A = 120 B = 90 – 60
A = 600 B = 300 =

19) Prove that 𝑡𝑎𝑛2𝜃 –𝑠𝑖𝑛2𝜃 = 𝑡𝑎𝑛2𝜃.𝑠𝑖𝑛2𝜃 =


Ans. :
L.H.S. = 𝑡𝑎𝑛2𝜃 –𝑠𝑖𝑛2𝜃
=
= - 𝑠𝑖𝑛2𝜃
. = x
=
[𝟒𝐬𝐢𝐧𝛉 𝐜𝐨𝐬𝛉 𝟏] 𝟏𝟑
( ) =
= [𝟒𝐬𝐢𝐧𝛉 𝐜𝐨𝐬𝛉 𝟏] 𝟐𝟏

= Sin2𝜃. Sin2𝜃 [ ∵ 1- Cos2𝜃 = Sin2𝜃 ] 2) If tan2𝐴 = cot(𝐴−18°) where 2𝐴 is an acute angle. Find the value of A.
Cos2𝜃 Ans. : MODEL-2019
= 𝑡𝑎𝑛2𝜃.𝑠𝑖𝑛2𝜃
tan2𝐴 = cot(𝐴−18°)
= R.H.S. cot(90 - 2𝐴) = cot(𝐴−18°) { ∵ tan𝜃 = cot(90-𝜃) }
90 – 2A = A - 18
THREE MARKS QUESTIONS : 90 + 18 = A + 2A
[𝟒𝐬𝐢𝐧𝛉 𝐜𝐨𝐬𝛉 𝟏] 108 = 3A
1) If 4tan𝜃 = 3 Evaluate MODEL-2019 A = 36
[𝟒𝐬𝐢𝐧𝛉 𝐜𝐨𝐬𝛉 𝟏]
Ans. : 4tan𝜃 = 3 A 𝟐𝐜𝐨𝐬 (𝟗𝟎𝟎 𝟑𝟎𝟎 ) 𝐭𝐚𝐧𝟒𝟓𝟎 √𝟑𝐜𝐨𝐬𝐞𝐜𝟔𝟎𝟎
3) Evaluate : June-2022
√𝟑 𝐬𝐞𝐜𝟑𝟎𝟎 𝟐𝐜𝐨𝐬𝟔𝟎𝟎 𝐜𝐨𝐭𝟒𝟓𝟎
tan𝜃 = ( ) √
Ans. :

tan𝜃 = 3k

By Pythagoras theorem, =

AC2 = AB2 + BC2 B 4k C √
2 2
AC2 = (3k) + (4k) = √

2 2 √
AC2 = 9k + 16k √

Sri.Veerendra.H.M. Asst.Teacher. G.P.U.C. Megaravalli. Thirthahalli Tq Ph: 8762624683 Page-141 Sri.Veerendra.H.M. Asst.Teacher. G.P.U.C. Megaravalli. Thirthahalli Tq Ph: 8762624683 Page-142
= Let L.H.S. =
=0 ( ) ( )
0 0 0 0 = √
[ ∵ sin2θ + cos2θ = 1 ]
4) Evaluate : 4Sin30 + tan48 . tan42 – 3tan45 MODEL-2022

Ans : Sin300 = , tan450 = 1, = 5 x + 4x -1


0 0 0 0 0
tan48 . tan42 = tan48 . cot(90-42) = tan48 . cot48 = 1
= + -1
4Sin300 + tan480 . tan420 – 3tan450 = 4x + 1 – 3x1
=2+1–3=0 =

5) Evaluate : 6cos600 – Sin300 + Sin2450 + cos2450 MODEL-2022 =


0 0 0 0
Ans : Cos60 = , Sin30 = , Sin45 = , Cos45 =
√ √
0 0 2 0 2 0
=
6cos60 – Sin30 + Sin 45 + cos 45 = 6x - + ( ) + ( )
√ √
= R.H.S.
=3- + +
6cos600 – Sin300 + Sin2450 + cos2450 = 3 𝐒𝐢𝐧𝛉 𝟏 𝐂𝐨𝐬𝛉
8) Prove that + = 2 Cosecθ MODEL-2020
𝐜𝐨𝐬𝟕𝟎𝟎 𝐜𝐨𝐬𝟓𝟓𝟎 .𝐜𝐨𝐬𝐞𝐜𝟑𝟓𝟎 𝟏 𝐂𝐨𝐬𝛉 𝐒𝐢𝐧𝛉
6) Prove that + =2 Ans. :
𝐬𝐢𝐧𝟐𝟎𝟎 𝐭𝐚𝐧𝟓𝟎 .𝐭𝐚𝐧𝟐𝟓𝟎 .𝐭𝐚𝐧𝟔𝟓𝟎 .𝐭𝐚𝐧𝟖𝟓𝟎
Ans. : L.H.S. = +
. ( )( )
L.H.S. = + =
. . . ( )

= + =
. . . ( )

( ) = [ ∵ 1-𝑠𝑖𝑛 θ = 𝑐𝑜𝑠 θ ]
( )
= + ).
( ) . ( . ( ) ( )
=
( )
= + ( )
. . . =
( )
= 1+1 [ ∵ tanθ.cotθ = 1 ] = 2cosecθ
= 2 = R.H.S.
= R.H.S. 𝐜𝐨𝐬𝛉 𝐬𝐢𝐧𝛉
𝟓 𝐜𝐨𝐬𝟐 𝟔𝟎𝟎 𝟒 𝐬𝐞𝐜 𝟐 𝟑𝟎𝟎 𝐭𝐚𝐧𝟐 𝟒𝟓𝟎 𝟔𝟕
9) Prove that + = sin θ + cos θ. PREP ∶ 2020
𝟏 𝐭𝐚𝐧𝛉 𝟏 𝐜𝐨𝐭𝛉
7) Prove that = PREP-2022
𝐬𝐢𝐧𝟐 𝟑𝟎𝟎 𝐜𝐨𝐬𝟐 𝟑𝟎𝟎 𝟏𝟐 Ans. :
0 0 0 0 0 √
Proof : cos60 = , sec30 = , tan45 = 1 , sin30 = , cos30 =
√ L.H.S. = +
Sri.Veerendra.H.M. Asst.Teacher. G.P.U.C. Megaravalli. Thirthahalli Tq Ph: 8762624683 Page-143 Sri.Veerendra.H.M. Asst.Teacher. G.P.U.C. Megaravalli. Thirthahalli Tq Ph: 8762624683 Page-144
= + = p2 tan2𝜃 + q2 sec2 𝜃 + 2.pq tan𝜃.sec𝜃 -----(1)
y = (p sec𝜃 + q tan𝜃)
2 2

= p2 sec2 𝜃 + q2 tan2𝜃 + 2.pq tan𝜃.sec𝜃 -----(2)


= +
Eqn(1) – Eqn(2)
x2 - y2 = p2 tan2𝜃 + q2 sec2 𝜃 + 2.pq tan𝜃.sec𝜃 - p2 sec2 𝜃 - q2 tan2𝜃 - 2.pq tan𝜃.sec𝜃
= -
= p2 (tan2𝜃 - sec2 𝜃) + q2 (sec2 𝜃 - tan2𝜃)
= [∵ a2 – b2 = (a+b)(a-b)] = p2 (tan2𝜃 - sec2 𝜃) - q2 (tan2𝜃 - sec2 𝜃)
= p2(-1) - q2(-1) [∵ tan2𝜃 - sec2 𝜃 = -1 ]
(𝐜𝐨𝐬 )( ) = - p 2 + q2
=
(𝐜𝐨𝐬 ) ∴ x2 - y2 = q2 - p2
= cosθ + sinθ = R.H.S.
𝐜𝐨𝐭 𝟐 (𝟗𝟎 𝛉) 𝐜𝐨𝐬𝐞𝐜 𝟐 𝛉 𝟏
12) Prove that + = . JUNE-2020
𝟏 𝐜𝐨𝐬𝛉 𝐬𝐢𝐧𝛉 𝐭𝐚𝐧𝟐 𝛉 𝟏 𝐬𝐞𝐜 𝟐 𝛉 𝐜𝐨𝐬𝐞𝐜 𝟐 𝛉 𝐬𝐢𝐧𝟐 𝛉 𝐜𝐨𝐬𝟐 𝛉
10) Prove that - = 2cot θ PREP ∶ 2020
𝐬𝐢𝐧𝛉 𝟏 𝐜𝐨𝐬𝛉 Ans. :
Ans. : ( )
L.H.S. = +
L.H.S. = -
( )( ) = +
=
( )

= = +
( )

= [ ∵ 1-𝑠𝑖𝑛 θ = 𝑐𝑜𝑠 θ ]
( ) = +
.
=
( )
( )
= +
=
( )
=
= 2cot θ
= R.H.S. = [ ∵ sin θ + cos θ = 1]
= R.H.S.
11) If x = p tan 𝜃 +q sec 𝜃 and y = p sec 𝜃 + q tan 𝜃 then prove that
x2 – y2 = q2 – p2. JUNE-2020 𝐒𝐢𝐧𝛉 𝐂𝐨𝐬𝛉
13) Prove that + = Sinθ + cosθ MODEL-I-2021
𝟏 𝐂𝐨𝐭𝛉 𝟏 𝐭𝐚𝐧𝛉
Ans. : Ans. :
x = p tan𝜃 + q sec𝜃 θ θ
L.H.S. = +
y = p sec𝜃 + q tan𝜃 θ θ
θ θ
x2 = (p tan𝜃 + q sec𝜃)2 = θ + θ
θ θ
Sri.Veerendra.H.M. Asst.Teacher. G.P.U.C. Megaravalli. Thirthahalli Tq Ph: 8762624683 Page-145 Sri.Veerendra.H.M. Asst.Teacher. G.P.U.C. Megaravalli. Thirthahalli Tq Ph: 8762624683 Page-146
θ θ 𝐒𝐢𝐧𝛉 𝐂𝐨𝐬𝛉 𝐒𝐢𝐧𝛉 𝐂𝐨𝐬𝛉 𝟐
= θ θ + θ θ
16) Prove that + = . MODEL-II-2021
𝐒𝐢𝐧𝛉 𝐂𝐨𝐬𝛉 𝐒𝐢𝐧𝛉 𝐂𝐨𝐬𝛉 𝟐𝐒𝐢𝐧𝟐 𝛉 𝟏
θ θ
Ans. :
θ θ
= +
θ θ θ θ L.H.S. = +
θ θ
= - ( ) ( )
θ θ θ θ =
θ θ ( )( )
= [∵ a2 – b2 = (a+b)(a-b)] . .
θ θ =
(𝐬𝐢𝐧θ θ)( θ θ)
=
( θ θ)
=
= sin𝛉 + cos𝛉 = R.H.S.
𝐂𝐨𝐬𝛉 𝟐𝐂𝐨𝐬𝟑 𝛉 = [ ∵ sin θ + cos θ = 1]
14) Prove that = Cotθ MODEL-I-2021 ( )
𝟐𝐒𝐢𝐧𝟑 𝛉 𝐒𝐢𝐧𝛉
Ans. : =
L.H.S. = = R.H.S
( )
= 𝐒𝐢𝐧𝛉 𝐭𝐚𝐧𝛉
( )
17) Prove that + = cotθ + secθ.cosecθ
( ) 𝟏 𝐂𝐨𝐬𝛉 𝟏 𝐜𝐨𝐬𝛉
=
( )
( )
Ans. :
=
( )
L.H.S. = +
=
( ) ( )
= cotθ = ( )( )
= R.H.S. . .
=
15) P.T. {Cosec (90 - θ) - Sin (90 - θ)} {(Cosecθ - Sin θ) (tanθ + cotθ)} = 1
.
Ans. : MODEL-II-2021 =
L.H.S. = [Cosec (90 - θ) - Sin (90 - θ)][(Cosecθ - Sin θ) (tanθ + cotθ)] .
= [sec θ - cos θ][(Cosecθ - Sin θ) (tanθ + cotθ)] =
=[ – cosθ][ – sinθ ][ + ] .
= +
=[ ][ ][ ]
.
= +
=[ ][ ][ ]
.

=[ ][ ][ ] = cotθ + secθ.cosecθ
.
= sin2θ + cos2θ = R.H.S.
= 1= R.H.S

Sri.Veerendra.H.M. Asst.Teacher. G.P.U.C. Megaravalli. Thirthahalli Tq Ph: 8762624683 Page-147 Sri.Veerendra.H.M. Asst.Teacher. G.P.U.C. Megaravalli. Thirthahalli Tq Ph: 8762624683 Page-148
18) Prove that
𝐬𝐢𝐧𝟐 𝛉
+
𝐜𝐨𝐬 𝟐 𝛉
= 𝑠𝑒𝑐2𝜃 + 𝑐𝑜𝑠𝑒𝑐2𝜃 −2
= 1 + 1 + cot2A + 1 +tan2A + 2 + 2
𝐜𝐨𝐬𝟐 𝛉 𝐬𝐢𝐧𝟐 𝛉 = 7 + cot2A + tan2A
Ans. : = R.H.S.
θ θ
L.H.S. = +
θ θ 21) Prove that Secθ(1 – sinθ)(secθ +tanθ) = 1. March-2022
= tan2𝜃 + cot2𝜃
Ans :
= 𝑠𝑒𝑐2𝜃 -1 + 𝑐𝑜𝑠𝑒𝑐2𝜃 -1 [ ∵ 1 + tan2𝜃 = sec2𝜃 & 1 + cot2𝜃 = 𝑐𝑜𝑠𝑒𝑐2𝜃 ] L.H.S. = Secθ(1 – sinθ)(secθ +tanθ)
= 𝑠𝑒𝑐2𝜃 + 𝑐𝑜𝑠𝑒𝑐2𝜃 −2 𝟏
= (1 – sinθ)( + )
= R.H.S.
( – ) ( )
0 𝐜𝐨𝐬 𝐀 𝟏 𝐜𝐨𝐬 𝐁 𝟐 = x
19) If A + B = 90 , prove that + = PREP-2022
𝟏 𝐜𝐨𝐬 𝐁 𝐜𝐨𝐬 𝐀 𝐜𝐨𝐬 𝐀
Proof : =
A + B = 90 , So B = 90 - A = [ ∵ 1 – Sin2θ = Cos2θ ]
Let L.H.S. = + =1
( ) L.H.S. = R.H.S.
= +
( )

= +
( )( )
= ( )

= ( )

= ( )

= ( )
( )
= ( )

= R.H.S

20) Prove that ( sinA + cosecA)2 + ( cosA + secA)2 = 7 + tan2A + cot2A.


Ans : March-2022
2 2
L.H.S. = ( sinA + cosecA) + ( cosA + secA)
= Sin2A + cosec2A + 2.SinA.cosecA + cos2A + sec2A + 2.cosA.secA
= Sin2A + cos2A + cosec2A + sec2A + 2.SinA. + 2.cosA.
Sri.Veerendra.H.M. Asst.Teacher. G.P.U.C. Megaravalli. Thirthahalli Tq Ph: 8762624683 Page-149 Sri.Veerendra.H.M. Asst.Teacher. G.P.U.C. Megaravalli. Thirthahalli Tq Ph: 8762624683 Page-150
UNIT-12. SOME APPLICATIONS OF TRIGONOMETRY: BD = 100√3 = 100x1.73 = 173m
TWO MARKS QUSTIONS : Distance between two ships = BC = BD – BC
1) The angle of elevation of the top of a tower from a point on the ground, Distance between two ships = BC = 173 – 100 = 73m
which is 30m away from the foot of the tower is 300. Find the height of the
3) The angle of elevation of the top of a vertical tower on a level ground from
tower. 2-MODEL-2019
Ans. : C Point, at a distance of 9√3 m from its foot on the same ground is 600. Find
Angle of elevation = 300 the height of the tower? 2-MODEL-2020
The distance of the tower from the point = AB = 30m Ans. :
Let the height of the tower = AB
Height of the tower = BC = ?
Distance from the foot of the tower to the point = 9√3 m
tanθ = = 300
Angle of elevation = 600 A
0
tan30 = A 30m B In ∆ ABC ,tanθ =


= tan600 = ?

BC = = = 10√3 m √3 =
√ √ √
BC = 10√𝟑 m AB = 9√3 x √3 C 600 B
AB = 9x3 = 27m
2) As observed from the top of a 100m high light house from the sea level the ∴ Height of the tower = AB = 27m.
angle of depression of two ships are 300 and 450. If one of the ship is exactly
behind the other on the same side of the light house, find the distance between 4) Find the diameter of the circular base of right circular cone whose slant
the two ships (√3 ≈1.73) 2-MODEL-2019 height is 8cm and semi vertex angle is 600 2-MODEL-2020
Ans. : A Ans. :
Height of the light house = 100m 450 Sinθ = = D B A
Let the positions of the ships B and C
0 0
Sin600 = 600 8m
Angle of depression = 45 and 30
√𝟑
=
In ∆ ACD, 𝟐
AB = 4√3 m C
tanθ = = 300 450
∴ Diameter = AD = 2x4√3 m = 8√3 m
tan450 = B C D
1= THREE MARKS QUSTIONS :
CD = 100m
In ∆ ABD, 1) From the top of a vertical building of 50√3 m height on a level ground the
angle of depression of an object on the same ground is observed to be 60°.
tan300 =
Find the distance of the object from the foot of the building. June-2019
= OR

Sri.Veerendra.H.M. Asst.Teacher. G.P.U.C. Megaravalli. Thirthahalli Tq Ph: 8762624683 Page-151 Sri.Veerendra.H.M. Asst.Teacher. G.P.U.C. Megaravalli. Thirthahalli Tq Ph: 8762624683 Page-152
From the top of a building 50√3 m high the angle of depression of a car on Ans. : Distance between wind mills = AB + BC = ?
the ground is observed to be 600 . Find the distance of the car from the foot In ∆ ABD,
of a building. MODEL-II-2021 tanθ =
P M
tan450 =
θ = 600 1=
AB = 50m
50√3
In ∆ BCE
tanθ =
Q ? R tan300 =

Ans. : Height of the building = 50√3 m



Angle of depression = 600 =

The distance of the car from the Foot of a building = QR = ? BC = 40x√3x√3


∟MPR = θ = 60
0 BC = 40x3
0
∴ ∟PRQ = θ = 60 [ ∵ PM ǁ QR, Alternate angles ] BC = 120m
tan600 = ∴ Distance between wind mills = AB + BC = 50 + 120 = 170 m.

√3 =
3) A man observes two vertical poles which are fixed opposite to each other on

QR = either side of the road. If the width of the road is 90 feet and heights of the


pole are in the ratio 1 : 2, also the angle of elevation of their tops from a point
QR = between the line joining the foot of the poles on the road is 600 . Find the

∴ The distance of the car from the Foot of a building = QR = 50m heights of the poles. 3-PREP-2020
Ans. :
E
2) Two windmills of height 50 m and 40√3 m are on either side of the field. A
D
person observes the top of the windmills from a point in between them. The
angle of elevation was found to be 45° and 30°. Find the distance between the h 2h
windmills. June-2019 600 600
E A 90 - x B x C
D Width of the road = AC = 90 feet
Height of the short pole = AD = h = ?
50 40√3
0 0 Height of the long pole = CE = 2h = ?
45 30
Angle of elevation = 600
A B C
Let BC = x, then AB = 90 - x

Sri.Veerendra.H.M. Asst.Teacher. G.P.U.C. Megaravalli. Thirthahalli Tq Ph: 8762624683 Page-153 Sri.Veerendra.H.M. Asst.Teacher. G.P.U.C. Megaravalli. Thirthahalli Tq Ph: 8762624683 Page-154
In ∆ ABD, Ans. : Height of the building = BC = 20m
0
tan 60 = Angle of Elevation = θ = 600 and 450
Height of the transmission tower = AB = ?
√3 =
h = (90- x)√3 ---------- (1) In ∆ BCD , tanθ =
In ∆ BCE, tan450 =
tan600 = 1=
√3 = ∴ CD = 20cm
2h = x.√3 In ∆ ACD, tanθ =

h = x.

---------- (2) tan600 =
From Eqn(1) and Eqn(2), √3 =

(90- x)√3 = x. √3x20 = AB + 20
2(90- x)√3 = x. √3 AB = 20(√3 -1) m
180 – 2x = x ∴ Height of the transmission tower = AB = 20(√3 -1) m
3x = 180 5) A tower and a building are standing vertically on the same level ground.
x = 60 The angle of elevation of the top of a building from the foot of the tower is 30°
Substitute x = 60 in Eqn (1) then and the angle of elevation of the top of the tower from the foot of the building
h = (90- x)√3 = (90 – 60)√3 = 30√3 m is 60°. If the tower is 50 m high, find the height of the building. June-2022
Height of the short pole = AD = h = 30√3 feet A
Height of the long pole = CE = 2h = 60√3 feet
50m

4) From a point on the ground, the angles of elevation of the top and bottom C
of a transmission tower fixed at the top of a 20m high building are 600 and 450
B 300 600 D
respectively. Find the height of the transmission tower. 3-MODEL-I-2021
A Ans. : In ∆ABD, tan600 =
√3 =
B
BD = --------(1)

In ∆BDC, tan300 =
20cm
=
600 450 √
BD = √3. DC ---------- (2)
C D From equation (1) and (2)
Sri.Veerendra.H.M. Asst.Teacher. G.P.U.C. Megaravalli. Thirthahalli Tq Ph: 8762624683 Page-155 Sri.Veerendra.H.M. Asst.Teacher. G.P.U.C. Megaravalli. Thirthahalli Tq Ph: 8762624683 Page-156
√3. DC = FOUR MARKS QUSTIONS :

1) The angles of elevation of the top of a tower from two points at a distance
DC =
√ √ of 4 m and 9 m from the base of the tower and in the same straight line with
𝟓𝟎
∴ Height of the building. = DC = m = 16.67m it are complementary. Find the height of the tower. APRIL-2019
𝟑
A
6) A cable tower and a building are standing vertically on the same level
ground. From the top of the building which is 7 m high, the angle of elevation
of the cable tower is 60° and the angle of depression of its foot is 45°. Find the x 90-x
height of the tower. ( Use 3 = 1·73 ) June-2022 C 4m B 9m D
C Ans. : Let AB be height of the tower
∟ACB = x°
∴ ADB = 90° – x
In Δ ABC
tan x =
600 tan x = ... (i)
A 45 0
E In Δ ADB
tan ( 90° – x ) =
7m 7m
cot x = ... (ii)
450
Eqn (i) x Eqn (ii)
B D
tan x × cot x = x
Ans. : Height of the building is 7 m. tan x × =
Height of the tower = CD = CE + DE = ?
AB and CD are perpendicular to the ground. 1=
2
∴ AB || CD. AB = DE = 7 m and AE = BD. AB = 36
Also ∠ EAD = ∠ BDA Alternate angles AB = 6
AE || BD ∴ ∠ BDA = 45° ∴ Height of the tower AB = 6 m.
In ∆ABD, tan450 = 2) The angle of elevation of the top of an unfinished vertical building on a
ground, at a point which is 100m from the base of the building is 450. How
1= much height the building must be raised, so that its angle of elevation from
BD = AB the same point be 600 ( Take √3 = 1.73) MODEL-2020 D
BD = 7m Ans. :
In ∆ACE, tan600 = Height of the unfinished building = 100m C
0
Angle of elevation = 45
√3 = In ∆ ABC,
CE = 7√3 tanθ = 450 600
∴ Height of the tower = CE + DE = 7√𝟑 + 7 = 7(√𝟑 + 1) m A B
Sri.Veerendra.H.M. Asst.Teacher. G.P.U.C. Megaravalli. Thirthahalli Tq Ph: 8762624683 Page-157 Sri.Veerendra.H.M. Asst.Teacher. G.P.U.C. Megaravalli. Thirthahalli Tq Ph: 8762624683 Page-158
tan450 = x= ----------(1)

1= From Eqn(1) and Eqn(2)
AB = 100m √3 (60 – h) =

In ∆ ABD, (60 – h) =
√ √
tanθ =
(60 – h) =
tan600 =
(60 – h) = 20
√3 = h = 60 – 20
BD = 100√3 m = 100x1.73 = 173m. h = 40 m
CD = BD- BC = 173 – 100 = 73m Height of the pole = CD = h = 40m
∴ 73m height the building must be raised to to get an angle of elevation 600
4)An aircraft flying parallel to the ground in the sky from the point A through
3) A tower and a pole stand vertically on the the point B is observed, the angle of elevation of aircraft at A from a point on
same level ground. It is observed that the angles the level ground is 60°, after 10 seconds it is observed that the angle of
of depression of top and foot of the pole from the elevation of aircraft at B is found to be 30° from the same point. Find at what
top of the tower of height 60 m is 300 and 600 height the aircraft is flying, if the velocity of aircraft is 648 km/hr.
respectively. Find the height of the pole. ( Use √3 = 1·73 ) SEPT-2020
Ans. : 𝑱𝒖𝒏𝒆 ∶ 𝟐𝟎20
Height of the tower = 60m.
Height of the pole = CD = h m. and BE = CD = h m.
Let BD = EC = x.
∴ AE = (60 – h) m.
∟ACE = 300
∟ADB = 600
In ∆ AEC, Ans. : Velocity = 648 km/h =
0
tan 30 = = 180 m/sec.
After10 seconds distance travelled by aircraft = 180x10 = 1800 m
=

In the diagram OC = x CD = 1800 m OD = 1800 + x
x = √3 (60 – h) ----------(1)
In ∆ ABD, In ∆ OAC , ∟C = 900 tanθ =

tan 600 = tan 600 =


√3 = √3 =
Sri.Veerendra.H.M. Asst.Teacher. G.P.U.C. Megaravalli. Thirthahalli Tq Ph: 8762624683 Page-159 Sri.Veerendra.H.M. Asst.Teacher. G.P.U.C. Megaravalli. Thirthahalli Tq Ph: 8762624683 Page-160
h = x√3 ---------- (i) In ∆ ABD, tanθ =
0
In ∆ ODB, ∟D = 90 tanθ = tan 300 =
tan300 = =

= AD = 50√3 m ----------(1)

In ∆ ADC,
h√3 = 1800 + x ---------- (ii)
Substitute (i) in (ii) tan 600 =

x√3.√3 = 1800 + x √3 =

3x = 1800 + x AC = 50√3x√3
3x – x = 1800 AC = 50x3 = 150m
2x = 1800 ∴ Height of the tower = AC = 150m
x= = 900 6) A building on the ground is in the form of a conical tomb surmounted by a
cylinder of height 10 feet as shown in the figure. From a point ‘P’ on the
∴ h = x√3 = 900 x √3 = 900 x 1.73
same ground the angle of elevation of the angle of elevation of the top edge
∴ h = 1557 m
of the cylinder is found to be 300 and the angle of elevation to the vertex of
the cone is found to be 450. If the diameter of the outer edge of circular base
5) The angle of depression from the top of a vertical tower to a point on the
of the cylinder is 9.4 feet, find the height of the conical shaped tomb.
ground is found to be 600 and from a point 50m above the foot of the tower
A PREP-2022
the angle of depression to the same point is found to be 300 as shown in the
figure find the height of the tower. MODEL-2022

C 600 E C

10ft

B 300 450 300


B D P
9.4 ft
A D Ans. :
Ans. : Height of the cylinder = CD = 10 ft
Height of the tower = AC = ? Height of the conical shaped tomb = AE = ?
Height of the tower from the foot to point B = AB = 50m. Angle of Elevations θ = 300 and 450
0 0
Angle of depression = 60 and 30 In ∆ PCD, tanθ =
∴ ∟ADB = 300
∟ADC = 600 tan 300 =

Sri.Veerendra.H.M. Asst.Teacher. G.P.U.C. Megaravalli. Thirthahalli Tq Ph: 8762624683 Page-161 Sri.Veerendra.H.M. Asst.Teacher. G.P.U.C. Megaravalli. Thirthahalli Tq Ph: 8762624683 Page-162
= AB =

PD = 10√3 = 10x 1.73 = 17.3 ft ∴ Height of the building = AB = m
Diameter of the cylinder = 9.4 ft
Radius = BD = 4.7 ft 8) As observed from the top of a 75 m high light house from the sealevel, the
So, BP = BD + PD = 4.7 + 17.3 = 22 ft angles of depression of two ships are 30° and 45°. If one ship is exactly behind
In ∆ ABP, tan 45 = 0 the other on the same side of the light house, then find the distance between
the two ships. March-2022
1 =
AB = 22 ft A 300 M
Height of the conical shaped tomb = AE = AB – EB = 22-10 = 12 ft 450
75m
7) The angle of elevation of the top of a building from the foot of a tower is
30° and the angle of elevation of the top of the tower from the foot of the
building is 60°. Both the tower and building are on the same level. If the 450 300
height of the tower is 50 m, then find the height of the building. March-2022 B P Q
Ans : Distance between two ships is PQ
Ans. : In ∆ABP, tan450 =
C
A 1=
BP = 75
50m
In ∆ABQ, tan300 =
600 300
=

B D =
0

In ∆ABC, tan60 =
75 + PQ = 75√3
√3 = PQ = 75√3 – 75
BD = --------(1) PQ = 75(√3 – 1) m

∴ Distance between two ships = PQ = 75(√3 – 1) m
In ∆ABD, tan300 =
= 9) A tower and a building are standing vertically on the same level ground. The

BD = √3. AB ---------- (2) angle of elevation of the top of a building from the foot of the tower is 30° and
From (1) and (2) the angle of elevation of the top of the tower from the foot of the building is 60°.
If the tower is 50 m high, find the height of the building. JUNE-2022
√3. AB =

Sri.Veerendra.H.M. Asst.Teacher. G.P.U.C. Megaravalli. Thirthahalli Tq Ph: 8762624683 Page-163 Sri.Veerendra.H.M. Asst.Teacher. G.P.U.C. Megaravalli. Thirthahalli Tq Ph: 8762624683 Page-164
A Ans. : Height of the building is 7 m.
Height of the tower = CD = CE + DE = ?
AB and CD are perpendicular to the ground.
∴ AB || CD. AB = DE = 7 m and AE = BD.
50m
Also ∠ EAD = ∠ BDA Alternate angles
C AE || BD ∴ ∠ BDA = 45°

B 300 600 D In ∆ABD, tan450 =


Ans. : In ∆ABD, tan600 = 1=
√3 = BD = AB
BD = 7m
BD = --------(1)
√ In ∆ACE, tan600 =
0
In ∆BDC, tan30 =
√3 =
=
√ CE = 7√3
BD = √3. DC ---------- (2) ∴ Height of the tower = CE + DE = 7√𝟑 + 7 = 7(√𝟑 + 1) m
From equation (1) and (2)
√3. DC =

DC =
√ √
𝟓𝟎
∴ Height of the building. = DC = m = 16.67m
𝟑
10)A cable tower and a building are standing vertically on the same level
ground. From the top of the building which is 7 m high, the angle of
elevation of the cable tower is 60° and the angle of depression of its foot is 45°.
Find the height of the tower. ( Use 3 = 1·73 ) JUNE-2022
C

600
A 450 E

7m 7m
450
B D
Sri.Veerendra.H.M. Asst.Teacher. G.P.U.C. Megaravalli. Thirthahalli Tq Ph: 8762624683 Page-165 Sri.Veerendra.H.M. Asst.Teacher. G.P.U.C. Megaravalli. Thirthahalli Tq Ph: 8762624683 Page-166
UNIT-13. STATISTICS : sum of the marks scored by him in 5 subjects is 86. Then the marks scored
by him in the sixth subject is MODEL-II
Mean : The mean of observations, is the sum of the values of all the observations A. 20 B. 21 C. 18 D. 22
divided by the total number of observations. 7. In the given frequency distribution table the median class is : MODEL-I
Ʃ. Ʃ .
Mean : X = or X= Class Interval 0-10 10-20 20-30 30-40 40-50
Ʃ
Mode : Mode is that value among the observations which occurs most often i.e. the Frequency 5 8 12 15 20
value of the observations having the maximum frequency.
A. 10-20 B. 20-30 C. 30-40 D. 0-10
Mode = l + [ ]xh
8. The median and mode of a grouped scores are 26 and 29 respectively then
Median : Median is the middle-most observation in the data when they are the mean of the score is MODEL-II
arranged in ascending or descending order. A. 27.5 B. 28.4 C. 25.8 D. 24.5
.. 9. In the cummulative frequency distribution table given below, the number of
Median : l + [ ]xh
families having income range of Rs.20000 and Rs.25000 is MODEL-II

3 x Median = Mode + 2 x Mean Monthly Income Number of families


More than Rs.5000 100
More than Rs.10000 85
MULTIPLE CHOICE QUESTIONS :
More than Rs.15000 69
1. The value among the observations of most repeated scores of the data is More than Rs.20000 50
(A) the mean (B) the mode (C) the median (D) the range JULY-2021 More than Rs.25000 37
2. The mean of the following scores is More than Rs.30000 15
Marks 1 3 5 7 A. 35 B. 22 C. 13 D. 19
(A) 16 (B) 5 (C) 1.6 (D) 4 10. The x-coordinate of the point of intersection of “ less than and more than”
3. The relation among the Mean, Mode and Median is types of ogive gives the PREP-2022
JULY-2021 & MODEL-I-2021, MODEL : 2022 A) Standard deviation B) Mode C) Mean D) Median
(A) 3 Median = 2 Mean + Mode (B) 3 Mean = 2 Median + Mode
(B) Mean = 3 Median + Mode (D) Mode = 3 Mean + 2 Median ONE MARK QUESTIONS :
4. The formula to find the mid-point of the class interval is JULY-2021
1) Find the mean for the following data. PREP-2019
(A) (B)
7, 6, 5, 0, 7, 8, 9
𝐔𝐩𝐩𝐞𝐫 𝐥𝐢𝐦𝐢𝐭 𝐥𝐨𝐰𝐞𝐫 𝐥𝐢𝐦𝐢𝐭
(C) (D) Ans : Sum of the observations = 7+6+0+7+8+9=42
𝟐
5. A student scored 65 marks in I language, 50 marks in Science, 55 marks in Number of observations = 7
Social Science and some marks in Mathematics. If the average marks scored Mean =
by him in all the four subjects is 60, then the marks scored by him in .

mathematics is A. 65 B. 60 C. 50 D. 70 MODEL-I-2021 Mean = = 6


6. The average marks scored by a student in a test of 6 subjects is 18. The Mean = 6
Sri.Veerendra.H.M. Asst.Teacher. G.P.U.C. Megaravalli. Thirthahalli Tq Ph: 8762624683 Page-167 Sri.Veerendra.H.M. Asst.Teacher. G.P.U.C. Megaravalli. Thirthahalli Tq Ph: 8762624683 Page-168
2) Find the median for the given data : 5, 3, 14, 16, 19 and 20. MODEL-1 Mode = l + (
𝐟𝟏 𝐟𝟎
)h
𝟐𝐟𝟏 𝐟𝟎 𝐟𝟐
Ans : Ascending order : 3, 5, 14, 16 , 19, 20
=3+( )x2
Median = = = 15
=3+( )x2
3) Find the median of the scores 6, 4, 2, 10 and 7. March-2022
= 3 + 0.28
Ans. : Ascending order : 2, 4, 6, 7, 10
Mode = 3.28
Median = 6
4) Find the mode of the following scores : 4, 5, 5, 6, 7, 7, 6, 7, 5, 5 June ∶ 2022
2) Calculate the mode for the following distribution : June- 2019 & MODEL-2019
Ans. : Mode = 5
5) Median of some data is 40 and mean is 39. Then find the mode. MODEL-2021 Class-interval 10-25 25-40 40-55 55-70 70-85 85-100
f 2 3 7 6 6 6
Ans : 3xMedian = Mode + 2xMean
3 x 40 = Mode + 2 x 39
Ans : Class- Frequency
120 = Mode + 78
interval (f)
Mode = 120-78 10-25 2
Mode = 42 25-40 3 f0
40-55 7 f1
55-70 6 f2
70-85 6
85-100 6

THREE MARKS QUESTIONS : Modal class : 40-55


1) Calculate the mode for the following data in the distribution table : l = 40, h = 15, f0 = 3, f1 = 7, f2 = 6
𝐟𝟏 𝐟𝟎
Family size 1-3 3-5 5-7 7-9 9-11 Mode = l + ( )h
𝟐𝐟𝟏 𝐟𝟎 𝐟𝟐
No. of families 7 8 2 2 1 = 40 + ( ) x 15

Ans : JUNE-2019
= 40 + ( ) x 15
Class- Frequency = 40 + 12
interval (f)
Mode = 52
1-3 7 f0
3-5 8 f1
5-7 2 f2 3) The following data gives the information on the observed lifetimes
7-9 2 (in hours) of 225 electrical components. Find modal life time PREP-2019
9-11 1
Lifetimes(in hour) 0-20 20-40 40-60 60-80 80-100 100-120
Modal class : 3-5 Frequency 10 35 52 61 38 29
l = 3, h = 2, f0 = 7, f1 = 8, f2 = 2
Ans :
Sri.Veerendra.H.M. Asst.Teacher. G.P.U.C. Megaravalli. Thirthahalli Tq Ph: 8762624683 Page-169 Sri.Veerendra.H.M. Asst.Teacher. G.P.U.C. Megaravalli. Thirthahalli Tq Ph: 8762624683 Page-170
Class-interval 5)Calculate the mean for the following distribution table from direct method
Frequency
0-20 10 Ans: MODEL-I-2021
20-40 35
40-60 52 f0 Class-
Frequency(f) Mid-point(xi) fi.xi
interval
60-80 61 f1 5-15 4 10 10x4 = 40
80-100 38 f2 15-25 3 20 20x3 = 60
100-120 29 25-35 6 30 30x6 = 180
35-45 5 40 40x5 = 200
Modal class : 60-80
45-55 2 50 50x2 = 100
l = 60, h = 20, f0 = 52, f1 = 61, f2 = 38
𝐟𝟏 𝐟𝟎
Ʃ fi = 20 Ʃ fi.xi = 580
Mode = l + ( )h
𝟐𝐟𝟏 𝐟𝟎 𝐟𝟐 Ʃ
Mean =
= 60 + ( ) x 20 Ʃ

= 60 + ( ) x 20 Mean =

= 60 + Mean = 29
= 60 + 5.625 6) In the following distribution, the mean is 10. Then find ‘x’
Class-interval 1-5 5-10 10-15 15-20
Mode = 65.625
f 2 3 x 1
4) Calculate the mean for the following distribution table from direct method
Ans :
MODEL-I-2020 Class-interval Frequency(fi) Mid-point(xi) fi.xi
Class-interval 0 - 10 10 - 20 20 - 30 30 - 40 40 - 50 1-5 2 3 6
Frequency 3 5 9 5 3 5-10 3 7.5 22.5
Ans: 10-15 x 12.5 12.5x
15-20 1 17.5 17.5
Class-
Frequency(f) Mid-point(xi) fi.xi N = (6+x) ∑ 𝒇𝒊 𝒙𝒊 = (46+12.5x)
interval
0-10 3 5 5x3 = 15 Ʃ
Mean =
10-20 5 15 15x5 = 75 Ʃ
.
20-30 9 25 25x9 = 225 10 =
30-40 5 35 35x5 = 175
10(6+ x) = 46 + 12.5x
40-50 3 45 45x3 = 135
10(6+ x) = 46 + 12.5x
N= 25 Ʃ fi.xi = 625
60 + 10x = 46 + 12.5x
Ʃ 60-46 = 12.5x - 10x
Mean =
Ʃ 2.5 x = 14
𝟏𝟒
Mean = Mean = 25 x= = 5.6
𝟐.𝟓

Sri.Veerendra.H.M. Asst.Teacher. G.P.U.C. Megaravalli. Thirthahalli Tq Ph: 8762624683 Page-171 Sri.Veerendra.H.M. Asst.Teacher. G.P.U.C. Megaravalli. Thirthahalli Tq Ph: 8762624683 Page-172
7) Calculate the median for the following distribution : April-2019 Class- Frequency Cumulative
Class-interval 1-4 4-7 7-10 10-13 13-16 16-19 interval (f) frequency
6 30 40 16 4 4 0-20 6 6
f
Ans : 20-40 8 14
Class- Frequency Cumulative 40-60 10 24 c.f
interval (f) frequency 60-80 12 f 36
1-4 6 6
80-100 6 42
4-7 30 36 c.f
100-120 5 47
7-10 40 f 76
10-13 16 92 120-140 3 50
13-16 4 96 N = 50
16-19 4 100
N = 100 N = 50, = = 25, Median = 25th score
Median class : 60-80
N = 100, = = 50, Median = 50th score
Lower limit of median class : l = 60
Median class : 7-10
C.F. of class preceding median class: c.f. = 24
Lower limit of median class : l=7
Frequency of median class: f = 12
C.F. of class preceding median class: c.f. = 36
Class size: h = 20
Frequency of median class: f = 40 𝐍
( 𝐜.𝐟.)
Class size: h=3 Median = l + 𝟐
xh
𝐍 𝐟
( 𝐜.𝐟.) ( )
Median = l + 𝟐
xh = 60 +
𝐟
( ) ( )
= 7+ = 60 +
( )
= 7+ = 60 +
= 7+ = 60 + 1.66
= 7 + 1.05 Median = 61.66
Median = 8.05
9) The mode of the following distribution table is 15. Find the mean for this
data, and then find the median value by using empirical formula relating
8) Calculate the median for the following distribution : MODEL-2019
mean, median and mode. 4-MODEL-II-2020
Class- 0-20 20-40 40-60 60-80 80-100 100-120 120-140 Ans: Mode = 15
interval
f 6 8 10 12 6 5 3 Calculation of Mean :

Ans :
Sri.Veerendra.H.M. Asst.Teacher. G.P.U.C. Megaravalli. Thirthahalli Tq Ph: 8762624683 Page-173 Sri.Veerendra.H.M. Asst.Teacher. G.P.U.C. Megaravalli. Thirthahalli Tq Ph: 8762624683 Page-174
4) Calculate the mode for the following distribution : MODEL-II-2021
Class- Frequency(fi) Mid-point fi.xi
interval (xi) Class-interval 0-10 10-20 20-30 30-40 40-50
1-5 7 3 3x7 = 21 frequency 6 9 15 9 1
5-9 2 7 7x2 = 14 Ans. : Mode = 25
9-13 2 11 11x2 = 22
13-17 8 15 15x8 = 120 5) Calculate the mode for the following data in the frequency distribution
17-21 1 19 19x1 = 19 table : MODEL-I-2021
N = 20 Ʃ fi.xi = 196
Class-interval 0 - 10 10 - 20 20 - 30 30 - 40 40 - 50
Ʃ 7 9 15 11 8
Mean = Frequency
Ʃ
Mean = Ans: Mode = 26
Mean = 9.8 6) Calculate the mode for the following distribution : JUNE-2020

3x median = 2x mean + mode Class-interval 1-3 3-5 5-7 7-9 9-11


3x median = 2x9.8 + 15 f 6 9 15 9 1
= 19.6 + 15
Ans : Mode = 6
= 34.6
. 7) Calculate the mode for the following distribution : SEPT-2020
Median =
Class-interval 0-5 5-10 10-15 15-20 20-25
Median = 11.53 f 8 9 5 3 1
EXERCISE :
1) Find the mode of scores in the following data. PREP-2022 Ans : Mode = 6
8) Find the mean for the following distribution table from direct method :
Class- 1-4 4-7 7-10 10-13 13-16
interval Class-interval 1-5 5-9 9-13 13-17 17-21
f 7 4 8 6 4 Frequency 4 3 5 7 1
Ans : Mode = 9
2) Find the mode of scores in the following data. March-2022 Ans: Mean = 10.6 MODEL-II-2021

Class-interval 5-15 15-25 25-35 35-45 45-55 9) Find the mean of the following data by “direct method”. MODEL-2022
f 3 4 8 7 3 Class-interval 10-30 30-50 50-70 70-90
Ans : Mode = 33 Frequency 2 6 10 2
3) Find the mode of scores in the following data. MODEL-2022 Ans : Mean = 52

1-3 3-5 5-7 7-9 9-11 10) Find the mean of the following data using direct method. PREP-2022
Class-interval
f 6 9 2 2 1 Class-interval 0-10 10-20 20-30 30-40 40-50
Ans: Mode = 3.6 Frequency 5 8 20 15 7
Ans : Mean = 27
Sri.Veerendra.H.M. Asst.Teacher. G.P.U.C. Megaravalli. Thirthahalli Tq Ph: 8762624683 Page-175 Sri.Veerendra.H.M. Asst.Teacher. G.P.U.C. Megaravalli. Thirthahalli Tq Ph: 8762624683 Page-176
11) Find the mean of the following data using direct method. March-2022 OGIVE GRAPH :
01) Construct ‘Less than type ogive’ for the following distribution. MODEL-2019
Class-
10-20 20-30 30-40 40-50 50-60
interval Class-interval 0-3 3-6 6-9 9 - 12 12 - 15
Frequency 2 3 5 7 3 Frequency 9 3 5 3 1
Ans :
Ans : Mean = 38
Class Frequency Cumulative
Upper limits Points
12) Find the median of the following data. PREP-2020 interval frequency
0-3 9 Less than 3 9 (3, 9)
C.I. 50-60 60-70 70-80 80-90 90-100
3-6 3 Less than 6 12 (6, 12)
Frequency 12 14 8 6 10
6-9 5 Less than 9 17 (9, 17)
Ans: Median = 69.28
9-12 3 Less than 12 20 (12, 20)
13) Calculate the median for the following distribution : JUNE-2020
12-15 1 21 (15, 21)
Less than 15
Class-interval 20-40 40-60 60-80 80-100
Frequency(f) 7 15 20 8
Ans : Median = 63
14) Find the mean of the following scores by direct method : June-2022

Class-
5-15 15-25 25-35 35-45 45-55
interval
Frequency 1 3 5 4 2
Ans. : Mean = 32
15) Find the median of the following scores: JUNE-2022

C.I. 0-20 20-40 40-60 60-80 80-100


Frequency 6 9 10 8 7
Median = 50
16) Calculate the median for the following data in the distribution table :

Weight (in K.G.) 15-20 20-25 25-30 30-35 35-40


No.of students 2 3 6 4 5
Ans : Median = 29.16 JUNE-2019

Sri.Veerendra.H.M. Asst.Teacher. G.P.U.C. Megaravalli. Thirthahalli Tq Ph: 8762624683 Page-177 Sri.Veerendra.H.M. Asst.Teacher. G.P.U.C. Megaravalli. Thirthahalli Tq Ph: 8762624683 Page-178
2) During the medical check
check-up
up of 35 students of a class, their weights were 03) The following table gives production yield per hectare of wheat of 100
recorded as follows. Draw a ‘less than typ
type’
e’ of ogive for the given data :− farms of a village.:− JUNE ∶ 2019 & MODEL-I-2021
June- 2019 & PREP-2019
June
Production yield 50-55
55 55-60 60-65 65-70 70-75 75-80
Weight Less Less Less Less Less Less Less Less
in kg/hectare
( in kg ) than 38 than 40 than 42 than 44 than 46 than 448 than 50 than 52 Number of farms 2 8 12 24 38 16
No. of 0 3 5 9 14 28 32 35
students Change the distribution
tion to a ‘more than type’ distribution, and draw its ogive.
Ans :
Production yield Number of Cumulative
Lower limits Points
in kg/hectare farms frequency
50-55 2 More than or equal to 50 100 (50, 100)
55-60 8 More than or equal to 55 98 (55, 98)
60-65 12 More than or equal to 60 90 (60, 90)
65-70 24 More than or equal to 65 78 (65, 78)
70-75 38 More than or equal to 70 54 (70, 54)
75-80 16 More than or equal to 75 16 (75, 16)

PRODUCTION YIELD

Sri.Veerendra.H.M. Asst.Teacher. G.P.U.C. Megaravalli. Thirthahalli Tq Ph: 8762624683 Page-179 Sri.Veerendra.H.M. Asst.Teacher. G.P.U.C. Megaravalli. Thirthahalli Tq Ph: 8762624683 Page-180
7) Yield of co-conuts grown in a village by ‘15’ farmers is as follows. Draw
EXERCISE :
“less than type” ogive. MODEL-2022
1) The following table gives the production yield per hectare of wheat of 100
Number of Less Less Less Less Less Less Less
Class 40-45 45-50 50-55 55-60 60-65 65-70 coconuts than 50 than 75 than than than than than
interval
Frequency 4 6 16 20 30 24 100 125 150 175 200
Cumulative 2 4 9 10 11 13 15
farms of a village. Draw ‘more than type’ ogive. MODEL-2020 frequency
2) Change the distribution to a ‘less than type’ distribution, and draw its ogive.
Class interval 0-10 10-20 20-30 30-40 40-50 50-60 8) The following table gives production yield per hectare of wheat of 100
No.labours 3 9 15 30 18 5 farms in a village. Draw a “less than type ogive” for the given data.
PREP-2022

3) The following table gives the information of daily income of 50 workers Production yield Less Less Less Less Less Less
of a factory. Draw a ‘less than type ogive’ for the given data. June ∶ 2020 (kg/ hectares) than than 55 than 60 than 65 than 70 than 75
Daily Income Less Less than Less than Less than Less than Less than 50
than100 120 140 160 180 200 Number of farms 2 10 22 46 74 100
No.workers 0 8 20 34 44 50
9) During a medical check recorded as follows : Draw “less than type” ogive
4) An insurance policy agent found the following data for distribution of for the given data : March-2022
ages of 35 policy holders. Draw a “less than type” ( below ) of ogive for the
Heights of Less Less Less Less Less Less than
given data : SEPT ∶ 2020
students than 140 than 145 than 150 than 155 than 160 165
Number of 5 10 15 25 40 50
Age(in Less Less Less Less Less Less Less
years) than 20 than 25 than 30 than 35 than 40 than 45 than 50 student
No. of policy 2 6 12 16 20 25 35
holders 10) The following table gives the information of heights of 60 students of class
X of a school. Draw a ‘less than type’ ogive for the given data : June-2022
5) The following table gives the information of daily income of 50 workers
Heights of Number of student
of a factory. Draw a ‘more than type ogive’ for the given data.:
students
Daily Income 100-120 120-140 140-160 160-180 180-200 Less than 130 04
No.workers 12 14 8 6 10 Less than 140 12
Less than 150 30
6) Draw a ‘less than type’ of ogive for the given frequency distribution table. 45
Less than 160
Class interval 0-10 10-20 20-30 30-40 40-50 Less than 170 56
Frequency 2 12 2 4 3 60
Less than 180
MODEL-II-2021

Sri.Veerendra.H.M. Asst.Teacher. G.P.U.C. Megaravalli. Thirthahalli Tq Ph: 8762624683 Page-181 Sri.Veerendra.H.M. Asst.Teacher. G.P.U.C. Megaravalli. Thirthahalli Tq Ph: 8762624683 Page-182
UNIT-14. PROBABILITY : TWO MARKS QUESTIONS :
1) A die is thrown once. Find the probability of getting a number lying
* The probability of an event E, is
( )
between 2 and 6. MODEL : 2018-19
P(E) = = Ans. : S = { 1,2,3,4,5,6 } n(s) = 6
( )
* The probability of a sure event(certain event) is 1. E = { 3,4,5 } n(E) = 3
( )
* The probability of an impossible event is 0. Probability = P(E) = = =
( )
* The probability of an event E is a number P(E) such that 0≤P(E) ≤ 1 2) A die is thrown twice. What is the probability that
* An event having only one outcome is called an elementary event. The sum of the (i ) 5 will not come up either time ?
Probabilities of all the elementary events of an experiment is 1. (ii) 5 will come up at least once ? PREP-2019
* For any event ‘E’, P(E) + P(E) = 1, where E and E are complementary events. Ans. : Total number of outcomes:
S = { (1 1), (1 2), (1 3), (1 4), (1 5), (1 6)
MULTIPLE CHOICE QUESTIONS :
.....................................................
1) Which one of the following cannot be the probability of an event.MODEL : 2018 (6 1), (6 2), (6 3) , (6 4) ,( 6 5) , (6 6) }
𝐴) 𝐵) -1.5 𝐶) 15% 𝐷) 0.7 n(s) = 36
2) P(E) + P (E) is equal to, PREP-2019 (i ) The possibility of 5 will not come either time = n(E1) = 25
( )
A) 1 B) 2 C) 0 D) 3 Probability = P(E) = =
( )
3) If P (A) = 0·05 then P (A) is APRIL-2019
(ii ) The possibility of 5 will comes at least once:
𝐴) 0.59 𝐵) 0.95 𝐶) 1 𝐷) 1.05
4) Faces of a cubical die numbered from 1 to 6 is rolled once. The probability E2 = { (1 5), (2 5), (3 5) , ( 4 5) ,( 5 5) , (6 5) , (5 1), (5 2), (5 3) , (5 4), (5 6)}
of getting an odd number on the top face is JUNE-2019
n(E2) = 11
𝟑 ( )
𝐴) 𝐵) 𝐶) 𝐷) Probability = P(E) = =
𝟔 ( )
5) The probability of an event ‘E’ is 0.05, then the probability of an event 3) A cubical die numbered from 1 to 6 are rolled twice. Find the probability
‘not E’ is MODEL -1 : 2019-20 of getting the sum of numbers on its faces is 10. APRIL : 2019
𝐴) 0.05 𝐵) 0.95 𝐶)
.
𝐷)
.
Ans. : S = { (1 1), (1 2), (1 3), (1 4), (1 5), (1 6)
6) 26 English alphabet cards ( without repeating any alphabet) are put in a box .....................................................
and shuffled well. If a card is chosen at random then the probability that the (6 1), (6 2), (6 3) , (6 4) ,( 6 5) , (6 6) }
card with an vowel is MODEL -2 : 2019-20 n (S) = 36
𝐴) 𝐵)
𝟓
𝐶) 𝐷) n(A) = { ( 5, 5 ) ( 4, 6 ) ( 6, 4 ) } = 3
𝟐𝟔 ( )
7) The probability of certain(sure) event is PREP-2020
P(A) =
( )
𝟑 𝟏
𝐴) 0 𝐵) 0.5 𝐶) 0.75 𝐷) 1 P(A) = =
𝟑𝟔 𝟏𝟐
𝟐
8) If P ( A ) = , then P ( A ) is SEPT-2020
4) A box contains 90 discs, which are numbered from 1 to 90. If one disc is
𝟑
𝟏
Drawn at random from the box, find the probability that it bears a perfect
A) B) 3 C) 1 D) square number. JUNE-2019
𝟑
Sri.Veerendra.H.M. Asst.Teacher. G.P.U.C. Megaravalli. Thirthahalli Tq Ph: 8762624683 Page-183 Sri.Veerendra.H.M. Asst.Teacher. G.P.U.C. Megaravalli. Thirthahalli Tq Ph: 8762624683 Page-184
Ans. : S = { 1,2,3,......89,90 } n(s) = 90 S = { Total bulbs }
E = { 1,4,9,16,25,36,49,64,81 } n(E) = 9 n(S) = 28
( ) Defective bulbs = 7
Probability = P(E) = = =
( ) E = Non-Defective bulbs = 21
5)Two cubical dice whose faces are numbered 1 to 6 are rolled simultaneously n(E) = 21
( )
once. Find the probability that the sum of the two numbers occurring on Probability = P(E) = = =
( )
their top faces is more than 7. MODEL-1 : 2019-20
8) Letters of English alphabets A ,B, C, D, E, I are marked on the faces of a
Ans. : S = { (1 1), (1 2), (1 3), (1 4), (1 5), (1 6)
..................................................... cubical die. If this die is rolled once, then find the probability of getting a
(6 1), (6 2), (6 3) , (6 4) ,( 6 5) , (6 6) } vowel on its top face. SEPT-2020
n (S) = 36
Ans. :
The sum of the two numbers occurring on their top faces is more than 7.
S = { A, B, C, D, E, I }
A = { ( 2, 6 ), ( 3, 5 ), ( 3, 6 ), ( 4, 4 ), ( 4, 5 ), ( 4, 6 ), ( 5, 3 ), ( 5, 4 ), ( 5, 5),
( 5, 6 ), ( 6, 2 ), ( 6, 3 ), ( 6, 4 ), ( 6, 5 ), ( 6, 6 )} n(S) = 6
n(A) = 15 E = Vowel on its top face. = { A, E, I }
Probability = P(A) =
( ) n(E) = 3
( ) ( )
Probability = P(E) = = =
P(A) = = ( )

6) If ‘A’ is an event of a random experiment, such that P(A) : P(A) = 1 : 2, 9) A game of chance consists of rotating an arrow which
find the value of P(A). MODEL-2 : 2019-20 comes to rest pointing at one of the numbers 1, 2, 3, 4,
Ans. : 5, 6, 7, 8 and these are equally possible outcomes.
( ) Find the probability that it will point at an odd
Given that, =
( ) number. SEPT-2020
P(A) = 2P(A) --------- (1) Ans. :
We know that, S = { 1, 2, 3, 4, 5, 6, 7, 8 }
P(A) + P(A) = 1 n(S) = 8
P(A) = 1 – P(A) ---------- (2) E = Getting an odd number = { 1, 3, 5, 7 }
Substitute Eqn(2) in Eqn(1), n(E) = 4
P(A) = 2(1 – P(A) ) ( )
Probability = P(E) = = =
P(A) = 2 – 2P(A) ( )
P(A) + 2P(A) = 2 10) Cards marked with numbers 2 to 101 are placed in a box and mixed
3P(A) = 2 thoroughly. One card is drawn from this box. Find the probability that
𝟐
P(A) = number of the is : a) An even number
𝟑
7) A box contains 28 bulbs of which 7 bulbs are are defective, a bulb is drawn b) A perfect square number
randomly from the box. Find the probability of picking a non-defective bulb. Ans. : S = { 2,3,......, 100,101 } n(s) = 100
Ans. : PREP-2020 𝑎) Getting an even number : A = { 2,4,6,..........100 } n(A) = 50

Sri.Veerendra.H.M. Asst.Teacher. G.P.U.C. Megaravalli. Thirthahalli Tq Ph: 8762624683 Page-185 Sri.Veerendra.H.M. Asst.Teacher. G.P.U.C. Megaravalli. Thirthahalli Tq Ph: 8762624683 Page-186
( )
Probability = P(A) = = =
( ) UNIT.15 : SURFACE AREA AND VOLUMES :
𝑏) A perfect square number : B = { 4,9,16,25,36,49,64,81,100 } n(E) = 9 IMPORTANT FORMULAE :
( )
Probability = P(B) = =
( ) Name of the
L.S.A. L.S.A. Volume
Solid
11) Write the probability of sure event and imposible event. V = lbh
Cuboid A = 2h(l + b) A = 2(lb+bh+hl)
Ans. : Probability of a sure event = 1 V = a3
Cube A = 4a2 A = 6a2
Probability of an imposible event = 0 V = πr2h
Cylinder A = 2πrh A = 2πr(r+h)
12) Two coins are tossed simultaneously. Find the probability of getting atleast V = πr2h
Cone A = πrl A = πr(r+l)
one head?
Ans. : S = { HH, HT, TH, TT } n(s) = 4 Frustum of V = πh(R2 + r2 + Rr)
A = π(R+ r)l A = π(R+ r)l + πR2 + πr2
cone
Getting atleast one head = E = { HH, HT, TH }
V = πr3
n(E) = 3 Sphere A = 4πr2 A = 4πr2
( )
Probability = P(E) = = Hemisphere A = 2πr2 A = 3πr2
V = πr3
( )

THREE MARKS QUESTIONS : Slant height of cone = l = √ℎ + 𝑟


1) A bag contains 3 red balls, 5 white balls and 8 blue balls. One ball is taken Slant height of frustum of cone = l = ℎ + (𝑅 − 𝑟)
out of the bag at random. Find the probability that the ball taken out is MULTIPLE CHOICE QUESTIONS :
(a) a red ball, (b) not a white ball. JUNE-2020
1) The formula to find the lateral surface area of the cone whose radius is (r),
Ans :
height(h) and slant height is (l) Model : 2019
Total number of balls in a bag = n(S) = 16
a) Number of red balls = n(R) = 3 𝐴) π𝑟l 𝐵) 2π(r + l) 𝐶) 2𝜋r(𝑟 + h) 𝐷)
( ) 2)The curved surface area of a cylinder of radius ‘r’ cm and height ‘h’ cm is
Probability of drawing red balls P(R) = =
( )
A. 2πr(r+h)cm2 B. πr2h cm3 C. cm3 D. 2πrh cm2
b) Number of not a white ball = n(A) = 8 + 3 = 11
( )
3) The relation between the slant height ‘l’ height ‘h’ and the radius of the
Probability of drawing not a white ball P(B) = = cone ‘r’ is Model-1: 2021
( )
2 2 2 2 2 2 2 2 2
A. l = h - r B. l = h + r C. h = l + r D. l = √ℎ − 𝑟
4) The formula to find the lateral surface area of a frustum of a cone whose
circular ends having radii r1 and r2 and slant height ‘l’ units, is PREP-2022
A) 𝜋 ( r1 - r2 )l sq.units B) 𝜋r2 sq.units
C) 𝜋 ( r1 + r2 )l sq.units D) 𝜋( r1 - r2 )h sq.units

Sri.Veerendra.H.M. Asst.Teacher. G.P.U.C. Megaravalli. Thirthahalli Tq Ph: 8762624683 Page-187 Sri.Veerendra.H.M. Asst.Teacher. G.P.U.C. Megaravalli. Thirthahalli Tq Ph: 8762624683 Page-188
5) Surface area of a sphere of radius ‘r’ unit is March-2022 13) In the given figure, the volume of the frustum of a cone is March-2022
A) 𝜋 r2 sq.units B) 2𝜋r2 sq.units A) 𝜋 ( r1 + r2 ) l r2
C) 3𝜋r sq.units
2 D) 4𝜋r2 sq.units B) 𝜋 ( r1 - r2 ) l
6) Volume of a sphere of radius ‘r’ unit is June-2022 C) 𝜋h ( r12 - r22 - r1 r2 ) l h
A) π r2 cubic units B) π r3 cubic units
D) 𝜋h ( r12 + r22 + r1 r2 )
3 2
C) π r cubic units D) π r cubic units. r1
7) If two solid hemispheres of same radius are joined together along their 14) A cone is cut by a plane parallel to its base and the small cone that
bases. The surface area of this new solid is, obtained is removed then the remaining part of the cone is MODEL-2022

A) 4πr2 B) 2π r2 + 2 π R C) 2π r2 D) π r3 A) a frustum of cone B) a frustum of cylinder


C) a Sphere D) a right circular cone
8) A cone is mounted on a hemisphere of radius ‘r’cm. and height of the
15) Prepare a cone from “model clay”. When wet, cut it with a knife parallel
cone h cm. Then the volume of the solid is PREP-2020
3 2 3 3 2 3 to its base, remove the smaller cone obtained. The solid left is a Model-1: 2021
A) ( 𝜋𝑟 + 𝜋𝑟 ℎ ) 𝑐𝑚 B) ( 𝜋𝑟 + 𝜋𝑟 ℎ ) 𝑐𝑚 A. Cylinder B. Cone C. Sphere D. Frustum of a cone
C) ( 𝜋𝑟3 + 𝜋𝑟2ℎ ) 𝑐𝑚3 3 2
D) ( 𝜋𝑟 + 𝜋𝑟 ℎ ) 𝑐𝑚
3 16) The volume and the surface area of a sphere is numerically equal, then
9) The solid is in the shape of a cylinder with two hemispheres stuck to each the radius of the sphere is Model-5: 2021
A. 3 units B. 2 units C. 2.5 units D. 6 units
of its ends asshown in the figure.The radius of the cylinder and hemispheres
are equal to ‘r’ cm, if the height of the cylinder is ‘h’ cm. The volume of the 17) A funnel given in the figure is the combination of Model-2: 2021

solid is Model-2: 2021 A. a cylinder and a cone


2
A. πr ( + h) cm 3 B. a cylinder and a frustum of a cone
C. a cone and a hemisphere
B. πr2( + h) cm3 D. a cylinder and a cuboid
C. πr2( + h) cm3
18)A cylinder made of wax is melted and recast
D. πr2( + h) cm3
completely in to a sphere. Then the volume of the sphere is JULY-2021
10) The volume of a cone as shown in the figure is JULY : 2021
(A) two times the volume of the cylinder
(A) 𝜋r2ℎ
(B) half the volume of the cylinder
(B) 𝜋r ( r +l)
(C) 3 times the volume of the cylinder
(C) 𝜋r2ℎ h l
(D) equal to the volume of the cylinder
(D) 𝜋rl
19) A frustum of a cone shaped Jaggery is melted and remoulded completely
r
11) The formula to find the total surface area of a right circular based to the shape of a sphere. Then the volume of the sphere is Model-2: 2021
cylindrical vessel of base radius r cm and height h cm opened at one end is A. 3 times the volume of the frustum
2
(A) (𝜋r + 2𝜋rh ) cm2
𝟏
(B) 2𝜋rh cm2 (C) 𝟑 𝜋r2ℎ cm3 (D) (𝜋r2 + h ) cm2 B. 2 times the volume of the frustum
C. half the volume of the frustum
12) To find the curved surface area of a frustum of a cone as shown in the
D. equal to the volume of the frustum
figure the formula used is JULY-2021
20) The circumference of a cylinder is 44cm and height is 10cm. Then its
(A) 𝜋l ( r1 + r2 )
curved surface area is PREP-2019
(B) 𝜋h ( r12 + r22 + r1 r2 ) A) 120cm 2
B) 440cm 2
C) 110cm 2
D) 220cm2

(C) 𝜋l ( r1 + r2 )
(D) 𝜋l ( r1 - r2 )
Sri.Veerendra.H.M. Asst.Teacher. G.P.U.C. Megaravalli. Thirthahalli Tq Ph: 8762624683 Page-189 Sri.Veerendra.H.M. Asst.Teacher. G.P.U.C. Megaravalli. Thirthahalli Tq Ph: 8762624683 Page-190
21) If the area of the circular base of a cylinder is 22 cm2 and its height is
ONE MARK QUESTIONS :
10 cm, then the volume of the cylinder is JUNE-2020
2 3 3 2
𝐴) 2200 𝑐𝑚 𝐵) 2200 𝑐𝑚 𝐶) 220 𝑐𝑚 𝐷) 220 𝑐𝑚 1) Write the formula to calculate the curved surface area of the frustum of a
22) The base radius and height of a right circular cylinder and a right circular cone. Ans. : C.S.A. of the frustum of a cone = π l (r1 + r2) PREP-2020
cone are equal and if the volume of the cylinder is 360 cm3 , then the 2) Write the formula to calculate the total surface area of the frustum of a
volume of cone is Sept ∶ 2020 cone. April-2019
3 3 3 3
𝐴) 120 𝑐𝑚 𝐵) 180 𝑐𝑚 𝐶) 90 𝑐𝑚 𝐷) 360 𝑐𝑚 Ans. : T.S.A. of the frustum of a cone = π l (r1 + r2) + π(r12 + r22)
23) The volume of a cylinder is 300 m3 then the volume of a cone having the 3) Write the formula to calculate the area of the sphere. PREP-2019
same radius and height as that of the cylinder is : Model-1: 2021 Ans : Area of the sphere = 4 π r2
3 3 3 3
A. 900 m B. 600 m C. 150 m D. 100 m
4) Write the formula to calculate the total surface area of the cone whose
24) The perimeter of the base of a right circular cylinder is 44cm and its
radius is ‘r’ units and slant height is ‘l’ units. June-2020
height is 10cm then its volume is Model-1: 2021
A. 490 π cm3 B. 440 π cm3 C. 374 π cm3 D. 980 π cm3 Ans: π r (𝑟 + l)
25) The height and the radius of the base of a cone are 12cm and 5cm 5) Write the formula to find the curved surface area of the frustum of a cone
respectively. Then the slant height of the cone is Model-2: 2021 As shown in the figure. r2 Model-1-2021
A. 12 cm B. 10 cm C. 13 cm D. 8 cm
h
26) The surface area of a sphere of radius 7 cm is. Sept ∶ 2020 & Model-1: 2021
2 2 2 2
A. 154 cm B. 308 cm C. 616 cm D. 770 cm r1
27) The surface area of a sphere is 616 sqm. The surface area of its
hemisphere is Model-1: 2021 Ans : C.S.A. of frustum of a cone = π(𝑟1 +𝑟2)l + π(r12 + r22)
A. 205.6 cm2 B. 308 cm2 C. 462 cm2 D. 38 cm2 6) Write the formula to find the total suface area of a right cylinder ?
28) The surface area of a sphere is 616 sq.cm. Then the radius of the same
Ans: Total suface area of a right cylinder = A = 2πr(r+h) Model-2-2021
sphere is JULY-2021
7) Write the formula to find the volume of a solid sphere ? Model-2-2021
(A) 49 cm (B) 14 cm (C) 21 cm (D) 7 cm
29) The total surface area of solid hemisphere is 462 cm2. If the curved Ans : Volume of a solid sphere = V = x πr3
surface area of it is 308 cm2 , then the area of the base of the hemisphere is 8) Write the mathematical relation between slant height (l) height (h) and
(A) 308 cm2 (B) 231 cm2 (C) 154 cm2 (D) 1078 cm2 JULY : 2021 radius (r) of a cone ? Model-2-2021
30) If the slant height and radius of a cone are 7cm and 2cm respectively, then 2 2
Ans : l = r + h 2

the Curved (Lateral) Surface Area of the Cone is, 9) Write the formula to find the volume of a cone. MODEL-2022
A) 56.57 cm2 B) 44 cm2 C) 88 cm2 D) 33 cm2
Ans : Volume of a cone = 𝜋r ℎ cubic units.
2
31) A cup is in the shape of a frustum of a Cone of height 6cm. The radii of its
10) Write the formula to find the surface area of sphere whose radius is ‘r’
two circular bases are 2 cm and 1 cm. Then the maximum water, that be
stored in this glass is, units. PREP-2022
A) 22 cm3 B) 44 cm3 C) 88 cm3 D) 100 cm3 Ans : Surface area of a sphere = 4𝜋r2 square units.
11) Find the surface area of a sphere of radius 7cm. MODEL-2022
Answers : 1.A, 2.D, 3.B, 4.C, 5.D, 6.C, 7.A, 8.D, 9.A, 10.C, 11.A, 12.C, 13.D, 14.A, 15.D, Ans : Surface area of a sphere = 4𝜋r2 = 4x x7x7 = 616cm2
16.A, 17.B, 18.D, 19.D, 20.B, 21.C, 22.A, 23.D, 24.A, 25.C, 26.C, 27.B, 28.D,
29.C, 30.B, 31.B

Sri.Veerendra.H.M. Asst.Teacher. G.P.U.C. Megaravalli. Thirthahalli Tq Ph: 8762624683 Page-191 Sri.Veerendra.H.M. Asst.Teacher. G.P.U.C. Megaravalli. Thirthahalli Tq Ph: 8762624683 Page-192
12) In the given figure, write the formula used to find the curved surface area 2) The faces of two cubes of volume 64 cm3 each are joined together to form a
of the cone. V March-2022 cuboid. Find the total surface area of the cuboid. June-2019
Ans : Volume of cube =V= 64 𝑐𝑚 3

h l Length of side = a = 4cm 4


Length of the cuboid = l = 4+4=8cm 4 64𝑐𝑚3 64𝑐𝑚3
Breadth of the cuboid = b = 4cm 4 4
A O r B Height of the cuboid = h = 4cm
Ans. : Curved surface area of cone = πrl sq units Total surface area of cuboid = 2(lb + bh + hl)
13) Find the length of the side of a cube whose volume is 64 cm3. PREP-2022
= 2(8x4 + 4x4 +8x4)
Ans : Volume of a cube = a3 = 2(32 + 16 +32)
a3 = 64
= 2x80
a = √64 Total surface area of cuboid = 160 cm2
∴ length of the side of a cube = a = 4 cm 3) A metallic sphere of radius 9 cm is melted and recast into the shape of a
14) The dimensions of cubiodal shape iron ball is 10cm X 5cm X 2cm. cylinder of radius 6 cm. Find the height of the cylinder. June-2019
Find its volume. Model : 2019 Ans : Radius of the metallic sphere = 9 cm, cylinder of radius = 6 cm.
Ans : Volume of a cubiod = lxbxh = 10cmX5cmX2cm = 100cm3 Volume of the sphere = Volume of the cylinder
15) The radius of the base and slant height of a cone is 21cm and 35cm
x πr3 = 𝜋R2ℎ
respectively, then find the height of the cone. Model : 2019
Ans : Radius of the cone = r = 21 cm x 93 = 62ℎ
Slant height of cone = l = 35 cm., Height of the cone = h = ? 4x243 = 36h
By Pythagoras theorem, r + h2 = l2
2 h = 27 cm.
h2 = l2 - r
2 4) The circumference of the base of a cylinder is 132cm and its height is 25cm.
h2 = 352- 212 h2 = 1225 - 441 h2 = 784 Find the volume of the cylinder? 2-MODEL-2021
Ans : Perimeter of the base of a cylinder = 132cm, Height = h = 25cm
h = 28 cm.
Height of the cone = 28 cm. 2πr = 132cm
πr = 66
16) Write the formula to find the volume ( V ) of the frustum of a cone of
height h and radii of two circular ends r1 and r2 . June-2022 r = 66 x h = 25cm
2 2
Ans. : V = πh( r1 + r2 + r1r2 ) cubic units r = 21 cm
TWO MARKS QUESTIONS : Volume of a cylinder =V = πr2h
1) Volume of a cube is 64𝑐𝑚3. Find the total surface area of the cube. Model-II-20 V= x212x25
Ans : Volume of a cube =V= a3 = 64 𝑐𝑚3 V = x21x21x25 = 22x21x3x25
a = √64 cm 3
64𝑐𝑚3 V = 34,650 cm3
Length of the side = a = 4 cm 5) The curved surface area of a right circular cylinder is 440cm2 and radius of
Total surface area of cube = 6a2 = 6x 42 = 6x16 the base is 7 cm. Find the volume of the cylinder. Model-II-2020
∴ Total surface area of cube = 96 cm
2 Ans : C.S.A of cylinder = 440cm2
Sri.Veerendra.H.M. Asst.Teacher. G.P.U.C. Megaravalli. Thirthahalli Tq Ph: 8762624683 Page-193 Sri.Veerendra.H.M. Asst.Teacher. G.P.U.C. Megaravalli. Thirthahalli Tq Ph: 8762624683 Page-194
C.S.A of cylinder = A = 2πrh = 440cm2 8) The radii of two circular ends of a frustum of a cone shaped dustbin are

2x x7xh = 440 15 cm and 8 cm. If its depth is 63 cm, find the volume of the dustbin.
Ans. : r1 = 15 cm r2 = 8 cm h = 63 cm April-2019
44h = 440
Volume of Frustum of cone = V = π h(r12 + r22 + r1r2)
h = 10 cm
Volume of cylinder = 𝜋r ℎ
2 7cm V= x x63(152 + 82 + 15x8)
= x72x10 V= x 22 x9(225 + 64 + 120)
= 22x7x10 V = 22x3 (409)
Volume of cylinder = 1540cm3
∴ Volume of dustbin =V = 26,994cm3
6) A copper rod of diameter 1 cm and length 8 cm is drawn into a wire of
9) The slant height of a frustrum of a cone is 4cm and perimeters of its
length 18 m of uniform thickness. Find the thickness of the wire. Model : 2019
Ans : Volume of cylinder = 𝜋r2ℎ circular bases are 18cm and 6cm, find the curved surface area of the
frustrum of a cone. MODEL-2021
𝜋r12ℎ1 = 𝜋r22ℎ2 8cm Ans : Perimeter of the base = 18cm.
r 1 ℎ 1 = r 2 ℎ2
2 2
⇒ 2𝜋r1 = 18 ⇒ 𝜋r1 = 9 cm
(0.5)2x8 = r22x(18x100) Perimeter of the top = 6 cm
0.25x8 = r22x1800 ⇒ 2𝜋r2 = 6 ⇒ 𝜋r2 = 3 cm
2 = r22x1800 Curved surface area of the frustrum of a cone = (𝜋r1 + 𝜋r2 )l
r22 = 2
∴ C.S.A. of the frustrum of cone = (9 + 3)4 = 48 cm
r22 =
r2 = cm THREE MARKS QUESTIONS :
1) A right circular metalic cone of height 20cm and base radius 5cm is melted
Thickness of the wire = Diameter of the wire = 2x cm
and recast into a solid sphere. Find the radius of the sphere. MODEL-1
∴ Thickness of the wire = cm Ans :
7) A drinking glass is in the shape of a frustum of cone of height 14 cm. The Radius of the cone = r = 5cm
diameters of its two circular ends are 4cm and 2 cm. Find the capacity of Height of the cone = h = 20cm
the glass. PREP-2019
Ans : Height of the glass = h = 14cm 4cm
Radius of solid sphere = R = ?
Diameters of frustum of cone = 4 cm , 2 cm Volume of the sphere = Volume of the cone
πR3 = πr2h
Radius = r1 = 2 cm h= 14cm
4R3 = r2h
r2 = 1 cm
4R3 = 52x20
Volume of frustum of cone = V = π h(r12 + r22 + r1r2) 2cm R3 = 52x5
V= x x14(22 + 12 + 2x1) R = 5 cm
∴ Radius of solid sphere = R = 5 cm
V= x 22 x2(4 + 1 + 2)
2) A solid sphere of radius 3cm is melted and reformed by stretching it into a
V = x22x2x7
cylindrical shaped wire of length 9m. Find the radius of the wire. MODEL-1
∴ Capacity of the glass = V = 102.66cm3
Sri.Veerendra.H.M. Asst.Teacher. G.P.U.C. Megaravalli. Thirthahalli Tq Ph: 8762624683 Page-195 Sri.Veerendra.H.M. Asst.Teacher. G.P.U.C. Megaravalli. Thirthahalli Tq Ph: 8762624683 Page-196
Ans : = x3.5x3.5x10 + x x3.5x3.5x3.5
Radius of solid sphere = R = 3cm = 22x3.5x5 + x22x3.5x3.5
Height of the cylinder = h = 9m = 900cm
= 77x5 + x77x3.5
Radius of the cylinder = r = ?
= 385 + 89.83
Volume of the cylinder = Volume of the sphere Volume of the toy = 474.83 cm3
πr2h = πR3
5) A tent is in the shape of a cylinder surrounded by a conical top as shown in
r2h = R3 the figure. The height and radius of the cylindrical part are 2.1m and 2m
r2x900 = x33 respectively. Also slant height of the cone is 2.8m. Find the area of the
canvas used for making the tent. PREP-2022
r2x900 = 4 x32
r2 = 2.8m

r2 =
2m
r=
r = =0.2cm
2.1 m
∴ Radius of the wire = 0.2cm

3) The slant height of a frustum of a cone is 4cm and the perimeters of its
circular ends are 18cm and 16cm, then find the curved surface area of the
frustum of the cone. MODEL-2022
Ans : Height of the cylinder = h = 2.1 m
Ans : Slant height = l = 4cm Radius = r = 2 m
Perimeter of the base = 18cm. Slant height of the cone = l = 2.8 m
Area of the canvas = C.S.A. of cylinder + C.S.A. of cone
⇒ 2𝜋r1 = 18 ⇒ 𝜋r1 = 9 cm
= 2πrh + πrl
Perimeter of the top = 16 cm
= πr(2h + l)
⇒ 2𝜋r2 = 16 ⇒ 𝜋r2 = 8 cm
Curved surface area of the frustrum of a cone = (𝜋r1 + 𝜋r2 )l = x2(2x 2.1 + 2.8)
∴ C.S.A. of the frustrum of cone = (9 + 8)4 = 68 cm
2 = x2(4.2 + 2.8)
= x2x7
4) A Toy is in the form of a hemisphere surmounted on a h= 10cm
cylinder of height 10cm as shown in the figure. If the radius ∴ Area of the canvas = 44 m2
of the cylinder is 3.5cm find the volume of the toy. r = 3.5cm
Ans : Height of the cylinder = h = 10cm MODEL-2022
6) The volume of a solid right circular cylinder is 2156 cm of the cylinder is
𝟐𝟐
Radius = r = 3.5 cm 14 cm, then find its curved surface area. [ Take π = ] March-2022
𝟕
Volume of the toy = volume of cylinder + volume of hemisphere Ans : V = 2156 cm 3

= πr2h + πr3 h = 14 cm
r=?
= x3.52x10 + x x3.53
Sri.Veerendra.H.M. Asst.Teacher. G.P.U.C. Megaravalli. Thirthahalli Tq Ph: 8762624683 Page-197 Sri.Veerendra.H.M. Asst.Teacher. G.P.U.C. Megaravalli. Thirthahalli Tq Ph: 8762624683 Page-198
Curved Surface Area = ? Volume of vessel = Volume of cylinder - Volume of cone
Volume of cylinder = πr h 2 Volume of vessel = 3080 - 154
Volume of vessel = 2926 cm3
2156 = x r2x14 2
1,000 cm3 = 1 litre
2156 = 22x r2 x2
r2 = ∴ 2926 cm3 = = 2.926 litre
r2 = 49 ∴ 𝐂𝐨𝐬𝐭 𝐨𝐟 𝐦𝐢𝐥𝐤 𝐭𝐨 𝐟𝐢𝐥𝐥 𝐭𝐡𝐢𝐬 𝐯𝐞𝐬𝐬𝐞𝐥 𝐚𝐭 𝐭𝐡𝐞 𝐫𝐚𝐭𝐞 𝐨𝐟 𝐑𝐬. 𝟐𝟎 𝐩𝐞𝐫 𝐥𝐢𝐭𝐫𝐞 = 2.926x 20
r = √49 = Rs. 58.52
r = 7 cm
Curved Surface Area of cylinder = 2πrh 2) A hemispherical vessel of radius 14 cm is fully filled with sand. This sand is
= 2x x 7x 14 poured on a level ground. The heap of sand forms a cone shape of height
7 cm. Calculate the area of ground occupied by the circular base of the
= 2x22x14
heap of the sand. April-2019
= 616 cm2
Ans. : Radius of hemisphere = r = 14 cm
Height of cone = h = 7cm
FOUR MARKS QUESTIONS :
Volume of the hemisphere = πr3
1) The bottom of a right cylindrical shaped vessel made from metallic sheet is
Volume of the cone = πr2h
closed by a cone shaped vessel as shown in the figure. The radius of the
circular base of the cylinder and radius of the circular base of the cone each Radius of cone = R = ?
is equal to 7 cm. If the height of the cylinder is 20 cm and height of cone is Volume of the hemisphere = Volume of the cone
3 cm, calculate the cost of milk to fill completely this vessel at the rate of
πr3 = πR2h
Rs. 20 per litre. April-2019
Ans : Radius of cylinder = 7cm 2 x (14)3 = R2x7
Height of cylinder = 20cm R2 =
Radius of cone = 7cm R2 = 2x14x14x2
R = √2𝑥2𝑥14𝑥14
Height of cone = 3cm
R = 2x14
Rate of one litre milk = Rs.20 R = 28cm
Volume of cylinder =V = πr2h ∴ The area occupied by the circular base of the heap of the sand on the ground = π r2
V= x72x20 = x 282
V = x7x7x20 = 22x7x20 = x 28x28
Volume of cylinder = V = 3080 cm3 Area = 2464 cm2
Volume of cone = V = πr2h
3) A milk tank is in the shape of a cylinder with hemispheres of same radii
= x x72x3 attached to both ends of it as shown in figure. If the total height of the tank is
= x x7x7x3 6 m and the radius is 1 m, calculate the maximum quantity of milk filled in
= 22x7 the tank in litres. ( π = ) JUNE-2019

Volume of cone = 154 cm3 Ans : Radius of the hemisphere = r = 1m.


Sri.Veerendra.H.M. Asst.Teacher. G.P.U.C. Megaravalli. Thirthahalli Tq Ph: 8762624683 Page-199 Sri.Veerendra.H.M. Asst.Teacher. G.P.U.C. Megaravalli. Thirthahalli Tq Ph: 8762624683 Page-200
Total height of the tank = 6m h = 5cm
∴ Height of the cylinder = h = (6-2) = 4m. ∴ Height of frustum of cone = BP = AP – AB = 20 - 5= 15 cm
Volume of = Volume of + Volume of + Volume of Volume of frustum of cone = V = π h(r12 + r22 + r1r2)
Milk tank hemisphere cylinder hemisphere
= x x15(32 + 122 + 3x12)
Volume of milk tank = πr3 + πr2h + πr3 Radius =1m = x x15(9 + 144 + 36)
3 2
Volume of milk tank = πr + πr h = x x15(189)
2
Volume of milk tank = πr ( r + h) 6m = 22x15x9
= 2
x1 ( x1 + 4) ∴ Volume of frustum of cone = 2970cm3
= ( )
5) A container opened from the top is in the form of a
Volume of milk tank = 16.762m3
frustum of a cone of height 16 cm with radii of its
1m3 = 1,000 litre. lower and upper ends are 8 cm and 20 cm. respecti –
∴ Quantity of milk filled in the tank in litres = 16.762x1,000 = 16,762litres vely. Find the cost of the milk which can completely
fill the container at the rate of Rs. 20 per litre. (π= 3.14)
4) A cone is having its base radius 12 cm and height 20 cm. If the top of this Ans : Height of frustum of cone = h = 16cm April∶ 2020
cone is cut in to form of a small cone of base radius 3 cm is removed, then the Radius = r1 = 8cm
remaining part of the solid cone becomes a frustum. Calculate the volume of
the frustum. JUNE-2019
Radius = r2 = 20cm
Volume of frustum of cone = V = π h(r12 + r22 + r1r2)
A

h1
= x3.14 x16(82 + 202 + 8x20)

B r1 C = x 3.14 x16(64 + 400 + 160)


20 cm.
= x 3.14 x16x624
= 3.14 x 208
Volume of frustum of cone = V = 10449.9 cm3
P r2 Q 1 Litre = 1000 cm3
∴ 10450 cm3 = = 10.45 Litre
Ans : In ∆APQ , PQ ║ BC, By corollary of Thales theorem, Rate of 1 litre milk = Rs.20
∴ Cost of 10.45 litre milk = 10.45x20 = Rs.209
=
6) A right circular cone of height 30 cm is cut and removed by a plane
= parallel to its base from the vertex. If the volume of smaller cone obtained
𝟏
= is 𝟐𝟕 of the volume of the given cone, calculate the height of the remaining
h = part of the cone. Sept ∶ 2020

Sri.Veerendra.H.M. Asst.Teacher. G.P.U.C. Megaravalli. Thirthahalli Tq Ph: 8762624683 Page-201 Sri.Veerendra.H.M. Asst.Teacher. G.P.U.C. Megaravalli. Thirthahalli Tq Ph: 8762624683 Page-202
Length of entire capsule = 14mm.
A
∴ Height of cylinder = h = (14-5) = 9mm.
h1
Surface Area = Surface Area + Lateral Surface + Surface Area
B r1 C of capsule of hemisphere Area of cylinder of hemisphere
30 cm. Surface area of = 2πr2 + 2πrh +2πr2
the capsule = 4πr2 + 2πrh
= 2πr(2r + h)

P r2 Q
= 2x x2.5(2x2.5 + 9)
= 2x x2.5x14
Ans : In ∆APQ, PQ ║ BC, By Thale’s corollary = 2x22x2.5x2
∴ Surface area of the capsule = 220 Sq.m.m.
=
= -----------------(1) 8) A toy is made in the shape of a cylinder with one hemisphere stuck to one
Volume of cone = x Volume of given cone. end and a cone to the other end, as shown in the figure, the length of the
cylindrical part of the toy is 20cm and its diameter is 10cm. If the slant height
π xr12xh1 = x πx r22xh2 of the cone is 13cm. Find the surface area of the toy. Model-1
r12 x h1 = x r22x 30
( )2 =
( )2 =

h13 =
h13 = 10x10x10 Ans : The length of the cylindrical part = h = 20cm
AB = h1 = 10 cm The diameter of the cylindrical part =10cm, Radius = r = 5cm
∴ Height of the remaining part of the cone is = BP = AP – AB = 30-10=20cm The slant height of the conical part = l = 13cm
Surface area = Curved surface area + Curved surface area + Curved surface area
7) A medicine capsule is in the shape of a cylinder with hemispheres stuck to
each of its ends. The length of the entire capsule is 14 mm and the diameter of of the toy of hemisphere of cylinder of cone
the capsule is 5 mm. Find its surface area. Sept ∶ 2020 = 2πr2 + 2πrh + πrl
= πr ( 2r + 2h + l)
= x 5(2x5+2x20+13)
5mm.
= x 5(10+40+13)
14mm = x 5(63)
Ans : Diameter of hemisphere = 5 mm. ∴ Surface area of the toy = 990cm2
∴ Radius = r = 2.5mm.
Sri.Veerendra.H.M. Asst.Teacher. G.P.U.C. Megaravalli. Thirthahalli Tq Ph: 8762624683 Page-203 Sri.Veerendra.H.M. Asst.Teacher. G.P.U.C. Megaravalli. Thirthahalli Tq Ph: 8762624683 Page-204
9) A toy is in the form of a cone mounted on a hemisphere with the some r22 = 18x18x2x2
radius is as shown in the figure. If the diameter of the conical portion is 6cm r2 = 18x2 = 36 cm
and its height is 4cm, then find the surface area of the toy. Model-2 Radius of the conical shaped heap = r2 = 36 cm
Ans : Diameter of the conical portion = 6cm , Slant height = l = √𝑟 + ℎ
Radius = r = 3cm, Height = h = 4cm = √36 + 24
Slant height of the cone = l = √ℎ + 𝑟 = √1296 + 576
l = √4 + 3 Slant height = l = √1872 = 12√𝟏𝟑 cm
l = √16 + 9
l = √25 11) A toy is in the form of a cone of radius 21 cm,
Slant height of the cone = l = 5cm mounted on a hemisphere of same radius, as
Surface area = Curved surface area + Curved surface area shown in the figure. The total height of the toy is
49 cm. Find the surface area of the toy. June-2022
of the toy of hemisphere of cone
2
= 2πr + πrl Ans. : Radius of cone = Radius of hemisphere = r = 21 cm
= πr (2r + l) Total height of the toy = 49 cm
Height of the cone = ( 49 – 21 ) cm = h = 28 cm
= 3.14x3( 2x3 + 5)
Slant height of the cone = l = √𝑟 + ℎ
= 3.14x3x11
= √21 + 28
∴ Surface area of the toy = 103.62 cm2
= √441 + 784
Slant height = l = √1225 = 35 cm
10) Sand is filled in a cylindrical vessel of height 32 cm and radius of its base is
18 cm. This sand is completely poured on the level Total surface area of the toy = Curved surface + Curved surface
ground to form a conical shaped heap of sand. If the area of the cone area of the hemisphere
height of the conical heap is 24 cm. Find the base = π r l + 2 π r2
radius and slant height of the conical heap. = π r (l + 2 r)
June-2022
Ans. : Height of the cylinder = h1 = 32 cm = x 21 (35 + 2 x 21)
Radius of the cylinder = r1 = 18 cm = 22 x 3 (35 + 42)
Height of the conical shaped heap = h2 = 24 cm = 66 x 77
Radius of the conical shaped heap = r2 = ? Total surface area of the toy is 5082 cm2 .
Slant height of the conical shaped heap = l = ? 12) A social Welfare Association decides to supply drinking water for the
Volume of the cylinder = Volume of the conical shaped heap flood affected people. The drinking water is filled in a water tanker which is
in the shape of a cylinder with hemispherical end as shown in the figure.
𝜋r12ℎ1 = 𝜋r22ℎ2 The whole length of the tanker is 4.2 metre and the diameter of base of the
r12ℎ1 = r22ℎ2 cylinder and two hemispheres are each 1.2 m. If they distribute drinking
water to 60 people in a container, each is in the shape of a cylinder of radius
182x32 = r22x24 21 cm and height 50cm, find the quantity of water left in the tanker
𝟐𝟐
r22 = after distribution in litre. (𝜋 = ) 5-PREP-2020
𝟕

Sri.Veerendra.H.M. Asst.Teacher. G.P.U.C. Megaravalli. Thirthahalli Tq Ph: 8762624683 Page-205 Sri.Veerendra.H.M. Asst.Teacher. G.P.U.C. Megaravalli. Thirthahalli Tq Ph: 8762624683 Page-206
APPLICATION QUESTIONS :
1. How many cubic metres of earth must be dug out to make a well 28m deep and
1.2m 2.8m in diameter ? Also, find the cost of plastering its inner surface at Rs. 4.50
per sq. metre. Ans.: 172.48 m3 , Rs.1108.80
2. The radius of a solid right circular cylinder increases by 20% and its height
4.2m.
decreases by 20%. Find the percentage change in this volume. Ans. : 15.2%
Ans : Diameter of hemisphere = 1.2m. , Radius = r = 0.6m. 3. A heap of wheat is in the form of a cone of diameter 16.8m and height 3.5m.
The whole length of the tanker = 4.2m Find its volume. How much cloth is required to just cover the heap ?
∴ Height of the cylinder = h = (4.2-1.2) = 3m Ans.: 258.72m3 ; 240.24m2
4. A vessel, in the form of an inverted cone, is filled with water to the brim, its
Volume of = Volume of + Volume of + Volume of height is 20 cm and diameter is 16.8cm. Two equal solid cones are dropped in it so
tanker hemisphere cylinder hemisphere that they are fully submerged. As a result, one-third of the water in the original cone
3 2
Volume of water tanker = πr + πr h + πr 3
overflows. What is the volume of each of the solid cones submerged ?Ans.246.4cm3
5. Eight metallic spheres; each of radius 2 mm, are melted and cast into a single
Volume of water tanker = πr3 + πr2h
sphere. Calculate the radius of the new sphere. Ans. : 4mm.
Volume of water tanker = πr2( r + h) 6. A girl fills a cylindrical bucket 32 cm in height and 18cm in radius with sand.
She empties the bucket on the ground and makes a conical heap of the sand.
= x(0.6)2( x0.6 + 3)
If the height of the conical heap is 24cm, find:
= x0.36x3.8 i) the radius and
Volume of water tanker = 4.299m3 ii) the slant height of the heap. Ans. : r = 36cm and l = 43.3cm
7. Water flows through a cylindrical pipe of internal diameter 7cm at 5 metre per
1m3 = 1,000 litre.
second. Calculate the volume, in litres, of water discharged by the pipe in
∴ Quantity of water in the tanker = 4.299x1,000 = 4299litre. one minute. Ans, : 1,155 litres
Volume of 60 cylindrical containers = 60 𝜋𝑟2ℎ 8. A conical tent is to accommodate 77 persons. Each person must have 16 m3 of air
to breathe. Given the radius of the tent as 7m, find the height of the tent and also
= 60 x x 21 x 21 x 50 its curved surface area. Ans. : h = 24m C.S.A = 550m2
= 60 x 22 x 63 x 50
Volume of 60 cylindrical containers = 41,58,000 cm3
1 Litre = 1000 cm3
41,58,000 cm3 = = 4158 Litre
Quantity of water left in the tanker
after distribution = 4,299 – 4,158 = 141 Litre

“ Nothing is impossible, the word itself says I’m possible ”


***** All the Best *****

Sri.Veerendra.H.M. Asst.Teacher. G.P.U.C. Megaravalli. Thirthahalli Tq Ph: 8762624683 Page-207 Sri.Veerendra.H.M. Asst.Teacher. G.P.U.C. Megaravalli. Thirthahalli Tq Ph: 8762624683 Page-208

You might also like